Вербальная часть

Page 1

CONTENTS INTRODUCTION .......................................................................................................................... 5 READING COMPREHENSION ................................................................................................... 7 READING COMPREHENSION BANK ..................................................................................... 24 CRITICAL REASONING ............................................................................................................ 60 CRITICAL REASONING BANK.............................................................................................. 110 SENTENCE CORRECTION...................................................................................................... 168 SENTENCE CORRECTION BANK ......................................................................................... 221 ANALYTICAL WRITING ASSESSMENT .............................................................................. 297 EXAMPLES OF ESSAYS ......................................................................................................... 303 WRITING BANK ....................................................................................................................... 306 PUTTING IT ALL TOGETHER ................................................................................................ 309


GMAT COURSE SYLLABI GMAT Fundamental Lesson

RC

CR

Session 1

RC basics; Reading strategy

Session 2

Question types

Session 3

SC

GMAT Overview; Pretest Argument structure; Assumption questions Strengthen q-s; Weaken q-s; Except q-s SC Basics; SubjectVerb Agreement

Session 4

Inference q-n

Session 5

Minor q-s: Flaw q-s; Explain q-s; Structure q-s

Session 6

Revision, practice and time management

Parallelism; Comparisons Modifiers, Verbs

CR revision, time management

Session 7

Pronouns; Minor topics (Quantity words, Word pairs)

Session 8

SC Revision

RC RC basics; Reading strategy Question types (Global, Detail, Inference, Attitude, Special question formats)

Lesson

CR Argument structure Assumption questions Strengthen questions Weaken questions Inference questions Flaw questions Explain questions Except questions

2

AWA; IR Final Test, Preparation strategy

Session 9

Lesson

Other

Lesson

SC

Lesson

Other

SC Basics

GMAT Overview

Subject-Verb agreement

Pretest

Modifiers

AWA

Parallelism

IR

Comparisons

Final test

Verbs

Preparation strategy

Pronouns Minor topics (quantity words, word pairs)


GMAT Adaptive Lesson #

RC

CR

Session 1

RC Basics; Reading Strategy; Question types

Session 2 Revision, practice and time management Session 3

SC

GMAT Overview; Pretest Argument structure; Assumption q-s; Strengthen q-n; Weaken q-n

SC Basics; SubjectVerb Agreement; Modifiers

Inference q-n; Minor Q-s: Flaw qn; Explain q-n; Except q-n

Parallelism; Comparisons, Verbs, Pronouns; Minor topics (Quantity words, Word pairs) AWA, IR, Final Test, Preparation Strategy

Session 4

Lesson

Other

RC RC basics; Reading strategy

Question types (Global, Detail, Inference, Attitude, Special question formats)

Lesson

CR

Lesson

SC

Lesson

Other

Argument structure; Assumption questions

SC Basics

GMAT Overview

Strengthen qn; Weaken qn

Subject-Verb agreement

Pretest

Inference questions

Modifiers

AWA

Flaw questions

Parallelism

IR

Explain questions

Comparisons

Final test

Except questions

Verbs

Preparation strategy

Minor topics (quantity words, word pairs)

3


Introduction

What is the GMAT? The Graduate Management Admission Test® (GMAT®) examination is a standardized test designed to measure basic verbal, mathematical, and analytical writing skills that have been developed over a long period of time through education and work. It does NOT measure:  knowledge of business  job skills  specific content in undergraduate or first university course work  subjective qualities such as motivation, creativity, and interpersonal skills  abilities in any other specific subject area The total score ranges from 200 to 800. Scores above 700 or below 250 are unusual. The average is 540.

What is the Analytical Writing Assessment Section? The Analytical Writing Assessment (AWA) is designed as a direct measure of the test taker's ability to think critically and communicate ideas. The AWA consists of one writing task— Analysis of an Argument. The task must be completed in 30 minutes. A specific knowledge of the essay topic is not necessary. Only the test taker’s capacity to write analytically is assessed. For the Analysis of an Argument section, test takers need to analyze the reasoning behind a given argument and write a critique of that argument. They are not asked to present their own views on the subject. The Analysis of an Argument tests their ability to formulate an appropriate and constructive critique of a specific conclusion based on a specific line of thinking.

What is the Verbal Section? The Verbal section of the GMAT® exam measures the test taker's ability to:  read and comprehend written material  reason and evaluate arguments  correct written material to make it conform to the norms of standard written English Three types of multiple-choice questions are used on the Verbal section of the GMAT (41 questions) —Reading Comprehension, Sentence Correction and Critical Reasoning. Time – 75 minutes. Approximately equal numbers of all question types will be on the test. Reading Comprehension (RC) Reading Comprehension passages are around 350 words long. Topics from the social sciences, physical or biological sciences, and business-related areas (marketing, economics, human resource management, etc.) are discussed. All questions are answered on the basis of what is stated or implied in the reading material. No specific knowledge of the material is required. Reading Comprehension passages are accompanied by interpretive, applied, and inferential questions. The questions measure the test taker’s ability to understand, analyze, and apply information and concepts presented in written form. Sentence Correction (SC) Sentence Correction questions ask, which of the five choices best expresses an idea or

4


relationship. The questions will require test takers to be familiar with the stylistic conventions and grammatical rules of standard written English. Test takers must also demonstrate their ability to improve incorrect or ineffective expressions. Critical Reasoning (CR) Critical Reasoning questions are designed to test the reasoning skills involved in making arguments, evaluating arguments, and formulating or evaluating a plan of action. Questions are based on materials from a variety of sources. No familiarity with the specific subject matter is needed. What is it important to know?  If you do not know for sure, try to guess. Eliminate the obviously wrong answer choices and choose the remaining.  Beware of penalty. Against all odds, try to complete the section as you will be penalized for not answering the questions.  Every question is important. Especially the first 15. There is a number (approximately one quarter) of experimental questions, which are not included in your result.  Do not hesitate to jot things down.  You have about 1 minute 40 seconds to answer each question. Try to keep the rhythm and not spend more than 2 minutes on any question. Keep an eye on the timer all the time.

What is the Integrated Reasoning Section? The Integrated Reasoning section measures your ability to understand and evaluate multiple sources and types of information – graphic, numeric, and verbal – as they relate to one another; use both quantitative and verbal reasoning to solve complex problems; and solve multiple problems in relations to one another. Four types of questions are used in the Integrated Reasoning section:  Multi-Source Reasoning  Table Analysis  Graphics Interpretation  Two-Part Analysis Integrated Reasoning questions may be quantitative, verbal or a combination of both. You will have to interpret graphics and sort tables to extract meaning from data, but advanced statistical knowledge and spreadsheet manipulation skills are not necessary. You will have access to an online calculator with basic functions for the Integrated Reasoning section, but note that the calculator is not available on the Quantitative section.

5


Part I Reading Comprehension RC consists of a passage and several questions. The questions in this group are based on the content of the passage. After reading the passage, choose the best answer to each question. Answer all the questions accompanying the passage on the basis of what is stated or implied in the passage.

2 important facts: - You are looking for the best answer among 5 provided, not just the right one - You must use only the given (stated or implied) information; applying additional knowledge can get you into trouble.

The types of passages: NATURAL SCIENCE: scientific facts, intimidating jargon. These passages look very complicated, but usually have a clear structure and provide precise, unambiguous information. Remember, you are not supposed to be familiar with DNA life cycle or mechanisms of volcano formation. Rather, your ability to deal with and understand ANY topic quickly is tested. SOCIAL SCIENCE: passages are diverse; they can range from history to a brief biography of a famous person. Although these passages usually use a more familiar wording than do NATURAL SCIENCE passages, they can be much more complicated – the ideas are more confusing, the sentence structures are tangled, the author’s point is less precise. BUSINESS: you might read about financial or economic theory, the ins and outs of investing, or the way certain financial institutions operate. These topics will frequently look familiar to you and you might have, on average, more correct answers than for other RC passages. The only thing to watch out is application of your own, out-of-the-passage-scope knowledge on the subject.

6


1) READING STRATEGY (RS) (GoGMAT Session 3) Reading strategies of test-takers differ according to the level of language, speed of reading and skills. RS 1 - The most common RS is to read the whole text. This one should be applied when the person has at least upper-intermediate level and is a quick reader. Advantages of reading the whole passage are that you can understand the main idea and know the details, as some people say :) on the other hand, there is no question on the test asking about our understanding of the information. RC is about finding necessary reference and using this reference to answer every particular question. Moreover, while reading everything, people often get confused with all the details given. So, it is advisable to read everything when the text consists of one paragraph. RS 2 - Another possibility is to read certain parts of the passage, to identify the key subject, and to make a map of the passage. This can give you the opportunity to know the main idea and structure of the text as well as to know to which paragraph to refer with a detailed question. Key subject of the passage - a particular someone or something that is depicted throuout the whole passage. It may be a person, a book, a theory, 2 controversial opinions, 3 studies etc. Map of the passage - a short plan of the passage that indicates key structural elements. For example: paragraph 1 (theory+contradiction), paragraph 2 (details on contradiction), paragraph 3 (validation of theory). Maps vary according to passages. Some text will have a problem, a study, an opinion, a phenomen etc. To find key subject and to make a map one needs to read the first paragraph and topic sentence ( first sentence) of every other paragraph. While reading try to restate sentences into 1 or 2 words ( short phrases) as is mentioned in the previous paragraph.

7


LIST OF KEYWORDS Below you will find a list of keywords in groups according to their function. This list is a usefull tool to find necessary reference to not only making a map of a passage but also answering detailed questions. If a paragraph begins with "However, the theory", then in a map we would sign this paragraph as "disadvantages/contradiction of a theory". If a detailed question refers to a sentence beginning with "For instance", then the purpose of mentioning this info is to "give an example/illustrate". Noticing keywords and knowing their functions will help you to know the structure of the passage and answer purpose questions. Illustration Keywords signal that an example is about to arrive. For example For instance To illustrate Reference Keywords indicate that we are sighting a source of information. According to In the words of Jean-Paul Sartre As Maya Angelou says Comparison Keywords provide signs of comparison. Like As does In contrast to Compare with Unlike Sequence Keywords announce that a logical arrangement of the author's reasoning is taking place. The order of One more Another Second (third, fourth, etc.) Next Finally Continuation Keywords announce that more of the same is about to come up. And is the most common one; others include: Also As well as Likewise Furthermore Moreover In addition At the same time Contrast/ Contradiction Keywords signal an opposition or shift, they are also called “turnarounds�. Although By contrast

8


Despite However But Conversely Whereas Nevertheless Still Yet Otherwise Though Conclusion Keywords signal that the author is about to sum up or announce his or her thesis. The most common one is therefore, to which we can add: Consequently Thus As a result Hence So In conclusion We can conclude that Evidence Keywords, which tell you that the author is about to provide support for a point. Here are the most important Evidence Keywords: Because Since Given that For Emphasis Keywords may be the most welcome. If we are supposed to read for the author’s point of view, and we are, what better than to stumble across words and phrases whose sole purpose is to announce "I, the author, find this important"? Note these well: Above all Indeed Most of all Essentially Primarily Moreover

9


MAPPING You might find it useful to make quick notes while you read. These notes will help you to derive a clear structure of the passage, to stay concentrated and to avoid memorizing important ideas that you will come across. Simply summarize the important ideas of each paragraph and write them down as briefly as possible in an understandable and helpful form. Do not include details in your map. In the example below you will see highlighted phrases that help to make a map based on RS 2. Below the passage we will provide our map to this passage. Remember that maps vary according to texts. Example (K) Congress has had numerous opportunities in recent years to reconsider the arrangements under which federal forest lands are owned and managed. New institutional structures merit development because federal forest lands cannot be efficiently managed under the hierarchical structure that now exists. The system is too complex to be understood by any single authority. The establishment of each forest as an independent public corporation would simplify the management structure and promote greater efficiency, control, and accountability. To illustrate how a system for independent public corporations might work, consider the National Forest System. Each National Forest would become an independent public corporation, operating under a federal charter giving it legal authority to manage land. The charter would give the corporation the right to establish its own production goals, land uses, management practices, and financial arrangements, within the policy constraints set by the Public Corporations Board. To assure economic efficiency in making decisions, the Public Corporations Board would establish a minimum average rate of return to be earned on assets held by each corporation. Each corporation would be required to organize a system for reporting revenues, costs, capital investments and recovery, profits, and the other standard measures of financial health. While the financial objective would not necessarily be to maximize profit, there would be a requirement to earn at least a public utility rateof-return on the resources under the corporation's control. Such an approach to federal land management would encourage greater efficiency in the utilization of land, capital, and labor. This approach could also promote a more stable work force. A positive program of advancement, more flexible job classifications, professional training, and above all, the ability to counter outside bids with higher salaries, would enable a corporation to retain its best workers. A third advantage to this approach is that federal land management would become less vulnerable to the politics of special interest groups. Instructions: read each paragraph and try to restate the information as short as possible. You should receive up to one phrase representing each paragraph with essential content/idea of that paragraph. Then write these phrases in the “Par.#� space below. These three phrases, taken together, should further constitute the main idea of the passage. Try to keep this map while dealing with questions and answer choices.

10


Key Subject:___________________________________________________________________ Par. 1:________________________________________________________________________ Par. 2:________________________________________________________________________ Par. 3:________________________________________________________________________ Sample Map: Key Subject: Management of federal forest lands Par. 1: Problem + solution Par. 2: Example of solution Par. 3: Advantages of solution

11


2) READING COMPREHENSION QUESTION TYPES (QT) 1. GLOBAL QUESTIONS (GoGMAT Session 3) These questions refer to the whole passage, but it doesn't mean that you have to read the text. We suggest you to use the map based on proper RS. Steps to answer the question: 1. Read the question 2. Identify QT (based on examples provided below) 3. Find the answer with key subject of the text (for main idea/purpose questions) or an answer that best agrees with your map (for structure questions). The subtypes of Global Questions are presented below: 1) main idea/purpose These questions usually sound like: • "Which of the following best states the central idea of the passage?" • "Which of the following most accurately states the main idea of the passage?" • "Which of the following is the principal topic of the passage?" • "The main topic of the passage..." • "The primary purpose of the passage as a whole is to..." • "The primary focus of this passage is on which of the following?" • "The main concern of the passage is to..." • "In the passage, the author is primarily interested in...." • "The passage is chiefly concerned with..." 2) structure (organization) (GoGMAT Session 6) These questions usually sound like: • "Which of the following best describes the organization of the passage?" • "Which of the following best describes the organization of the first paragraph of the passage?" • "One function of the third paragraph is..." Eliminate answers that: • Are too specific, dealing with just one portion of the passage (no details – factors, examples, illustrations – will be mentioned in the answer; the answer should include as many important ideas that you have found mapping as possible); • Are too broad, going outside the scope of the passage (avoid out of scope information, i.e. information not mentioned in the passage); • Contradict the passage • Contain extreme language; if an answer choice contains extreme wording, ask yourself whether this wording is clearly supported by the passage (if not, the answer is wrong).

12


Global questions practice Instructions: choose one answer out of five given. All the questions are asked according to reading passage on page 6. You should spend no more than 1,5 minutes per question. 1.

The primary purpose of this passage is to

(A) (B) (C) (D) (E)

suggest that the National Forest System is plagued by many problems. argue that it is necessary to restructure the management of federal forest lands. insist that private corporations be allowed to manage the country's natural resources. discuss the role of private corporations in the management of the National Forest System. highlight the competing needs of public agencies managing national resources.

2.

Which of the following best describes the organization of the passage?

(A) (B) (C) (D) (E)

A proposal is made and then supporting arguments are set forth. One claim is evaluated and then rejected in favor of another claim. A point of view is stated and then evidence for and against it is evaluated. A problem is outlined and then various solutions are discussed. Opposing opinions are introduced and then debated. 2. DETAIL QUESTIONS (GoGMAT Session 9)

Many Reading Comprehension questions will ask you about specific details in the passage. On detail questions, you ALWAYS have to go back to the passage to research the answer. These questions usually sound like: "According to the passage,...." "The passage states that ...." “The passage mentions which of the following as…. ” Steps to answer the question:     

choose a keyword in the question; scan the passage and locate all cases in which this keyword (or its synonym) is mentioned; pick 1 case that is most relevant to the question; Read only that 1 sentence with reference and look for a proper answer If that one sentence is not enough, go back and read previous and following sentence (do not read all the three sentences at once as wrong answers speculate other peaces of information to misguide you). Three sentences within one paragraph should be enough to answer a detailed question answer the question.

Very often the correct answer will be a paraphrase of the information that you have found. Detail questions can be of two subtypes: Line Reference: Sometimes the test maker is kind enough to tell you exactly where to look in the passage to research the answer (boldfaced words, words in “quotation marks”, number of line). Since you are told exactly where to go in the passage, go ahead and reread the sentence containing the line reference. Try to formulate your own answer to the question before you look

13


at the answer choices. Sometimes you will have to read a line or two before and after the reference to get a sense of the context. But not all the information you read is important, sometimes one sentence is sufficient to answer the question! Usual detail: These questions only give you a hint for which information to look in the passage. Here the keywords in the question or in the answer choices will help you. Pay attention to the question’s main subject and to its description. Do not try to answer these questions without referring to the passage! Eliminate answers that: • Contradict the passage • Use similar wording, but actually change the meaning of what is said in the passage • Go beyond the scope of the passage, stating things that are not said • are given in the text, but do not answer the question being asked • Use extreme language Detail questions practice Instructions: choose one answer out of five given. All the questions are asked according to reading passage on page 6. You should spend no more than 1,5 minutes per question. 3.

According the passage, the present problems of federal forest lands derive mainly from

(A) (B) (C) (D) (E)

overuse by the population. inefficient organization. hostility from special interest groups. the corporate mentality of Congress. opposition to reform by government employees. 3. INFERENCE QUESTIONS (GoGMAT Session 9)

Inference questions are also quite common on Reading Comprehension, and may be either global or detail. Any time you are asked to determine what a passage suggests or what can be inferred from a passage, you are asked to draw an inference, rather than find what is explicitly stated in the passage. These questions usually sound like: "It can be inferred from the passage…" “The author implies that….” “The passage suggests that….” "Which of the following, if true, would most strengthen the hypothesis mentioned in lines 17-19?" "With which of the following statements regarding chaos theory would the author be most likely to agree?" “Which of the following would most likely be the next topic the author would discuss?” Steps to answer the question:  identify the type of inference (either global or detailed)  If it’s a global inference find the answer that agrees with the key subject and the map  If it’s a detailed inference use the steps to answer detailed questions  Remember that the answer should be indirect and sometimes vague As we will discuss later in the Critical Reasoning chapter, the GMAT writers do not expect you to

14


infer too much when they ask an inference question. Eliminate answers that: • Contradict the passage • Are too strongly worded • Go beyond the scope of the passage, stating things that can not be directly inferred from the information given Inference questions practice Instructions: choose one answer out of five given. All the questions are asked according to reading passage on page 6. You should spend no more than 1,5 minutes per question. 4.

The author suggests that administrators of federal forest lands have been handicapped by which of the following?

(A) The public expectation that federal forest lands will remain undeveloped (B) The failure of environmental experts to investigate the problems of federal forest lands (C) The inability of the federal government to compete with private corporations for the services of skilled professionals (D) The unwillingness of Congress to pass laws to protect federal forest lands from private developers (E) The difficulty of persuading citizens to invest their capital in a government-run endeavor 4. ATTITUDE QUESTIONS (GoGMAT Session 6) These questions ask you about the author’s attitude towards the main subject of the passage or towards a detail mentioned in the passage. Because most passages are written in a scientific, official tone and deal with serious subjects, and because the test-makers adhere to political correctness, the correct answers to attitude questions must use emotionally neutral wording. The commonly correct answers for a negative attitude are “critical” and “skeptical”. These questions usually sound like: "The author’s attitude towards …. can be described as" “The tone of the passage can be described as” “The author most likely regards…. with” Steps to answer the question:  Find the subject from the question in the text  Identify whether the authors attitude is positive or negative  Choose a formal answer with the closest attitude to the one given in the text Eliminate answers that:  Are emotional and show feelings (angry, happy, bitter, anxious etc.)  Provide “indifferent” or “ambivalent” as an answer  Contradict the passage  Give extreme language Attitude questions practice Instructions: choose one answer out of five given. All the questions are asked according to

15


reading passage on page 6. You should spend no more than 1,5 minutes per question. 5.

The author's attitude towards the "hierarchical structure" can best be characterized as

(A) resigned (B) admiring (C) skeptical (D) bitter (E) ambivalent 5. SPECIAL QUESTION FORMATS While they have all the similar contents to the regular question types discussed above, these special format questions will require a little bit more of your attention. 1) Roman numeral questions Roman Numeral questions are not so often on the GMAT, but are generally considered harder than average. This is because these questions require you to look for three pieces of information in the passage with which to evaluate the three enumerated options. We recommend that you consider each option one at a time and that you eliminate answer choices as you go along (sometimes you will get lucky and will not have to evaluate all three options). Steps to answer the question:  choose a keyword in the question and identify a paragraph with reference;  read the first roman numeral statement and check it with that paragraph;  do the same with the rest of statements (do not be reluctant to check every statement)  remember that we are to choose answers tht we cannot deny 2) EXCEPT questions EXCEPT questions are considered the biggest time-consumers of all Reading Comprehension questions. This is because these questions require you to verify all answer choices and find an answer choice that IS NOT correct. In other words, you are looking for the wrong answer choice. Steps to answer the question:  choose a keyword in the question and identify a paragraph with reference;  read the first sentence with reference, find answer with the same meaning, eliminate it;  do the same with the rest of the paragraph;  you should be left with one answer that doesn’t have a reference; Special question formats practice Instructions: choose one answer out of five given. All the questions are asked according to reading passage on page 6. You should spend no more than 1,5 minutes per question. 6.

According to the passage, the responsibilities of a corporation authorized to manage a National Forest should include which of the following?

I.

Establishing formal procedures for making official reports of the corporation's financial wellbeing Earning at least a minimum return on the corporation's resources and assets

II.

16


III.

Achieving a maximum level of profits on the corporation's capital investments

(A) (B) (C) (D) (E)

I only II only III only I and II only I, II, and III

7.

According to the passage, all of the following are potential benefits of forming public corporations to manage federal forest lands EXCEPT:

(A) (B) (C) (D) (E)

less turnover of personnel more effective management of natural resources the ability to offer competitive salaries less vulnerability to special interest group expansion of federal forest land boundaries

17


GoGMAT Passage

18


2.

The primary purpose of the passage above is

(A) (B) (C) (D)

to explain the effect of the Catholic Church on South American cultures to show the difference between the Olmec culture and the following civilizations to introduce Olmec culture, its origin and traces in subsequent civilizations in South America to hypothesize how South American cultures would have developed, had they not been affected by colonization (E) to compare accomplishments of Olmec civilization with those of other contemporary civilizations. 3. Which of the following titles would best summarize the content of the passage? (A) (B) (C) (D) (E)

The South American Civilizations: The Olmecs Evolution of South American Civilizations West African Origin of Olmec Civilizations Religious Beliefs of Olmec People Olmec Civilization: The First Democratic Society

4.

It can be determined from the passage that the Olmecs shared all the following with later civilizations EXCEPT:

(A) (B) (C) (D) (E)

a 365-day year fertility and war gods growing corn human sacrifice pyramid construction

5.

According to the passage, which of the following statements is true of the Almecs?

(A) (B) (C) (D)

They were the earliest known culture They had a democratic political organization They were literate They had one god whom they worshiped and the ruling elite had to make people believe they were godlike to maintain power (E) Their culture was under significant influence of Spanish colonists. 6.

The passage implies that two peoples inhabiting different parts of the earth may be considered of the same descent if

I. they have similar religious beliefs and rituals II. they speak languages that could have originated from the same dialect III. their architectural systems are similar (A) I only (B) II only (C) III only

19


(D) I and III (E) I, II and III 7.

The author’s attitude towards the consequences of the spread of Catholicism throughout South America can be described as

(A) (B) (C) (D) (E)

Complimentary Critical historically unjustified biased impartial

20


GoGMAT Passage Key 1.

OA: D

Explanation: In the last paragraph, the author discusses how the religion, language, and architecture of the Olmecs lasted through at least three other civilizations, implying that the people themselves were no longer around. The same lines also mean that Choice (C) is not correct. The fourth paragraph, which links a person’s spirit with the Jaguar, disputes choice (A). The end of the passage, which links the introduction of Catholicism with the coming of the Spaniards and questions whether it helped or hurt the people, contradicts choice (B). Choice (E) is not correct, because the symbolic language of the Olmecs was mentioned. 2.

OA: C

Explanation: Choice A deals only with the last passage, and the Catholic Church is mentioned only to show what brought end to traces of Olmec culture in South America. Choice B is simply not true. If anything, the passage points out some similarities. Choice C is correct. The passage introduces Olmec culture, talks about its possible origin and about traces it left in subsequent civilizations of Teotihuacans, Zapotecs and Maya. Choice D is not correct as no such hypothesis is mentioned or implied. Choice E. At some point the author does show similarities, but mostly with later civilizations and only a small portion of passages is concerned with those similarities. 3.

OA: A

Explanation: Choice B is only slightly implied, but the passage as a whole deals with one particular civilization—the Olmecs. Choices C and D describe only part of the passage and are too narrow. Choice E is simply wrong, nothing implies Olmecs were democrats. Choice A is the best option. 4.

OA: D

Explanation: All the choices are mentioned in the passage except human sacrifice. Religious rituals are mentioned, but they are not specified. 5.

OA: C

Explanation: Choice (A) is too broad, “the Olmecs are believed to be the basis of all subsequent cultures in that region of the Americas” not the earliest known culture in general. Choice (B). No they had ruling elite. Choice (C) is correct. The last passage states they had a symbolic language. Choice (D) is partially correct. Ruling elite had to make people believe they were godlike, but there was no single god they worshiped. Choice (E) is not true. Their culture no longer existed when Spanish colonists came to America.

21


6. OA: B Explanation: Take a look at the third paragraph. Of the three statements only similarity in language is mentioned as reason that allows to hypothesize that Olmecs came from Africa. Other two options—similar religious beliefs and architectural systems—are also mentioned in the passage, but as traces of Olmec civilization in later civilizations in the same area, which is not what we need. Note that similar religious rituals and architectural systems might in real life be used to assume common descent; however, the passage does not mention them in this regard, and thus they are not correct. 7. OA: E

Explanation: The author states a historical fact that the Catholic Church replaced numerous gods with the new One. He then points out two opposing opinions: some think that Catholicism changed South American life for better, while other point out the poverty in the Catholic South America. The author does not support or criticize either position, he never states his own view on the issue and his attitude can be best described as impartial.

22


READING COMPREHENSION BANK

(PR) Passage 1 Within most animal species, the males must do their best to attract females by showing off—by attempting either to demonstrate sexual prowess or to intimidate rivals. A new study, however, suggests that males are actually submitting to a more genetic imperative. By displaying their most prized attributes or talents, hopeful males do the best they can to show off superior genetic qualities that lesser males cannot mimic. Biologists at the University of Missouri conducted this new study by analyzing the mating calls of the gray tree frog. Females have been shown to gravitate toward males whose calls last longest, and it has long been theorized that a male's lengthy mating call is linked to superior fitness and energy. Tree frogs were chosen for the experiment for two very important reasons. Since frogs fertilize their eggs externally, it is easy to trace the genealogy of each tadpole. Secondly, male frogs are utterly uninvolved with raising their offspring. This allows the scientists to address the "nature vs. nurture" conundrum directly by removing any chance that the tadpoles are "learning" anything from their fathers. Whatever strength or weakness the offspring displays, that characteristic must have been passed down through the father's genes. To start the experiment, eggs were harvested from several females and then split into two groups. The first cluster was fertilized with the sperm of a long-calling male, the second with that of a male whose call was demonstrably shorter. The scientists compared the progress of all tadpoles with different fathers but the same mother. Mating calls are strictly a male trait, so the biologists later planned to compare the mating calls of certain males with those of their male offspring. The results were astounding. The physical characteristics of all the male tadpoles were virtually identical at first, but within weeks the children of long-calling males grew into faster and stronger tadpoles who eventually would

1.

The primary focus of the passage is on which of the following?

(A) supporting a new theory by providing a new explanation for an accepted mode of behavior (B) evaluating the results of two separate experiments and contrasting the relative merits of each one (C) pursuing evidence in the "nature vs. nurture" debate (D) dismissing a current phenomenon as inconsistent with common trends (E) embracing a new system of analysis that is likely to overturn much of today's accepted knowledge 2.

According to the passage, which of the following is an accurate statement about the gray tree frog?

(A) Females teach their offspring to fend for themselves in the wild (B) Males are more energetic and physically fit than are females (C) Mating calls are restricted to the males of the species (D) Offspring acquire more of their genetic information from their mothers than from their fathers (E) The mating call of the female is hardly distinguishable from that of the male 3.

The passage suggests which of the following about the male tree frog?

(A) It is now no longer prudent to assume that the length of a frog's mating call is linked to its sexual prowess (B) Some females are attracted to long-calling males because of the quality of "parenting" that these males can provide (C) A male that exhibits a demonstrably shorter mating call is probably also betraying its physical inadequacy

23


metamorphose into frogs much sooner than the (D) A study comparing tadpoles with the same offspring of short-calling males. Such a finding father but different mothers would yield lends credence to the theory that calling is an similar results honest and reliable indicator of genetic quality. (E) A frog that is unable to sound a mating call will never reproduce

(PR) Passage 2 Most Americans are fascinated with their own history, particularly that of the colonial era. Whenever a crisis affects the current government, pundits and plebeians alike invoke the writings and teachings of the eighteenth century in order to support or denounce modern viewpoints. Many citizens wax nostalgic for the glorious times of their nascent union, when some of the most shrewd and free-thinking minds came together to construct "a more perfect Union." A new book by Nathan Parker, however, suggests that colonial New England was never the egalitarian Eden that modern Americans make it out to be. Popular imagination holds that the Puritans were a virtuous group determined to create a new government through direct democracy. Communities convened town-hall meetings, at which policies were debated and decisions were made by the will of the people. Many renowned international thinkers, such as France's Alexis de Tocqueville and Hector St. John Crevecoeur, praised this new American commitment to the voice of the common man. According to Parker, admiration for New England's first settlers is profoundly misplaced. In a typical show of their historical revisionism, Americans have mythologized these town meetings to the point of embarrassment. Parker asserts that town-hall meetings were open only to a select few male property owners who wielded a strong financial influence on the community. Thus, the laws that were put into effect as a result of these meetings hardly reflected the "consent of the governed." Therefore, there is vast evidence of voter apathy among the colonists. Citing the disparity between the roll calls of several meetings and the voter registries of the towns in which they took place, Parker demonstrates that attendance at town-hall meetings rarely exceeded 30

24

1.

The primary purpose of the passage as a whole is to

(A) (B) (C) (D) (E)

critique a system of logic clarify an ambiguity contrast two diverse notions discredit a commonly held perception question a dubious explanation

2.

Which of the following best describes the purpose of the second paragraph?

(A) It extols the virtue of the first American settlers who were not daunted by the prospect of creating an egalitarian society (B) It first introduces de Tocqueville and Crevecoeur, by whom Parker was first inspired to write (C) It lists several perceptions about the early American colonies that Parker believes to be more myth than fact (D) It serves to emphasize the massive impact that French thinking had on New England's first settlers (E) It provides evidence that the Puritans were not nearly as virtuous as they asserted themselves to be 3.

According to the passage, it is common practice to refer to the colonial era in order to

(A) advocate the importance of pure democracy to a fledgling capitalist nation (B) illustrate how American society has always depended upon the family unit that was so highly esteemed in the eighteenth century (C) establish a historical context for the celebrated writings of de Tocqueville and Crevecoeur


percent of all registered voters. (D) praise the perseverance of the Puritans, In making these points, Parker hopes to lie who never receive the recognition they so to rest the notion that simple, family-oriented richly deserve colonial New England was far preferable to the (E) indicate that the country's rampant growth modern America that many perceive has since its creation has caused it to stray outgrown pure democracy. His objective is to from its original path toward absolute lay bare the true nature of eighteenth-century democracy governance and thus assure Americans that progress can't kill an equality that never was. 4. According to the passage, Parker asserts which of the following about early colonial town meetings? (A) Those who owned property in the area served as representatives for everyone in the community (B) Attendance at these meetings was restricted to wealthy landowners (C) All registered voters were permitted to attend, but fewer than one-third of them actually did (D) They became the inspiration for what is known today as direct democracy (E) They were looked upon as models by the framers of the Constitution

(PR) Passage 3 Anyone who thinks that rabbits make cute and cuddly pets has never owned one, and has most definitely never worked as a farmer or gardener. To people whose livelihood depends on agribusiness, rabbits are nothing more than ravenous vermin that inflict millions of dollars in damage to crops meant for both animal and human consumption. Until now, no one had undertaken to quantify the annual cost to a farmer’s output for which a single rabbit is accountable. Great Britain’s Ministry of Agriculture, however, has shown itself to be up to the challenge. Gordon McKillop, a biologist at the Central Science Laboratory in York, England, just finished a study that monitored the appetites of rabbits let loose to graze on several crops. As a result, farmers can gauge rabbit damage more effectively, allowing them to anticipate the crops they will lose and make necessary compensation. During his three-year study, McKillop

1.

Which of the following statements best sums up the purpose of McKillop's experiment?

(A) He contrasted several methods for establishing more credible methods for controlling rabbit populations (B) He set out to express the damage inflicted by rabbits on farmers' crops in a more tangible, monetary sense (C) He endeavored to prove that rabbits are more destructive than most people perceive them to be (D) He hoped to determine the crop for which rabbits showed the most ardent appetite (E) He wanted to portray the rabbit in a less flattering manner 2.

The passage supplies information about each of the following EXCEPT

(A) the population density of rabbits

25


released a set number of rabbits into several enclosed regions, each containing one type of vegetation on which the rabbits subsisted. To keep numbers constant, each enclosure was surrounded by fence that was entrenched ten feet into the ground, and all rabbits released in a certain area were of the same sex. The rabbits did the least damage in the pens containing grass, which many farmers cultivate as grazing land for their livestock. The average rabbit ate almost 300 pounds of grass in one year, which reduced the yield of one hectare (about two and a half acres) by half a percent. This translates to more than $3 worth of damage per rabbit per year—a seemingly nominal sum until one considers that most grasslands are home to as many as forty rabbits per acre. The rabbits’ taste for barley was about the same as that for grass in terms of percentage, but the cost was calculated to be almost $7 per rabbit. By far, the most endangered crop was wheat, which rabbits munch at a rate that depleted normal yields by more than 1 percent of the maximum. Since wheat is also the most expensive on the open market, McKillop’s group calculated that one rabbit can eat almost $1 worth of the crop in one month. This can mean financial ruin for wheat farmers in areas with abnormally high rabbit populations. Farmers may now be able to attach a dollar value to the crops that rabbits feed on, but they still lack the most important piece of information that Dr. McKillop’s study did not reveal: how to stop them. Shooting and trapping rabbits is too time-consuming and inefficient to keep up with the approximate 2 percent increase in rabbit populations every year, and most rabbits have developed resistance to viral diseases such as myxomatosis and viral hemorrhagic fever that have been introduced to curb reproduction. Even the age-old remedy of releasing foxes on the property has been blocked by chicken farmers, whose commodity, according to the Ministry, contributes almost 14 percent of Britain’s gross domestic product.

(B) the best way to prevent rabbits from decimating a certain crop (C) the duration of McKillop's study (D) the rate at which rabbits normally reproduce (E) the amount of grass usually grown annually upon a hectare of land 3.

Each of the following can be inferred from the passage EXCEPT

(A) in the agricultural marketplace, barley is at least twice as expensive as grass (B) at one point, myxomatosis and viral hemorrhagic fever were more effective than they are now (C) the power wielded by a certain type of farmer is at least partly influenced by financial impact of that farmer's product (D) the cost incurred by farmers to rid themselves of large rabbit populations far exceeds the monetary damage done to the farmers' crops (E) rabbits are unable to tunnel through the ground at a depth that is greater than ten feet 4.

Which of the following hypothetical situations best exemplifies a potential problem that would most seriously undermine the merit of McKillop's study?

(A) Several rabbits develop a new strain of myxomatosis that renders each completely sterile. (B) Due to a decrease in supply, the price of barley suddenly doubles. (C) It is determined that younger, more energetic rabbits consume almost double the food that an older rabbit does. (D) A rare drought inhibits plant growth in the enclosures for several months. (E) Soon after the experiment begins, a predatory animal finds its way into some of the rabbit enclosures. 5. In the last paragraph, the author is primarily concerned with (A) exposing a problem to which McKillop's

26


study has failed to supply a solution (B) suggesting that rabbit farmers and chicken farmers are often at odds when it comes to agricultural legislation (C) citing evidence that McKillop's study is woefully incomplete (D) comparing the various methods that farmers have used in order to keep rabbit populations under control (E) establishing that foxes have an equal appetite for rabbits as they do for chickens

(K) Passage 4 The search for an explanation of the historically weak status of U.S. third-party movements is illuminated by examining the conditions that have favored the growth of a strong two-party system. Different interests and voting blocs predominate in different regions, creating a geographically fragmented electorate. This heterogeneity is complemented by a federal political structure that forces the major parties to find voter support at state and local levels in separate regions. For example, the Democratic Party long sought and drew support simultaneously from northern black urban voters and from segregationists. Such pressures encourage the major parties to avoid political programs that are too narrowly or sharply defined. The nondoctrinal character of U.S. politics means that important new issues and voting blocs tend to be initially ignored by the major parties. Such issues—opposition to immigration and the abolition of slavery are two historic examples—tend to gain political prominence through third parties. Ironically, the same factors that lead to the emergence of third parties contribute to the explanation of their failure to gain national political power. Parties based on narrow or ephemeral issues remain isolated or fade rapidly. At the same time, those that raise increasingly urgent social issues also face inherent limits to growth. Long before a third party can begin to broaden substantially its base of voter support, the major parties are able to move to attract the minority of voters that it represents. The Democratic Party, for instance, appropriated the

1.

The primary purpose of this passage is to

(A) examine the appeal of U.S. national third parties to the electorate at state and local levels (B) trace the historical rise and decline of third party movements in the United States (C) explain why most U.S. third party movements have failed to gain majorparty status (D) demonstrate that U.S. politics has traditionally been non-ideological in character (E) suggest a model to explain why certain U.S. third party movements have succeeded while others have failed 2.

Which of the following does the author suggest was an important factor in the establishment of the Republican Party as a major national party?

(A) the polarization of national opinion at the time of a major social crisis (B) the unique appeal of its program to significant sectional interests (C) the acceptance of its program by a large bloc of voters unrepresented by a major party (D) the simultaneous decline of an established major party (E) the inability of the major parties of the era to appeal to all sectional interests

27


agrarian platform of the Populist Party in 1896, and enacted Socialist welfare proposals in the 3. 1930s, in both cases winning much of the popular bases of these parties. Except for the Republican Party, which gained national prominence as the Whigs were declining in the 1850s, no third party (A) has ever achieved national major-party status. Only at state and local levels have a handful of (B) third parties been sustained by a stable voting bloc that remains unrepresented by a major party. (C)

According to the author, the major factor responsible for the rise of third parties in the United States has been the

domination of major parties by powerful economic interests inability of major parties to bring about broad consensus among a variety of voters and interest groups slow response of major parties to new issues and voting groups (D) exclusion of immigrants and minorities from the mainstream of U.S. politics (E) variety of motivations held by voting blocs in different regions

(K) Passage 5 Despite increasing enrollments of women in medical schools, feelings of isolation among women medical students persist. Women students still have to contend with the social stereotype of a male doctor. In addition, institutions themselves may intentionally foster feelings of separateness. Comparatively few women are hired for faculty positions, thus offering women students few role models. The pervasive sexual humor of male doctors and students further intensifies the woman students' alienation. Alienation, in turn, negatively affects individual self-perception. As women enter medical study in increasing numbers, they may feel less at odds with their peers and the teaching establishment. Institutional bias will, no doubt, also change in response to changing societal values. However, we should not wait passively for gradual social processes to bring changes; schools must provide current students with support services designed to meet women's needs. In a recent study, 48 percent of the women questioned rated a student support group as the most important support service a school can provide.

28

1.

The passage cites all of the following as causing psychological problems for women medical students EXCEPT

(A) feelings of alienation from teachers and fellow students (B) prevailing societal conceptions about doctors (C) declining admissions of women to medical schools (D) expressions of sexism by peers and superiors (E) absence of positive female models for women students 2.

The author of the passage would be most likely to agree with which of the following?

(A) Medical schools practice widespread discrimination on the basis of sex (B) Gender-based stereotyping encourages feelings of isolation among women medical students (C) Some medical school policies are deliberately designed to make women students feel isolated (D) Social norms must change before institutional bias can decline (E) The majority of women medical


students have a negative self-image 3.

The passage suggests that which of the following would be likely to reduce the isolation felt by women medical students?

I.

An increased proportion of women in medical schools II. An increase in the number of women filling faculty positions in medical schools III. An increase in support services available to women students

(A) (B) (C) (D) (E)

I only II only I and II only I and III only I, II, and III

4.

In terms of its tone and form, the passage is most appropriately described as

(F) (G) (H) (I) (J)

an impassioned polemic an indignant protest a reasoned appeal a detached summary a biased speculation

1.

Which of the following best describes the organization of the passage?

(K) Passage 6 For many years, the observation that certain intensely bright young stars are concentrated along the spiral arms of disk-shaped galaxies remained unexplained. But recent research suggests both a solution to the puzzle of these "0stars," which are a million times brighter than the sun, and a mechanism that may partially explain the process of star formation in general. Astronomers have long been aware that stars are made up of interstellar gas and dust, but until recently the specific sequence of events that signaled their birth was a mystery. Today, however, the stars in spiral arms of disk-shaped galaxies are thought to result from density waves induced by gravitational fluctuations at the galactic center. These waves appear to function as

(A) A puzzle is presented and then two possible scientific solutions are discussed (B) A new phenomenon is described and then the scientific methods used to study it are discussed (C) Recent scientific research is described and then applied to solve an existing problem (D) A previously known phenomenon is described and then explained by scientific observations

29


the lines along which scattered clouds of interstellar gas and dust collect into much larger clouds, which then coalesce into clumps of high concentration, out of which different types of stars, including 0-stars, eventually emerge. Extensive mapping of these cloud complexes, or nebulas, has established a correlation between these complexes and 0-stars—a coincidence too striking, in view of the expanse of empty space within galaxies, to be the result of chance. Since they produce a red florescence, 0-stars are usually found in glowing nebulas that astronomers have labeled H II regions. 0-stars cannot migrate out of these regions because their lifespans are too short. Therefore, astronomers have studied H II regions to determine how clouds and 0-stars interact. These observations suggest that the interaction between clouds and 0-stars is a self-perpetuating cycle in which stars will be produced until the cloud material is used up. 0-stars consume their fuel rapidly and release huge amounts of energy. Moreover, 0-star radiation contributes to driving a shock wave into these clouds, compressing gas and dust there. Out of this tremendous compression of gas and dust arises a second generation of young stars, among them new 0stars.

(E) A number of scientific hypotheses are discussed and then observations concerning their validity are described 2.

The passage states all of the following about 0-stars EXCEPT:

(A) (B) (C) (D) (E)

They consume their fuel quickly They are found in glowing nebulas They are much bigger than the sun They emit large amounts of radiation They cannot migrate out of H II regions

3.

According to the passage, interstellar gas and dust coalesce into "clumps of high concentration"

(A) only in H II regions of space (B) because of gravitational fluctuations in galactic arms (C) when 0-stars migrate out of H II regions (D) after releasing huge amounts of energy (E) before new stars are formed 4.

The author mentions "0-star radiation" in order to

(A) explain why 0-stars have short lifespans (B) outline the role of 0-stars in star formation (C) emphasize the reddish glow of certain nebulas (D) prove that 0-stars interact with cloud complexes (E) predict the rate at which nebulas use up their gas and dust

(K) Passage 7 The relevance of formal economic models to real-world policy has been a topic of some dispute. The economists R. D. Norton and S. Y. Rhee achieved some success in applying such a model retrospectively to the Korean economy over a fourteen-year period; the model's figures for output, prices, and other variables closely matched real statistics. The model's value in policy terms, however, proved less clearcut. Norton and Rhee performed simulations in which, keeping long-term factors constant, they

30

1.

The author is primarily concerned with

(A) proposing a new type of economic analysis (B) criticizing an overly formal economic model (C) advocating the use of statistical models in determining economic policy (D) suggesting an explanation for Korean inflation


tried to pinpoint the effect of short-term policy changes. Their model indicated that rising prices for imported oil would increase inflation; reducing exports by five percent would lower Gross Domestic Product and increase inflation; and slowing the growth of the money supply would result in slightly higher inflation. These findings are somewhat startling. Many economists have argued that reducing exports will lessen, not increase, inflation. And while most view escalating oil costs as inflationary, few would think the same of slower monetary growth. The Norton-Rhee model can perhaps be viewed as indicating the pitfalls of a formalist approach that stresses statistical "goodness of fit" at the expense of genuine policy relevance.

(E) determining the accuracy of Norton and Rhee's analysis 2.

The author mentions "a fourteen-year period" in order to

(A) indicate how far into the future Norton and Rhee's model can make accurate predictions (B) acknowledge the accuracy of Norton and Rhee's model in accounting for past events (C) explain the effect of reducing exports on inflation (D) demonstrate the startling nature of Norton and Rhee's findings (E) expose the flaws in Norton and Rhee's model 3.

The most significant criticism leveled against Norton and Rhee's model is that it

(A) excludes key statistical variables (B) is too abstract to be useful in policy making (C) fails to adjust for Korea's high rate of inflation (D) underestimates the importance of economic growth (E) fails to consider the effect of short-term variations in the economy 4.

It can be inferred that the most surprising finding of the Norton-Rhee study is that

(A) reducing exports would reduce inflation (B) high oil prices worsen inflation (C) an increase in exports can slow the rate of growth (D) slower monetary expansion would worsen inflation (E) long-term factors do not affect economic growth

(K) Passage 8 A basic principle of ecology is that population size is partly a reflection of available food 1.

The

author

provides

specific

31


resources. Recent experiments suggest that the relationship is more complex than formerly thought. Specifically, the browsing of certain rodents appears to trigger chemical reactions in food plants which, in turn, affect the size of the rodent populations. Two examples of such regulation have been reported. Berger has demonstrated the power of a naturally occurring chemical called 6-MBOA to stimulate reproductive behavior in the mountain vole, a small mouselike rodent. 6-MBOA forms in young grass in response to browsing by voles. Berger experimented by feeding oats coated with 6-MBOA to non-breeding winter populations of voles. After three weeks, she found a high incidence of pregnancy among females. Since the timing of reproduction is crucial to the short-lived vole in an environment in which the onset of vegetative growth may be considerably delayed, the phytochemical triggering of reproductive behavior represents a significant biological adaptation. In an example reported by Bryant, plants appear to have developed a phytochemical defense against the depredations of snowshoe hares in Canada. Every ten years, for reasons that are unclear, the hare population swells. The result is overbrowsing of certain deciduous trees and shrubs. Bryant found that trees favored by the hare produce young shoots high in terpene and phenolic resins, which discourage hare browsing. After treating non-resinous willow twigs with resinous extracts and placing treated and untreated samples at hare feeding stations, Bryant found that samples containing at least half of the resin concentration of natural twigs were untouched. The avoidance of resinous shoots, he concludes, may play a role in the decline of the hare population to normal levels. Both of these reports suggest areas for further research. For example, data should be reviewed to determine if periodic population explosions among lemmings (another small rodent living in a northern environment) occur during years in which there is an early onset of vegetative growth; if so, a triggering mechanism similar to that prompted by the vole may be involved.

32

information to answer which of the following questions? (A) What factors other than food supply affect the population size of rodents? (B) Why is the timing of the voles' reproductive effort important? (C) Are phytochemical reactions found only in northern environments? (D) How does 6-MBOA trigger reproductive activity in the mountain vole? (E) What are the causes of the periodic increase in the snowshoe hare population? 2.

The passage describes the effect of 6MBOA on voles as a "significant biological adaptation" because it

(A) limits reproductive behavior in times of food scarcity (B) prompts the vole population to seek new food sources (C) supports species survival during periods of fluctuating food supply (D) maximizes the number of offspring in individual litters (E) minimizes territorial competition 3.

It can be inferred that the study of lemmings proposed by the author would probably

(A) (B) (C) (D)

strengthen the conclusions of Bryant cast doubt on the conclusions of Bryant support the specific findings of Berger provide evidence as to whether Berger's conclusions can be generalized (E) disprove common beliefs about the relationship between population size and food supply 4.

The author of the passage is primarily concerned with

(A) reviewing findings about phytochemical regulation of rodent populations (B) outlining the role of 6-MBOA in


regulating population size (C) summarizing knowledge on population size of rodents (D) explaining why earlier studies of population size were wrong (E) describing mechanisms used by plants to protect themselves 5.

Bryant's interpretation of the results of his experiment (italics) depends on which of the following assumptions?

(A) The response of hares to resinous substances may be different in nature than under experimental conditions (B) The periodic rise in the hare population is triggered by an unknown phytochemical response (C) Many hares will starve to death rather than eat resinous shoots (D) Hares alter their breeding behavior in response to the declining availability of food (E) Significant numbers of hares die from ingesting the resins in shoots 6.

The experiments described in the passage did each of the following EXCEPT

(A) measure changes in the behavior of test animals (B) measure changes in the populations of experimental animals (C) simulate a hypothesized phytochemical effect in nature (D) measure the consumption of foods by test animals (E) analyze the effects of food on breeding behavior

(K) Passage 9 There is an intriguing note to the current call upon civil rights law to help remedy the 1. undervaluation of women's work. Until fairly recently, government was not expected to solve workers' economic grievances, however valid (A) they might be. Many assumed that the

In the first paragraph, "this" most likely refers to increasing the wages of women and men in a single industry

33


responsibility lay with workers themselves. Collective bargaining was the preferred instrument for pursuing pay equity for women. Rather than call upon the law to regulate the market from the outside, one could try to reshape or otherwise influence the market so that women themselves would be better able to address the problem. This could be done by raising absolute wage levels in low-paying, predominantly female industries (such as retail clothing) or by changing the pay relationship between largely female and largely male occupations within a single industry, such as auto manufacturing. Through union representation, employees in traditionally female jobs in an industry could identify the actual degree of underpayment of their work and then, as a group, pressure their employer to remedy it. In addition, this process would encourage those affected—men and women alike—to be sensitive to the limits of available resources, to be pragmatic about the pace at which the wage structure could be revised. I do not mean to suggest that collective bargaining is a foolproof means for closing the gender gap in wages. To the extent that the problem involves the undervaluation of nonunion female occupations in an otherwise unionized industry, political hurdles will discourage unionized employees from supporting revisions in the wage structure. And to the extent that the problem is the concentration of women in lowpaying industries—textiles, for example—the product market imposes serious economic constraints on a substantial closing of the wage gap. Despite the imperfections of tools like collective bargaining for redressing wage disparities between men and women, a reliance on law or government is favorable for neither individual firms nor our economy as a whole. Nonetheless, although opponents of mandatory public remedies may correctly fear those remedies as being a cure worse than the disease, they are wrong when they imply that the current system of wage determination by business management is perfectly healthy.

(B) bringing about changes in market conditions (C) changing the dynamic of collective bargaining (D) relying on civil rights law to remedy economic grievances (E) applying group pressure on an employer 2.

According to the author, the process of unionization and collective bargaining could do all of the following EXCEPT

(A) overcome market pressures that keep wages in some industries lower than in others (B) encourage worker flexibility in adjusting a new pay scale to economic conditions (C) help workers to apply group pressure on employers (D) aid in determining the degree to which women are being underpaid (E) sensitize workers to the limits of their industry's ability to institute change 3.

Which of the following best summarizes the author's main point?

(A) Pay inequity for women exists because of the lack of unionization in traditionally female occupations. (B) Government regulation of industry to achieve pay equity for women is unnecessary because management has the power to effectively determine wages. (C) Unionization would solve all industry problems relating to the valuation of women's work. (D) Government regulation of women's wages is necessary only in those industries where collective bargaining is ineffective. (E) Collective bargaining is preferable to government actions in redressing the undervaluation of women's work. 4.

The author mentions textiles in order to

(A) demonstrate the potential harm of

34


(B) (C) (D) (E)

5.

government regulation of industry outline a strategy for achieving pay equity for women indicate how quickly employees can reasonably expect to achieve pay equity give an example of a situation in which collective bargaining may be ineffective show why civil rights laws are the most important tool for increasing women's wages It can be inferred that the author's attitude towards opponents of government regulation of wage determination mentioned in the last paragraph is characterized by which of the following?

I. Distrust of their motives II. Sympathy with some of their concerns III. Disagreement with some of their assumptions IV. Opposition to their political principles (A) (B) (C) (D) (E) 6.

I only III only I and II only II and III only I, II, and IV In the final paragraph, the author addresses "opponents of mandatory public remedies" by

(A) arguing that those remedies would benefit the economy (B) implying that alternative methods of correcting wage disparities would be worse (C) asserting that the present approach to setting wages is flawed (D) defending civil rights legislation as a solution to social problems (E) insisting that those remedies are a viable means of correcting wage disparities 7.

The passage refers to which of the following as reasons for preferring collective bargaining to legislation as a

35


method of ending the undervaluation of women's work? I. The greater responsiveness of collective bargaining to existing conditions that affect wage levels II. The general desirability of using private rather than public remedies III. The potential of collective bargaining for achieving a uniform national solution to the problem of gender wage disparities

(K)

I only III only I and II only II and III only I, II, and III

1.

The primary purpose of this passage is to discuss

Passage 10

The debt crisis affecting many developing countries has had three causes: imprudent management and borrowing by debtor countries; imprudent lending by banks; and rising interest rates. The unprecedented rise in real interest rates to about 6 percent by 1982 increased the burden on borrowers and completely changed the nature of the debt problem. In past debt crises, when loans were made at fixed rates, real interest rates rose with deflation. But once price levels stabilized, the interest burden would be higher only to the extent of the proportional decline in price levels, and it remained quite possible that inflation would eventually reduce the burden. In this crisis, though, the real interest rate has risen and stayed high, and inflation has brought no relief. During the 1980s, fear of financial loss led U.S. commercial banks to sharply curtail their lending activity in debtor countries. In 1982, nine large banks had over 250 percent of their capital in loans to developing countries; by mid-1986, the nine banks had reduced their activities to the point

36

(A) (B) (C) (D) (E)

(B) the factors that have led to the current levels of debt carried by developing countries (C) the deleterious effects of rising interest rates (D) American banking in the 1980s and its impact on debtor nations (E) the future of U.S. banking with respect to its interest rates and loan policies (F) economic conditions in developing countries that might prevent them from or enable them to repay U.S. loans 2.

Which of the following characterizes responses to the debt crisis in the 1980s?

(A) increased pressure on debtor countries to pay interest due on loans


where they had sufficient equity and reserves to withstand potential losses. Although banks have stabilized their positions, many continue to carry developing-country debt at face value. Present bank strategies deal with the debt crisis by extending the effective maturity of loans. Although any method that reduces the flow of resources from debtor countries will help in the short run, further lending promises little relief to the debt problem. So long as real interest rates remain high, developing countries will remain in debt.

(B) an increase in the percentage of their total capital large banks devoted to foreign loans (C) a decrease in the funds designated by banks to cover potential losses (D) reliance by banks on inflationary pressure to reduce debt levels (E) a decline in bank lending and an increase in capital reserves 3.

The author suggests that methods currently in place for dealing with the debt crisis are inadequate because they

(A) increase the upward pressure on real interest rates without allowing any opportunity for reduction (B) allow real wages to rise at the expense of economic growth in debtor countries (C) fail to address problems of mismanagement in debtor and creditor countries (D) lessen the immediate burden of debt service but do not promote long-term growth (E) sacrifice a reduction of real interest rates for a short-term increase in loan maturity

4.

In the passage, the author identifies all of the following as contributing to the current debt crisis EXCEPT

(A) careless borrowing by developing countries (B) sustained high real interest rates (C) unwillingness of banks to transfer the burden of loans to other countries (D) unwise decisions made by commercial lending institutions (E) failure of inflation to reduce the interest burden 5.

The passage provides information that helps to answer which of the following questions?

I.

Did errors of economic management by developing countries contribute to the

37


II. III.

(A) (B) (C) (D) (E) 6.

debt crisis? Are steps currently being taken to alleviate the debt crisis? Do taxpayers in lending countries support the notion of debt relief? I only II only I and II only II and III only I, II, and III If the passage were to continue, the next topic the author would logically discuss would most likely be

(A) possible steps which might bring about effective debt relief (B) options other than debt relief that might alleviate the debt crisis (C) current attitudes of bankers toward international lending (D) measures currently taken by debtor countries to reduce inflation (E) the effects of 1980s banking activities on debtor countries

(K) Passage 11 Desert plant populations have evolved sophisticated physiological behavioral traits that aid survival in arid conditions. Some send out long, unusually deep taproots; others utilize shallow but widespread roots, which allow them to absorb large, intermittent flows of water. Certain plants protect their access to water. The creosote bush produces a potent root toxin which inhibits the growth of competing root systems. Daytime closure of stomata exemplifies a further genetic adaptation; guard cells work to minimize daytime water loss, later allowing the stomata to open when conditions are more favorable to gas exchange with the environment. Certain adaptations reflect the principle that a large surface area facilitates water and gas exchange. Most plants have small leaves, modified leaves (spines), or no leaves at all. The

38

1.

The passage refers to the spines and thorns of desert plants as

I.

genetically evolved structural adaptations that protect against predation genetic modifications that aid in the reduction of water loss structures that do not participate directly in food production

II. III.

(A) (B) (C) (D) (E) 2.

I only III only I and II only II and III only I, II, and III The passage suggests that which of the


main food-producing organ is not the leaf but the following weather-related conditions stem, which is often green and non-woody. Thick, would most benefit plants with shallow waxy stems and cuticles, seen in succulents such root systems? as cacti and agaves, also help conserve water. Spines and thorns (modified branches) protect (A) An unusually prolonged drought against predators and also minimize water loss. (B) A windstorm (C) A flash flood (D) A light spring rain (E) A winter snowfall 3.

The adaptations of desert plants to their environment would tend to support the statement that

(A) the rate of genetic evolution is greater in the desert than in more temperate surroundings (B) structures in a plant which usually perform one function may, under certain conditions, perform different functions (C) while the amount of leaf surface area is critical for a desert plant, it is much less so for plants in most other environments (D) desert plants do not have many physiological and behavioral traits in common with other plants (E) desert plants could probably adapt to life in a variety of harsh ecosystems

4.

All of the following are mentioned as examples of adaptation by desert plants EXCEPT

(A) (B) (C) (D) (E)

deep roots shallow roots poisonous roots food-producing leaves spines and thorns

(K) Passage 12 The great migration of European intellectuals to the United States in the second quarter of the 1. The author's main concern in the twentieth century prompted a transformation in passage is to the character of Western social thought. The influx of Continental thinkers fleeing fascist (A) characterize the effects of migration on

39


regimes had a great impact on American academic circles, leading to new developments in such diverse fields as linguistics and theology. But the greatest impact was on the émigrés themselves. This "migration experience" led expatriates to reexamine the supposedly selfevident premises inherited from the Continental intellectual tradition. The result, according to H. Stuart Hughes in The Sea-Change, was an increased sophistication and deprovincialization in social theory. One problem facing newly arrived émigrés in the U.S. was the spirit of anti-intellectualism in much of the country. The empirical orientation of American academic circles, moreover, led to the conscious tempering by many European thinkers of their own tendencies towards speculative idealism. In addition, reports of oppression in Europe shook many Old World intellectuals from a stance of moral isolation. Many great European social theorists had regarded their work as separate from all moral considerations. The migration experience proved to many intellectuals of the following generations that such notions of moral seclusion were unrealistic, even irresponsible. This transformation of social thought is perhaps best exemplified in the career of the German theologian Paul Tillich. Migration confronted Tillich with an ideological as well as a cultural dichotomy. Hughes points out that Tillich's thought was "suspended between philosophy and theology, Marxism and political conformity, theism and disbelief." Comparable to the fusion by other expatriate intellectuals of their own idealist traditions with the Anglo-American empiricist tradition was Tillich's synthesis of German Romantic religiosity with the existentialism born of the twentieth-century war experience. Tillich's basic goal, according to Hughes, was to move secular individuals by making religious symbols more accessible to them. Forced to make his ethical orientation explicit in the context of American attitudes, Tillich avoided the esoteric academic posture of many Old World scholars, and was able to find a wide and sympathetic audience for his sometimes difficult theology. In this way, his experience in America, in his own words, "deprovincialized" his thought.

40

(B)

(C) (D) (E)

2.

U.S. history show how Paul Tillich's career was representative of the migration experience discuss the effects of the great migration on modern social thought reveal the increased sophistication of postmigration thought contrast European social thought with that of the United States The author probably mentions H. Stuart Hughes in order to

(A) give an example of a European intellectual who migrated to America (B) cite an important source of information about the migration experience (C) demonstrate how one American academic was influenced by European scholars (D) pay tribute to Americans who provided European thinkers with a refuge from fascism (E) name a leading disciple of Paul Tillich 3.

Which of the following statements describe Tillich's achievement?

I.

He elucidated religious symbols in a secular context without sacrificing their impact. II. He shunned the esotericism of much theological scholarship. III. He adapted a traditional religiosity to the temper of the modern world. (A) (B) (C) (D) (E)

I only II only I and II only II and III only I, II, and III

4.

According to the passage, "reports of oppression in Europe" affected social thinkers by forcing them to

(A) rethink their moral responsibilities (B) re-examine the morality of European leaders


(C) analyze the effects of migration on morality (D) reconsider their antisocial behavior (E) justify the moral value of social thought 5.

It can be inferred that postmigration social thought is distinguished from premigration thought by its

(A) (B) (C) (D) (E)

less secular nature greater social consciousness more difficult theology diminished accessibility more theoretical nature

6.

The passage suggests that the migration experience

(A) had little major effect on American academic circles (B) led to the abandonment of the idealist philosophical tradition (C) made American intellectuals sensitive to oppression in Europe (D) caused ĂŠmigrĂŠ social thinkers to question certain of their beliefs (E) negated Tillich's influence on modern social thought

(OG) Passage 13 The majority of successful senior managers do not closely follow the classical rational model of first clarifying goals, assessing the problem, formulating options, estimating likelihoods of success, making a decision, and only then taking action to implement the decision. Rather, in their day-by-day tactical maneuvers, these senior executives rely on what is vaguely termed "intuition" to manage a network of interrelated problems that require them to deal with ambiguity, inconsistency, novelty, and surprise; and to integrate action into the process to thinking. Generations of writers on management have recognized that some practicing managers rely heavily on intuition. In general, however, such writers display a poor grasp of what intuition is. Some see it as the opposite of rationality:

1.

According to the passage, senior managers use intuition in all of the following ways EXCEPT to

(A) speed up of the creation of a solution to a problem (B) identify a problem (C) bring together disparate facts (D) stipulate clear goals (E) evaluate possible solutions to a problem 2.

The passage suggests which of the following about the "writers on management"?

(A) They have criticized managers for not following the classical rational model

41


others view it as an excuse for capriciousness. Isenberg's recent research on the cognitive processes of senior managers reveals that managers' intuition is neither of these. Rather, senior managers use intuition in at least five distinct ways. First, they intuitively sense when a problem exists. Second, managers rely on intuition to perform well-learned behavior patterns rapidly. This intuition is not arbitrary or irrational, but is based on years of painstaking practice and hands-on experience that build skills. A third function of intuition is to synthesize isolated bits of data and practice into an integrated picture, often in an "Aha!" experience. Fourth, some managers use intuition as a check on the results of more rational analysis. Most senior executives are familiar with the formal decision analysis models and tools, and those who use such systematic methods for reaching decisions are occasionally leery of solutions suggested by these methods which run counter to their sense of the correct course of action. Finally, managers can use intuition to bypass in-depth analysis and move rapidly to engender a plausible solution. Used in this way, intuition is an almost instantaneous cognitive process in which a manager recognizes familiar patterns. One of the implications of the intuitive style of executive management is that "thinking" is inseparable from acting. Since managers often "know" what is right before they can analyze and explain it, they frequently act first and explain later. Analysis is inextricably tied to action in thinking/acting cycles, in which managers develop thoughts about their companies and organizations not by analyzing a problematic situation and then acting, but by acting and analyzing in close concert. Given the great uncertainty of many of the management issues that they face, senior managers often instigate a course of action simply to learn more about an issue. They then use the results of the action to develop a more complete understanding of the issue. One implication of thinking/acting cycles is that action is often part of defining the problem, not just of implementing the solution.

42

(B)

(C)

(D)

(E)

3.

of decision analysis They have not based their analyses on a sufficiently large sample of actual managers They have relied in drawing their conclusions on what managers say rather than on what managers do They have misunderstood how managers use intuition in making business decisions. They have not acknowledged the role of intuition in managerial practice Which of the following best exemplifies "an 'Aha!' experience" as it is presented in the passage?

(A) A manager risks taking an action whose outcome is unpredictable to discover whether the action changes the problem at hand (B) A manager performs well-learned and familiar behavior patterns in creative and uncharacteristic ways to solve a problem (C) A manager suddenly connects seemingly unrelated facts and experiences to create a pattern relevant to the problem at hand (D) A manager rapidly identifies the methodology used to compile data yielded by systematic analysis (E) A manager swiftly decides which of several sets of tactics to implement in order to deal with the contingencies suggested by a problem 4.

According to the passage, the classical model of decision analysis includes all of the following EXCEPT

(A) evaluation of a problem (B) creation of possible solutions to a problem (C) establishment of clear goals to be reached by the decision (D) action undertaken in order to discover more information about a problem (E) comparison of the probable effects of different solutions to a problem


5.

It can be inferred from the passage that which of the following would most probably be one major difference in behavior between Manager X, who uses intuition to reach decisions, and Manager Y, who uses only formal decision analysis?

(A) Manager X analyzes first and then acts; Manager Y does not (B) Manager X checks possible solutions to a problem by systematic analysis; Manager Y does not (C) Manager X takes action in order to arrive at the solution to a problem; Manager Y does not (D) Manager Y draws on years of hands-on experience in creating a solution to a problem; Manager X does not (E) Manger Y depends on day-to-day tactical maneuvering; manager X does not 6.

It can be inferred from the passage that "thinking/acting cycles" in managerial practice would be likely to result in which of the following?

I.

A manager analyzes a network of problems and then acts on the basis of that analysis A manager gathers data by acting and observing the effects of action A manager takes action without being able to articulate reasons for that particular action

II. III.

(A) (B) (C) (D) (E)

I only II only I and II only II and III only I, II, and III

43


(OG) Passage 14 In 1988 services moved ahead of manufacturing as the main product of the United States economy. But what is meant by "services'? Some economists define a service as something that is produced and consumed simultaneously: for example, a haircut. The broader, classical definition is that a service is an intangible something that cannot be touched or stored. Yet electric utilities can store energy, and computer programmers save information electronically. Thus, the classical definition is hard to sustain. The United States government's definition is more practical: services are the residual category that includes everything that is not agriculture or industry. Under this definition, "services" includes activities as diverse as engineering and driving a bus. However, besides lacking a strong conceptual framework, this definition fails to recognize the distinction between service industries and service occupations. It categorizes workers based on their company's final product rather than on the actual work the employees perform. Thus, the many service workers employed by manufacturers— bookkeepers or janitors, for example—would fall under the industrial rather than the services category. Such ambiguities reveal the arbitrariness of this definition and suggest that, although practical for government purposes, it does not accurately reflect the composition of the current United States economy.

1.

(A) discussing research data underlying definitions (B) arguing for the adoption of a particular definition (C) exploring definitions of a concept (D) comparing the advantages of several definitions (E) clarifying some ambiguous definitions 2.

In comparing the United States government's definition of services with the classical definition the author suggests that the classical definition

(A) (B) (C) (D) (E)

more pragmatic more difficult to apply less ambiguous more widely used more arbitrary

3.

The passage suggests which of the following about service workers in the United States?

(A) The number of service workers may be underestimated by the definition of services used by the government (B) There were fewer service workers than agricultural workers before 1988 (C) The number of service workers was almost equal to the number of workers employed in manufacturing until 1988 (D) Most service workers are employed in service occupations rather than in service industries (E) Most service workers are employed in occupations where they provide services that do not fall under the classical definition of services 4.

44

The author of the passage is primarily cone with

The author of the passage mentions which of the following as one disadvantage of the United States


government's definition of services? (A) It is less useful than the other definitions mentioned in the passage (B) It is narrower in scope than the other definitions mentioned in the passage (C) It is based on the final product produced rather than on the type of work performed (D) It does not recognize the diversity of occupations within the service industries (E) It misclassifies many workers who are employed in service industries 5.

The author refers to "service workers employed by manufacturers" primarily in order to point out

(A) a type of worker not covered by the United States government's system of classifying occupations (B) a flaw in the United States government's definition of services (C) a factor that has influenced the growth of the service economy in the United States (D) a type of worker who is classified on the basis of work performed rather than on the basis of the company's final product (E) the diversity of the workers who are referred to as service workers

(K) Passage 15 The basic theory of plate tectonics recognizes two ways continental margins can grow seaward. Where two plates move away from a midocean rift that separates them, the continental margins on those plates are said to be passive. Such continental margins grow slowly from the accumulation of riverborne sediments and of the carbonate skeletons of marine organisms. Since most sequences of such accretions, or miogeoclinal deposits, are undeformed, passive margins are not associated with mountain building. Along active margins continents tend to grow

1.

Which one of the following best expresses the main idea of the passage?

(A) The margin of the west coast of North America developed through a combination of active and passive mechanisms (B) The growth of continental margins is only partially explained by the basic theory of plate tectonics (C) Continental margins can grow seaward in two ways, through sedimentation or

45


much faster. At an active margin an oceanic plate plunges under a continental plate, fragments of which then adhere to the continental margin. The process is met with extensive volcanism and mountain-building. A classic example is the Andes of the west coast of South America. In the original plate-tectonic model western North America was described as being initially passive and then active. It was assumed that the continent grew to a limited extent along this margin as oceanic rocks accreted in places such as the Coast Ranges of California. The model was successful in explaining such disparate features as the Franciscan rocks of the California Coast Ranges, created by seduction, and the granite rocks of the Sierra Nevada that originated in volcanoes. The basic plate-tectonic reconstruction of the geologic history of western North America remains unchanged in the light of microplate tectonics, but the details are radically changed. It is now clear that much more crust was added to North America in the Mesozoic era than can be accounted for by volcanism and by the simple accretion of sediments. Further, some adjacent terrenes are not genetically related, as would be expected from simple plate tectonics, but have almost certainly traveled great distances from entirely different parts of the world.

volcanism (D) The introduction of micro plate tectonics poses a fundamental challenge to the existing theory of how continental margins are formed (E) Continental margins grow more rapidly along active margins than along passive margins 2.

The passage supplies information for answering all of the following questions regarding continental margins EXCEPT:

(A) How have marine organisms contributed to the formation of passive continental margins? (B) What were some of the processes by which the continental margin of the west coast of North America was formed? (C) Are miogeoclinal deposits associated with mountain building along continental margins? (D) How was the continental margin of the west coast of South America formed? (E) How much crust added to North America in the Mesozoic era can be accounted for by the accretion of sediments from the ocean floor? 3.

The author mentions the Franciscan rocks of the California Coast Ranges in order to make which one of the following points?

(A) The basic theory of plate tectonics accounts for a wide variety of geologic features (B) The original plate tectonic model falls short of explaining such features (C) Subduction processes are responsible for the majority of the geologic features found along the west coast of North America (D) Passive margins can take on many geologic forms (E) The concept of microplate tectonics was first introduced to account for such phenomena

46


4.

Which one of the following does the author mention as evidence for the inadequacy of the original plate tectonic model to describe the formation of continental margins?

(A) Nearly flat, undeformed crystal blocks have been found along some continental margins where there are mountains further inland (B) Sediments and fragments from the depths of the ocean accumulate along continental margins (C) Large pieces of the Earth's crust that appear to be completely unrelated are found in the same area today (D) Undeformed miogeoclinal deposits are usually not linked to mountain building (E) Oceanic plates drop beneath continental plates along active margins

(K) Passage 16 A tiny fraction of binary systems belong to a curious subclass whose radiation has a wavelength distribution so peculiar that it long defied explanation. Such systems radiate strongly in the visible region of the spectrum, but some of them do so even more strongly at both shorter and longer wavelengths: in the ultraviolet region and in the infrared and radio regions. This odd distribution of radiation is best explained by the pairing of a cool red-giant star and an intensely hot small star, known as symbiotic stars, that travel around a common center. Recently two symbiotic-star systems, the first to be detected outside our galaxy, have been observed in the Large Cloud of Magellan. The spectra of symbiotic stars indicate that the cool red giant is surrounded by a very hot ionized gas which satellite observations finally identified as radiating from an invisible hot companion. It is possible that symbiotic stars represent a transitory phase in the evolution of certain types of binary systems in which a substantial amount of matter transfers from the larger partner to the smaller. The exact evolutionary course that turns a binary system into a symbiotic one is unknown. The comparative scarcity of known symbiotics in

1.

The passage implies that symbiotic star systems differ from other binary systems in which one of the following ways?

(A) Symbiotically paired stars emit a radiation pattern different from that of most binary stars (B) In symbiotic star systems, one star is the center of the other's orbit (C) Symbiotically paired stars are the only binary stars which are capable of exchanging matter (D) Symbiotic star systems are more common than other binary systems (E) Symbiotic star systems are the only binary systems that can be detected by satellite-borne instruments 2.

The primary purpose of the passage is to

(A) argue that a great percentage of binary star systems are symbiotic (B) criticize the theory of symbiotic stars as overly speculative

47


our galaxy suggests that if all binaries of modest mass pass through a symbiotic phase in their evolution, the phase must be extremely brief, perhaps as short as a million years. It is suspected that the evolutionary course of binary stars is predetermined by the initial mass and angular momentum of their gas clouds. Since red giants and Mira variables are thought to be stars with a mass of one or two suns, it seems plausible that the original cloud from which a symbiotic system is formed can consist of no more than a few solar masses of gas.

(C) describe symbiotic stars as a distinct type of binary system (D) present evidence that binary star systems have evolved from gas clouds (E) compare symbiotic stars to red giants and Mira variables 3.

According to the passage, the radiation emitted by symbiotic stars is distinctive in that it

(A) (B) (C) (D)

generates standard wavelengths consists partly of visible waves is transferred from one star to its partner is strongest at the extreme ends of the spectrum (E) emanates primarily from the larger star 4.

The author suggests that

(A) the detection of radiation from an invisible hot companion star prompted scientists to investigate the peculiar ionized gas surrounding cool red giants (B) small hot stars attach to cool red giants because red giants have a mass of one or two suns (C) a million years is a brief period of time for the occurrence of many solar events (D) the only symbiotic star systems to be detected outside of our galaxy are in the Large Cloud of Magellan (E) if binary stars of modest mass passed through symbiotic phases lasting much more than a million years, it is likely that more of them would have been detected

(K) Passage 17 Every day the mailboxes of America are filled with solicitations provided by the direct 1. marketing industry. America's response to this deluge has been strangely mixed. On the negative side, poorly executed direct marketing produces unwanted, annoying and wasteful solicitations, also known as "junk mail." Also, aggressive direct (A) marketing techniques, aided by new tools in technology, represent a serious threat to

48

Which one of the following, if true, would best strengthen the author's explanation of the "seeming contradiction"? Awareness of commercial infringements on the rights of citizens has never been higher.


informational privacy. Sophisticated computer matching programs can produce intrusive personal profiles from information which, standing alone, does not threaten individual privacy. The 1991 Harris-Equifax Consumer Privacy Survey addressed popular attitudes towards direct mailing practices and their impact on informational privacy. When asked how they viewed direct mail offers in general, 46 percent of the respondents said they were "nuisance," 9 percent considered them to be "invasions of privacy," and only 6 percent said they were "useful." But if Americans have such a negative opinion of the direct marketing industry, they have a strange way of showing it. Direct mail advertising expenditures rose from $7.6 billion in 1980 to $23.4 billion in 1990. The laws of the market dictate that companies would not have made these efforts without prospects of success. Moreover, almost half of the citizens surveyed who considered direct mail offers to be "invasions of privacy" had themselves bought something in response to a direct mail ad in the past year. Analysis of this seeming contradiction reveals the central problem of regulation in this industry: everyone hates receiving "junk mail," and everyone ought to be concerned about informational privacy. Still, direct marketing offers real advantages over other means of shopping. Even those who believe that the direct mailing industry has a generally negative societal impact probably would prefer to remain on some mailing lists. We like shopping by mail, and we don't want to throw out the good with the bad.

(B) The number of people on more than one mailing list has increased in direct proportion to the increase in direct marketing expenditures. (C) Consumers do not perceive a connection between their individual purchasing behavior and infringements on their personal rights. (D) Some people believe that the benefits associated with the recent success of the direct marketing industry will filter down to consumers over time. (E) Some opinion polls on other topics indicate a similar discrepancy between what people say about an issue and how they act in relation to that issue. 2.

Which one of the following critiques most approximates the logic underlying the author's concern regarding the effects of the computer matching?

(A) An ecologist who states that since each of three species individually would not damage an ecosystem, it is safe to introduce all three into the ecosystem overlooks the possibility that the dominance of one species may lead to the extinction of one or both of the other two species (B) An ecologist who states that since each of three species individually would not damage an ecosystem, it is safe to introduce all three into the ecosystem overlooks the possibility that the three species taken together may very well pose a serious threat to the ecosystem (C) An ecologist who states that since each of three species individually would not damage an ecosystem, it is safe to introduce all three into the ecosystem overlooks the possibility that the addition of the three species to the ecosystem may preclude the addition of any further species (D) An ecologist who states that since each of three species individually would not damage an ecosystem, it is safe to

49


introduce all three into the ecosystem overlooks the possibility that the ecosystem may not be the optimal environment for the species in question (E) An ecologist who states that since each of three species individually would not damage an ecosystem, it is safe to introduce all three into the ecosystem overlooks the possibility that any one of three species may have posed a risk to the previous ecosystem in which it lived 3.

Which one of the following can be inferred from the passage about direct mail advertising expenditures in the years between 1980 and 1990?

(A) The rise in expenditures during this period is suggestive of the expectations of companies engaged in direct marketing at the time (B) The profit derived from sales linked to these expenditures in 1990 was more than double the profit derived from such sales in 1980 (C) The lowest yearly expenditure on direct mail advertising during this period occurred in 1980 (D) Direct marketing companies expect the pattern of expenditures during this period to continue in the decades to come (E) The rise in expenditures during this period closely parallel the laws of the market

(OG) Passage 18 Caffeine, the stimulant in coffee, has been called "the most widely used psychoactive substance on Earth." Snyder, Daly, and Bruns have recently proposed that caffeine affects behavior by countering the activity in the human brain of a naturally occurring chemical called adenosine. Adenosine normally depresses neuron firing in many areas of the brain. It apparently does this by inhibiting the release of neurotransmitters, chemicals that carry nerve

50

1.

The primary purpose of the passage is to

(A) discuss a plan for investigation of a phenomenon that is not yet fully understood (B) present two explanations of a phenomenon and reconcile the differences between them (C) summarize two theories and suggest a


impulses from one neuron to the next. Like many other agents that affect neuron firing, adenosine must first bind to specific receptors on neuronal membranes. There are at least two classes of these receptors, which have been designated A1, and A2. Snyder, Daly, and Bruns propose that caffeine, which is structurally similar to adenosine, is able to bind to both types of receptors, which prevents adenosine from attaching there and allows the neurons to fire more readily than they otherwise would. For many years, caffeine's effects have been attributed to its inhibition of the production of phosphodiesterase, an enzyme that breaks down the chemical called cyclic AMP. A number of neurotransmitters exert their effects by first increasing cyclic AMP concentrations in target neurons. Therefore, prolonged periods at the elevated concentrations, as might be brought about by a phosphodiesterase inhibitor, could lead to a greater amount of neuron firing and, consequently, to behavioral stimulation. But Snyder et al point out that the caffeine concentrations needed to inhibit the production of phosphodiesterase in the brain are much higher than those that produce stimulation. Moreover, other compounds that block phosphodiesterase's activity are not stimulants. To buttress their case that caffeine acts instead by preventing adenosine binding, Snyder et al compared the stimulatory effects of a series of caffeine derivatives with their ability to dislodge adenosine from its receptors in the brains of mice. "In general," they reported, "the ability of the compounds to compete at the receptors correlates with their ability to stimulate locomotion in the mouse; i.e., the higher their capacity to bind at the receptors, the higher their ability to stimulate locomotion." Theophylline, a close structural relative of caffeine and the major stimulant in tea, was one of the most effective compounds in both regards. There were some apparent exceptions to the general correlation observed between adenosinereceptor binding and stimulation. One of these was a compound called 3-isobutyl-l methylxanthine (IBMX), which bound very well but actually depressed mouse locomotion. Snyder et all suggest that this is not a major stumbling block to their hypothesis. The problem is that the

third theory that overcomes the problems encountered in the first two (D) describe an alternative hypothesis and provide evidence and arguments that support it (E) challenge the validity of a theory by exposing the inconsistencies and contradictions in it 2.

According to Snyder et all, caffeine differs from adenosine in that caffeine

(A) stimulates behavior in the mouse and in humans, whereas adenosine stimulates behavior in humans only (B) has mixed effects in the brain, whereas adenosine has only a stimulatory effect (C) increases cyclic AMP concentrations in target neurons, whereas adenosine decreases such concentrations (D) permits release of neurotransmitters when it is bound to adenosine receptors, whereas adenosine inhibits such release (E) inhibits both neuron firing and the production of phosphodiesterase when there is a sufficient concentration in the brain, whereas adenosine inhibits only neuron firing 3.

In response to experimental results concerning IBMX, Snyder et al contended that it is not uncommon for psychoactive drugs to have

(A) mixed effects in the brain (B) inhibitory effects on enzymes in the brain (C) close structural relationships with caffeine (D) depressive effects on mouse locomotion (E) the ability to dislodge caffeine from receptors in the brain 4.

According to Snyder et all, all of the following compounds can bind to specific receptors in the brain EXCEPT:

(A) IBMX (B) caffeine (C) adenosine

51


compound has mixed effects in the brain, a not (D) theophylline unusual occurrence with psychoactive drugs. (E) phosphodiesterase Even caffeine, which is generally known only for its stimulatory effects, displays this property, 5. Snyder et all suggest that caffeine's depressing mouse locomotion at very low ability to bind to A1 and A2, receptors concentrations and stimulating it at higher ones. can be at least partially attributed to which of the following? (A) The chemical relationship between caffeine and phosphodiesterase (B) The structural relationship between caffeine and adenosine (C) The structural similarity between caffeine and neurotransmitters (D) The ability of caffeine to stimulate behavior (E) The natural occurrence of caffeine and adenosine in the brain 6.

The author quotes Snyder et all in bold lines most probably in order to

(A) reveal some of the assumptions underlying their theory (B) summarize a major finding of their experiments (C) point out that their experiments were limited to the mouse (D) indicate that their experiments resulted only in general correlations (E) refute the objections made by supporters of the older theory

(OG) Passage 19 All of the cells in a particular plant start out with the same complement of genes. How then can these cells differentiate and form structures as different as roots, stems, leaves, and fruits? The answer is that only a small subset of the genes in a particular kind of cell are expressed, or turned on at a given time. This is accomplished by a complex system of chemical messengers that in plants include hormones and other regulatory molecules. Five major hormones have been identified: auxin, abscisic acid, cytokinin, ethylene, and gibberellin. Studies of plants have now identified a new class of regulatory molecules called oligosaccharins.

52

1.

According to the passage, the five wellknown plant hormones are not useful in controlling the growth of crops because

(A) it is not known exactly what functions the hormones perform (B) each hormone has various effects on plants (C) none of the hormones can function without the others (D) each hormone has different effects on different kinds of plants (E) each hormone works on only a small


Unlike the oligosaccharins, the five wellknown plant hormones are pleiotropic rather than specific; that is, each has more than one effect on the growth and development of plants. The five have so many simultaneous effects that they are not very useful in artificially controlling the growth of crops. Auxin, for instance, stimulates the rate of cell elongation, causes shoots to grow up and roots to grow down, and inhibits the growth of lateral shoots. Auxin also causes the plant to develop a vascular system, to form lateral roots, and to produce ethylene. The pleiotropy of the five well-studied plant hormones is somewhat analogous to that of certain hormones in animals. For example, hormones from the hypothalamus in the brain stimulate the anterior lobe of the pituitary gland to synthesize and release many different hormones, one of which stimulates the release of hormones from the adrenal cortex. These hormones have specific effects on target organs all over the body. One hormone stimulates the thyroid gland, for example, another the ovarian follicle cells, and so forth. In other words, there is a hierarchy of hormones. Such a hierarchy may also exist in plants. Oligosaccharins are fragments of the cell wall released by enzymes: different enzymes release different oligosaccharins. There are indications that pleiotropic plant hormones may actually function by activating the enzymes that release these other, more specific chemical messengers from the cell wall.

subset of a cell's genes at any particular time 2.

The passage suggests that the place of hypothalamic hormones in the hormonal hierarchies of animals is similar to the place of which of the following in plants?

(A) Plant cell walls (B) The complement of genes in each plant cell (C) A subset of a plant cell's gene complement (D) The five major hormones (E) The oligosaccharins 3.

The passage suggests that which of the following is a function likely to be performed by an oligosaccharin?

(A) To stimulate a particular plant cell to become part of a plant's root system (B) To stimulate the walls of a particular cell to produce other oligosaccharins (C) To activate enzymes that release specific chemical messengers from plant cell walls (D) To duplicate the gene complement in a particular plant cell (E) To produce multiple effects on a particular subsystem of plant cells 4.

The author mentions specific effects that auxin has on plant development in order to illustrate the

(A) point that some of the effects of plant hormones can be harmful (B) way in which hormones are produced by plants (C) hierarchical nature of the functioning of plant hormones (D) differences among the best-known plant hormones (E) concept of pleiotropy as it is exhibited by plant hormones 5.

According to the passage, which of the following best describes a function

53


performed by oligosaccharins? (A) Regulating the daily functioning of a plant's cells (B) Interacting with one another to produce different chemicals (C) Releasing specific chemical messengers from a plant's cell walls (D) Producing the hormones that cause plant cells to differentiate to perform different functions (E) Influencing the development of a plant's cells by controlling the expression of the cells' genes 6.

The passage suggests that, unlike the pleiotropic hormones, oligosaccharins could be used effectively to

(A) trace the passage of chemicals through the walls of cells (B) pinpoint functions of other plant hormones (C) artificially control specific aspects of the development of crops (D) alter the complement of genes in the cells of plants (E) alter the effects of the five major hormones on plant development

(OG) Passage 20 (This passage was adapted from an article written in 1992.) Some observers have attributed the dramatic growth in temporary employment that occurred in 1. The primary purpose of the passage is to the United States during the 1980’s to increased participation in the workforce by certain groups, (A) present the results of statistical analyses such as first-time or reentering workers, who and propose further studies. supposedly prefer such arrangements. However, (B) explain a recent development and statistical analyses reveal that demographic predict its eventual consequences. changes in the workforce did not correlate with (C) identify the reasons for a trend and variations in the total number of temporary recommend measures to address it. workers. Instead, these analyses suggest that (D) outline several theories about a factors affecting employers account for the rise phenomenon and advocate one of them. in temporary employment. One factor is product (E) describe the potential consequences of demand: temporary employment is favored by implementing a new policy and argue employers who are adapting to fluctuating in favor of that policy. demand for products while at the same time seeking to reduce overall labor costs. Another 2. According to the passage, which of the

54


factor is labor’s reduced bargaining strength, which allows employers more control over the terms of employment. Given the analyses, which reveal that growth in temporary employment now far exceeds the level explainable by recent workforce entry rates of groups said to prefer temporary jobs, firms should be discouraged from creating excessive numbers of temporary positions. Government policymakers should consider mandating benefit coverage for temporary employees, promoting pay equity between temporary and permanent workers, assisting labor unions in organizing temporary workers, and encouraging firms to assign temporary jobs primarily to employees who explicitly indicate that preference.

following is true of the “factors affecting employers”? (A) Most experts cite them as having initiated the growth in temporary employment that occurred during the 1980’s. (B) They may account for the increase in the total number of temporary workers during the 1980’s. (C) They were less important than demographic change in accounting for the increase of temporary employment during the 1980’s. (D) They included a sharp increase in the cost of labor during the 1980’s. (E) They are more difficult to account for than at other factors involved in the growth of temporary employment during the 1980’s. 3.

The passage suggests which of the following about the use of temporary employment by firms during the 1980’s?

(A) It enabled firms to deal with fluctuating product demand far more efficiently than they before the 1980’s. (B) It increased as a result of increased participation in the workforce by certain demography groups. (C) It was discouraged by governmentmandated policies. (D) It was a response to preferences indicated by certain employees for more flexible working arrangements. (E) It increased partly as a result of workers’ reduced ability to control the terms of their employment. 4.

The passage suggests which of the following about the workers who took temporary jobs during the 1980’s?

(A) Their jobs frequently led to permanent positions within firms. (B) They constituted a less demographically diverse group than has been suggested.

55


(C) They were occasionally involved in actions organized by labor unions. (D) Their pay declined during the decade in comparison with the pay of permanent employees. (E) They did not necessarily prefer temporary employment to permanent employment. 5.

The first sentence in the passage suggests that the observers would be most likely to predict which of the following?

(A) That the number of new temporary positions would decline as fewer workers who preferred temporary employment entered the workforce. (B) That the total number of temporary positions would increase as fewer workers were able to find permanent positions. (C) That employers would have less control over the terms of workers’ employment as workers increased their bargaining strength. (D) That more workers would be hired for temporary positions as product demand increased. (E) That the number of workers taking temporary positions would increase as more workers in any given demographic group entered the workforce. 6.

In the context of the passage, the word “excessive� most closely corresponds to which of the following phrases?

(A) Far more than can be justified by worker preferences. (B) Far more than can be explained by fluctuations in product demand. (C) Far more than can be beneficial to the success of the firms themselves. (D) Far more than can be accounted for by an expanding national economy. (E) Far more than can be attributed to increases in the total number of people in the workforce.

56


7.

The passage mentions each of the following as an appropriate kind of governmental action EXCEPT

(A) getting firms to offer temporary employment primarily to a certain group of people (B) encouraging equitable pay for temporary and permanent employees (C) facilitating the organization of temporary workers by labor unions (D) establishing guidelines on the proportion of temporary workers that firms should employ (E) ensuring that temporary workers obtain benefits from their employers

57


Reading Comprehension ANSWER KEY

58

Text\Qs# 1 1 A

2

3

4

5

6

C

C

2

D

C

E

B

3

B

B

D

E

4

C

D

C

5

C

B

E

C

6

D

C

E

B

7

B

B

B

D

8

B

C

D

A

C

E

9

B

A

E

D

D

C

10

A

E

D

C

C

A

11

E

C

B

D

12

C

B

E

A

B

D

13

D

D

C

D

C

D

14

C

B

A

C

B

15

B

E

A

C

16

A

C

D

E

17

C

B

A

18

D

D

A

E

B

B

19

B

D

A

E

E

C

20

C

B

E

E

E

A

7

A

C

D


Part II Critical Reasoning Critical reasoning is the logical section of the test that includes a task, question and answer choices, among which we have to choose the best one. To accomplish this section a basic understanding of common terms and question types is needed. 2 important facts:  You are looking for the best answer among 5 provided, not just the right one  You must use only the given (stated or implied) information; applying additional knowledge can get you into trouble. As any typical task of CR has an argument, question, and answers, we will discuss every element separately. Most of the questions refer to the argument or conclusion, so the key to success is to locate conclusion correctly. Knowing the Structure of an Argument is vital! If a question wants us to infer something from the argument, while we think we are to strengthen it, we fail. Identifying the Question type is crucial! When we are left with two answer choices and consider both correct, we may often miss some tiny difference. Using Elimination methods is necessary!

59


1) STRUCTURE OF AN ARGUMENT (GoGMAT Session 2) The task and its structure: ARGUMENT: is a statement or a set of statements on which the question is based. Generally, an argument is composed of certain facts, descriptions and an opinion of the author. In terms of this section, an argument consists of evidence and conclusion. EVIDENCE: is the factual, objective, true by default part of the argument that can be identified by the use of evidence keywords such as because, since, for, etc. This structural component of the argument gives different descriptions, dates, events, and other types of data. CONCLUSION: represents the author’s point of view, an idea derived from evidence. This information is subjective, not necessarily true and can be doubted or supported. Conclusion does not necessarily come at the end of the argument and can be identified by such keywords as therefore, thus, as a result, hence, clearly, consequently, so, etc. ASSUMPTION: is never stated explicitly in the argument and answers the question ‘why else does the author come to such a conclusion?’ If the assumption is not true, it follows that the conclusion may not be true. To find the assumption one should answer the question – what is missed in the argument? By definition, the assumption will not contain any absolutely new information. The function of an assumption is to explain why using such evidence the author comes to such a conclusion. EVIDENCE (written/why the author came to conclusion) – ASSUMPTION (not written/why else the author came to conclusion) – CONCLUSION (written/the author’s idea based on evidence and assumption)

Scheme of an Argument Evidence

Conclusion

- Facts

- Opinion

(why does the author say something?) - Keywords to locate Evidence

(what does the say?) Assumption

- Keywords to locate Conclusion:

Because

- is additional evidence

Therefore

For

- explains why based on

Thus

Since

evidence the author made

Hence etc.

As

such a conclusion

Should Must

60

- Data (numbers and percentages)

Claim

- Dates (In 1985)

Contention


Sample Argument: John wants to get 730 on GMAT in the next 3 months. (Evidence) John should do a lot of practice. (Conclusion) Assumption to this argument should explain why based on the fact that John wants 730, he should do practice. Assumption: a lot of practice helps to get 730 on GMAT. Home Practice 1 1. Although the earliest surviving Greek inscriptions written in an alphabet date from the eighth century B.C., the fact that the text of these Greek inscriptions sometimes run from right to left and sometimes from left to right indicates that the Greeks adopted alphabetic writing at least two centuries before these inscriptions were produced. After all, the Greeks learned alphabetic writing from the Phoenicians, and presumably, along with the alphabet, they also adopted the then-current Phoenician practice with respect to the direction of text. And although Phoenician writing was originally inconsistent in direction, by the eights century B.C. Phoenician was consistently written from right to left and had been for about two centuries. In the argument given, the two portions in boldface play which of the following roles? (A) The first and the second each describe evidence that has been used to challenge the position that the argument seeks to establish. (B) The first evidence that forms the basis for an objection to the position that the argument seeks to establish; the second is that position. (C) The first evidence that forms the basis for an objection to the position that the argument seeks to establish; the second is a consideration that is produced to counter the force of that evidence. (D) The first and the second each provide evidence in support of the position that the argument seeks to establish. (E) The first provides evidence in support of the position that the argument seeks to establish; the second is that position. 2. In countries where automobile insurance includes compensation for whiplash injuries sustained in automobile accidents, reports of having suffered such injuries are twice as frequent as they are in countries where whiplash is not covered. Presently, no objective test for whiplash exists, so it is true that spurious reports of whiplash injuries cannot be readily identified. Nevertheless, these facts do not warrant the conclusion drawn by some commentators that in the countries with the higher rates of reported whiplash injuries, half of the reported cases are spurious.

61


Clearly, in countries where automobile insurance does not include compensation for whiplash, people often have little incentive to report whiplash injuries that they actually have suffered. In the argument given, the two portions in boldface play which of the following roles? (A) The first is a claim that the argument disputes; the second is a conclusion that has been based on that claim. (B) The first is a claim that has been used to support a conclusion that the argument accepts; the second is that conclusion. (C) The first is evidence that had been used to support a conclusion for which the argument provides further evidence; the second is the main conclusion of the argument. (D) The first is finding whose implications are at issue in the argument; the second is a claim presented in order to argue against deriving certain implications from that finding. (E) The first is a finding whose accuracy is evaluated in the argument; the second is evidence presented to establish that the finding is accurate. 3. Networks of blood vessels in bat’s wings serve only to disperse heat generated in flight. This heat is generated only because bats flap their wings. Thus paleontologists’ recent discovery that the winged dinosaur Sandactylus had similar networks of blood vessels in skin of its wings provides evidence for the hypothesis that Sandactylus flew by flapping its wings, not just by gliding. In the passage, the author develops an argument by (A) forming the hypothesis that best explains several apparently conflicting pieces of evidence (B) reinterpreting evidence that had been used to support an earlier theory (C) using an analogy with a known phenomenon to draw a conclusion about an unknown phenomenon (D) speculating about how structures observed in present-day creatures might have developed from similar structures in creatures now extinct (E) pointing out the differences in the physiological demands that flight makes on large, as opposed to small, creatures 4. Keith: Compliance with new government regulations requiring the installation of smoke alarms and sprinkler systems in all theaters and arenas will cost the entertainment industry $ 25 billion annually. Consequently, jobs will be lost and profits diminished. Therefore, these regulations will harm the country’s economy. Laura: The $25 billion spent by some businesses will be revenue for others. Jobs and profits will be gained as well as lost. Laura responds to Keith by

62


(A) Demonstrating that Keith’s conclusion is based on evidence that is not relevant to the issue at hand (B) Challenging the plausibility of the evidence that serves as the basis for Keith’s argument (C) Suggesting that Keith’s argument overlooks a mitigating consequence (D) Reinforcing Keith’s conclusion by supplying a complementary interpretation of the evidence Keith cites (E) Agreeing with the main conclusion of Keith’s argument but construing that conclusion as grounds for optimism rather than for pessimism.

QUESTION TYPES 1.

ASUMPTION QUESTIONS (GoGMAT Session 2) These questions usually sound like:

• "The study’s conclusion is based upon which of the following assumptions?" • “The argument above depends upon which of the following assumptions?” • “Which of the following identifies an assumption in the argument above?” • “The argument above assumes that” • “The conclusion above would be more reasonably drawn if which of the following were inserted into the argument as an additional premise?” Assumption questions are asked to different types of arguments and according to argument’s type assumptions bring about different information. A couple of possible assumption arguments are provided below.  Global and particular arguments talk about trends and groups based on information about specific subjects or objects or vice versa. In this case either the global or the particular is introduced into the argument in conclusion and can be considered the new subject of the argument. Assumptions for such arguments should indicate that those subjects or objects represent a part of trend, group and so on.  Comparison arguments compare two subjects/objects that have various characteristics (price, size, etc.) and make a conclusion about a difference or preference between the two. The task is to find the one feature that lets these two subjects/objects be comparable in the first place. Assumption for such arguments should indicate a specific common quality of the two subject/objects to show that they are comparable. Very often this quality will be the main function/characteristic of these objects.  Ideas substitution arguments describe a subject, its quality, characteristics in the evidence. Conclusion shifts these features to another subject that was not mentioned in the previously.

63


The point is that two subjects have the same characteristics or purpose even though the argument says nothing concrete about the second subject. This second subject from conclusion can be considered the new subject of the argument. Assumption to this argument should indicate that the second subject has the same features and that the subjects are almost equal. Steps to answer the question: 1. 2.

Read the Q Identify Question Type (phrases from question: assume, assumption, enables to draw a conclusion, additional premise) 3. Read the Argument 4. Find Conclusion (conclusion keywords: therefore, thereby, so, should, must, claim, contention that) 5. Identify conclusion type (to do so ask yourself “Do we have a comparison in this argument?” – can be indicated with the help of comparison keywords: like, unlike, compared to etc.)  YES – Comparison argument (an argument comparison two subjects or objects based on some qualities) Answer will give a new quality based on which the subjects from Argument can be compared.  NO – New subject ( a subject is given in conclusion, but not in the evidence) Answer will talk about this new subject from conclusion. Eliminate answers that:  Are out of scope  Provide the same information as is given in the argument  Contain extreme language  Do not support/ explain conclusion Sample Assumption arguments: Instructions: read attentively each argument (task) below. Identify which sentences of the argument represent evidence and which sentence is conclusion. In the “Evidence/Conclusion” space below you can simply put the number of sentence you consider either evidence or conclusion. Pay attention, you do not need to make them up. Evidence and conclusion are written in the argument. After you identify these key elements of the argument, try to find assumption, which can explain why based on evidence the author made such a conclusion, and write it down in the “Assumption” space below. (A) Doctors say that the best boost for your brain in the morning is a cup of sweet tea or coffee. Otherwise you get sleepy at work and cannot concentrate in the mornings. Therefore, Mark should drink either sweet tea or coffee. Evidence:_______________________________________________________________ Conclusion:______________________________________________________________

64


Assumption:____________________________________________________________ _______________________________________________________________________ _______________________________________________________________________

(B) I want to get the best score on the exam. Therefore, I should take individual preparatory classes. Evidence:_______________________________________________________________ Conclusion:_____________________________________________________________ Assumption:_____________________________________________________________ ___________________________________________________________________________ ___________________________________________________________________________ (C) If you sleep 8 hours a day, you will feel less tired. Thus, Alex sleeps 8 hours a day. Evidence:_______________________________________________________________ Conclusion:______________________________________________________________ Assumption:_____________________________________________________________ ___________________________________________________________________________ ___________________________________________________________________ (D) Bridal agency, Happiness, is known for creating unforgettable weddings for the newlyweds. Ashley should address this agency. Evidence:_______________________________________________________________ Conclusion:______________________________________________________________ Assumption:_____________________________________________________________ ___________________________________________________________________________ ___________________________________________________________________ (E) During the last 3 years the company has implemented two new processes, so the boss is interested in innovations. Evidence:__________________________________________________________________ Conclusion:________________________________________________________________ Assumption:________________________________________________________________

___________________________________________________________________________ ___________________________________________________________________________

65


Assumption questions practice

GoGMAT Problem

66


GoGMAT Problem Explanation

Instructions: chose one answer out of five given in each task. You should spend no more than 2 minutes per question. 1. (K) World War II had a profound effect on the growth of nascent businesses. The Acme Packaging Company netted only $10,000 in the year before the war. By 1948 it was earning almost ten times that figure. The argument above depends upon which of the following assumptions? (A) (B) (C) (D) (E)

Acme's growth rate is representative of other nascent businesses. An annual profit of $10,000 is not especially high. Wars inevitably stimulate a nation's economy. Rapid growth for nascent businesses is especially desirable. Acme is not characterized by responsible, far-sighted managers.

2. (K)Magnetic resonance imaging (MRI)—a noninvasive diagnostic procedure—can be used to identify blockages in the coronary arteries. In contrast to angiograms—the invasive procedure customarily used—MRI’s pose no risk to patients. Thus, to guarantee patient safety in the attempt to diagnose arterial blockages MRI’s should replace angiograms in all attempts at diagnosing coronary blockages. Which of the following identifies an assumption in the argument above?

67


(A) (B) (C) (D) (E)

Angiograms can be used to diagnose conditions other than blockages in arteries. MRI’s were designed primarily in order to diagnose blockages in the coronary arteries. Angiograms reveal more information about the nature of a blockage than an MRI can. An MRI is just as likely as an angiogram to identify an arterial blockage. Some patients for whom an angiogram presents no risk are unwilling to undergo an MRI.

3. (K) Enrollment in graduate and professional programs tends to be high in a strong economy and much lower during recessions. The perceived likelihood of future job availability, therefore, affects people's willingness to pass up immediate earning potential in order to invest in career-related training. The argument above assumes that (A) the perceived likelihood of job availability has decreased in recent years (B) all those who avoid graduate and professional school during an economic slump do so because of the perceived lack of future jobs (C) perceptions of the likelihood of job availability are related to the state of the economy (D) those who enroll in graduate and professional schools during a strong economy help increase the economy's strength (E) graduate and professional programs admit fewer students during recessions Home Practice 2 1. Although computers can enhance people’s ability to communicate, computer games are a cause of underdeveloped communication skills in children. After-school hours spent playing computer games are hours not spent talking with people. Therefore, children who spend all their spare time playing theses games have less experience in interpersonal communication than other children have. The argument depends on which of the following assumptions? (A) Passive activities such as watching television and listening to music do not hinder the development of communication skills in children. (B) Most children have other opportunities, in addition to after-school hours, in which they can choose whether to play computer games or to interact with other people. (C) Children who do not spend all of their after-school hours playing computer games spend at least some of that time talking with other people. (D) Formal instruction contributes little or nothing to children’s acquisition of communication skills. (E) The mental skills developed through playing computer games do not contribute significantly to children’s intellectual development. 2. Because no employee wants to be associated with bad news in the eyes of a superior, information about serious problems at lower levels is progressively softened and distorted as it goes up each step in the management hierarchy. The chief executive is, therefore, less well-informed about problems at lower levels than are his or her subordinated at those levels.

68


The conclusion is drawn above is based on the assumption that (A) problems should be solved at the level in the management hierarchy at which they occur (B) employees should be rewarded for accurately reporting problems to their superiors (C) problem-solving ability is more important at higher levels that it is at lower levels of the management hierarchy (D) chief executives obtain information about problems at lower levels from no source other than their subordinates (E) some employees are more concerned about truth than about the way they are perceived by their superiors 3. Many people suffer an allergic reaction to certain sulfites, including those that are commonly added to wine as preservatives. However, since there are several winemakers who add sulfites to none of the wines they produce, people who would like to drink wine but are allergic to sulfites can drink wines produced by these winemakers without risking an allergic reaction to sulfites. Which of the following is an assumption on which the argument depends? (A) These winemakers have been able to duplicate the preservative effect produced by adding sulfites by means that do not involve adding any potentially allergenic substances to their wine. (B) Not all forms of sulfite are equally likely to produce the allergic reaction. (C) Wine is the only beverage to which sulfites are commonly added. (D) Apart from sulfites, there are no substances commonly present in wine that give rise to an allergic reaction. (E) Sulfites are not naturally present in the wines produced by these winemakers in amounts large enough to produce an allergic reaction in someone who drinks these wines. 4. Commentator: The theory of trade retaliation states that countries closed out of any of another country's markets should close some of their own markets to the other country in order to pressure the other country to reopen its markets. If every country acted according to this theory, no country would trade with any other. The commentator's argument relies on which of the following assumptions? (A) No country actually acts according to the theory of trade retaliation. (B) No country should block any of its markets to foreign trade. (C) Trade disputes should be settled by international tribunal. (D) For any two countries, at least one has some market closed to the other. (E) Countries close their markets to foreigners to protect domestic producers.

69


5. Although parapsychology is often considered a pseudoscience, it is in fact a genuine scientific enterprise, for it uses scientific methods such as controlled experiments and statistical tests of clearly stated hypotheses to examine the questions it raises. The conclusion above is properly drawn if which of the following is assumed? (A) If a field of study can conclusively answer the questions it raises, then it is a genuine science. (B) Since parapsychology uses scientific methods, it will produce credible results. (C) Any enterprise that does not use controlled experiments and statistical tests is not genuine science. (D) Any field of study that employs scientific methods is a genuine scientific enterprise. (E) Since parapsychology raises clearly statable questions, they can be tested in controlled experiments. 6. Last year all refuse collected by Shelbyville city services was incinerated. This incineration generated a large quantity of residual ash. In order to reduce the amount of residual ash Shelbyville generates this year to half of last year's total, the city has revamped its collection program. This year city services will separate for recycling enough refuse to reduce the number of truckloads of refuse to be incinerated to half of last year's number. Which of the following is required for the revamped collection program to achieve its aim? (A) This year, no materials that city services could separate for recycling will be incinerated. (B) Separating recyclable materials from materials to be incinerated will cost Shelbyville less than half what it cost last year to dispose of the residual ash. (C) Refuse collected by city services will contain a larger proportion of recyclable materials this year than it did last year. (D) The refuse incinerated this year will generate no more residual ash per truckload incinerated than did the refuse incinerated last year. (E) The total quantity of refuse collected by Shelbyville city services this year will be no greater than that collected last year. 2.

STRENGTHEN QUESTIONS (GoGMAT Session 5) These questions usually sound like:

• "Which of the following, if true, would most strengthen the argument above?" • “Which of the following, if true, would provide the most support for the argument above?” • “The argument above would be more persuasive if which one of the following was found to be true?”

70


• “Which of the following, if discovered to be true, would lend the most support to the conclusion above?” • “Which of the following statements, if true, would be of the most use to…?” • “Which of the following statements, if true, would help to validate the…?” Strengthen questions are intent on showing that the information given in the argument “may really be true”. You have to remember the following notions: 1. 2. 3. 4.

Strengthening is not about proving, but about showing the possibility of occurrence. Evidence does not need support as facts are doubtless by default. You can strengthen only the conclusion. To strengthen the conclusion means to show that “maybe, conclusion is really true”.

According to the argument/conclusion type there are various strengthening answers. A couple of possible strengthen argument types are provided below. Usual arguments provide common argument structure as is seen from sample tasks in Assumption questions block in this book. Strengthen answers for such arguments particularize assumption in support of the conclusion. Cause and Effect arguments provide a clear cause and effect connection throughout the evidence and conclusion. You can indicate the cause from conclusion. Critical reasoning allows two possible strengthen answers for such arguments. 1. Showing that the cause exists. By providing information about the cause of the cause from conclusion, the answer shows that the cause-effect connection may really exist. Cause (new) – Cause (in conclusion) – Effect (in either evidence or conclusion) 2. Showing that the opposite connection does not exist. Between two objects A and B there are only two possible connections: from A to B or from B to A. When we deny “B to A”, if the connection took place, we show that it was more likely “A to B”. Steps to answer the question: 1. 2. 3. 4.

Read the Q Identify Question Type (phrases from question: support, strengthen, validate, persuasive) Read the Argument Find Conclusion (conclusion keywords: therefore, thereby, so, should, must, claim, contention that) 5. Identify conclusion type (to do so ask yourself “Do we have direct cause and effect in this argument?” – can be indicated with the help of keywords: due to, as a result of, because of etc.)  NO - Without Cause and Effect (without due to, caused by) Answer will directly or indirectly support assumption, which we have to make ourselves  YES - With Cause and Effect (with due to, caused by) Answer will give:

71


New cause of the cause from the argument to show that the cause from argument exists Effect doesn't lead to the cause (the answer will deny the opposite connection to show that the initial connection is correct)

Eliminate answers that:  Are out of scope  Contain extreme language  Contradict evidence Sample Strengthen arguments: Instructions: read attentively each argument (task) below. Identify which sentences of the argument represent evidence and which sentence is conclusion. After you identify these key elements of the argument, try to formulate a strengthening answer, which can show that conclusion is, probably, really true, and write it down in the “Strengthen” space below. Remember, it should be something in the scope of the argument. 1) Doctors say that the best boost for your brain in the morning is a cup of sweet tea or coffee. Otherwise you get sleepy at work and cannot concentrate in the mornings. Therefore, Mark should drink either sweet tea or coffee. Strengthen:______________________________________________________________ ___________________________________________________________________________ ___________________________________________________________________

2) I want to get the best score on the exam. Therefore, I should take individual preparatory classes. Strengthen:______________________________________________________________ ___________________________________________________________________________ ___________________________________________________________________

3) Alex sleeps 8 hours a day. As a result he feels less tired. Strengthen:______________________________________________________________ ___________________________________________________________________________ ___________________________________________________________________

4) Mother came home yesterday and discovered a broken vase. She concluded that her daughter had broken the vase. Strengthen:______________________________________________________________

72


___________________________________________________________________________ ___________________________________________________________________

5) As I am smart, I will solve this problem. Strengthen:______________________________________________________________ ___________________________________________________________________________ ___________________________________________________________________

73


Strengthen questions practice

GoGMAT Problem

74


GoGMAT Problem Explanation

Instructions: chose one answer out of five given in each task. You should spend no more than 2 minutes per question. 4. (K) The local high school students have been clamoring for the freedom to design their own curricula. Allowing this would be as disastrous as allowing three-year-olds to choose their own diets. These students have neither the maturity nor the experience to equal that of the professional educators now doing the job. Which of the following statements, if true, would most strengthen the above argument? (A) (B) (C) (D) (E)

High school students have less formal education than those who currently design the curricula. Three-year-olds do not, if left to their own devices, choose healthful diets. The local high school students are less intelliÂŹgent than the average teenager. Individualized curricula are more beneficial to high school students than are the standard curricula, which are rigid and unresponsive to their particular strengths and weaknesses. The ability to design good curricula develops only after years of familiarity with educational life.

75


5. (K) Recent surveys show that many people who have left medical school before graduating suffer from depression. Clearly, depression is likely to cause withdrawal from medical school. Which of the following, if true, would most strengthen the conclusion above? (A) Many medical schools provide psychological counseling for their students. (B) About half of those who leave medical school report feeling depressed after they make the decision to leave. (C) Depression is very common among management consultants who have a similarly difficult work schedule to those of many young doctors. (D) Medical students who have sought depression counseling due to family problems leave at a higher rate than the national average. (E) Career change has been shown to be a strong contributing factor in the onset of depression.

6. (K) A recently published article on human physiology claims that enzyme К contributes to improved performance in strenuous activities such as weightlifting and sprinting. The article cites evidence of above-average levels of enzyme К in Olympic weightlifters and sprinters. Which of the following, if true, would most strengthen the articles conclusion? (A) Enzyme К levels are the most important factor affecting the performance of strenuous activities. (B) Enzyme К has no other function in the human body. (C) Enzyme К is required for the performance of strenuous activities. (D) Enzyme К helps weightlifters more than it helps sprinters. (E) Strenuous activities do not cause the human body to produce unusually high levels of enzyme K. Home Practice 3 1. Most employees in the computer industry move from company to company, changing jobs several times in their careers. However, Summit Computers is known throughout the industry for retaining its employees. Summit credits its success in retaining employees to its informal, nonhierarchical work environment. Which of the following, if true, most strongly supports Summit's explanation of its success in retaining employees? A) Some people employed in the computer industry change jobs if they become bored with their current projects. (B) A hierarchical work environment hinders the cooperative exchange of ideas that computer industry employees consider necessary for their work. (C) Many of Summit's senior employees had previously worked at only one other computer company.

76


(D) In a nonhierarchical work environment, people avoid behavior that might threaten group harmony and thus avoid discussing with their colleagues any dissatisfaction they might have with their jobs. (E) The cost of living near Summit is relatively low compared to areas in which some other computer companies are located. 2. Studies in restaurants show that the tips left by customers who pay their bill in cash tend to be larger when the bill is presented on a tray that bears a credit card logo. Consumer psychologists hypothesize that simply seeing a credit-card logo makes many credit card holders willing to spend more because it reminds them that their spending power exceeds the cash they have immediately available. Which of the following, if true, most strongly supports the psychologists' interpretation of the studies? (A) The effect noted in the studies is not limited to patrons who have credit cards. (B) Patrons who are under financial pressure from their credit-card obligations tend to tip less when presented with a restaurant bill on a tray with a credit-card logo than when the tray has no logo. (C) In virtually all of the cases in the studies, the patrons who paid bills in cash did not possess credit cards. (D) In general, restaurant patrons who pay their bills in cash leave larger tips than do those who pay by credit card. (E) The percentage of restaurant bills paid with a given brand of credit card increases when that credit card's logo is displayed on the tray with which the bill is presented. 3. Which of the following most logically completes the argument? The irradiation of food kills bacteria and thus retards spoilage. However, it also lowers the nutritional value of many foods. For example, irradiation destroys a significant percentage of whatever vitamin B1 a food may contain. Proponents of irradiation point out that irradiation is no worse in this respect than cooking. However, this fact is either beside the point, since much irradiated food is eaten raw, or else misleading, since ___________________ (A) many of the proponents of irradiation are food distributors who gain from foods' having a longer shelf life (B) it is clear that killing bacteria that may be present on food is not the only effect that irradiation has (C) cooking is usually the final step in preparing food for consumption, whereas irradiation serves to ensure a longer shelf life for perishable foods (D) certain kinds of cooking are, in fact, even more destructive of vitamin B1 than carefully controlled irradiation is (E) for food that is both irradiated and cooked, the reduction of vitamin B1 associated with either process individually is compounded

77


4. It is true of both men and women that those who marry as young adults live longer than those who never marry. This does not show that marriage causes people to live longer since, as compared with other people of the same age, young adults who are about to get married have fewer of the unhealthy habits that can cause a person to have a shorter life, most notably smoking and immoderate drinking of alcohol. Which of the following, if true, most strengthens the argument above? (A) Marriage tends to cause people to engage less regularly in sports that involve risk of bodily harm. (B) A married person who has an unhealthy habit is more likely to give up that habit than a person with the same habit who is unmarried. (C) A person who smokes is much more likely than a nonsmoker to marry a person who smokes at the time of marriage, and the same is true for people who drink alcohol immoderately. (D) Among people who marry as young adults, most of those who give up an unhealthy habit after marriage do not resume the habit later in life. (E) Among people who as young adults neither drink alcohol immoderately nor smoke, those who never marry live as long as those who marry. 3. WEAKEN QUESTIONS (GoGMAT Session 5) These questions usually sound like: • "Which of the following, if true, would most weaken the argument above?" • “Which are the following, if true, would most seriously damage the argument above?” • “Which of the following, if true, casts the most doubt on the argument above?” • “Which of the following, if true, most seriously undermines the validity of the explanation in the argument above?” Weaken questions are intent on showing that the information given in the argument “may not really be true”. You have to remember the following notions: 1. 2. 3. 4.

Weakening is not about denying, but about showing the possibility of absent occurrence. Evidence cannot be denied as facts are doubtless by default. You can weaken only the conclusion. To weaken the conclusion means to show that “maybe, conclusion is not really true”.

According to the argument/conclusion type, there are various weakening answers. A couple of possible weaken arguments are provided below.

78


Usual arguments provide common argument structure as is seen from sample tasks in Assumption and Strengthen question blocks in this book. Weaken answers for such arguments directly or indirectly deny the assumption. Cause and Effect arguments provide a clear cause and effect connection throughout the evidence and conclusion. You can indicate the cause from conclusion. Critical reasoning allows two possible weaken answers for such arguments. 1. Denying the cause. By showing that the effect from the conclusion is possible without the cause in question, the answer questions the cause from the argument. 2. Providing an alternative cause. In this way the answer again questions the cause mentioned in the argument. Steps to answer the question: 1. Read the Q 2. Identify Question Type (phrases from question: weaken, cast doubt, question, undermine validity, damage) 3. Read the Argument 4. Find Conclusion (conclusion keywords: therefore, thereby, so, should, must, claim, contention that) 5. Identify conclusion type (to do so ask yourself “Do we have direct cause and effect in this argument?” – can be indicated with the help of keywords: due to, as a result of, because of etc.)  NO - Without Cause and Effect (without due to, caused by) Answer will directly or indirectly deny assumption, which we have to make ourselves  YES - With Cause and Effect (with due to, caused by) Answer will give Alternative cause of the effect from the argument Effect without the cause from the argument in another situation Eliminate answers that:  Are out of scope  Contain extreme language  Contradict evidence Sample Weaken arguments: Instructions: read attentively each argument (task) below. Identify which sentences of the argument represent evidence and which sentence is conclusion. After you identify these key elements of the argument, try to formulate a weakening answer, which can show that conclusion is, probably, not true, and write it down in the “Weaken” space below. Remember, it should be something in the scope of the argument.

79


1. Doctors say that the best boost for your brain in the morning is a cup of sweet tea or coffee. Otherwise you get sleepy at work and cannot concentrate in the mornings. Therefore, Mark should drink either sweet tea or coffee. Weaken:______________________________________________________________ ___________________________________________________________________________ ___________________________________________________________________

2. I want to get the best score on the exam. Therefore, I should take individual preparatory classes. Weaken:______________________________________________________________ ___________________________________________________________________________ ___________________________________________________________________

3. Alex sleeps 8 hours a day. As a result he feels less tired. Weaken:______________________________________________________________ ___________________________________________________________________________ ___________________________________________________________________

4. Mother came home yesterday and discovered a broken vase. She concluded that her angry daughter had broken the vase. Weaken:______________________________________________________________ ___________________________________________________________________________ ___________________________________________________________________

5. As I am smart, I will solve this problem. Weaken:______________________________________________________________ ___________________________________________________________________________ ___________________________________________________________________

80


Weaken questions practice

GoGMAT Problem

81


GoGMAT Problem Explanation

Instructions: chose one answer out of five given in each task. You should spend no more than 2 minutes per question. 7. (K) An investigation must be launched into the operations of the private group that is training recruits against the Balaland Republic. The United States Neutrality Act plainly forbids United States from engaging in military campaigns against anŃƒ nation with which we are not at war. Since no war has been declared between the United States and Balaland Republic, we should bring charges for these fanatics. Which of the following, if true, would most weaken the argument above? (A) The Balaland Republic is currently engaged in a bloody and escalating civil war. (B) Diplomatic relations between the United States and the Balaland Republic were severed last year. (C) The recruits are being trained to fight only in the event the United States goes to war against the Balaland Republic. (D) The training of recruits is funded not by United States citizens, but rather by a consortium of individuals from abroad.

82


(E)

Charges cannot be brought against the private group that is training the recruits unless an investigation is first launched.

8. (K) The owner of a four-story commercial building discovered termites in the building's first and second floors and called an exterminator. The exterminator pumped gas into the walls on both the first and second floors. Due to the exterminator's work, the termites on those floors were killed quickly. Which of the following, if true, most seriously undermines the validity of the explanation for the speed with which the termites were killed? (A) The third floor had no termite infestation. (B) Even though the exterminator did not pump gas into the walls of the fourth story, the termites there died as quickly as they did on the first and second stories. (C) The speed at which termites are killed increases as the concentration of an exterminator's gas increases. (D) The speed with which the exterminator's gas kills termites drops off sharply as the gas dissipates throughout the building's walls. (E) The exterminator's gas-pumping system works efficiently even when pumping gas into both the first and second stories of the building simultaneously. 9. (K) The rate of violent crime in this state is up 30 percent from last year. The fault lies entirely in our court system. Recently our judges' sentences have been so lenient that criminals can now do almost anything without fear of a long prison term. The argument above would be weakened if it were true that (A) (B) (C) (D) (E)

85 percent of the other states in the nation have lower crime rates than does this state white collar crime in this state has also increased by over 25 percent in the last year 35 percent of the police in this state have been laid off in the last year due to budget cuts polls show that 65 percent of the population in this state oppose capital punishment the state has hired 25 new judges in the last year to compensate for deaths and retirements

Home Practice 4 1. In comparison to the standard typewriter keyboard, the EFCO keyboard, which places the mostused keys nearest the typist's strongest fingers, allows faster typing and results in less fatigue. Therefore, replacement of standard keyboards with the EFCO keyboard will result in an immediate reduction of typing costs. Which of the following, if true, would most weaken the conclusion drawn above? (A) People who use both standard and EFCO keyboards report greater difficulty in the transition from the EFCO keyboard to the standard keyboard than in the transition from the standard keyboard to the EFCO keyboard.

83


(B) EFCO keyboards are no more expensive to manufacture than are standard keyboards and require less frequent repair than do standard keyboards. (C) The number of businesses and government agencies that use EFCO keyboards is increasing each year. (D) The more training and experience an employee has had with the standard keyboard, the more costly it is to train that employee to use the EFCO keyboard. (E) Novice typists can learn to use the EFCO keyboard in about the same amount of time that it takes them to learn to use the standard keyboard. 2. In the United States, of the people who moved from one state to another when they retired, the percentage who retired to Florida has decreased by three percentage points over the past ten years. Since many local businesses in Florida cater to retirees, these declines are likely to have a noticeably negative economic effect on these businesses and therefore on the economy of Florida. Which of the following, if true, most seriously weakens the argument given? (A) People who moved from one state to another when they retired moved a greater distance, on average, last year than such people did ten years ago. (B) People were more likely to retire to North Carolina from another state last year than people were ten years ago. (C) The number of people who moved from one state to another when they retired has increased significantly over the past ten years. (D) The number of people who left Florida when they retired to live in another state was greater last year than it was ten years ago. (E) Florida attracts more people who move from one state to another when they retire than does any other state. 3. A new law gives ownership of patents - documents providing exclusive right to make and sell an Invention - to universities, not the government when those patents result from government sponsored university research. Administrators at Logos University plan to sell any patents they acquire to corporations in order to fund programs to improve undergraduate teaching. Which of the following, if true, would cast the most doubt on the viability of the college administrators' plan described above? (A) Profit-making corporations interested in developing products based on patents held by universities are likely to try to serve as exclusive sponsors of ongoing university research projects. (B) Corporate sponsors of research in university facilities are entitled to tax credits under new federal tax-code guidelines. (C) Research scientists at Logos University have few or no teaching responsibilities and participate little if at all in the undergraduate programs in their field. (D) Government-sponsored research conducted at Logos University for the most part duplicates research already completed by several profit-making corporations.

84


(E) Logos University is unlikely to attract corporate sponsorship of its scientific research. 4. Guidebook writer: I have visited hotels throughout the country and have noticed that in those built before 1930 the quality of the original carpentry work is generally superior to that in hotels built afterward. Clearly carpenters working on hotels before 1930 typically worked with more skill, care, and effort than carpenters who have worked on hotels built subsequently. Which of the following, if true, most seriously weakens the guidebook writer's argument? (A) The quality of original carpentry in hotels is generally far superior to the quality of original carpentry in other structures, such as houses and stores. (B) Hotels built since 1930 can generally accommodate more guests than those built before 1930. (C) The materials available to carpenters working before 1930 were not significantly different in quality from the materials available to carpenters working after 1930. (D) The better the quality of original carpentry in a building, the less likely that building is to fall into disuse and be demolished. (E) The average length of apprenticeship for carpenters has declined significantly since 1930. 4. INFERENCE QUESTIONS (GoGMAT Session 5) These questions usually sound like: • "The facts above best support which of the following conclusions?" • “Which of the following conclusions can be properly drawn from the information above?” • “If the statements above are all true, which of the following can be properly inferred on the basis of them?” • “If the statements above are true, which of the following must be true?” • “Which of the following best expresses the argument above?” Inference questions are intent on summarizing/paraphrasing the argument. You have to remember the following notions: 1. Inference is not about analyzing deeply and concluding, but about making a probable statement out of the argument. 2. A good inference answer should not contain any new information that is not 100% true on the base of the facts from the argument. This is a point of difference between an inference and a conclusion. 3. As anything you infer, even if it is simply a paraphrase, is never just the same thing as the argument states, the best answers often contain probability key words, such as “may, might, can, probably, likely” etc. 4. If the argument has both evidence and conclusion, the best inference will summarize the whole argument; otherwise, the best answer may just paraphrase either conclusion or some evidence. 5. Inference should not contradict the argument.

85


Steps to answer the question: 1. Read the Q 2. Identify Question Type (phrases from question: support, inference, infer, must be true, can be true, express the argument, draw a conclusion) 3. Read the Argument 4. If the argument provides some general info and an opinion, the answer will focus on the key subject of the argument and paraphrase the same idea. 5. If the argument gives a chain of ideas, chose an answer with first and last elements of the chain 6. If the argument compares two groups in numbers or percentages, the answer will infer that that one is bigger, more….than the other group.

Eliminate answers that:  Are out of scope  Contain extreme language  Require additional information  Contradict evidence Sample Inference arguments: Instructions: read attentively each argument (task) below. Try to make a probable statement, based on the information in argument, and write your answer in the “Inference” field. Remember, although it should be something in the scope of the argument, additional information is possible. 1. It is known that cats can be either black or white. Inference:______________________________________________________________ ___________________________________________________________________________ ___________________________________________________________________

2. People can be either small or tall. It is not true that this person is not tall. Inference:______________________________________________________________ ___________________________________________________________________________ ___________________________________________________________________

3. More than a dozen of rare birds have been found in nature. All of the found birds are red. Inference:______________________________________________________________

86


___________________________________________________________________________ ___________________________________________________________________

4. It is not true that this cat is big and white. Inference:______________________________________________________________ ___________________________________________________________________________ ___________________________________________________________________

5. If you read a book, you will get smarter. If you get smarter, you will soon find a good job. Inference:______________________________________________________________ ___________________________________________________________________________ ___________________________________________________________________ Inference questions practice

GoGMAT Problem

87


GoGMAT Problem Explanation

Instructions: chose one answer out of five given in each task. You should spend no more than 2 minutes per question. 10. (K) The performance of Southport's two high schools has been quite consistent over the past five years. In each of those years, Suburban High has enrolled 40 percent of Southport's students and produced 75 percent of the town's high school graduates, while Lakeside High has accounted for the remainder. Which of the following can properly be inferred regarding the past five years from the passage above? (A) The total number of students attending each high school has remained roughly constant. (B) Students attending Suburban High come from a larger geographical area than students attending Lakeside High. (C) Lakeside High has graduated a lower percentage of its attendees than has Suburban High. (D) The respective geographic areas from which the schools draw their student populations have remained unchanged. (E) Students attending magnet programs accounted for a higher percentage of the graduating students at Lakeside High than at Suburban High.

88


11. (K) Techniques to increase productivity in the performance of discrete tasks, by requiring less human labor in each step of the production process, are widely utilized. Consultants on productivity enhancement point out, however, that although these techniques achieve their specific goal, they are not without drawbacks. They often instill enough resentment in the workforce eventually to lead to a slowdown in the production process as a whole. Which of the following can be reasonably inferred from the statements above? (A) Productivity enhancement techniques do not attain their intended purpose and should not be employed in the workplace. (B) The fact that productivity enhancement techniques are so widely employed has led to a decline in the ability of American businesses to compete abroad. (C) If productivity enhancement consultants continue to utilize these techniques, complete work stoppages will eventually result. (D) Ironically, an increase in the productivity of discrete tasks may result in a decrease in the productivity of the whole production process. (E) Production managers are dissatisfied with the efforts that productivity enhancement consultants have made to increase productivity. 12.

(K) All German philosophers, except for Marx, are idealists.

From which of the following can the statement above be most properly inferred? (A) (B) (C) (D) (E)

Except for Marx, if someone is an idealist philosopher, then he or she is German. Marx is the only non-German philosopher who is an idealist. If a German is an idealist, then he or she is a philosopher, as long as he or she is not Marx. Marx is not an idealist German philosopher. Aside from the philosopher Marx, if someone is a German philosopher, then he or she is an idealist.

Home Practice 5 1. The fewer restrictions there are on the advertising of legal services, the more lawyers there are who advertise their services, and the lawyers who advertise a specific service usually charge less for that service than the lawyers who do not advertise. Therefore, if the state removes any of its current restrictions, such as the one against advertisements that do not specify fee arrangements, overall consumer legal costs will be lower than if the state retains its current restrictions. If the statements above are true, which of the following must be true? (A) Some lawyers who now advertise will charge more for specific services if they do not have to specify fee arrangements in the advertisements. (B) More consumers will use legal services if there are fewer restrictions on the advertising of legal services.

89


(C) If the restriction against advertisements that do not specify fee arrangements is removed, more lawyers will advertise their services. (D) If more lawyers advertise lower prices for specific services, some lawyers who do not advertise will also charge less than the currently charge for those services. (E) If the only restrictions on the advertising of legal services were those that apply to every type of advertising, most lawyers would advertise their services. 2. To protect certain fledgling industries, the government of Country Z banned imports of the types of products those industries were starting to make. As a direct result, the cost of those products to the buyers, several export-dependent industries in Z, went up, sharply limiting the ability of those industries to compete effectively in their export markets. Which of the following conclusions about Country Z's adversely affected export-dependent industries is best supported by the passage? (A) Profit margins in those industries were not high enough to absorb the rise in costs mentioned above. (B) Those industries had to contend with the fact that other countries banned imports from Country Z. (C) Those industries succeeded in expanding the domestic market for their products. (D) Steps to offset rising materials costs by decreasing labor costs were taken in those industries. (E) Those industries started to move into export markets that they had previously judged unprofitable. 3. Environmentalist: The commissioner of the Fish and Game Authority would have the public believe that increases in the number of marine fish caught demonstrate that this resource is no longer endangered. This is a specious argument, as unsound as it would be to assert that the ever-increasing rate at which rain forests are being cut down demonstrates a lack of danger to that resource. The real cause of the increased fish-catch is a greater efficiency in using technologies that deplete resources. The environmentalist's statements, if true, best support which of the following as a conclusion? (A) The use of technology is the reason for the increasing encroachment of people on nature. (B) It is possible to determine how many fish are in the sea in some way other than by catching fish. (C) The proportion of marine fish that are caught is as high as the proportion of rain forest trees that are cut down each year. (D) Modern technologies waste resources by catching inedible fish. (E) Marine fish continue to be an endangered resource. 4. An experiment was done in which human subjects recognize a pattern within a matrix of abstract designs and then select another design that completes that pattern. The results of the experiment were surprising. The lowest expenditure of energy in neurons in the brain was found in those subjects who performed most successfully in the experiments.

90


Which of the following hypotheses best accounts for the findings of the experiment? (A) The neurons of the brain react less when a subject is trying to recognize patterns than when the subject is doing other kinds of reasoning. (B) Those who performed best in the experiment experienced more satisfaction when working with abstract patterns than did those who performed less well. (C) People who are better at abstract pattern recognition have more energy-efficient neural connections. (D) The energy expenditure of the subjects' brains increases, when a design that completes the initially recognized pattern is determined. (E) The task of completing a given design is more capably performed by athletes, whose energy expenditure is lower when they are at rest. 5. FLAW QUESTIONS (GoGMAT Session 8) These questions usually sound like: • "Which of the following would reveal most clearly the absurdity of the conclusion drawn above?" • “Which one of the following most accurately describes a flaw in the skeptic's reasoning?” Flaw questions are intent on showing that the conclusion is invalid. The answer should indicate “why the conclusion is incorrect”. You have to remember the following notion: Finding a flaw is different from finding a weakening answer as weakening answers should not contradict or disagree with conclusion. Steps to answer the question: 1. Read the Q 2. Identify Question Type (phrases from question: undermine validity, flaw, absurdity, criticism) 3. Read the Argument 4. Find Conclusion (conclusion keywords: therefore, thereby, so, should, must, claim, contention that) 5. Standard flaw is that conclusion is not directly supported by evidence 6. If the argument compares 2 groups in numbers, the flaw will be that total numbers of both groups are not given

Eliminate answers that:  Are out of scope  Contradict the argument

91


Sample Flaw arguments: Instructions: read attentively each argument (task) below. Identify sentence that is conclusion, as on flaw questions you always work with conclusions. Try to make a statement, based on the information in argument, explaining why conclusion is improper and write your answer in the “Flaw� field. It should be not a fact, but a reason. 1. 80% of white cats are fluffy and only 20% of black cats possess the same feature. Clearly, White cats are fluffier. Flaw:______________________________________________________________ ___________________________________________________________________________ ___________________________________________________________________

2. If you are sick, you have a headache. I have a headache, so I am sick. Flaw:______________________________________________________________ ___________________________________________________________________________ ___________________________________________________________________

3. Sausages, bananas, and soda water are all my favorite meals. So together they will make a very good dinner. Flaw:______________________________________________________________ ___________________________________________________________________________ ___________________________________________________________________

4. 20 teenagers and 15 adults came to see the show. Obviously, teenagers are more interested in the show than are adults. Flaw:______________________________________________________________ ___________________________________________________________________________ ___________________________________________________________________

92


Flaw questions practice

GoGMAT Problem

93


GoGMAT Problem Explanation

Instructions: chose one answer out of five given in each task. You should spend no more than 2 minutes per question. 13. (K) A study found that last year roughly 6,000 homeless people in the United States were admitted to hospitals because of malnutrition. In the same year, a little more than 10,000 nonhomeless people were admitted to hospitals for the same reason. These findings clearly show that the non-homeless are more likely to suffer from malnutrition than are the homeless. The answer to which of the following questions would be most likely to point out the illogical nature of the conclusion drawn above? (A) What is the relative level of severity of the malnutrition suffered by each group cited in the study? (B) To what extent, on average, are the non-homeless better off financially than are the homeless? (C) To what extent are the causes of malnutrition in the non- homeless related to ignorance of proper dietary habits? (D) What percentage of each group cited in the study suffered from malnutrition last year?

94


(E)

What effect would a large increase in the number of homeless shelters have on the incidence of malnutrition among the homeless?

14. (K) Attention Deficit Disorder (ADD) is a condition characterized by an inability to focus on any topic for a prolonged period of time, and is especially common among children five to ten years old. A recent study has shown that 85 percent of seven-year-old children with ADD watch, on average, more than five hours of television a day. It is therefore very likely that Ed, age seven, has ADD, since he watches roughly six hours of television a day. The argument above is flawed because it (A) cites as a direct causal mechanism a factor that may only be a partial cause of the condition in question (B) fails to indicate the chances of having ADD among seven-year-old children who watch more than five hours of television a day (C) limits the description of the symptoms of ADD to an inability to focus for a prolonged period of time (D) fails to consider the possibility that Ed may be among the 15 percent of children who do not watch more than five hours of television a day (E) does not allow for other causes of ADD besides television watching 15. (K) The average normal infant born in the United States weighs between twelve and fourteen pounds at the age of three months. Therefore, if a three-month-old child weighs only ten pounds, its weight gain has been below the United States average. Which of the following indicates a flaw in the reasoning above? (A) (B) (C) (D) (E)

Weight is only one measure of normal infant development. Some three-month-old children weigh as much as seventeen pounds. It is possible for a normal child to weigh ten pounds at birth. The phrase “below average� does not necessarily mean insufficient. Average weight gain is not the same as average weight.

95


6. EXPLAIN QUESTIONS (GoGMAT Session 8) These questions usually sound like: • "Which of the following, if true, would explain the discrepancy described above?" • “Which of the following, if true, contributes most to an explanation for the apparent contradiction noted above?” • “Which of the following, if true, would best account for…?” Explain questions are intent on giving a new fact that would explain the paradoxical situation described in the argument. You have to remember the following notions: 1. You can recognize the paradox by contrast/contradiction keywords such as “Although, but, however” etc. 2. The best answer can contain new, out-of-the-argument-scope information. Steps to answer the question: 1. 2. 3. 4. 5. 6.

Read the Q Identify Question Type (phrases from question: explain, account for, paradox) Read the Argument Find Contradiction (contradiction keywords: however, yet, but, although, while) Restate the contradiction In simple words Find answer that explains the contradiction with some new relevant info

Eliminate answers that:  Are irrelevant to the situation  Contradict the argument  Worsen the contradiction Sample Explain arguments: Instructions: read attentively each argument (task) below. Identify the contradiction and restate the discrepancy in an understandable language to make sure that you understand what you need to explain. Try to make a statement, based on the information in argument, explaining the contradiction and write your answer in the “Explain” field. Remember, answers to these questions mostly use new or additional information. 1. In school Greg mostly studied chemistry and biology. Yet, he was enrolled into a university. Explain:______________________________________________________________ _________________________________________________________________________________ _________________________________________________________________________

96


2. Tom usually gets to work in 30 minutes on feet. However, today he got to work in 10 minutes. Explain:______________________________________________________________ ___________________________________________________________________________ ___________________________________________________________________

3. During this year the sales of Coca-cola have increased, yet the number of people drinking it has not. Explain:______________________________________________________________ ___________________________________________________________________________ ___________________________________________________________________

4. Last year demand for new computers has fallen, but the average price of a computer sold has risen. Explain:______________________________________________________________ ___________________________________________________________________________ ___________________________________________________________________

97


Explain questions practice

GoGMAT Problem

98


GoGMAT Problem Explanation

Instructions: chose one answer out of five given in each task. You should spend no more than 2 minutes per question. 16. (K) Saguaro kangaroo rats generally leave watering holes, where food and water are abundant, during the day, while Sonoran kangaroo rats remain near these same watering holes and continue foraging throughout the day. Although Sonoran kangaroo rats have larger and more frequent litters, they are generally outnumbered by the Saguaro kangaroo rat. Which of the following, if true, would best resolve the apparent paradox described above? (A) Several species of successful rodents also leave the streams and watering holes during the day. (B) The Saguaro kangaroo rat matures much more slowly than the Sonoran kangaroo rat because of its relatively limited food supply. (C) Many of the predators of kangaroo rats, such as falcons and rattlesnakes, are only active around streams during the day. (D) Saguaro kangaroo rats are more sensitive to sunlight than are Sonoran kangaroo rats.

99


(E)

Sonoran kangaroo rats are reproductive to a greater age than are Saguaro kangaroo rats.

17. (K) A poll has revealed that 95 percent of the residents of Essex County believe that the way to reduce violent crime is to build larger maximum-security prisons. When a referendum for the construction of a maximum-security prison in Essex County was added to the ballot the following November, however, the proposal was voted down by a three-to-one margin. Which of the following, if true, forms the best basis for at least a partial explanation of the apparent discrepancy described above? (A) The threat of a life sentence in a maximum-security prison has been shown to be an adequate deterrent of violent crime. (B) The prison would have been constructed by a private company, and it would have been impossible for the county government to oversee the company's finances. (C) Fewer than half of the registered voters in Essex County voted. (D) Much of the substantial cost of the prison could have been offset by increasing the tolls required to cross the bridge over nearby Lake Essex. (E) Maximum-security prisons are a boon to the overall well-being of the state, but they also pose a considerable risk in the areas in which they are located. 18. (K) In January there was a large drop in the number of new houses sold, because interest rates for mortgages were falling and many consumers were waiting to see how low the rates would go. This large sales drop was accompanied by a sharp rise in the average price of new houses sold. Which of the following, if true, best explains the sharp rise in the average price of new houses? (A) Sales of higher-priced houses were unaffected by the sales drop because their purchasers have fewer constraints limiting the total amount they pay. (B) Labor agreements of builders with construction unions are not due to expire until the next January. (C) The prices of new houses have been rising slowly over the past three years because there is an increasing shortage of housing. (D) There was a greater amount of moderate-priced housing available for resale by owners during January than in the preceding three months. (E) Interest rates for home mortgages are expected to rise sharply later in the year if predictions of increased business activity in general prove to be accurate. Home Practice 6 1. During the 1980s and 1990s, the annual number of people who visited the Sordellian Mountains increased continually, and many new ski resorts were built. Over the same period, however, the number of visitors to ski resorts who were caught in avalanches decreased, even though there was no reduction in the annual number of avalanches in the Sordellian Mountains. Which of the following, if true in the Sordellian Mountains during the 1980s and 1990s, most helps to explain the decrease?

100


(A) Avalanches were most likely to happen when a large new snowfall covered an older layer of snow. (B) Avalanches destroyed at least some buildings in the Sordellian Mountains in every year. (C) People planning new ski slopes and other resort facilities used increasingly accurate information about which locations are likely to be in the path of avalanches. (D) The average length of stay for people visiting the Sordellian Mountains increased slightly. (E) Construction of new ski resorts often led to the clearing of wooded areas that had helped to prevent avalanches. 2. When there is less rainfall than normal, the water level of Australian rivers falls and the rivers flow more slowly. Because algae whose habitat is river water grow best in slow-moving water, the amount of algae per unit of water generally increases when there has been little rain. By contrast, however, following a period of extreme drought, algae levels are low even in very slow-moving river water. Which of the following, if true, does most to explain the contrast described above? (A) During periods of extreme drought, the populations of some of the species that feed on algae tend to fall. (B) The more slowly water moves, the more conducive its temperature is to the growth of algae. (C) When algae populations reach very high levels, conditions within the river can become toxic for some of the other species that normally live there. (D) Australian rivers dry up completely for short intervals in periods of extreme drought. (E) Except during periods, of extreme drought, algae, levels tend to be higher; in rivers in which the flow has been controlled by damming than in rivers that flow freely.

101


7. EXCEPT QUESTIONS These questions usually sound like: • "Any of the following, if introduced into the argument as an additional premise, makes the argument above logically correct EXCEPT" • “Which of the following, if true, would LEAST strengthen the argument above?” • “Which of the following, if true, would NOT weaken the conclusion drawn above?” • “If the argument above is true, then each of the following must also be true EXCEPT” Except questions are intent on showing an “outsider” among the other choices. You have to remember the following notions: 1. Answering such a question, you should not forget that it is an “EXCEPT”. 2. Identify the common question type for the other 4 answers and when a choice suits the type, cross it out. 3. The EXCEPT answer for the weakening question does not have to strengthen the argument, and vice versa. It can just be irrelevant to the argument as a whole. Steps to answer the question: 1. 2. 3. 4. 5. 6.

Read the Q Identify Question Type (phrases from question: except, not) Identify Except subtype (e.g. strengthen, weaken, assumption, inference, explain, flaw) Read the Argument Use the steps to answer the question of a relevant subtype Eliminate answers that fit the subtype (it is always easier to find a weaken answer than a nonweaken answer) 7. Choose answer, which is left

Eliminate answers that:  Fit the Except subtype (if you have “strengthen EXCEPT”, eliminate all strengthening answers)

102


Except questions practice

GoGMAT Problem

103


GoGMAT Problem Explanation

Instructions: chose one answer out of five given in each task. You should spend no more than 2 minutes per question. 19. (K) Sport-utility vehicles have become extremely popular because of the robust and energetic image they project. Although these vehicles look sturdy, they are subject to only a small fraction of the safety standards the government imposes on ordinary passenger cars. Consequently, a highimpact collision involving both a passenger car and a sport-utility vehicle is much more likely to injure the latter and not the former. Each of the following serves to strengthen the conclusion above EXCEPT (A) those who design vehicles are inclined to make them safe only if government rules dictate that they must (B) sport-utility vehicles have a higher center of gravity, which makes them more susceptible to turning over in a collision (C) the government rigorously enforces its standards for maximum roof strength and impact resistance in all passenger cars

104


(D) sport-utility vehicles are less aerodynamic than passenger cars, and this extra bulk hinders their ability to accelerate (E) people who drive sport-utility vehicles are often instilled with a false sense of security and therefore neglect to wear their seatbelts 20. (K) Over the past several years, Running River Water Park has experienced a serious decline in attendance and sales despite the addition of several state-of-the-art water slides. This year the Board of Directors lowered the park's weekday admission prices in order to attract more customers. Attendance during the first two months of this year's season has been 30 percent higher than the attendance during the same two months last year. Clearly, the price cut has had the desired effect. Each of the following, if true, weakens the conclusion above EXCEPT (A) Nationwide, the number of people attending amusement parks has increased by 30 percent this year over last year. (B) Grand Excursions Amusement Park, located ten miles from Running River, has been closed during most of this year's season due to unexpected equipment problems. (C) The most popular movie released this summer, The Big Chase, features a long action sequence that was filmed at Running River. (D) Several large businesses relocated near Running River during the past year bringing with them many employees and their families. (E) Most amusement park visitors are aware of the admission prices before they arrive at an amusement park. 21. (K) In the United States, injuries to passengers involved in automobile accidents are typically more severe than in Europe, where laws require a different kind of safety belt. It is clear from this that the United States needs to adopt more stringent standards for safety belt design to protect automobile passengers better. Each of the following, if true, weakens the argument above EXCEPT: (A) Europeans are more likely to wear safety belts than are people in the United States. (B) Unlike United States drivers, European drivers receive training in how best to react in the event of an accident to minimize injuries to themselves and to their passengers. (C) Cars built for the European market tend to have more sturdy construction than do cars built for the United States market. (D) Automobile passengers in the United States have a greater statistical chance of being involved in an accident than do passengers in Europe. (E) States that have recently begun requiring the European safety belt have experienced no reduction in the average severity of injuries suffered by passengers in automobile accidents.

105


WRONG ANSWERS DO THE FOLLOWING: 1) 2) 3) 4)

Contradict the evidence Describe a different group from the one given in the argument Give out-of-scope information, not related directly to the argument Use extreme words, such as “all, always, inevitably, will, must”, etc. (only when extreme language is not given in the reference) 5) Make exaggerations 6) Give irrelevant comparisons CONCEPTS FOR REVIEW: 1) ENTITY – main argument/passage element (protagonist). Can be minor or major. From the point of view of entities, an argument or a passage represents a net of relations between entities (correlation, causation, comparison, proportion, etc.) 2) EVIDENCE – factual information upon which further speculations are made. In other words, a premise that is stated explicitly in the argument/passage. 3) PREMISE – proposition upon which an argument is drawn. May or may not be stated explicitly in the argument/passage. 4) ASSUMPTION – premise that is not stated explicitly in the argument/passage. 5) CONCLUSION – position or opinion that is drawn from a premise or a number of premises, whether stated explicitly in the argument/passage or not. 6) INFERENCE, IMPLICATION, SUGGESTION – a guess that certain information is true, based on information already present. 7) EXAMPLE – factual information used to illustrate a point. 8) COUNTEREXAMPLE – factual information used to weaken a point. 9) COUNTERARGUMENT – another name for counterexample. In a way is more conceptual than a counterexample – may contain a position or an opinion besides factual information. 10) JUDGEMENT – personal opinion drawn from a premise or a number of premises. 11) TO SUPPORT, TO STRENGTHEN, TO PROVE – to help a conclusion be more reliably drawn with help of additional evidence. 12) TO UNDERMINE, TO WEAKEN, TO CAST DOUBT – to disturb or question a conclusion with help of additional evidence.

106


CRITICAL REASONING BANK (K) Set 1 1. The extent to which a society is really free can be gauged by its attitude towards artistic expression. Freedom of expression can easily be violated in even the most outwardly democratic of societies. When a government arts council withholds funding from a dance performance that its members deem "obscene," the voices of a few bureaucrats have in fact censored the work of the choreographer, thereby committing the real obscenity of repression. Which of the following, if true, would most seriously weaken the argument above? (A) (B) (C) (D) (E)

Members of government arts councils are screened to ensure that their beliefs reflect those of the majority. The term obscenity has several different definitions that should not be used interchangeably for rhetorical effect. Failing to provide financial support for a performance is not the same as actively preventing or inhibiting it. The council's decision could be reversed if the performance were altered to conform to public standards of appropriateness. The definition of obscenity is something on which most members of a society can agree.

2. The increase in the number of newspaper articles exposed as fabrications serves to bolster the contention that publishers are more interested in boosting circulation than in printing the truth. Even minor publications have staffs to check such obvious fraud. The argument above assumes that (A) (B) (C) (D) (E)

newspaper stories exposed as fabrications are a recent phenomenon everything a newspaper prints must be factually verifiable fact checking is more comprehensive for minor publications than for major ones only recently have newspapers admitted to publishing intentionally fraudulent stories the publishers of newspapers are the people who decide what to print in their newspapers

3. Our architecture schools must be doing something wrong. Almost monthly we hear of domes and walkways collapsing in public places, causing great harm to human life. In their pursuit of some dubious aesthetic, architects design buildings that sway, crumble, and even shed windows into our cities' streets. This kind of incompetence will disappear only when the curricula of our architecture schools devote less time to so-called artistic considerations and more time to the basics of good design. Which of the following, if true, would most seriously weaken the argument above? (A) All architecture students are given training in basic physics and mechanics. (B) Most of the problems with modern buildings stem from poor construction rather than poor design. (C) Less than 50 percent of the curriculum at most architecture schools is devoted to aesthetics. (D) Most buildings manage to stay in place well past their projected life expectancies. (E) Architects study as long and as intensively as most other professionals.

107


4. This editorial cannot be a good argument because it is barely, literate. Run-on sentences, slang, and perfectly dreadful grammar appear regularly throughout. Anything that poorly written cannot be making very much sense. Which of the following identifies an assumption in the argument above? (A) (B) (C) (D) (E)

This editorial was written by someone other than the usual editor. Generally speaking, very few editorials are poor in style or grammar. The language of an argument is indicative of its validity. Generally speaking, the majority of editorials are poor in style and grammar. The author of the editorial purposely uses poor grammar to disguise what he knows is a bad argument.

5. Attempts to blame the mayor's policies for the growing inequality of wages are misguided. The sharp growth in the gap in earnings between college and high school graduates in this city during the past decade resulted from overall technological trends that favored the skills of more educated workers. Nor can the mayor's response to this problem be criticized, for it would hardly be reasonable to expect him to attempt to slow the forces of technology. Which of the following, if true, casts the most serious doubt on the conclusion drawn in the last sentence above? (A) The mayor could have initiated policies that would have made it easier for less educated workers to receive the education necessary for better-paying jobs. (B) Rather than cutting the education budget, the mayor could have increased the amount of staff and funding devoted to locating employment for graduating high school seniors. (C) The mayor could have attempted to generate more demand for products from industries that paid high blue-collar wages. (D) Instead of reducing the tax rate on the wealthiest earners, the mayor could have ensured that they shouldered a greater share of the total tax burden. (E) The mayor could have attempted to protect the earnings of city workers by instigating policies designed to reduce competition from foreign industries. 6. A team of efficiency consultants conducted a study of Company X and found that 85 percent of its employees suffered a "midafternoon slump" between the hours of 2:00 p.m. and 4:00 p.m. During this slump, each employee's productivity went down an average of 30 percent. The consultants recommended, therefore, that management institute a policy encouraging employees to take their lunch breaks sometime between the hours of 2:00 p.m. and 4:00 p.m., since employees do not need to be productive as they eat lunch. The consultants' conclusion relies on which of the following assumptions? (A) The consultants found no correlation between consumption of food and the feelings of lethargy experienced by the employees of Company X during the midafternoon slump. (B) Some of the employees of Company X do not eat breakfast until they arrive at the office at 9:00 a.m. (C) The consultants had seen the same slump phenomenon at Company R and had made the same recommendation to change the lunch hour. (D) Most of the employees of Company X expressed a preference to eat lunch sometime between

108


(E)

the hours of 1:00 p.m. and 3:00 p.m. The consultants also suggested adjusting the work schedules of half of the employees of. Company X so that they would come in early in the morning and leave by 2:00 p.m.

7. Ms. S, an adoptive mother, recently read an article about a birth father who is suing the adoptive parents for custody of his child, who had been offered up for adoption without the birth father's permission. Ms. S adopted her child several years later through the same agency, but she should not be worried that her child's birth father will sue her for custody. Which of the following, if discovered to be true, would lend the most support to the conclusion above? (A) The birth mother of Ms. S's child never informed the birth father that she was pregnant, so the birth father does not know of the child's existence. (B) In the last twenty years in the state in which Ms. S lives, adoptive parents have won 90 percent of the cases in which birth parents have sued for custody of an adopted child. (C) The adoption in the article was completed two months before a policy was instituted by the agency requiring the signatures of both birth parents on the paperwork releasing the child for adoption. (D) The birth father in the case described in the article was over the age of eighteen at the time the adoption was completed. (E) Ms. S adopted one child through the agency mentioned in the article and another child through a lawyer specializing in adoptions. 8. Staff members at the Willard Detention Center typically oversee students' schedules and make all final decisions regarding the required activities in which students participate. Students are permitted, however, to make their own decisions regarding how they spend their free time. Therefore, students should be permitted to make their own decisions regarding the elective courses that they wish to take. The conclusion above would be more reasonably drawn if which of the following were inserted into the argument as an additional premise? (A) Decisions regarding required activities are more important than decisions regarding the elective courses that students take. (B) Students are more willing to take elective courses than to participate in required Center activities. (C) Required activities contribute more to the students' rehabilitation than do their free-time activities. (D) Staff members at Willard have found that elective courses are more beneficial for students than the available free-time activities. (E) When compared for decision-making purposes, elective courses are more like free-time activities than required activities. 9. The average math score on a state-wide proficiency exam for students attending Middlebury High School last year was 20 points higher than the average math score for students attending nearby Ellingsford High School. Therefore, any student at Ellingsford High School wishing to achieve a better math score on next September's proficiency exam should transfer to Middlebury High School over the summer.

109


Which of the following statements, if true, would most significantly strengthen the conclusion drawn in the passage? (A) Middlebury High School offers its students a unique, week-long course just before they take the proficiency exam that has consistently proven effective in raising student scores. (B) One third of all the students who have transferred to Middlebury High School the summer before taking the test got scores that are at least 20 points higher than the average score at Ellingsford High School. (C) Middlebury High School students who transfer to Ellingsford High School in the summer before they take the proficiency exam get average scores that are comparable to the average scores of students who remain at Middlebury. (D) In the past five years, the average score at Ellingsford High School has been rising at a faster rate than has the average score at Middlebury High School. (E) Students wanting better proficiency-exam scores are transferring to Middlebury High School at a high rate, which will ultimately result in a lowering of the school's average score. 10. Barnes: The two newest employees at this company have salaries that are too high for the simple tasks normally assigned to new employees and duties that are too complex for inexperienced workers. Hence, the salaries and the complexity of the duties of these two newest employees should be reduced. Which one of the following is an assumption on which Barnes's argument depends? (A) The duties of the two newest employees are not less complex than any others in the company. (B) It is because of the complex duties assigned that the two newest employees are being paid more than is usually paid to newly hired employees. (C) The two newest employees are not experienced at their occupations. (D) Barnes was not hired at a higher-than-average starting salary. (E) The salaries of the two newest employees are no higher than the salaries that other companies pay for workers with a similar level of experience. (K)Set 2

1. Ordinary mountain sickness, a common condition among mountain climbers, and one from which most people can recover, is caused by the characteristic shortage of oxygen in the atmosphere at high altitudes. Cerebral edema, a rarer disruption of blood circulation in the brain that quickly becomes life-threatening if not correctly treated from its onset, can also be caused by a shortage of oxygen. Since the symptoms of cerebral edema resemble those of ordinary mountain sickness, cerebral edema is especially dangerous at high altitudes. Which one of the following is an assumption on which the argument depends? (A) The treatment for ordinary mountain sickness differs from the treatment for cerebral edema. (B) Cerebral edema can cause those who suffer from it to slip into a coma within a few hours. (C) Unlike cerebral edema, ordinary mountain sickness involves no disruption of blood circulation in the brain. (D) Shortage of oxygen at extremely high altitudes is likely to affect thinking processes and cause errors of judgment. (E) Most people who suffer from ordinary mountain sickness recover without any special

110


treatment. 2. Smith Products fabricates machine tools that are essentially identical to those produced by Jackson Manufacturing. For both companies raw materials represent about two-thirds of the cost of manufacturing the machine tools. To gain an edge over Jackson Manufacturing, Smith Products should purchase its raw materials from a new supplier advertising much lower prices. Which of the following, if true, would most weaken the argument above? (A) (B) (C) (D) (E)

Smith Products spends more on employee wages than Jackson Manufacturing does. Smith's current supplier provides raw materials of exceedingly high quality. The market for machine tools has been declining for several years. The new supplier's materials are of low quality and would reduce the lifespan of Smith machine tools by half, causing sales to decline. The plant manager for Smith Products is planning to increase the plant's efficiency.

3. The price of a full-fare coach ticket from Toronto to Dallas on Breezeway Airlines is the same today as it was a year ago, if inflation is taken into account by calculating prices in constant dollars. However, roughly 90 percent of the Toronto-to-Dallas coach tickets that Breezeway sells are discount tickets and only 10 percent are full-fare tickets, whereas a year ago half were discount tickets and half were full-fare tickets. Therefore, on average, people pay less today in constant dollars for a Breezeway Toronto-to-Dallas coach ticket than they did a year ago. Which one of the following, if assumed, would allow the conclusion above to be properly drawn? (A) A Toronto-to-Dallas full-fare coach ticket on Breezeway Airlines provides ticket-holders with a lower level of service today than such a ticket provided a year ago. (B) A Toronto-to-Dallas discount coach ticket on Breezeway Airlines costs about the same amount in constant dollars today as it did a year ago. (C) All full-fare coach tickets on Breezeway Airlines cost the same in constant dollars as they did a year ago. (D) The average number of coach passengers per flight that Breezeway Airlines carries from Toronto to Dallas today is higher than the average number per flight a year ago. (E) The criteria that Breezeway Airlines uses for permitting passengers to buy discount coach tickets on the Toronto-to-Dallas route are different today than they were a year ago. Questions 4-5 are based on the following: In 2001, a local high school implemented a new program designed to reduce the incidence of teenage pregnancy. The program, however, failed to produce the desired result. If the program had been successful, the dropout rate for female students would not have increased substantially in 2001. 4.

The argument in the passage depends on which of the following assumptions?

(A) The number of teen pregnancies nationwide increased in 2001. (B) The number of teen pregnancies in 2001 was greater than the number of teen pregnancies in 1991. (C) Teenage pregnancy is a leading reason that female students leave school. (D) The program was mandatory for all female students. (E) Most 2001 female dropouts were not pregnant at any time during the year.

111


5.

The argument in the passage would be most seriously weakened if it were true that

(A) the number of female students in the school has remained constant for the past decade nationwide, (B) the teen pregnancy rate and the female high school dropout rate both increased significantly in 2001 (C) some female students who dropped out were pregnant at the time (D) many female high school dropouts said that poor economic conditions forced them to leave school and find jobs (E) the school also implemented a program designed to reduce the incidence of drug use among teenagers in 2001 6. According to a recent study, a diet that is free of meat and dairy products greatly reduces the risk of suffering a heart attack. The study cites the fact that only 10 percent of those who consume such a diet suffer a heart attack at some point in their lives. Which of the following would most seriously weaken the argument above? (A) Diets free of meat and dairy are low in calcium, which can lead to bone density decreases. (B) Those who consume only dairy but not meat are twice as likely to suffer a heart attack as those who consume neither meat nor dairy. (C) Some people who consume neither dairy nor meat suffer two or more heart attacks over the course of a lifetime. (D) Meat and dairy products are high in low-density cholesterol, which is known to harden arteries and cause other heart problems. (E) Seven percent of those who consume dairy and meat regularly suffer heart attacks over the course of their lifetime. 7. If participation in the honors creative writing class were limited to graduate students and those undergraduates who had received at least a B+ in composition, most of the undergraduate students would be forced to take the regular creative writing class. Such a reduction in undergraduate enrollment would reduce the percentage of failing grades in the honors class. Which of the following, if true, would most strengthen the conclusion drawn in the second sentence above? (A) Graduate students have all scored at least B+ in composition. (B) The honors creative writing course is experiencing overcrowding due to increases in graduate enrollment. (C) Many undergraduates would work harder to score B+ in composition rather than be excluded from honors creative writing. (D) The number of failing grades in honors creative writing has decreased in recent years. (E) Undergraduates who scored lower than B+ in composition are responsible for a disproportionate percentage of failing grades in honors creative writing. 8. A researcher has discovered that steel containing Element X is stronger and more flexible than ordinary steel because Element X reduces the occurrence of microscopic fractures. The level of

112


Element X in much of the steel produced in Canada is naturally high because the ore deposits from which the steel is produced also contain Element X. Which of the following can be correctly inferred from the statements above? (A) Steel from Canada is stronger and more flexible than steel from any other country. (B) Steel that is not from Canada is highly likely to develop microscopic fractures after years of use. (C) Producing steel from ore deposits containing Element X is the best way to make steel that is stronger and more flexible. (D) Some steel produced in Canada is less likely to develop microscopic fractures than other steel. (E) Steel produced from Canadian ore deposits contains the highest levels of Element X found in any steel. 9. (PR) A series of glitches within the satellite infrastructure of a cellular phone service company has resulted in service interruptions and several complaints from the company's clients. The company has responded by offering its clients free months of service and other rebates in order to keep its clients from changing to another service provider. Because of this new policy, the company's profits are destined to keep falling for years to come. Which of the following, if true, taken together with the information above, best supports the conclusion that the company's financial situation will only worsen as long as this new policy is in place? (A) Clients who experience technical difficulties with their cellular phones are unlikely to recommend the service to friends and business associates. (B) Since satellite technology and construction is still a relatively new industry, it's unwise to assume that every satellite will always work perfectly. (C) The money that the company passes on as rebates to its clients whose service has been interrupted had been previously budgeted to be spent on repairing the satellites. (D) In order to balance out the lost and forfeited revenue, the company will have to lay off at least 10 percent of its employees. (E) The company has no plans to launch any new satellites anytime soon.

10. Privatization of the large state enterprises that comprise the industrial sector in Russia is proceeding slowly, due to competing claims of ownership by various groups. Continued government subsidization of these enterprises creates large deficits that drive up the inflation rate and cause the rubles value to decline. It is therefore unlikely that the government will make the ruble freely convertible to Western currencies until the question of ownership of state enterprises has been resolved. If the author's prediction concerning the ruble is accurate, which of the following conclusions can most reliably be drawn? (A) The industrial sector accounts for at least fifty percent of Russia's economic activity. (B) Making the Russian ruble freely convertible to Western currencies will cause the ruble's value to decline. (C) The Russian government can indefinitely withstand the expense of subsidiary state enterprises. (D) The Russian government is among the groups claiming ownership of certain state enterprises.

113


(E)

The Russian government is under pressure from the West to make the ruble freely convertible to Western currencies.

(K) Set 3 1. An ecology magazine regularly publishes articles on tree diseases. This year, the number of articles on Ophiostoma ulmi, the fungus that causes Dutch elm disease, is significantly smaller than the number of such articles that appeared last year. Clearly, fewer researchers studied Ophiostoma ulmi this year than did so last year. Which of the following, if true, weakens the above conclusion? (A) Many researchers publishing articles are currently studying Stegophora ulmea, a fungus that causes elm leaf spot. (B) Since its introduction, Dutch elm disease has killed half of the elm trees in North America. (C) Research on Dutch elm disease that focuses on prevention receives more funding than research that focuses on finding a cure. (D) A new strain of the fungus Rhytisma acerinum infested maple trees at an unprecedented rate this year. (E) All articles go through at least a one-year review process before publication. 2. Critics of strict "promotional gates" at the grade school level point to a recent study comparing students forced to repeat a grade with those promoted despite failing scores on an unscheduled, experimental competency test. Since there was no significant difference between the two groups' scores on a second test administered after completion of the next higher grade level, these critics argue that the retention policy has failed in its expressed purpose of improving students' basic skills. Which of the following best expresses the argument made by critics of promotional gates? (A) Anxiety over performance on standardized tests often hinders a student's ability to master challenging new material. (B) A student's true intellectual development cannot be gauged by his score on a standardized competency test. (C) The psychological damage a child suffers by repeating a grade outweighs the potential intellectual benefits of a second chance at learning. (D) Strict requirements for promotion do not lead to harder work and greater mastery of fundamentals among students fearful of being held back. (E) Socioeconomic factors as well as test scores influenced whether a given student in the study was promoted or forced to repeat a grade. Questions 3-4 are based on the following: In a certain state, the rate at which inhabitants of city X contract a certain disease is significantly lower than the rate at which inhabitants of city Y contract the disease. So if a couple originally from city Y relocates to city X and raises a family there, their children will be significantly less likely to contract this disease than they would had they remained in city Y. 3. Which of the following, if true, would most seriously weaken the conclusion drawn in the passage?

114


(A) Many health experts do not believe that moving to city X will lead to a significant increase in the average person’s immunity to the disease. (B) The mayor of city Y has falsely claimed that statistics relating to the incidence of the disease in his city are not accurate. (C) The lower incidence of the disease in city X can be ascribed mostly to genetically determined factors. (D) Some inhabitants of city Y possess a greater immunity to the disease than do the healthiest inhabitants of city X. (E) Smog levels in city X are significantly lower than those of any other city in the state.

4. Which of the following statements, if true, would most significantly strengthen the conclusion drawn in the passage? (A) The rate at which inhabitants of city X contract the disease will increase as the overall population of city X increases. (B) In contrast to city Y, city X is characterized by an abundance of environmental factors that tend to inhibit the occurrence of the disease. (C) Twenty-three percent of those inhabitants of city Y who move to city X live beyond the average life span of native inhabitants of city X. (D) Over the last two decades, the incidence of the disease has decreased in city Y but remained unchanged in city X. (E) Studies indicate that the incidence of the disease among inhabitants of city X who move to city Y is roughly equal to that of inhabitants of city X who remain in city X. 5. In the state of Michigan, from 1980 to 1989, total spending on books purchased from all sources increased by 34 percent. But during the same period, spending on fiction books, most of which were purchased from bookstores selling only new books, grew just 16 percent. Which of the following statements about the period from 1980 to 1989 is best supported by the statements above? (A) Spending on nonfiction books increased by more than 34 percent. (B) Shoppers were more likely to buy fiction books when they went to a bookstore than they were to buy nonfiction. (C) The prices of books purchased at bookstores are higher than those of books purchased elsewhere. (D) Individual spending on books increased, while institutional spending declined. (E) The number of people who bought books from secondhand bookstores increased during this period. 6. Over the last century, paleontologists have used small differences between fossil specimens to classify triceratops into sixteen species. This classification is unjustified, however, since the specimens used to distinguish eleven of the species come from animals that lived in the same area at the same time. Which of the following, if true, would enable the conclusion of the argument to be properly drawn? (A) Not every species that lived in a given area is preserved as a fossil. (B) At least one individual of every true species of triceratops has been discovered as a fossil

115


specimen. (C) No geographical area ever supports more than three similar species at the same time. (D) In many species, individuals display quite marked variation. (E) Differences between fossil specimens of triceratops that came from the same area are no less distinctive than differences between specimens that came from different areas. 7. Naturally occurring chemicals cannot be newly patented once their structures have been published. Before a naturally occurring chemical compound can be used as a drug, however, it must be put through the same rigorous testing program as any synthetic compound, culminating in a published report detailing the chemical’s structure and observed effects. If the statements above are true, which of the following must also be true on the basis of them? (A) Any naturally occurring chemical can be reproduced synthetically once its structure is known. (B) Synthetically produced chemical compounds cannot be patented unless their chemical structures are made public. (C) If proven no less effective, naturally occurring chemicals are to be preferred to synthetic compounds for use in drugs. (D) Once a naturally occurring compound has been approved for use as a drug, it can no longer be newly patented. (E) A naturally occurring chemical cannot be patented unless its effectiveness as a drug has been rigorously established. 8. A public-service advertisement advises that people who have consumed alcohol should not drive until they can do so safely. In a hospital study, however, subjects questioned immediately after they consumed alcohol underestimated the time necessary to regain their driving ability. This result indicates that many people who drink before driving will have difficulty following the advertisement’s advice. Which of the following, if true, most strongly supports the argument above? (A) Many people, if they plan to drink alcohol, make arrangements beforehand for a nondrinker to drive them home. (B) The subjects in the hospital study generally rated their abilities more conservatively than would people drinking alcohol outside a hospital setting. (C) Some people refrain from drinking if they will have to drive to get home afterward. (D) The subjects in the hospital study were also questioned about the time necessary to regain abilities that do not play an important role in driving safely. (E) Awareness of the public-service advertisement is higher among the general population than it was among the subjects in the hospital study. 9. These days, drug companies and health professionals alike are focusing their attention on cholesterol in the blood. The more cholesterol we have in our blood, the higher the risk that we shall die of a heart attack. The issue is pertinent since heart disease kills more North Americans every year than any other single cause. At least three factors—smoking, drinking, and exercise— can each influence levels of cholesterol in the blood. Which one of the following can be properly concluded from the passage? (A) If a person has low blood cholesterol, then that person's risk of fatal heart disease is low.

116


(B) Smoking in moderation can entail as great a risk of fatal heart disease as does heavy smoking. (C) A high-cholesterol diet is the principal cause of death in North America. (D) The only way that smoking increases one's risk of fatal heart disease is by influencing the levels of cholesterol in the blood. (E) The risk of fatal heart disease can be altered by certain changes in lifestyle. 10. Political opinion and analysis outside the mainstream rarely are found on television talk shows, and it might be thought that this state of affairs is a product of the political agenda of the television stations themselves. In fact, television nations are driven by the same economic forces as sellers of more tangible goods. Because they must attempt to capture the largest possible share of the television audience for their shows, they air only those shows that will appeal to large numbers of people. As a result, political opinions and analyses aired on television talk shows are typically bland and innocuous. An assumption made in the explanation offered by the author of the passage is that (A) most television viewers cannot agree on which elements of a particular opinion or analysis are most disturbing (B) there are television viewers who might refuse to watch television talk shows that they knew would be controversial and disturbing (C) each television viewer holds some opinion that is outside the political mainstream, but those opinions are not the same for everyone (D) there are television shows on which economic forces have an even greater impact than they do on television talk shows (E) the television talk shows of different stations resemble one another in most respects (K) Set 4

1. The education offered by junior colleges just after the Second World War had a tremendous practical effect on family-run businesses throughout the country. After learning new methods of marketing, finance, and accounting, the sons and daughters of merchants returned home, often to increase significantly the size of the family's enterprise or to maximize profits in other ways. Which of the following statements is best supported by the information above? (A) The junior colleges principally emphasized methods of increasing the size of small businesses. (B) The business methods taught in the junior colleges were already widespread before the second World War (C) The business curricula at junior colleges did not include theoretical principles of management. (D) Without the influence of junior colleges, many family-run businesses would have been abandoned as unprofitable. (E) Business methods in many postwar family-run businesses changed significantly as a result of the junior colleges. 2. Time and time again it has been shown that students who attend colleges with low faculty/student ratios get the most well-rounded education. As a result, when my children are ready for college, I'll be sure they attend a school with a very small student population. Which of the following, if true, identifies the greatest flaw in the reasoning above?

117


(A) (B) (C) (D) (E)

A low faculty/student ratio is the effect of a well-rounded education, not its source. Intelligence should be considered the result of childhood environment, not advanced education. A very small student population does not, by itself, ensure a low faculty/student ratio. Parental desires and preferences rarely determine a child's choice of a college or university. Students must take advantage of the low faculty/student ratio by intentionally choosing small classes.

11. National regulations that limit the sale of meat to within 5 days of packing should be changed. Under optimal conditions, meat kept at 40 degrees Fahrenheit will not spoil for 16 days. If the regulations were changed, prices for meat would drop due to increased shelf life and reduced waste, but the safety of the food supply would not be compromised. Which of the following, if true, would most seriously weaken the conclusion drawn above? (A) Most consumers keep meat for up to a week before eating it. (B) 7 of 10 shopkeepers favor extending the limitation on meat to 9 days. (C) Approximately 65 percent of the meat display cases nationally maintain temperatures between 47 and 54째 F. (D) Approximately half the meat stored for 25 days is still safe to consume. (E) Meat packing operations are more efficient when they can make fewer, larger deliveries than when they must make more frequent, smaller deliveries. 12. If a poor harvest season in a major corn-producing state results in higher prices for a bushel of corn, corn prices in other states will rise as well, whether or not those states are net importers of corn. Which of the following conclusions is best supported by the statement above? (A) Agricultural commodities companies in states that are not net importers of corn are excluded from the national corn market when there is a disruption in the national corn supply. (B) National corn supply disruptions have little, if any, effect on the price of local corn as long as the locality is in a state that is not a net importer of corn. (C) The corn market in any state is part of the national corn market even if most of the corn consumed in the state is produced in the state. (D) Poor harvesting seasons come at predictable regular intervals. (E) Higher prices for corn tend to lead to increased prices for livestock, which rely on corn feed 13. A chemical company recently introduced a new type of foam spray that it claims will reduce the rate of erosion from the walls of road cuts. A study by the company showed that the rate of erosion was low on a road cut where the foam was applied. Which of the following, if true, would most seriously weaken the company's conclusion? (A) Road cuts similar to the one studied typically show low rates of erosion without the foam. (B) Because the foam itself weathers, the foam would have to be reapplied every four years to order to maintain protection against erosion. (C) Studies by the company that produces the material are sometimes unreliable because of conflicts of interest. (D) The rate of erosion from the road cut in the study was greater than expected based on computer simulation models.

118


(E)

Other foams made from similar materials have failed to halt erosion from certain types of road cuts

14. General: The commander of the Air Force has recommended that we deploy the G28 aircraft in the reconnaissance mission, because the G28 can fly lower to the ground without being detected and could therefore retrieve the necessary information more efficiently than the currently stationed D12. But the D12 is already in the area and poised for takeoff, and would have just enough time to accomplish the mission if deployed immediately, while the G28 would require four days just to arrive in the area and get outfitted for the mission. Since the mission's deadline is immovable, I am forced to overrule the commander's recommendation and order the deployment of the D12. Which of the following is assumed in the general's argument? (A) The quality of information retrieved from the mission would be higher if the D12 were deployed than if the G28 were deployed. (B) By the time the G28 arrived in the area and was outfitted for the mission, the D12 would have already completed the mission if deployed immediately. (C) The ability of an aircraft to fly low to the ground is not a significant consideration when choosing aircraft for a reconnaissance mission. (D) It would take longer for any aircraft not currently in the area besides the G28 to arrive in the area and get outfitted for the mission. (E) Any time saved during the mission due to the operation of the more efficient G28 would not offset the additional time required to deploy the G28. 15. (PR) The Sports and Exhibition Authority has announced that the next election will feature a statewide referendum for the allotment of public funds to build a new football stadium in the state's capital city. Although the current facility is small and outdated, voters should reject this proposal because it stands to benefit only those who live in or near the capital at the expense of all of the state's taxpayers. Which of the following, if true, could supporters of the new stadium cite in response to the author's criticism? (A) A new facility will attract new retail businesses whose taxes will be used to fund statewide individual tax cuts. (B) The owner of the state's football franchise has threatened to move his team to another state across the country if a new stadium is not built. (C) During football season, the state's team plays only eight home games. (D) In each of the past five seasons, the football team has improved its record and increased its appeal among the state's football fans. (E) Five years ago, a neighboring state built a brand-new football stadium and successfully convinced an out-of-state team to relocate there. 16. Countries A and Đ’ are in competition to draw tourists to their countries. In Country A, about 2,500 violent crimes are reported per year. In Country B, only about 1,000 violent crimes are reported per year. Trying to draw tourists away from Country A, officials in Country Đ’ use these violent crime statistics to claim it has a lower violent crime rate than Country A. Which of the following, if true, would expose the flaw in Country B's argument that it has the lower violent crime rate? (A) Most violent criminals in Country Đ’ are repeat offenders

119


(B) (C) (D) (E)

White-collar crime is higher in Country Đ’ than in Country A. The population of Country A is twenty times greater than the population of Country B. Country Đ’ has fewer tourists than Country A. Country A has a better prison system than Country B.

17. In order to help the most famine-stricken areas of the world genetic engineers have proposed injecting certain animals with the growth hormone lipazine, which increases the animals' ratio of meat to body fat. Those who oppose this plan are concerned that the hormone could cause health problems in the humans who eat the meat. A physician has defended the proposal, however, by asserting that humans can ingest as much as fifteen milligrams of lipazine daily, and no animals would ever receive a dosage higher than ten milligrams. Which of the following statements, if true, would be of the most use to the plan's critics in response to the physician's claim? (A) Lipazine occurs naturally in many animals that are farmed for human consumption. (B) Each package of meat that had been treated with lipazine would bear a label warning consumers of that fact. (C) Lipazine is widely used among weight lifters who desire to build muscle mass in a brief period of time. (D) Some religions that are prevalent in famine-stricken countries have deemed that consumption of meat is sinful. (E) Lipazine has been shown to cause sterility in certain animals.

18. Psychologists who wish to have one of their book reviews nominated for the prestigious Boatwright Psychology Review award should not submit book review articles that review more than three books at a time. This is because editors for the Boatwright Psychology Review will not publish a book review article if it is too lengthy and cumbersome to read. In their submission guidelines, the editors explicitly state that review articles that cover more than three books at a time are considered too lengthy and cumbersome to read. Which of the following statements represents an assumption upon which the argument above depends? (A) The books review article that covers the most books must be the lengthiest and most cumbersome article to read. (B) If a book review article is published in the Boatwright Psychology Review, that article will receive the prestigious Boatwright Psychology Review award. (C) All articles published in the Boatwright Psychology Review must be limited to a certain length specified by the editors. (D) The Boatwright Psychology Review editors generally prefer book review articles that cover one book rather than two books. (E) To be nominated for the Boatwright Psychology Review award, a psychologist's book review article must be published in the Boatwright Psychology Review.

120


(K) Set 5 1. Statistics show that more than half of the nation's murder victims knew their assailants; in fact, 24 percent last year were killed by relatives. Nor was death always completely unexpected. In one study, about half the murder victims in a particular city had called for police protection at least five times during the 24 months before they were murdered. Nonetheless, most people are more likely to fear being killed by a stranger in an unfamiliar situation than by a friend or relative at home. Which of the following, if true, best explains the reaction of most people to the likelihood of being murdered? (A) Statistics are likely to be discounted no matter what the source, if their implication seems to run counter to common sense. (B) In the face of such upsetting problems as murder and assault, most people are more likely to react emotionally than rationally. (C) A study taken in only one city is not likely to have an effect on attitudes until similar studies have been undertaken at the national level and yielded similar results. (D) Most people do not consider themselves to be in the high-risk groups in which murder occurs frequently between relations, but do see themselves as at least minimally susceptible to random violence. (E) People who seek police protection from relatives and friends are often unwilling to press charges when the emotions of the moment have cooled. 2. A brochure for City X highlights the reasons why residents should move there rather than to other cities in the state. One reason that the brochure mentions is the relative ease of finding a job in City X, where the unemployment rate is 4.7 percent. Which of the following statements, if true, casts the most doubt on the validity of the reason to move to City X mentioned above? (A) Most of the jobs in City X are hourly rather than salary jobs. (B) The state where City X is located has an average unemployment rate of 3.9 percent. (C) Other reasons to move to City X include the school system and easy access to recreational activities. (D) The national unemployment rate, calculated during the last census, is 4.3 percent. (E) City Y, located in the same state as City X, recently built a new factory that will employ 5,000 workers. 3. (PR) Monique: Whenever I get depressed about the direction in which my life is going, I seem to receive another bit of horrible news. It's almost as if someone up there waits until I am at my most emotionally vulnerable and decides to make things worse. The flaw in Monique's reasoning is the possibility that (A) the phrase "emotionally vulnerable" is difficult to quantify. (B) Monique would feel better if she braced herself for bad news whenever she felt depressed. (C) people deal with bad news in many different ways because some people are more stable than others. (D) when Monique is emotionally vulnerable, the bad news she hears seems much more horrible than it actually is.

121


(E)

her feelings of depression might be treatable if she sought professional counseling.

4. The Laysan Rail, an insectivore bird once present on several of the Hawaiian Islands, can no longer be found and is thought to be extinct. Scientists originally thought that a decrease in the amount of ground vegetation available for nesting was responsible for the decline of the bird. However, they now believe that increased competition for food was ultimately responsible for the Laysan Rail's inability to survive. Which of the following would best help to account for the change in the accepted explanation for the Laysan Rail's extinction? (A) The vegetation on the Laysan Rail's home island was decimated when rabbits were introduced to the island in the 1910s. (B) When attempts were made to relocate the Laysan Rail to other islands, the birds lost the geographical cues that they relied on for finding mating sites. (C) The Laysan Rail builds nests under dense ground cover to protect the eggs. (D) An increase in the use of pesticides resulted in a decrease in the numbers of flies and moths present in the Laysan Rail's territory. (E) Many species nested in the same types of vegetation as the Laysan Rail. 5. The state legislature has proposed a new law that should provide a tax credit to people who install alarm systems in their homes. Members of the legislature claim that the new law will reduce crime, computer studies showing that crime rates fall as the percentage of homes with alarm systems rises. Which of the following, if true, would cast the most doubt on the claim that the new law will reduce crime? (A) No law can prevent crime altogether. (B) The amount of the tax credit is so low relative to the cost of alarm systems that very few people will install alarm systems in order to obtain this credit (C) Neighborhood crime prevention programs can reduce crime as effectively as alarm systems can. (D) The state would have to build more prisons to house all the people caught by the new alarm systems. (E) The state cannot afford to reduce taxes any further. 6. The occurrence of schizophrenia in the general population is 1 percent. If one parent is schizophrenic, however, the incidence rises to 12 percent, and a child with two schizophrenic parents has a 45 percent chance of also suffering from the disease. Which of the following can be most reasonably inferred from the passage above? (A) One's risk of developing schizophrenia is greater if one has a schizophrenic grandparent than if one has a grandparent with no diagnosed mental disease. (B) Early diagnosis of schizophrenia may reduce the severity of the impact of the disease on the patient's life. (C) One's risk of developing schizophrenia is higher if one has a full sibling with the disease. (D) Over the past forty years, psychiatrists have advanced significantly in their understanding of the causes and treatments of schizophrenia.

122


(E)

A person's risk of developing schizophrenia is at least partially determined by genetic factors.

7. Due to a string of dismal performances, a touring band has begun to lose its audience. News of the disappointing concerts has traveled quickly via cable stations and the Internet, and has negatively influenced ticket sales for future performances. Due to the poor ticket sales, a number of promoters have canceled the band's upcoming shows, forcing the band to attempt to recoup its touring and recording expenses from fewer total performances. Which of the following, if true, taken together with the information above, best supports the prediction that more of the band's shows will be canceled? (A) The promoters who canceled shows did so with the promise that they would monitor the band's reception in other cities before deciding whether or not to reschedule the canceled shows. (B) The pressure to restore its diminishing fan base and recoup its overall expenses from a decreased number of performing opportunities is likely to cause the band to perform poorly in future concerts. (C) Because of the canceled shows, it will be impossible for the band to earn a profit on the current tour. (D) If the band cannot salvage the tour, its next CD will likely fail economically unless the band can restore its image through music videos. (E) It is impossible for the management of a rock band to predict accurately the success of a tour because fans of rock bands are notoriously fickle in their tastes. 8. Investigator: XYZ Coins has misled its clients by promoting some coins as “extremely rare” when in fact those coins are relatively common and readily available. XYZ agent: That is ridiculous. XYZ Coins is one of the largest coin dealers in the world. We authenticate the coins we sell through a nationally recognized firm and operate a licensed coin dealership. The XYZ agent’s reply is most vulnerable to the criticism that it (A) (B) (C) (D) (E)

exaggerates the investigator’s a claims in order to make them appear absurd accuses the investigator of bias but presents no evidence to support that accusation fails to establish that other coin dealers do not also authenticate the coins those dealers sell lists strengths of XYZ Coins while failing to address the investigator’s charge provides no definition for the inherently vague phrase “extremely rare”

9. University systems that use graduation rates to determine which campuses are allotted additional funds are acting counter to their stated goals. The universities say they are trying to raise academic standards, yet they are actually encouraging campuses to graduate students regardless of achievement. Which of the following statements, if true, would help to validate the approach taken by the university systems mentioned above? (A) Graduation rates for university systems with this policy are among the highest in the nation, but the graduates from these systems score poorly on tests of basic skills. (B) The campuses that need the additional funds the most are the ones that have the lowest graduation rates.

123


(C)

The new funds will be allotted for facility upgrades, not new faculty positions or staff pay increases. (D) Graduation examinations currently exist that require every graduate at each university to demonstrate a minimum level of achievement. (E) An opposing plan focuses on providing extra funding based not on graduation rates, but on the percent of students that pass basic skills tests. 10. Many adults, no matter what their age, respond to adversity by seeking advice only from their parents. Consciously or not, they regress to a psychological state of childhood dependence in which the parent is seen as the only source of wisdom and comfort. Adults who do not regress to this childhood mode turn for advice in adversity only to other loved ones—a spouse or a best friend— whom they perceive and relate to as peers. If all of the above statements are true, which of the following must also be true? (A) (B) (C) (D) (E)

One's parents offer more wisdom in adversity than those whom one perceives as peers. Adults who do not suffer adversity look only to their parents for advice. No adults seek advice in adversity from total strangers. Adults who seek advice in adversity from their parents do not expect to receive wisdom and comfort. Adults who regress to a state of childhood dependence lose touch with their peers.

(PR) Set 6 1. Citing a year-long downward trend in global sales of personal computers, the CEO of Farmer Computer has announced that his company will shift much of its emphasis from PC production to corporate networking systems. Personal computers are now much more able to adapt to new networking software, but the software itself must be updated frequently. Which of the following, if true, casts the most serious doubt onto this new strategy? (A) The Farmer brand name is well respected in the computer industry, but few industry analysts believe that Farmer will have the same success with net-working. (B) Sales of Farmer PCs have increased dramatically over each of the past six quarters, and they show no signs of slowing down. (C) A slew of new competitors in the PC production business will inevitably result in widespread cost cutting to preserve market share. (D) Sales of cellular phones and other mobile communications are expanding at three times the rate of networking companies. (E) Two other companies have approached Farmer with the prospect of a three-way merger to form a dominant company that would own market share in the PC business.

2. At many universities in the United States, the average price of a used computer has risen dramatically. This rise has come about mostly because of increased demand from students entering college who need a computer but cannot afford to purchase a new one. In order to take advantage of this market, college seniors have been selling the computers that they have used throughout their college careers to incoming freshmen. This trend is sure to exert an upward pressure on the price of new computers as well.

124


To support a conclusion that the average price of a new computer will rise, it would be most important to establish which of the following? (A) The proliferation of e-mail and the Internet has made buying and selling used merchandise much easier. (B) Since most students need a computer to perform only basic duties, used computers are usually just as useful as new ones. (C) Most students who sell their used computers are inclined to replace them with new computers. (D) College seniors are more likely to wait until they secure employment before making any expensive purchases. (E) The majority of students who purchase new computers before entering college purchase another one before they graduate. 3. In 1994, the most common eye-related disease from which Americans suffered was conjunctivitis, and glaucoma was a distant second. Glaucoma is much more common among patients who are more than fifty years old than it is among those who are fifty or younger, but the incidence rate for conjunctivitis is the same for people of all ages. The average age of all Americans is expected to exceed fifty by the year 2010. Which of the following conclusions can be most properly drawn about eye-related diseases from the information given? (A) Conjunctivitis will remain the most common eye- related disease among Americans in 2010. (B) By the year 2010, glaucoma will overtake conjunctivitis as the most common eye-related disease. (C) More people will suffer from conjunctivitis in 2010 than did in 1994. (D) Most Americans will encounter either conjunctivitis or glaucoma by 2010. (E) The average age of Americans suffering from conjunctivitis will increase between 1994 and 2010. 4. (K) History has shown that severe and sudden political instability strikes the Republic of Balanda roughly once every 50 years. The most recent example was the attempt on the President's life in 1992. The reaction of average investors in Balanda to crisis situations in the country cannot be predicted in advance. The government's fiscal affairs department has introduced an electronic protection mechanism into the market in the hopes of avoiding a prolonged large-scale selloff. The mechanism is triggered in specific instances based on estimations of how average investors will react to changes in corporate data and economic indicators. If the statements above are true, which of the following conclusions can be drawn regarding the electronic protection mechanism? (A) Sometime within the next 50 years severe and sudden political instability in Balanda will trigger the protection mechanism. (B) Whether the protection mechanism will function appropriately in response to a sudden political event depends on whether the event is seen by investors as positive or negative. (C) It is unclear how well the protection mechanism would work in the event of a sudden political coup if such an event were partially or wholly unrelated to changes in corporate data and

125


economic indicators. (D) There would be no way for the protection mechanism to differentiate between market fluctuations resulting from economic factors and those that are caused by political instability. (E) The protection mechanism would be purposely destroyed by political insurgents if they were able to infiltrate the government's fiscal affairs department.

5. Both Writewell and Express provide round-the-clock telephone assistance to any customer who uses their word-processing software. Since customers only call the hot lines when they find the software difficult to use, and the Writewell hot line receives four times as many calls as the Express hot line, Writewell’s word-processing software must be more difficult to use than Express’s. Which of the following, if true, most strengthens the argument above? (A) Calls to the Express hot line are almost twice as long, on average, as are calls to the Writewell hot line. (B) Express has three times the number of word-processing software customers that Writewell has. (C) Express receives twice as many letters of complaint about its word-processing software as Writewell receives about its word-processing software. (D) The number of calls received by each of the two hot lines has been gradually increasing. (E) The Writewell hot-line number is more widely publicized than the Express hot-line number.

6. Cats spend much of their time sleeping; they seem to awaken only to stretch and yawn. Yet they have a strong, agile musculature that most animals would have to exercise strenuously to acquire. Which one of the following, if true, most helps to resolve the apparent paradox described above? (A) Cats have a greater physiological need for sleep than other animals. (B) Many other animals also spend much of their time sleeping yet have a strong, agile musculature. (C) Cats are able to sleep in apparently uncomfortable positions. (D) Cats derive ample exercise from frequent stretching. (E) Cats require strength and agility in order to be effective predators.

7. Mechanicorp’s newest product costs so little to make that it appears doubtful the company will be able to sell it without increasing the markup the company usually allows for profit: potential clients would simply not believe that something so inexpensive would really work. Yet Mechanicorp’s reputation is built on fair prices incorporating only modest profit margins. The statements above, if true, most strongly support which of the following? (A) Mechanicorp will encounter difficulties in trying to set a price for its newest product that will promote sales without threatening to compromise the company’s reputation. (B) Mechanicorp achieves large annual profits, despite small profits per unit sold, by means of a high volume of sales. (C) Mechanicorp made a significant computational error in calculating the production costs for its

126


newest product. (D) Mechanicorp’s newest product is intended to perform tasks that can be performed by other devices costing less to manufacture. (E) Mechanicorp’s production processes are designed with the same ingenuity as are the products that the company makes. 8. In Debbie's magic act, a volunteer supposedly selects a card in a random fashion, looks at it without showing it to her, and replaces it in the deck. After several shuffles, Debbie cuts the deck and supposedly reveals the same selected card. A skeptic conducted three trials. In the first, Debbie was videotaped, and no sleight of hand was found. In the second, the skeptic instead supplied a standard deck of cards. For the third trial, the skeptic selected the card. Each time, Debbie apparently revealed the selected card. The skeptic concluded that Debbie uses neither sleight of hand, nor a trick deck, nor a planted "volunteer" to achieve her effect. Which one of the following most accurately describes a flaw in the skeptic's reasoning? (A) The skeptic failed to consider the possibility that Debbie did not always use the same method to achieve her effect. (B) The skeptic failed to consider the possibility that sleight of hand could also be detected by some means other than videotaping. (C) The skeptic failed to consider the possibility that Debbie requires both sleight of hand and a trick deck to achieve her effect. (D) The skeptic failed to consider the possibility that Debbie used something other than sleight of hand, a trick deck, or a planted "volunteer" to achieve her effect. (E) The skeptic failed to consider the possibility that Debbie's success in the three trials was something other than a coincidence. 9. The symptoms of hepatitis A appear no earlier than 60 days after a person has been infected. In a test of a hepatitis A vaccine, 50 people received the vaccine and 50 people received a harmless placebo. Although some people from each group eventually exhibited symptoms of hepatitis A, the vaccine as used in the test is completely effective in preventing infection with the hepatitis A virus. Which one of the following, if true, most helps resolve the apparent discrepancy in the information above? (A) The placebo did not produce any side effects that resembled any of the symptoms of hepatitis A. (B) More members of the group that had received the placebo recognized their symptoms as symptoms of hepatitis A than did members of the group that had received the vaccine. (C) The people who received the placebo were in better overall physical condition than were the people who received the vaccine. (D) The vaccinated people who exhibited symptoms of hepatitis A were infected with the hepatitis A virus before being vaccinated. (E) Of the people who developed symptoms of hepatitis A, those who received the vaccine recovered more quickly, on average, than those who did not.

10. (K) In order to promote off-season business, Mt. Dunmore Lodge made the following "Welcome Back" offer to their winter guests: guests who rent a room for at least a week during ski season can come back during the summer and get 25% off the standard summer price of any room they rent. After the summer passed, the owners of the lodge determined that the majority of their

127


guests had taken advantage of the "Welcome Back" offer and paid the reduced rates. However, they were surprised to find they still managed to rent more rooms at full price than they did at the discount rate. Which of the following, if true, most helps to explain the apparent discrepancy in the passage? (A) Most of the guests who stayed at Mt. Dunmore Lodge during the winter did not stay for a full week. (B) Those guests taking advantage of the "Welcome Back" discount were more likely to bring their families with them than were those guests who were paying full price. (C) Some of the guests who received the "Welcome Back" discount also received a 10% rate reduction through their auto club. (D) In order to pay for the construction of a new gymnasium and a new pool, the owners of the lodge raised their summer prices considerably. (E) On average, guests who took advantage of the "Welcome Back" discount spent more money at the hotel on additional goods and services than guests who paid full price for their rooms.

(K) Set 7 1. In 1984, almost 2 percent of humans who were admitted to hospital emergency rooms after suffering a scorpion bite in the southwestern United States died from the attack. Ten years later, this figure had jumped to 4 percent. Clearly, the venom of the scorpion has become much more toxic to humans. Which of the following statements, if true, most seriously weakens the above conclusion? (A) The scorpion population in the southwestern United States has remained steady since 1984. (B) There have been few innovations in the treatment of scorpion bites since 1984. (C) Most people who suffer scorpion bites are inexperienced hikers who are unaware of the best methods to avoid coming in contact with a scorpion. (D) Since 1984, people have learned that scorpion bites can be treated in the home as long as they are detected early. (E) People who survive one scorpion bite have a better than average chance of surviving a second bite. 2. An experimental microwave clothes dryer heats neither air nor cloth. Rather, it heats water on clothes, thereby saving electricity and protecting delicate fibers by operating at a lower temperature. Microwaves are waves that usually heat metal objects, but developers of a microwave dryer are perfecting a process that will prevent thin metal objects such as hairpins from heating up and burning clothes. Which of the following, if true, most strongly indicates that the process, when perfected, will be insufficient to make the dryer readily marketable? (A) Metal snap fasteners on clothes that are commonly put into drying machines are about the same thickness as most hairpins. (B) Many clothes that are currently placed into mechanical dryers are not placed there along with hairpins or other thin metal objects. (C) The experimental microwave dryer uses more electricity than future, improved models would

128


be expected to use. (D) Drying clothes with the process would not cause more shrinkage than the currently used mechanical drying process causes. (E) Many clothes that are frequently machine-dried by prospective customers incorporate thick metal parts such as decorative brass studs or buttons. 3. There should be a greater use of gasohol. Gasohol is a mixture of alcohol and gasoline, and has a higher octane rating and fewer carbon monoxide emissions than straight gasoline. Burning gasohol adds no more carbon dioxide to the atmosphere than plants remove by photosynthesis. Each of the following, if true, strengthens the argument above EXCEPT: (A) (B) (C) (D) (E)

Cars run less well on gasoline than they do on gasohol. Since less gasoline is needed with the use of gasohol, an energy shortage is less likely. Cars burn on the average slightly more gasohol per kilometer than they do gasoline. Gasohol is cheaper to produce and hence costs less at the pump than gasoline. Burning gasoline adds more carbon dioxide to the atmosphere than plants can remove.

4. Nutritionist: Many people claim that simple carbohydrates are a reasonable caloric replacement for the fatty foods forbidden to those on low-fat diets. This is now in doubt. New studies show that, for many people, a high intake of simple carbohydrates stimulates an overproduction of insulin, a hormone that is involved in processing sugars and starches to create energy when the body requires energy, or, when energy is not required, these products result in fat. Which one of the following is most strongly supported by the nutritionist's statements? (A) People on low-fat diets should avoid consumption of simple carbohydrates if they wish to maintain the energy that their bodies require. (B) People who produce enough insulin to process their intake of simple carbohydrates should not feel compelled to adopt low-fat diets. (C) People who consume simple carbohydrates should limit their intake of foods high in fat. (D) People who wish to avoid gaining body fat should limit their intake of foods high in simple carbohydrates. (E) People who do not produce an excessive amount of insulin when they consume foods high in simple carbohydrates will not lose weight if they restrict only their intake of these foods. 5. Prosecutor: Dr. Yuge has testified that, had the robbery occurred after 1:50 A.M., then, the moon having set at 1:45 A.M., it would have been too dark for Klein to recognize the perpetrator. But Yuge edged that the moon was full enough to provide considerable light before it set. And we were conclusively shown that the robbery occurred between 1:15 and 1:30 A.M. So there was enough light for Klein to make a reliable identification. The prosecutor's reasoning is most vulnerable to criticism because it overlooks which one of the following possibilities? (A) Klein may be mistaken about the time of the robbery and so it may have taken place after the moon had set. (B) The perpetrator may closely resemble someone who was not involved in the robbery. (C) Klein may have been too upset to make a reliable identification even in good light.

129


(D) Without having been there, Dr. Yuge has no way of knowing whether the light was sufficient. (E) During the robbery the moon's light may have been interfered with by conditions such as cloud cover. 6. The indigenous people of Tasmania are clearly related to the indigenous people of Australia, but were separated from them when the land bridge between Australia and Tasmania disappeared approximately 10,000 years ago. Two thousand years after the disappearance of the land bridge, however, there were major differences between the culture and technology of the indigenous Tasmanians and those of the indigenous Australians. The indigenous Tasmanians, unlike their Australian relatives, had no domesticated dogs, fishing nets, polished stone tools, or hunting implements like the boomerang and the spear-thrower. Each of the following, if true, would contribute to an explanation of differences described above EXCEPT: (A) After the disappearance of the land bridge the indigenous Tasmanians simply abandoned certain practices and technologies that they had originally shared with their Australian relatives. (B) Devices such as the spear-thrower and the boomerang were developed by the indigenous Tasmanians more than 10,000 years ago. (C) Technological innovations such as fishing nets, polished stone tools, and so on, were imported to Australia by Polynesian explorers more recently than 10,000 years ago. (D) Indigenous people of Australia developed hunting implements like the boomerang and the spear-thrower after the disappearance of the land bridge. (E) Although the technological and cultural innovations were developed in Australia more than 10,000 years ago, they were developed by groups in northern Australia with whom the indigenous Tasmanians had no contact prior to the disappearance of the land bridge. 7. Some planning committee members – those representing the construction industry – have significant financial interests in the committee's decisions. No one who is on the planning committee lives in the suburbs, although many of them work there. If the statements above are true, which one of the following must also be true? (A) No persons with significant financial interests in the planning committee's decisions are not in the construction industry. (B) No person who has a significant financial interest in the planning committee's decisions lives in the suburbs. (C) Some persons with significant financial interests in the planning committee's decisions work in the suburbs. (D) Some planning committee members who represent the construction industry do not work in the suburbs. (E) Some persons with significant financial interests in the planning committee's decisions do not live in the suburbs. 8. Rats are generally more active than mice. But since gerbils are generally more active than hamsters, it follows that rats are generally more active than hamsters. Any of the following, if introduced into the argument as an additional premise, makes the argument above logically correct EXCEPT:

130


(A) (B) (C) (D) (E)

Gerbils are generally more active than rats. Mice are generally more active than hamsters. Mice are generally more active than gerbils. Mice and gerbils are generally equally active. Rats and gerbils are generally equally active.

9. Columnist: It is impossible for there to be real evidence that lax radiation standards that were once in effect at nuclear reactors actually contributed to the increase in cancer rates near such sites. The point is a familiar one: who can say if a particular case of cancer is due to radiation, exposure to environmental toxins, smoking, poor diet, or genetic factors. The argument's reasoning is most vulnerable to criticism on which one of the following grounds? (A) The argument fails to recognize that there may be convincing statistical evidence even if individual causes cannot be known. (B) The argument inappropriately presupposes that what follows a certain phenomenon was caused by that phenomenon. (C) The argument inappropriately draws a conclusion about causes of cancer in general from evidence drawn from a particular case of cancer. (D) The argument ignores other possible causes of the increase in cancer rates near the nuclear reactor complexes. (E) The argument concludes that a claim about a causal connection is false on the basis of a lack of evidence for the claim. 10. Tsumi bats are a rare breed of omnivorous bat found only in highly temperate climates. Most Tsumi bats living in captivity develop endocrine imbalances from their normal zoo diets, which consist mostly of fruits and berries. The healthiest way to feed the bats, therefore, is to provide them primarily with nuts, grubs, and vegetables and only minimal amounts of fruits and berries. Which of the statements below does NOT reflect an assumption upon which the argument depends? (A) Those who care for Tsumi bats in captivity should avoid feeding them diets that produce endocrine imbalances. (B) Tsumi bats living in captivity will not be malnourished on diets that contain minimal fruits and berries. (C) Tsumi bats living in captivity will consume diets that consist of nuts, grubs, and vegetables but no fruits or berries. (D) Tsumi bats living in captivity will be adequately nourished on a diet that consists primarily of nuts, grubs, and vegetables. (E) For Tsumi bats living in captivity, no health problem stemming from diets consisting mostly of nuts, grubs, and vegetables would surpass in severity the health problems associated with endocrine imbalances. (K) Set 8 1. In interview with jurors inquiring how they arrived at their verdicts, researchers found that 40 percent of the references jurors made were to factors that had not been included in courtroom testimony. To improve the jury system, the researchers suggested that judges give instructions to the jury at the beginning of a trial rather than at the end. They argued that this would permit jurors to

131


concentrate on the most relevant evidence rather than filling in gaps with their own assumptions, which have little to do with the legality of a case. The answer to which of the following questions is LEAST directly relevant to evaluating the researchers' suggestion above? (A) Is it possible for a judge to instruct a jury at the end of a trial in such a way that jurors will disregard any irrelevant factors they had been using to weigh the evidence? (B) Will a jury that hears a judge's instructions at the beginning of a trial be able to weigh the evidence accordingly once that evidence has actually been presented? (C) Will having judges give instructions at the beginning of a trial rather than at the end significantly alter the customary procedures employed by the judicial system? (D) When jurors are queried as to how they arrived at their verdicts, does their interpretation of their decision-making process include many references to factors that were not, in fact, influential? (E) If jurors hear the judge's instructions at the beginning of a trial, what percentage of the factors that influence their decisions will be matters that were not presented in the evidence? 2. (PR) After several states negotiated a $195 billion settlement with the tobacco industry. State F used some of its money to create a youth-advocacy group whose antismoking advertising campaign was written entirely by high school-age students. The governor of State F has urged the state legislature to increase funding of this group because only teenagers know how to persuade other teenagers. Which of the following, assuming that it could be carried out, would be most useful in order to evaluate the governor's desire to provide further funding to the youth advocacy group? (A) comparing the short-term effects of State F's antismoking campaign to the project long-term success the campaign will probably enjoy (B) breaking down the statistics for the decline of teen smoking into a month-by-month analysis and determining which months saw the steepest decreases (C) comparing the decline in teen smoking in State F to the decline of teen smoking in other states that received money from the settlement and implemented advertising campaigns written by adults (D) requiring that more adults be hired to supervise the group's finances (E) comparing the aggregate number of smoking products bought the year before the group was set up to the number of smoking products bought during the year since the group was set up 3. Country X complains that Country Y's high tariffs on imported goods have artificially inflated the price of cars imported from Country X into Country Y, and that this is the reason that few of Country X's cars are sold in Country Y. On the other hand, Country X's very low tariffs allow Country Y to sell many cars there at relatively low prices. Country X says that if Country Y would lower its tariffs, then Country X's cars would be able to compete in Country Y and an equitable balance of trade would be achieved. Which of the following, if true, would most undermine the validity of Country X's explanation for the poor sales of its cars in Country Y? (A) In places where the tariffs on goods from both countries are equal, Country Y's cars far outsell cars from Country X.

132


(B) (C)

Cars imported from Country Z sell poorly in Country Y. In countries where tariffs on imported goods are higher than in Country Y, Country X sells more cars than does Country Y. (D) Other goods from Country X sell poorly in Country Y. (E) Sales of Country Y's cars are high even in countries that have higher tariffs on imported goods than does Country X. 4. A publisher is now providing university professors with the option of ordering custom textbooks for their courses. The professors can edit out those chapters of a book they are not interested in and add material of their own choosing. The widespread use of the option mentioned above is LEAST likely to contribute to fulfilling which of the following educational objectives? (A) Coverage of material relevant to a particular student body’s specific needs (B) Offering advanced elective courses that pursue in-depth investigation of selected topics in a field (C) Ensuring that students nationwide engaged in a specific course of study are uniformly exposed to a basic set of readings (D) Making the textbooks used in university courses more satisfactory from the individual teacher’s point of view (E) Keeping students’ interest in a course by offering lively, well-written reading assignments

5. Companies in the country of Kollontay can sell semiconductors in the country of Valdivia at a price that is below the cost to Valdivian companies of producing them. To help those Valdivian com-panies, the Valdivian legislature plans to set a minimum selling price in Valdivia for semiconductors manufactured in Kollontay that is ten percent greater than the average production costs for companies in Valdivia. Which of the following, if true, most seriously threatens the success of the plan? (A) The annual rate of inflation in Kollontay is expected to exceed ten percent within the next year. (B) Valdivia is not the only country where companies in Kollontay currently sell semiconductors. (C) Some Valdivian companies that sell semiconductors have announced that they plan to decrease their price for semiconductors. (D) The government of Kollontay will also set a minimum price for selling semiconductors in that country. (E) Emerging companies in countries other than Kollontay will still be able to sell semiconductors in Valdivia at a price below the cost to Valdivian companies to manufacture them. 6. The top priority of the school administration should be student attendance. No matter how good the teachers, texts, and facilities are, none of these does any good if few students come to school. The pattern of reasoning in the argument above is LEAST similar to that in which one of the following? (A) The top priority of a salesperson should be not to alienate customers. Honesty and a good knowledge of the product line are useful to a salesperson only if the customer feels at ease.

133


(B)

The top priority of a person lost in the wilderness should be food-gathering. Knowing how to find one's way back or how to build a comfortable shelter does one no good if one does not have enough food to survive. (C) The top priority of a detective should be to gather physical evidence. High-tech crime lab equipment and the most sophisticated criminological analysis are of no use if crucial clues are not gathered. (D) The top priority of a library should be to maintain its collection of books. A knowledgeable staff and beautiful facilities are of no value if there is an inadequate supply of books to lend. (E) The top priority of a criminal defense lawyer should be to ensure that innocent clients are found not guilty. Such clients can justly be released from jail and resume their normal lives if they are found not guilty.

7. We can learn about the living conditions of a vanished culture by examining its language. Thus, it is likely that the people who spoke Proto-Indo-European, the language from which all IndoEuropean languages descended, lived in a cold climate, isolated from ocean or sea, because ProtoIndo-European lacks a word for "sea", yet contains words for "winter", "snow", and "wolf". Which one of the following, if true, most seriously weakens the argument? (A) (B) (C) (D) (E)

A word meaning "fish" was used by the people who spoke Proto-Indo-European. Some languages lack words for prominent elements of the environments of their speakers. There are no known languages today that lack a word for "sea." Proto-Indo-European possesses words for "heat." The people who spoke Proto-Indo-European were nomadic.

8. Some judges complain about statutes that specify mandatory minimum sentences for criminal offenses. These legal restrictions, they complain, are too mechanical and prevent judges from deciding when a given individual can or cannot be rehabilitated. But that is precisely why mandatory minimum sentences are necessary. History amply demonstrates that when people are free to use their own judgment they invariably believe themselves to act wisely when in fact they are often arbitrary and irrational. There is no reason to think that judges are an exception to this rule. Which one of the following sentences most accurately expresses the main point of the passage? (A) (B) (C) (D) (E)

People believe that they have good judgment but never do. Mandatory minimum sentences are too mechanical and reduce judicial discretion. Judges should be free to exercise their own judgment. Judges are often arbitrary and irrational. Mandatory minimum sentences are needed to help prevent judicial arbitrariness.

9. Choi: All other factors being equal, children whose parents earned doctorates are more likely to earn a doctorate than children whose parents did not earn doctorates.

134


Hart: But consider this: Over 70 percent of all doctorate holders do not have a parent that also holds a doctorate. Which of the following is the most accurate evaluation of Hart's reply? (A) (B) (C) (D) (F)

It establishes that Choi's claim is an exaggeration. If true, it effectively demonstrates that Choi's claim cannot be accurate. It is consistent with Choi's claim. It provides alternative reasons for accepting Choi's claim. It mistakes what is necessary for an event with what is sufficient to determine that the event will occur.

10. The paintings of French painter Trianne Dejere sold best in the period following the production of La Triumph, now Dejere's most famous piece. In the twelve month period preceding the unveiling of this piece, Dejere sold 57% of the works she produced in this period, a far greater percentage than in previous years. In the twelve month period following a glowing review of La Triumph in a popular magazine, however, Dejere sold 85% of the paintings she produced. Interestingly, Dejere's revenue from painting sales was roughly the same in both periods, since she sold the same number of paintings in the twelve months before presenting La Triumph as she did in the twelve months following the favorable review. Which of the following statements can be properly concluded from the passage, if the information above is true? (A) Due to the positive review, Dejere was able to charge substantially more for the works produced after La Triumph than the works produced before it. (B) Dejere was more concerned with positive reviews than with increasing the prices of her paintings. (C) The positive review of La Triumph brought Dejere's work to the attention of more art collectors than were previously aware of her work. (D) Dejere painted fewer works in the twelve month period following the review of La Triumph than she had in the twelve month period preceding its unveiling. (E) Dejere paid more attention to marketing her paintings after La Triumph received such a positive reception.

(K) Set 9 1. It is mistaken to attribute Zanco's failure to the publicity about the supposedly inhuman working conditions in the foreign factories that furnish Zanco with many of its parts. Zanco's failure has more to do with defects in its products than with any boycott on moral grounds. After all, plenty of other companies are supplied by factories with working conditions just as bad as those in Zanco's suppliers, and the public does not hesitate to buy their products. The argument in the passage is based on which of the following assumptions? (A) People are unlikely to let moral considerations affect what products they decide to purchase. (B) People who patronize companies supplied by factories where working conditions are as bad as those of Zanco's suppliers are aware of those conditions. (C) The working conditions in the factories that supply Zanco with parts are not as bad as has been

135


claimed. (D) Zanco's sales did not dip sharply after the poor working conditions in its suppliers' factories became known. (E) The poor quality of Zanco's products is not a result of the working conditions in the foreign factories where its parts are manufactured.

2. Reading skills among high school students in Gotham have been steadily declining, which can only be the result of overcrowding in the schools. Which of the following, if true, most seriously weakens the argument expressed above? (A) The high school system in Gotham succeeds in giving students a good education at considerably less cost than do most systems. (B) Several cities have found that overcrowding in the schools is not always associated with lower reading scores. (C) Gotham schools have a greater teacher-to-student ratio than most other school systems. (D) Students' reading skills have not declined in other cities where the high schools are just as crowded as those of Gotham. (E) Schools are not overcrowded in many cities where high school reading scores have declined more than they have in Gotham.

3. Before the advent of writing, each of the isolated clans of the Comaquogue tribe had master storytellers whose function was to orally transmit the clan's tradition from one generation to the next. When writing was developed within certain clans of the tribe, their master storytellers disappeared within a few generations. This stands to reason, since the availability of written records obviated the need for masterful oral communicators to keep the tradition of literate clans alive. What has puzzled anthropologists, however, is the total lack of masterful storytellers in modern illiterate Comaquogue clans. Which of the following, if true, best helps to explain the puzzling situation mentioned above? (A) Modern illiterate Comaquogue clan members display personality characteristics that resemble their ancestors more closely than they resemble the characteristics of modern literate Comaquogue clan members. (B) Modern illiterate Comaquogue clans participate in more ritual gatherings than most modern literate Comaquogue clans do, but they participate in fewer ritual gatherings than did their common ancestors. (C) Modern illiterate Comaquogue clans are recently descended from longtime literate clans that failed to pass on the skills of reading and writing due to a devastating 75-year war. (D) The celebrations of modern illiterate Comaquogue clans involve a great deal of singing and dancing, and children are taught clan songs and dances from a very young age. (E) The traditions of modern illiterate Comaquogue clans are an amalgamation of the cumulative experiences of previous generations plus innovations to the heritage added by the current generation of clan members.

136


4. An economic or political crisis in a poor country can lead to a lack of faith in the country's leaders, which is often followed by violent behavior, dissent, and even revolt among specific segments of the population. In many cases, propaganda is immediately issued from media outlets that quells such reactions by downplaying the extent of the recent crisis, thereby helping to restore belief in the efficacy of the government. However, the habitual violence exhibited by certain groups of disaffected youths in such countries generally has nothing to do with a lack of faith in their leaders, but rather is the consequence of an endemic boredom and lack of any vision of a positive future for themselves. Which of the following statements follows most logically from the statements in the passage above? (A) It is easier to quell periodic revolts in poor countries than it is to solve the habitual problem of youth violence. (B) In all poor countries, propaganda alone cannot entirely diffuse dissent stemming from an economic or political crisis. (C) Economic and political crises do not lead to any instances of youth violence in poor countries. (D) The effect that propaganda has in putting down revolts in poor countries is primarily related to its ability to alter people's fundamental beliefs. (E) To the extent that propaganda may help to decrease youth violence in a poor country, it is probably not the result of restoring the youths' faith in their country's leadership.

5. The plastics commonly used in household garbage bags take, on average, 110 years to decompose in landfills. From an environmental standpoint, the plastic bag industry should be forced to switch to newly developed plastics, which begin to decompose after only 20 years. Which of the following pieces of information would be most helpful in evaluating the argument above? (A) (B) (C) (D) (E)

the rate of growth or decline in sales of plastic garbage bags the number of plastic garbage bags sold last year that eventually wound up in landfills the feasibility of enforcing legislation that regulates the plastics used in garbage bags the length of time it takes the newly developed plastic to fully decompose in landfills. a comparison of the production cost of one bag made with the old plastics and of one bag made with the new plastics

6. Membership in the Theta Delta Psi fraternity is easily obtained by those who have previously had strong social connections with existing fraternity members before college. However, one must have attended high school with one or more of the members in order to forge such strong social connections. People who lack these social connections because they have not attended high school with one or more current fraternity members will therefore find it difficult to join the fraternity. This argument displays flawed reasoning because it neglects to consider the possibility that (A) many of those who went to high school with Theta Delta Psi fraternity members did not themselves become members of the fraternity (B) it is more important in the long run to socialize with non-fraternity members than to develop strong connections with fraternity members (C) it is more difficult to forge social connections with fraternity members than with non-fraternity

137


members (D) one may easily obtain membership in the fraternity through means other than having strong social connections with existing fraternity members (E) some current members of the fraternity did not go to high school with other members

7.

Which of the following best completes the passage below?

In opposing government regulation of business, conservatives often appeal to the Jeffersonian ideal of limited government, expressing the wish that government would “get off the backs of the American people.� Yet, paradoxically, many of these same conservatives address questions of private morality, such as those dealing with sexual behavior, by calling for______ (A) a return to the restrictive sexual morality of the Victorian era (B) a strengthening of the role of the family in setting moral norms for society (C) a limitation on the amount of sexually provocative material appearing in books, motives, and television shows (D) greater freedom for individuals to choose their own way of handling sexual issues (E) an increased governmental role in the regulation and control of private sexual behavior

8. Determining the authenticity of purported pre-Columbian artifacts is never easy. Carbon-14 dating of these artifacts is often impossible due to contamination by radioactive palladium (which occurs naturally in the soils of Central and South America). However, historians and anthropologists have evolved two reliable criteria, which, utilized in combination, have proven effective for dating these artifacts. First, because authentic pre-Columbian artifacts characteristically occur in a coarse, granular matrix that is shifted by major earthquakes, they often exhibit the unique scratch patterns known as gridding. In addition, true pre-Columbian artifacts show a darkening in surface color that is caused by centuries of exposure to the minute amounts of magnesium in the soil of the Americas. The criteria above would be LEAST useful in judging the authenticity of which of the following? (A) (B) (C) (D) (E)

An ax head of black obsidian, unearthed from a kitchen midden A pottery bowl with a red ocher design, found in the ruins of a temple A set of gold ear weights, ornamented with jasper pendants A black feather cape from a king’s burial vault A multicolored woven sash found near the gravesite of a slave

9. Scientists previously believed that human cells could divide an infinite number of times, as long as the tissue harboring those cells remained perfectly healthy. However, human spleen cells grown in an artificial, yet healthy, environment were shown to divide exactly twenty-four times before they died off. The bold phrases play which of the following roles in the argument? (A) The first phrase is the author's conclusion, and the second phrase provides an additional

138


premise for that conclusion. (B) The first phrase states a theory, and the second phrase weakens that theory by providing a counterexample. (C) The second phrase clarifies an ambiguity that the first phrase neglects to address. (D) The first phrase asserts a possible phenomenon, and the second phrase offers a condition for that phenomenon to take place. (E) The second phrase provides evidence that the theory presented in the first phrase is incomplete.

10. Sometimes when their trainer gives the hand signal for "Do something creative together," two dolphins circle a pool in tandem and then leap through the air simultaneously. On other occasions the same signal elicits synchronized backward swims or tail-waving. These behaviors are not simply learned responses to a given stimulus. Rather, dolphins are capable of higher cognitive functions that may include the use of language and forethought. Which one of the following, if true, most strengthens the argument? (A) Mammals have some resemblance to one another with respect to bodily function and brain structure. (B) The dolphins often exhibit complex new responses to the hand signal. (C) The dolphins are given food incentives as part of their training. (D) Dolphins do not interact with humans the way they interact with one another. (E) Some of the behaviors mentioned are exhibited by dolphins in their natural habitat. (K) Set 10 1. In a survey of freshmen at University X, two-thirds claimed never to have plagiarized while in high school. However, the survey may overstate the proportion of freshmen at University X who did not plagiarize in high school because_______ Which of the following best completes the passage above? (A) (B) (C) (D) (E)

some people who do not attend University X probably plagiarized in high school. some people who plagiarized in high school may not do so in college. some people who claimed to have plagiarized once may have done so many times. at University Z one half of the freshmen admitted to having plagiarized in high school. some freshmen who did plagiarize in high school might have claimed on the survey that they did not do so.

2. The workers' union of GrainCorp, a grain-processing plant, is attempting to obtain a pay raise from GrainCorp management. To pressure GrainCorp management into accepting the union's proposal, the president of the union has proposed a boycott against SquareMart food stores, which are owned by MegaFood, the parent company of GrainCorp. The answer to which of the following questions is LEAST directly relevant to the union president's consideration of whether a boycott of SquareMart will lead to acceptance of their pay rate proposal?

139


(A) Would the loss of business at SquareMart stores materially affect MegaFood? (B) Are the staple food products purchased by consumers at SquareMart stores readily available at other stores not owned by MegaFood? (C) How many SquareMarts are within the region of the GrainCorp plant? (D) Have other unions successfully employed the same strategy? (E) Is MegaFood the only corporation that operates both grain-processing plants and food stores?

3. Malcolm: I refuse to feel any contrition about failing to report all of my income on my income tax return last year. I have discussed this topic extensively with many friends, family members, and business associates, and it is clear to me that most Americans have bent the truth on their income tax returns at one time or another. Luka: It is improper for you to rationalize your actions that way. Regardless of how often it occurs, an illegal deed is still illegal and should be punished. Which of the following statements summarizes Luka's reasoning in response to Malcolm's admission? (A) She questions the credibility of the sources that Malcolm has consulted. (B) She offers evidence that Malcolm's actions were much more severe than he perceived them to be. (C) She demonstrates that Malcolm's rationalizations are based on insufficient evidence. (D) She asserts that the frequency of a crime does not lessen its severity. (E) She introduces the possibility that the moral convictions of different people can differ greatly.

4. At a certain investment bank that specializes in mergers and acquisitions, the highest percentage of potential deals that are never completed are those in which the bank's senior partner was the lead negotiator. Each of the senior partner's colleagues, however, states unequivocally that she is the most adept negotiator at the bank. Which one of the following, if true, goes furthest toward showing that these two statements could both be correct? (A) The current senior partner has a better record of success than her immediate predecessor did. (B) Many of the junior partners were trained by the senior partner when they first joined the firm. (C) The senior partner works only on potential deals that have the least chance of coming to fruition. (D) The number of mergers in which the investment bank has been involved has declined slightly in each of the last three years. (E) The senior partner was chosen by the board of directors of the bank's parent company, a large publishing conglomerate.

5. The university's decision to scale back significantly its teaching of the literary and philosophical classics of the Western Tradition is misguided. Proponents of the move argue that today's students are not interested in these works and desire more practical business-related courses

140


that will help them in their future careers. But any student lacking a sufficient grounding in the thought and tradition that underlie the present civilization cannot be said to be fully educated. The classics are the primary vehicle for instilling such knowledge. Which of the following best expresses the relationship between the two bolded statements above? (A) The first statement offers a hypothesis and the second statement offers conflicting evidence. (B) The first statement suggests an alternative explanation for the phenomenon described in the second statement. (C) The second statement provides evidence for a conclusion drawn in the first statement. (D) The second statement must be true for the first statement to be true. (E) The second statement is an inference drawn from the first statement.

6. Recent experiments in the Southern Ocean offer the promise of controlling the threat of global warming by creating organic "sponges" for carbon dioxide, which is widely considered the main culprit for rising global temperatures. Scientists were able to grow a lush strip of phytoplankton 150 kilometers long by fertilizing a patch of the ocean with hundreds of kilograms of an iron compound. Phytoplankton presently accounts for over half of the photosynthesis on Earth, the process by which carbon dioxide is absorbed and converted into oxygen. A major limiting factor in the production of phytoplankton is lack of iron, but by fertilizing oceans with iron compounds scientists hope to be able to reduce carbon dioxide levels and reverse the greenhouse effect.

Which of the following, if true, casts the most serious doubt upon the advisability of using the fertilization method described above to control carbon dioxide levels? (A) In some oceans, the growth of phytoplankton is also limited by how much nitrogen, phosphorus, and silicon are available. (B) The cost of fertilizing the oceans with sufficient quantities of iron to reverse the greenhouse effect is likely to be very high. (C) Iron naturally reaches the seas in the form of windblown mineral dust, which becomes more or less abundant as conditions on land change. (D) Fertilization efforts will do nothing to curb the production of more carbon dioxide emissions. (E) The oceans are a complex system, and the long-term consequences of ocean fertilization may be the opposite of what is predicted.

Questions 7-8 are based on the following: It is well known that many species adapt to their environment, but it is usually assumed that only the most highly evolved species alter their environment in ways that aid their own survival. However, this characteristic is actually quite common. Certain species of plankton, for example, generate a gas that is converted in the atmosphere into particles of sulfate. These particles cause water vapor to condense, thus forming clouds. Indeed, the formation of clouds over the ocean largely depends on the presence of these particles. More cloud cover means that sunlight is reflected and so the Earth

141


absorbs less heat. Thus plankton cause the surface of the Earth to be cooler and this benefits the plankton.

7.

Of the following, which one most accurately expresses the main point of the argument?

(A) The Earth would be far warmer than it is now if certain species of plankton became extinct. (B) By altering their environment in ways that improve their chances of survival, certain species of plankton benefit the Earth as a whole. (C) Improving their own chances of survival by altering the environment is not limited to the most highly evolved species. (D) The extent of the cloud cover over the oceans is largely determined by the quantity of plankton in those oceans. (E) Species such as plankton alter the environment in ways that are less detrimental to the wellbeing of other species than are the alterations to the environment made by more highly evolved species.

8.

Which one of the following accurately describes the argumentative strategy employed?

(A) A general principle is used to justify a claim made about a particular case to which that principle has been shown to apply. (B) An explanation of how a controversial phenomenon could have come about is given in order to support the claim that this phenomenon did in fact come about. (C) A generalization about the conditions under which a certain process can occur is advanced on the basis of an examination of certain cases in which that process did occur. (D) A counterexample to a position being challenged is presented in order to show that this position is incorrect. (E) A detailed example is used to illustrate the advantage of one strategy over another.

9. Damaged nerves in the spinal cord do not regenerate themselves naturally, nor even under the spur of nerve-growth stimulants. The reason, recently discovered, is the presence of nerve-growth inhibitors in the spinal cord. Antibodies that deactivate those inhibitors have now been developed. Clearly, then, nerve repair will be a standard medical procedure in the foreseeable future.

Which of the following, if true, casts the most serious doubt on the accuracy of the prediction above? (A) Prevention of the regeneration of damaged nerves is merely a by-product of the main function in the human body of the substances inhibiting nerve growth. (B) Certain nerve-growth stimulants have similar chemical structures to those of the antibodies against nerve-growth inhibitors. (C) Nerves in the brain are similar to nerves in the spinal cord in their inability to regenerate themselves naturally. (D) Researchers have been able to stimulate the growth of nerves not located in the spinal cord by using only nerve-growth stimulants.

142


(E)

Deactivating the substances inhibiting nerve growth for an extended period would require a steady supply of antibodies.

10. Candidate: I am worried about the effects that the recent media coverage of my personal life will have on my chances of gaining office. Even though the reports are untrue, some voters interviewed on television, in response to these reports, have already expressed doubts regarding my ability to lead. Campaign manager: Your concern is unfounded. Of 1,000 people in this city randomly surveyed by e-mail, only 25 have responded that their perception of your ability to lead has been negatively impacted by the recent media coverage. The campaign manager's argument is most vulnerable to criticism on the ground that it fails to acknowledge the possibility that (A) future media reports that follow up on the story of the candidate's personal life will further damage the public's perception of the candidate's ability to lead (B) the candidate's main opponent will use the opportunity created by the recent media coverage to conduct her own survey to assess the damage done to her opponent's credibility (C) the voting public would understand that its reaction to the recent media coverage of the candidate's personal life was the intended primary focus of the survey (D) opinions expressed in television interviews are not always the most reliable indicator of how interviewees are likely to act in given situations (E) many of those surveyed who are skeptical of the candidate's ability to lead due to the recent reports did not respond to the survey

(K) Set 11 Questions 1-2 are based on the following: A consumer survey of independent feature films revealed that the percentage of action films that received the survey's highest rating was greater than the percentage of romance films that received the highest rating. Yet, the survey organizers were probably erroneous in their conclusion that subject matter determines a feature film's popular appeal, since the action films were all directed by filmmakers with at least one hit film to their credit, while the romance films were directed by newer filmmakers, many of whom had not produced a previous film.

1. The statements above, if true, support which of the following inferences? (A) Fewer romance films than action films received the survey's highest rating. (B) There is no relationship between the popular appeal of the feature films evaluated in the survey and any previous successes of the directors of those films. (C) If consumers were surveyed regarding their impressions of big-budget mainstream films, the percentage of romance films that would receive the survey's highest rating would be lower than

143


the percentage of action films that would receive the highest rating. (D) Experienced filmmakers are more likely to produce hit films than are new filmmakers. (E) Among directors with the same number of hit films to their credit, differences in the subject matter of their feature films may not affect the way the films are popularly rated.

2. Each of the following, if true, supports the author's contention that the organizers misinterpreted the survey data EXCEPT: (A) The fact that one has directed a previous hit film is a positive indicator of that director's filmmaking talent. (B) Consumer ratings of a new film are influenced by the previous history of success of the film's director. (C) Action films generally require larger budgets than romance films and are thus prohibitive for many first-time film directors. (D) It is rare for the films of first-time directors to attain the popular appeal of films directed by filmmakers with at least one hit film to their credit. (E) Directors who have produced a previous hit film generally obtain the largest budgets and attract the most talented and well-known actors for their subsequent films.

3. Over the past seven years, private college tuition rates have increased, resulting in a large decrease in private college attendance across the country. Private college revenues, however, have progressively increased in each of the seven years during this period, and researchers predict further increases in the years to come. Which of the following, if true, offers the best explanation for the situation described above? (A) Most private colleges increase tuition rates approximately once every two years. (B) Attendance at vocational schools generally exceeds attendance at private colleges in most cities. (C) The increase in tuition rates at private colleges has influenced many prospective students to seek a state scholarship to attend a public university. (D) The decrease in students attending private colleges over the last seven years has been more than offset by the increases in tuition. (E) Private colleges gain a larger percentage of their revenue from alumni contributions than do public universities.

4. Airplane manufacturer: I object to your characterization of our X-387 jets as dangerous. No X-387 in commercial use has ever crashed or even had a serious malfunction. Airline regulator: The problem with the X-387 is not that it, itself, malfunctions, but that it creates turbulence in its wake that can create hazardous conditions for aircraft in its vicinity.

The airline regulator responds to the manufacturer by doing which of the following?

144


(A) Characterizing the manufacturer’s assertion as stemming from subjective interest rather than from objective evaluation of the facts (B) Drawing attention to the fact that the manufacturer’s interpretation of the word “dangerous” is too narrow (C) Invoking evidence that the manufacturer has explicitly dismissed as irrelevant to the point at issue (D) Citing statistical evidence that refutes the manufacturer’s claim (E) Casting doubt on the extent of the manufacturer’s knowledge of the number of recent airline disasters

5. The human body secretes more pain-blocking hormones late at night than during the day. Consequently, surgical patients operated on at night need less anesthesia. Since larger amounts of anesthesia pose greater risks for patients, the risks of surgery could be reduced if operations routinely took place at night.

Which of the following, if true, argues most strongly against the view that surgical risks could be reduced by scheduling operations at night? (A) Energy costs in hospitals are generally lower at night than they are during the day. (B) More babies are born between midnight and seven o’clock in the morning than at any other time. (C) Over the course of a year, people’s biological rhythms shift slightly in response to changes in the amounts of daylight to which the people are exposed. (D) Nurses and medical technicians are generally paid more per hour when they work during the night than when they work during the day. (E) Manual dexterity and mental alertness are lower in the late night than they are during the day, even in people accustomed to working at night.

Questions 6-7 are based on the following: Walter: A copy of an artwork should be worth exactly what the original is worth if the two works are visually indistinguishable. After all, if the two works are visually indistinguishable, they have all the same qualities, and if they have all the same qualities, their prices should be equal. Marissa: How little you understand art! Even if someone could make a perfect copy that is visually indistinguishable from the original, the copy would have a different history and hence not have all the same qualities as the original.

6.

Which of the following is a point at issue between Walter and Marissa?

(A) Whether a copy of an artwork could ever be visually indistinguishable from the original (B) Whether the reproduction of a work of art is ever worth more than the original is worth

145


(C) Whether a copy of a work of art is ever mistaken for the original (D) Whether a copy of a work of art could have all the same qualities as the original (E) Whether originality is the only valuable attribute that a work of art can possess

7.

Marissa uses which of the following techniques in attempting to refute Walter’s argument?

(A) Attacking his assumption that the price of an artwork indicates its worth (B) Raising a point that would undermine one of the claims on which his conclusion is based (C) Questioning his claim that a perfect copy of a work of art would be visually indistinguishable from the original (D) Giving reason to believe that Walter is unable to judge the quality of a work of art because of his inadequate understanding of the history of art (E) Proposing alternative criteria for determining whether two works of art are visually indistinguishable

8. Many consumers are concerned about the ecological effects of wasteful packaging. This concern probably explains why stores have been quick to stock new cleaning products that have been produced in a concentrated form. The concentrated form is packaged in smaller containers that use less plastic and require less transportation space. Which of the following, if true, most seriously undermines the explanation offered above? (A) Few consumers believe that containers of concentrated cleaning products are merely small packages of regular cleaning products. (B) The containers in which concentrated cleaning products are packaged are no harder to recycle than those in which regular cleaning products are packaged. (C) Those concentrated cleaning products that are intended to be used diluted have clear instructions for dilution printed on their labels. (D) The smaller containers of concentrated cleaning products enable supermarkets and drugstores to increase their revenues from a given shelf space. (E) Consumer pressure has led to the elimination of wasteful cardboard packaging that was used for compact discs.

9. Jean: Our navigational equipment sells for $ 1,100 and dominates the high end of the market, but more units are sold by our competitors in the $700 to $800 range. We should add a low-cost model, which would allow us to increase our overall sales while continuing to dominate the high end. Tracy: I disagree. Our equipment sells to consumers who associate our company with quality. Moving into the low-cost market would put our competitors in the high-cost market on an equal footing with us, which could hurt our overall sales. Jean's and Tracy's statements most strongly suggest that they disagree over which one of the following propositions? (A) There is a greater potential for profits in the low-cost market than there is in the high-cost

146


market. (B) The proposed cheaper model, if it were made available, would sell to customers who would otherwise be buying the company's present model. (C) The company could dominate the low-cost market in the same way it has dominated the highcost market. (D) The company would no longer dominate the high- cost market if it began selling a low-cost model. (E) Decreased sales of the high-cost model would result in poor sales for the proposed low-cost model.

10. Arbitrator: The shipping manager admits that he decided to close the old facility on October 14 and to schedule the new facility's opening for October 17, the following Monday. But he also claims that he is not responsible for the business that was lost due to the new facility's failing to open as scheduled. He blames the contractor for not finishing on time, but he, too, is to blame, for he was aware of the contractor's typical delays and should have planned for this contingency. Which one of the following principles underlies the arbitrator's argument? (A) (B) (C) (D) (E)

A manager should take foreseeable problems into account when making decisions. A manager should be able to depend on contractors to do their jobs promptly. A manager should see to it that contractors do their jobs promptly. A manager should be held responsible for mistakes made by those whom the manager directly supervises A manager, and only a manager, should be held responsible for a project's failure

(K) Set 12 Questions 1-2 are based on the following: Politician: The funding for the new nationwide health-awareness campaign should come from an increase in taxes on cigarettes. It is well established that cigarette smoking causes many serious health problems, and it is only reasonable that people whose unhealthful habits cause so many health problems should bear the costs of that campaign. Smoker: But it is equally well established that regularly eating high-fat, high-cholesterol foods causes as many serious health problems as does smoking, yet it would be manifestly unreasonable to force those who purchase such foods to bear the burden of financing this campaign.

1.

Which one of the following is the point at issue between the politician and the smoker?

(A) whether the politician's proposal for financing the health-awareness campaign is an unreasonable one (B) whether smokers are more aware of the harmful effects of their habit than are people who regularly eat high-fat, high-cholesterol foods

147


(C)

whether the effects of smoking constitute a greater health hazard than do the effects of regularly eating high-fat, high-cholesterol foods (D) whether it is unreasonable to require people who do not benefit from certain governmental programs to share the costs of those programs (E) whether the proposed increase on cigarette taxes is an efficient means of financing the healthawareness campaign

2. (A) (B) (C) (D) (E)

The smoker's response to the politician's argument offers a counterexample that calls into question the politician's reasoning presents an alternative solution to that proposed by the politician argues that the method proposed by the politician would be inadequate for its intended purpose questions the accuracy of the information cited by the politician in reaching a conclusion illustrates how the politician's proposal could aggravate the problem it is intended

3. Automobile-emission standards are enforced through annual inspection. At those inspections cars are tested while idling; that is, standing still with their engines running. Testing devices measure the levels of various pollutants as exhaust gases leave the tail pipe.

Which one of the following, if true, most strongly indicates that current enforcement of automobileemission standards might be ineffective in controlling overall pollutant levels? (A) As an emission-control technology approaches its limits, any additional gains in effectiveness become progressively more expensive. (B) The testing devices used must be recalibrated frequently to measure pollutant levels with acceptable accuracy. (C) The adjustments needed to make a car idle cleanly make it likely that the car will emit high levels of pollutants when moving at highway speeds. (D) Most car owners ask their mechanics to make sure that their cars are in compliance with emission standards. (E) When emission standards are set, no allowances are made for older cars.

4. A career in dermatology is still a safe bet for medical school graduates. In the U.S., the number of cases of skin cancer linked to ultraviolet (UV) radiation in sunlight has remained relatively constant every year even though far fewer adults are intentionally exposing themselves to UV sunlight now than were doing so at the height of the suntan craze 20 years ago.

Each of the following, if true, could explain the relative stability in the incidence of skin cancer each year despite the decrease in intentional exposure to UV sunlight EXCEPT: (A) Because of decreasing levels of ozone in the upper atmosphere, more people are now exposed

148


accidentally to excessive UV sunlight. (B) People who continue to intentionally expose themselves to UV sunlight are absorbing larger doses of the harmful radiation than the average sun- tanner did in the past. (C) Levels of UV radiation from sources other than sunlight are increasing every year. (D) While fewer women are intentionally exposing themselves to UV sunlight, the number of men doing so has increased significantly. (E) In most victims, skin cancer is linked to exposures to UV sunlight that occurred up to 30 years before the onset of the disease.

5. A team of pediatricians recently announced that dogs are more likely to bite children under age 13 than any other age group. Their finding was based on a study showing that the majority of all dog bites requiring medical attention involved children under 13. The study also found that the dogs most likely to bite are German shepherds, males, and non-neutered dogs.

Which of the following, if true, would most weaken the pediatricians' conclusion that dogs are more likely to bite children under age 13 than any other age group? (A) More than half of dog bites not requiring medical attention, which exceed the number requiring such attention, involve people aged 13 and older. (B) The majority of dog bites resulting in the death of the bitten person involve people aged 65 and older. (C) Many serious dog bites affecting children under age 13 are inflicted by female dogs, neutered dogs, and dogs that are not German shepherds. (D) Most dog bites of children under age 13 that require medical attention are far less serious than they initially appear. (E) Most parents can learn to treat dog bites effectively if they avail themselves of a small amount of medical information.

6. An antique and curiosity shop is weighing the advantages of renewing its regular advertisement in a monthly trade publication. The shop originally placed the advertisement in order to increase business, but found that the majority of its sales are made to those who do not read the publication and have not seen the ad. The shop is considering canceling the advertisement in that publication in order to save money.

The answer to which of the following questions would be LEAST relevant to the shop's decision? (A) How does the cost of the advertisement compare to the profit from purchases made by those who have responded to it? (B) Are there other trade publications that attract the shop's typical clientele in which an advertisement would be likely to reach prospective customers at the same cost? (C) Is a significant proportion of the shop's competitors satisfied with the effect of their advertisements in trade publications?

149


(D) How many buyers who responded to the advertisement were professional antique dealers who generally make relatively expensive purchases? (E) Is any significant change expected in the pattern of the trade publication's circulation?

Questions 7-8 are based on the following: The proliferation of colloquialisms is degrading the English language. A phrase such as she was like, "no way!" you know?—a meaningless collection of English words just a few decades ago—is commonly understood by most today to mean she was doubtful. No language can admit imprecise word usage on a large scale without a corresponding decrease in quality.

7.

The argument relies on which of the following assumptions?

(A) Colloquialisms always evolve out of a meaningless collection of words. (B) The colloquialisms appearing in the English language introduce imprecision into the language on what would be considered a large scale. (C) The Russian, French, and German languages cannot admit imprecise word usage on a large scale without an inevitable decrease in the quality of those languages. (D) The English language would not be degraded if there did not exist an alternative informal way to express the sentiment "she was doubtful." (E) The widespread use of colloquialisms represents the most serious form of language degradation.

8.

Which of the following, if true, most weakens the argument above?

(A) Linguists have shown that the use of imprecise language on a small scale does not generally impair understanding. (B) Many colloquialisms that appeared in earlier forms of the English language disappeared over time as the people who used those particular phrasings were assimilated into larger groups with different language patterns. (C) Dissemination of a new word or phrase by the mass media is an important factor in whether or not the new word or phrase will become a colloquialism. (D) Colloquialisms are more likely to be coined by the youth in a culture than by any other segment of the population. (E) Languages of the highest quality often evolve over time out of a collection of colloquial usages woven into the formal dialect of a given people.

9. In the years 1971 to 1980, the population of the state prison system was on average about 82 percent of maximum occupancy. During those years, the average number of prisoners entering the system each year was equivalent to 9.1 percent of maximum occupancy. From the years 1981 to 1984, the average number of prisoners entering the system each year fell to 7.3 percent of maximum

150


occupancy, yet the population of the state prison system rose to almost 89 percent of maximum occupancy. Which of the following, if true, helps to resolve the apparent discrepancy? (A) The average sentence of a prisoner in the state system increased from 1981 to 1984. (B) Beginning in 1981, many of those entering the state prison system had been transferred from prisons in other states. (C) Between 1981 and 1984, the percentage of prisoners incarcerated for violent crimes increased by 26 percent. (D) In 1981, a legislative fact-finding committee proposed a revision of the state's parole and work release programs. (E) Between 1971 and 1984, the proportion of active criminals actually caught and incarcerated in the state prison system has steadily increased.

10. A social worker surveyed 200 women who recently had given birth to their first child. Half of these women had chosen to give birth in a hospital or obstetric clinic; the other half had chosen to give birth at home under the care of certified midwives. Of the 100 births that occurred at home, only five had presented any substantial complications, whereas 17 of the hospital births had required extra attention because of complications during delivery. The social worker concluded from this survey that the home is actually a safer environment in which to give birth than is a hospital or clinic. Which of the following, if true, most seriously calls the social worker's conclusion above into question? (A) All of the women in the study who were diagnosed as having a high possibility of delivery complications elected to give birth in a hospital. (B) Many obstetricians discourage their patients from giving birth in their own homes. (C) Women who give birth in their own homes tend to experience less stress during labor and delivery than do those who deliver in hospitals. (D) Women who give birth in hospitals and clinics often have shorter periods of labor than do those who give birth at home. (E) Pregnant doctors prefer giving birth in a hospital.

(K) Set 13 Questions 1-2 are based on the following: Plant manager: We could greatly reduce the amount of sulfur dioxide our copper-smelting plant releases into the atmosphere by using a new process. The new process requires replacing our open furnaces with closed ones and moving the copper from one furnace to the next in solid, not molten, form. However, not only is the new equipment expensive to buy and install, but the new process also costs more to run than the current process, because the copper must be reheated after it has cooled. So overall, adopting the new process will cost much but bring the company no profit. Supervisor: I agree with your overall conclusion, but disagree about one point you make, since the latest closed furnaces are extremely fuel-efficient.

151


1.

The point about which the supervisor expresses disagreement with the plant manager is

(A) whether the new copper-smelting process releases less sulfur dioxide gas into the atmosphere than the current process (B) whether the new copper-smelting process is more expensive to run than the current process (C) whether the new process should be adopted in the copper-smelting plant (D) whether closed copper-smelting furnaces are more fuel-efficient than open furnaces (E) whether cooling and reheating the copper will cost more than moving it in molten form

2.

The plant manager's argument is most vulnerable to criticism on which one of the following grounds?

(A) The overall conclusion is about a net effect but is based solely on evidence about only some of the factors that contribute to the effect. (B) The support for the overall conclusion is the authority of the plant manager rather than any independently verifiable evidence. (C) The overall conclusion reached merely repeats the evidence offered. (D) Evidence that is taken to be only probably true is used as the basis for a claim that something is definitely true. (E) Facts that are not directly relevant to the argument are treated as if they supported the overall conclusion.

3. Studies of trauma victims suggest that shock brought on by violent or life threatening situations causes damage to the hippocampi, structures in the brain that play a crucial role in learning and memory. Researchers found that in combat veterans suffering from post-traumatic stress symptoms, which include nightmares and vivid flashbacks, the hippocampi were eight percent smaller in volume than in combat veterans who suffered no such symptoms. The researchers concluded that the hippocampi had lost cell mass as a result of trauma. Which of the following, if true, would most seriously weaken the researchers' conclusion drawn above? (A) In another study, subjects who had experienced the death of a close relative were found to have no reduction in the volume of their hippocampi when compared to those whose close relatives were all still living. (B) In the study, the traumatized veterans were compared with other veterans of similar background, body size, and other characteristics that might have a bearing on brain size. (C) Some individuals are born with hippocampi whose volume is smaller than average, and this reduced volume makes them more susceptible to post-traumatic stress symptoms. (D) Combat veterans who experience post-traumatic stress symptoms perform significantly worse on tests of verbal memory compared with veterans who surfer no such symptoms. (E) Further study revealed that veterans who had seen more intense combat and had more severe post-traumatic symptoms exhibited even greater reduction in the volume of their hippocampi.

152


4. A certain laboratory is studying the incidence of fatal liver damage in rats. Sixty-five percent of all rats whose environments exposed them to low levels of the toxin sulfur dioxide died of liver disorder. Ninety percent of all rats who died of liver disorder, however, were not exposed to any environmental toxins. Which of the following would provide a feasible explanation for the statistics above? (A) (B) (C) (D) (E)

Environmental and nonenvironmental causes of liver disease in rats are mutually exclusive. There is only one cause of fatal liver disease in rats. Environmental toxins are not particularly dangerous to the livers of rats. Only a small portion of the entire group of rats studied was exposed to environmental sulfur dioxide. Most rats will not suffer from exposure to low levels of sulfur dioxide.

Questions 5-6 are based on following: The incidence of suicide in the country of Travonia has increased dramatically in recent years, as evidenced by the fact that since the introduction of several nonprescription brands of sleeping pills, the number of deaths from overdoses alone has nearly doubled. However, certain types of suicides have not increased in number during this period. It is true that elderly suicides have seen a greater than 70 percent increase, but teen suicides now account for only 30 percent of all suicides in the country. This is a significant decrease over 1985, when teen cases represented 65 percent of all country-wide suicides. 5. The argument above is most vulnerable to criticism on the grounds that it does which of the following? (A) It discounts the possibility of suicides occurring in groups other than the elderly and teenagers. (B) It takes for granted that the introduction of non-prescription sleeping pills has had the same effect on two different demographic groups. (C) It assumes that a decrease in the percentage of teen suicides necessarily signifies a decrease in the number of teen suicides. (D) It overlooks the possibility that the total number of deaths in Travonia has increased since 1985. (E) It relies on evidence that contradicts its conclusion.

6. The assertion that suicides are increasing in Travonia is most justified if which of the following is assumed? (A) The elderly suffered the greatest number of overdoses from the non-prescription sleeping pills. (B) Overdosing on sleeping pills was not the most pervasive method of suicide in Travonia ten years ago. (C) The number of deaths from natural causes in Travonia has decreased in recent years. (D) The majority of deaths resulting from overdosing on non-prescription sleeping pills were not accidental. (E) Travonia's suicide rate is higher than the world-wide average suicide rate.

153


7. An international study recently examined the effects of second-hand smoke on health. Surprisingly, although the dosages of harmful chemicals from secondhand smoke are so small that their effect should be negligible, the study found that nonsmoking spouses of smokers displayed an incidence of heart disease that was significantly greater than that of nonsmokers who were not as regularly exposed to second-hand smoke. Each of the following, if true, could contribute to an explanation of the unexpectedly high incidence of heart disease in smokers' spouses EXCEPT: (A) A disproportionately high number of people married to smokers are among the older segment of the married population, a group that inherently has a higher-than-average risk of heart disease. (B) On average, more alcohol and coffee, both of which have been linked to heart disease, are consumed in the homes of smokers than in the homes of nonsmokers. (C) A disproportionately high number of smokers are married to other smokers, and the risk of heart disease increases in proportion to the number of smokers living in a household. (D) Smokers generally tend to live in higher-stress environments than do non-smokers, and stress is a factor associated with above average incidence of heart disease. (E) A disproportionately high number of smokers live in areas with a high level of industrial pollutants, which have been shown to be a factor in increased risk of heart disease.

8. A local department store hires college students for one month every spring to audit its unsold inventory. It costs the department store 20 percent less to pay wages to students than it would cost to hire outside auditors from a temporary service. Even after factoring in the costs of training and insuring the students against work-related injury, the department store spends less money by hiring the student auditors than it would by hiring auditors from the temporary service The statements above, if true, best support which of the following assertions? (A) The amount spent on insurance for college-student auditors is more than 20 percent of the cost of paying the college students' basic wages. (B) It takes 20 percent less time for the college students to audit the unsold inventory than it does for the outside auditors. (C) The department store pays its college-student auditors 20 percent less than the temporary service pays its auditors. (D) By hiring college students, the department store will cause 20 percent of the auditors at the temporary service to lose their jobs. (E) The cost of training its own college-student auditors is less than 20 percent of the cost of hiring auditors from the temporary service.

9. At many colleges today, regulations have been imposed that forbid the use in speech or print of language that “offends” or “insults” the members of any group, especially women and racial, ethnic, and religious minorities. Although these regulations are defended in the name of “democracy,” they restrict freedom of speech and the press in a way that opposes the true spirit of democracy.

154


The argument above attempts to prove its case primarily by (A) (B) (C) (D) (E)

impugning the credentials of an opponent providing examples that support a theoretical principle taking advantage of inconsistencies in the definition of “democracy� revealing a contradiction in an opposing point of view appealing to the patriotic feelings of its audience

10. (OG) A recent report determined that although only three percent of drivers on Maryland highways equipped their vehicles with radar detectors, thirty-three percent of all vehicles ticketed for exceeding the speed limit were equipped with them. Clearly, drivers who equip their vehicles with radar detectors are more likely to exceed the speed limit regularly than are drivers who do not. The conclusion drawn above depends on which of the following assumptions? (A) Drivers who equip their vehicles with radar detectors are less likely to be ticketed for exceeding the speed limit than are drivers who do not. (B) Drivers who are ticketed for exceeding the speed limit are more likely to exceed the speed limit regularly than are drivers who are not ticketed. (C) The number of vehicles that were ticketed for exceeding the speed limit was greater than the number of vehicles that were equipped with radar detectors. (D) Many of the vehicles that were ticketed for exceeding the speed limit were ticketed more than once in the time period covered by the report. (E) Drivers on Maryland highways exceeded the speed limit more often than did drivers on other state highways not covered in the report.

(K) Set 14 1. The U.S. census is not perfect: thousands of Americans probably go uncounted. However, the basic statistical portrait of the nation painted by the census is accurate. Certainly some of the poor go uncounted, particularly the homeless; but some of the rich go uncounted as well, because they are often abroad or traveling between one residence and another. Which of the following is an assumption on which the argument above depends? (A) Both the rich and the poor have personal and economic reasons to avoid being counted by the census. (B) All Americans may reasonably be classified as either poor or rich. (C) The percentage of poor Americans uncounted by the census is close to the percentage of rich Americans uncounted. (D) The number of homeless Americans is approximately equal to the number of rich Americans (E) The primary purpose of the census is to analyze the economic status of the American population

155


2. A wave of incidents of unusual violence, from murder to acts of self-destruction, plagued the small medieval town for a period of five years, nearly wiping out the population. At the same time, there was an unusual shift in the area's weather pattern. Rainfall was so heavy and continuous that the wheat crop probably fell prey to the ergot fungus. When eaten, grain thus affected can cause ergotism, a disease associated with hallucinations and other disturbing psychological side effects. In the end we can conclude that the violence was the result of freakish weather conditions. Which of the following is the most effective rebuttal to the contention made above? (A) It is based upon a series of plausible suppositions rather than upon contemporary evidence. (B) No clear distinction is drawn between cause and effect. (C) Explanations of historical events cannot be convincing when too great a role is assigned to chance or the irrational. (D) The author makes no distinction between probable occurrence and actual occurrence. (E) Such crucial terms as "unusual violence" are not adequately defined in regard to the specific historical event.

3. According to some sports historians, professional tennis players develop unique playing styles that result from a combination of the peculiarities of each player's physical attributes and the influence of coaches during their early adaptation to the game. But when the increase in strength and endurance of modem players is discounted, it becomes readily apparent that the playing styles of the current crop of professional tennis players are no different from the styles of players from previous generations. Clearly, there is a universally efficient tennis style to which all professional tennis players conform. The argument above is most weakened by which of the following statements? (A) The differences in physical attributes among tennis players are even more pronounced than the sports historians believe. (B) Few current professional tennis players are familiar with the professional tennis players of fifty years ago. (C) The increased strength of current tennis players contributes more to the development of individual playing styles than does increased endurance. (D) All of the early coaches of today's professional tennis players were professional tennis players themselves earlier in their lives. (E) Weight training and greater attention to diet are the primary factors in the increased strength and stamina of the current generation of professional tennis players.

4. A political candidate committed to the principal tenets of a political party may not always explain the implications of his or her party commitment to the voters in full detail. Adele Richardson, for example, is a minor-party candidate in contention for a seat on the school board. She is not likely to inform conservative voters in her district that the national leadership of her party has recently recommended that school curricula be more closely monitored by agencies of the federal government. Each of the following is assumed in the argument above EXCEPT:

156


(A) A political candidate is likely to be more interested in winning an election than in proselytizing the electorate. (B) The candidate of any party is likely to support the policy decisions made by the national leadership. (C) All candidates for such community positions as membership on the school board must have commitments to national parties. (D) Conservatives in Adele Richardson's district do not support federal intervention in decisions made by community school officials. (E) Voters in Adele Richardson's district are not fully aware of the policy statements made by the national leadership of her party.

5. Movie pirating, the illegal videotaping of a new theater release and subsequent selling of the tape on the black market, is a major concern to the film studios that produce today's mainstream movies. When pirating sales are high, individual studios whose movies are being taped and sold illegally lose a large amount of revenue from black-market viewers who would otherwise pay the full theater price. A low level of pirating sales during a specific period, however, is a fairly reliable indicator of an economic downturn in the movie industry as a whole during that period. Which of the following, if true, most helps to reconcile the discrepancy noted above? (A) The film studios that produce today's mainstream movies occasionally serve as distribution outlets for smaller budget independent films that are also susceptible to pirating. (B) Movie pirates exclusively target blockbuster hits, the existence of which is inextricably tied to the financial success of the movie industry during any given period. (C) Most movie pirates use small, handheld video cameras that are specially designed to record images in the darkened environment of a movie theater. (D) The five largest film studios take in a disproportionate amount of movie revenue compared to hundreds of smaller and independent film studios, regardless of whether pirating activity during a specific period is high or low. (E) A movie pirate who is highly active in selling movies on the black market can sometimes make a full living doing so, while a less active pirate will usually have to supplement the income generated from pirated movies.

6. Kopke: In the past ten years, most of the new clothes that I have purchased have fallen apart within a few short years. However, all of the clothes that I have purchased at vintage clothing shops are still in excellent condition, despite the fact that they were all over thirty years old at the time that I bought them. Clearly, clothes are not manufactured as well today as they were when those vintage clothes were made. Which of the following is a weakness in the argument above? (A) It fails to demonstrate that the clothes manufactured thirty years ago were of higher quality than clothes of all other eras. (B) It neglects the possibility that the clothes of thirty years ago, when prices are adjusted for inflation, cost more than clothes manufactured today. (C) It confuses the number of clothing items sold with the proportion of those items that are no

157


longer useful. (D) It does not explain why clothing manufacturing standards have fallen over time. (E) It fails to take into account clothes made over thirty years ago that are no longer fit for sale.

7. Sarrin monks practice the Pran meditation technique only when extremely damaging weather conditions confront the farming villages surrounding Sarrin monasteries. Pran meditation is a more highly disciplined form of the ritual meditation that the monks practice daily, and involves unique practices such as isolation and fasting. Which of the statements below does NOT follow logically from the passage above? (A) Some meditation practices are less disciplined than Pran meditation. (B) Pran meditation among Sarrin monks does not take place according to a precisely regulated schedule. (C) The ritual meditation that a typical Sarrin monk practices daily does not take place in an atmosphere of isolation. (D) Monks practice some types of meditation in response to threats faced by the local population. (E) The ritual meditation that Sarrin monks practice daily is largely undisciplined.

8. The earth’s resources are being depleted much too fast. To correct this, the United States must keep its resource consumption at present levels for many years to come. The argument above depends on which of the following assumptions? (A) (B) (C) (D) (E)

Per capita resource consumption in the United States is at an all-time high. The United States wastes resources. The United States uses more resources than any other country. The United States imports most of the resources it uses. Curbing U.S. resource consumption will significantly retard world resource depletion.

9. For years, scientists have believed that a certain hormone produced by the human liver was triggered by Enzyme U, which is released in the pancreas. Recently, however, researchers in Belgium discovered that Enzyme U is always preceded by the release of Enzyme W in the brain. Based on this, these researchers hypothesize that Enzyme W, not Enzyme U, triggers the production of the liver hormone. If a second research project were set up to verify the findings of the Belgian researchers, which of the following test results would most seriously weaken their hypothesis? (A) Enzyme W is released, but not followed by the release of Enzyme U, although the liver hormone is produced. (B) Enzyme U is released, but neither preceded by the release of Enzyme W, nor followed by the production of the liver hormone. (C) Neither Enzyme W nor Enzyme U is released and the liver hormone is not produced.

158


(D) Enzyme W is released, followed by the release of Enzyme U and the production of the liver hormone. (E) Enzyme U is released and followed by the production of the liver hormone, although Enzyme W is not released.

10. In the years since the city of London imposed strict air-pollution regulations on local industry, the number of bird species seen in and around London has increased dramatically. Similar air-pollution rules should be imposed in other major cities. Each of the following is an assumption made in the argument above EXCEPT: (A) In most major cities, air-pollution problems are caused almost entirely by local industry. (B) Air-pollution regulations on industry have a significant impact on the quality of the air. (C) The air-pollution problems of other major cities are basically similar to those once suffered by London. (D) An increase in the number of bird species in and around a city is desirable. (E) The increased sightings of bird species in and around London reflect an actual increase in the number of species in the area.

(OG) Set 15 1. No nation can long survive unless its people are united by a common tongue. For proof, we need only consider Canada, which is being torn asunder by conflicts between French-speaking Quebec and the other provinces, which are dominated by English speakers. Which of the following, if true, most effectively challenges the author’s conclusion? (A) Conflicts over language have led to violent clashes between the Basque-speaking minority in Spain and the Spanish-speaking majority. (B) Proposals to declare English the official language of the United States have met with resistance from members of Hispanic and other minority groups. (C) Economic and political differences, along with linguistic ones, have contributed to the provincial conflicts in Canada. (D) The public of India, in existence since 1948, has a population that speaks hundreds of different, though related, languages. (E) Switzerland has survived for nearly a thousand years as a home for speakers of three different languages.

2. The program to control the entry of illegal drugs into the country was a failure in 1987. If the program had been successful, the wholesale price of most illegal drugs would not have dropped substantially in 1987. The argument in the passage depends on which of the following assumptions? (A) The supply of illegal drugs dropped substantially in 1987.

159


(B)

The price paid for most illegal drugs by the average consumer did not drop substantially in 1987. (C) Domestic production of illegal drugs increased at a higher rate than did the entry of such drugs into the country. (D) The wholesale price of a few illegal drugs increased substantially in 1987. (E) A drop in demand for most illegal drugs in 1987 was not the sole cause of the drop in their wholesale price.

3. The fewer restrictions there are on the advertising of legal services, the more lawyers there are who advertise their services, and the lawyers who advertise a specific service usually charge less for that service than lawyers who do not advertise. Therefore, if the state removes any of its current restrictions, such as the one against advertisements that do not specify fee arrangements, overall consumer legal costs will be lower than if the state retains its current restrictions. If the statements above are true, which of the following must be true? (A) Some lawyers who now advertise will charge more for specific services if they do not have to specify fee arrangements in the advertisements. (B) More consumers will use legal services if there are fewer restrictions on the advertising of legal services. (C) If the restriction against advertisements that do not specify fee arrangements is removed, more lawyers will advertise their services. (D) If more lawyers advertise lower prices for specific services, some lawyers who do not advertise will also charge less than they currently charge for those services. (E) If the only restrictions on the advertising of legal services were those that apply to every type of advertising, most lawyers would advertise their services.

4. Oxygen-18 is a heavier-than-normal isotope of oxygen. In a rain cloud, water molecules containing oxygen-18 are rarer than water molecules containing normal oxygen. But in rainfall, a higher proportion of all water molecules containing oxygen-18 than of all water molecules containing ordinary oxygen descends to earth. Consequently, scientists were surprised when measurements along the entire route of rain clouds’ passage from above the Atlantic Ocean, the site of their original formation, across the Amazon forests, where it rains almost daily, showed that the oxygen-18 content of each of the clouds remained fairly constant. Which one of the following statements, if true, best helps to resolve the conflict between scientists’ expectations, based on the known behavior of oxygen-18, and the result of their measurements of the rain clouds’ oxygen-18 content? (A) Rain clouds above tropical forests are poorer in oxygen-18 than rain clouds above unforested regions. (B) Like the oceans, tropical rain forests can create or replenish rain clouds in the atmosphere above them. (C) The amount of rainfall over the Amazon rain forests is exactly the same as the amount of rain originally collected in the clouds formed above the Atlantic Ocean. (D) The amount of rain recycled back into the atmosphere from the leaves of forest vegetation is

160


(E)

exactly the same as the amount of rain in river runoffs that is not recycled into the atmosphere. Oxygen-18 is not a good indicator of the effect of tropical rain forests on the atmosphere above them.

5. Vitamin XYZ has long been a favorite among health food enthusiasts. In a recent large study, those who took large amounts of vitamin XYZ daily for two years showed on average a 40 percent lower risk of heart disease than did members of a control groups. Researchers corrected for differences in relevant health habits, such as diet. Which one of the following inference is most supported by the passage? (A) Taking large amount of vitamins is probably worth risking the side effects. (B) Those who take large doses of vitamin XYZ daily for the next two years will exhibit on average an increase in the likelihood of avoiding heart disease. (C) Li, who has taken large amounts of vitamin XYZ daily for the past two years, has a 40 percent lower risk. (D) Taking large amounts of vitamin XYZ daily over the course of one’s adult life should be recommended to most adults. (E) Health food enthusiasts are probably correct in believing that large daily doses of multiple vitamins promote good health.

6. It is even more important that we criticize democracies that have committed human rights violations than that we criticize dictatorships that have committed more violent human rights offenses. Human rights violations are always inexcusable, but those committed by governments that represent the will of the people are even more reprehensible than those committed by dictators. Further, our criticism is more likely to have an effect on the former than on the later. Which one of the following is a proper inference from the passage? (A) All governments commit same inexcusable and reprehensible acts. (B) Some human rights violations are more reprehensible than other, more violent human rights violations. (C) Criticism of human rights violations is certain to have no effect on a dictatorship. (D) Human rights violations are more likely to occur in democracies than in dictatorship. (E) Those who do represent the will of the people are less likely to be moved by criticism than are those who merely claim to represent the will of the people.

7. An easy willingness to tell funny stories or jokes about oneself is the surest mark of supreme self-confidence. This willingness, often not acquired until late in life, is even more revealing than is good-natured acquiescence in having others poke fun at one. Which one of the following inference is most supported by the statements above? (A) A person who lacks self-confidence will enjoy neither telling nor hearing funny stories about

161


himself or herself. (B) People with high self-confidence do not tell funny stories or jokes about others. (C) Highly self-confident people tell funny stories and jokes in order to let their audience know that they are self-confident. (D) Most people would rather tell a funny story or joke than listen to one being told. (E) Telling funny stories or jokes about people in their presence is a way of expressing one’s respect for them.

8. Industrialists from the country Distopia were accused of promoting the Distopian intervention in the Arcadian civil war merely to insure that the industrialists’ facilities in Arcadia made substantial profits during the war. Yet this cannot be the motive since, as the Distopians foresaw, Distopia’s federal expenses for the intervention were eight billion dollars, whereas, during the war, profits from the Distopian industrialists’ facilities in Arcadia totaled only four billion dollars. Which of the following, if true, exposes a serious flaw in the argument made in the second sentence above? (A) During the Arcadian war, many Distopian industrialists with facilities located in Arcadia experienced a significant rise in productivity in their facilities located in Distopia. (B) The largest proportion of Distopia’s federal expenses is borne by those who receive no significant industrial profits. (C) Most Distopian industrialists’ facilities located in Arcadia are expected to maintain the level of profits they achieved during the war. (D) Distopian industrialists’ facilities in Arcadia made substantial profits before the events that triggered the civil war. (E) Many Distopians expressed concern over the suffering that Arcadians underwent during the civil war.

9. With Proposition 13, if you bought your house 11 years ago for $75,000, your property tax would be approximately $914 a year (1 percent of $75,000 increased by 2 percent each year for 11 years); and if your neighbor bought an identical house next door to you for $200,000 this year, his tax would be $2,000 (1 percent of $200,000). Without Proposition 13, both you and your neighbor would pay $6,000 a year in property taxes (3 percent of $200,000). Which of the following is the conclusion for which the author most likely is arguing in the passage above? (A) Proposition 13 is unconstitutional because it imposes an unequal tax on properties of equal value. (B) If Proposition 13 is repealed, every homeowner is likely to experience a substantial increase in property taxes. (C) By preventing inflation from driving up property values, Proposition 13 has saved homeowners thousands of dollars in property taxes. (D) If Proposition 13 is not repealed, identical properties will continue to be taxed at different rates. (E) Proposition 13 has benefited some homeowners more than others.

162


10. In metropolitan areas, almost 60 percent of all fires are set by children, while in rural areas about 40 percent are. A psychological survey discovered that all children who play with fire believe that there will be no consequences if their parents catch them doing it. Which one of the following inferences can be most reliably drawn from the passage above? (A) Most children who believe there will be no consequences if they are discovered playing with fire do play with fire. (B) Parents who discover their children playing with fire will prevent those children from playing with fire in the future. (C) If parents have successfully instilled in their children the belief that there will be consequences if they are caught playing with fire, these children have not been among those playing with fire. (D) Children who play with fire attach no sense of right or wrong to this action. (E) Most children who do not play with fire believe there will be consequences if their parents discover them playing with fire.

163


Critical Reasoning ANSWER KEY

HP/Qs# HP1 HP2 HP3 HP4 HP5 HP6 HP7

1 D D B D C C E

2 D C B C A D C

3 C D E D E

4 C E E D C

C

C

C

Set/Qs# Set 1 Set 2 Set 3 Set 4 Set 5 Set 6 Set 7 Set 8 Set 9 Set 10 Set 11 Set 12 Set 13 Set 14 Set 15

1 C A E E D B D C B E E A B C E

2 E D D C B C E C E E C A A D E

3 B B C C D E C A C D D C C D C

4 C C B C D C D C E C B D D C B

5 A D A A B B E E D C E A C B B

164

5

6

7

D

D

D

6 A E C E E D B E D E D C D E B

7 C E D A B A E B E C B B C E A

8

9

10

8 E D B C D A A E D D D E E E B

9 A C E A D D A C B A D A D E B

10 C B B E C B C D B E A A B A C


Part III Sentence Correction

SC task is a sentence that has some of its portion underlined (sometimes, 100% are underlined). The first answer always repeats the original underlined portion – that is, if you choose the first answer, you decide against changing anything. The other 4 answers represent variations of the underlined portion in 3 criteria: 1. Standard Written English grammar rules 2. Preferences 3. Meaning Commonly tested grammar topics: I.

Sentence Structure

Parts of sentence: A correct English clause must contain a subject (a person or thing that acts in the clause) and a verb (a predicate, an action undertaken by the subject). Tom is reading. Some clauses also contain objects (people or things at which the action of the subject is targeted). Tom is reading a book. And some clauses contain modifiers (details about the person or thing, or about his or her action). Tom, a careful student, is quickly reading a very interesting book, making notes every two minutes. Parts of sentence practice: (identify subject, verb, object, modifiers) The new office manager was forced to discipline several employees during his first week at work. The playful puppy, a terrier and shepherd mix, pounced on the flowers and chased the birds. Even more important is the proper use of power in the department. Even though they faced the obvious obstacles, veterinarians and oncologists administered chemotherapy to Bubba, a 154-pound Queensland grouper, at the Shedd Aquarium in Chicago. Because the school board concentrated on English and mathematics as the fundamental cores of education, rather than supplementing the curriculum with a music program, many students from East Lake High were robbed of an opportunity to apply for the Lake County Scholarship, as it was awarded to those who excelled in orchestra. A sentence can contain more than 1 subject-verb pairs. Such sentences can contain either Independent clauses, as in: We must not complain about the problem, but they still should help us put it right. Or Independent and Dependent clauses, as in: The picking of the fruit, for which they receive no money, takes about a week. Conjunction between 2 Independent clauses:

165


1) Semicolon: I looked into the window; it started raining. 2) Comma + 1 of 7 coordinate conjunctions: For, And, Nor, But, Or, Yet, So I looked into the window, and it started raining. Mistakes: no coordinate conjunction: I looked into the window, it started raining (run-on sentence) no comma: I looked into the window and it started raining (sentences not separated) Conjunction between Independent and Dependent clauses: Relative pronouns: that, which, who, whom, whose Subordinating conjunctions: after, before, although, if, unless, as, inasmuch, until, when, where, how, as long as (and the like), since, while, because, so that, than, that, though !NOTE! THAT vs. WHICH In most of the cases WHICH refers to the last mentioned noun, whereas THAT may refer to any noun in the sentence. Pencils on the table, which is clean, are mine. (WHICH refers to the table) Pencils on the table that are mine can be useful. (THAT refers to pencils) If a dependent clause refers to 1 noun, it agrees with it in number; “that” and “which” have all the same properties here: A box, which is on the table. – Matches, which are on the table. A box that is on the table. – Matches that are on the table. But if a dependent clause refers to a prepositional phrase (Noun-Preposition-Noun), the properties of “which” and “that” differ: - A box of matches, which ARE on the table. “Which” always refers to the closest noun. - A box of matches that IS/ARE on the table. “That” can refer both to the 1st and to the 2nd noun in a prepositional phrase. Both variants are grammatically correct.

166


Sentence Structure practice:

GoGMAT Problem

167


GoGMAT Problem Explanation

Instructions: chose one answer out of five given in each task according to the rules you’ve read in the chapter. You should spend no more than 1,5 minutes per question.

1. The set of propositions which was discussed by the panel have been published in the society journal. (A) (B) (C) (D) (E)

which was discussed by the panel have which were discussed by the panel have that was discussed by the panel has which were discussed by the panel has which was discussed, by the panel, has

2. In The Matrix, Neo can choose either the blue pill, which will allow him to forget all that he has learned and return to life as he knew it, and the red pill, which will keep him in reality. (A) and return to life as he knew it, and the red pill, which will keep him in reality (B) and return to life as he knew it, or the red pill, which will keep him in reality (C) or return to life as he knew it, and the red pill, which will keep him in reality

168


(D) but also return to life as he knew it, and the red pill, which will keep him in reality (E) and return to life as he knew it, for the red pill, which will keep him in reality 3. It was extremely discouraging to learn that the bank, with whom I have had a savings account for over twenty years, could not open a checking account for me because I did not have proper identification. (A) (B) (C) (D) (E)

that the bank, with whom I have had a savings account for over twenty years that the bank, with which I have had a savings account for over twenty years that the bank, with who I have had a savings account for over twenty years that the bank, with that I have had a savings account for over twenty years that the bank, where I have had a savings account for over twenty years

4. The schedule for the skate park’s construction, which will serve skateboarders, rollerbladers, and bikers, was detailed at the city council meeting. (A) (B) (C) (D) (E)

The schedule for the skate park’s construction, which will serve The schedule for the skate park’s construction, that which will serve The schedule for the skate park’s construction, planned to serve The skate park’s construction schedule, which will serve The construction schedule for the skate park, which will serve Subject-Verb agreement (GoGMAT Session 1)

Subject and Verb of a clause must agree in number and person. If Subject and Verb disagree – it is always the verb that uses an incorrect form and must be changed. Common mistakes: 1) Subject is hard to find (subject and verb are set far apart, subject is an element of a prepositional phrase) 2) Subject is of a confusing type Confusing subject types: * Latin words Singular Plural Crisis Crises Datum Data Phenomenon Phenomena Criterion Criteria Correct: The latest data show a wide variation in numbers. Correct: A new phenomenon is noticeable everywhere on the market. * Group (collective) nouns – team, group, family, company, etc. When such a group is viewed as a single unit, the collective noun is used with a singular verb and singular pronouns. Correct: The committee has reached its decision. But when the focus is on the individual members of the group, a plural verb and plural pronouns are used. Correct: The committee have been arguing all morning. This is the same as saying: The people in the committee have been arguing all morning.

169


* Some nouns like Means, News, Ethics, Mathematics, Physics, Measles, Politics, Statistics seem to be plural, but they are singular and so take singular verbs. Correct: The news these days is always boring. Measles is a deadly disease. * Some nouns like Species, Series, Deer, Fish, Sheep have the same singular and plural form. They take singular or plural verb depending on the meaning. Correct: This species of monkey lives only in India. There are many species of monkeys. * Some prepositions have the meaning of “and”, a conjunction. They definitely cannot function to add 2 subjects requiring a plural verb. In sentences with such phrases we will always agree the verb with subject before the phrase. Here are some of them: With Along with Together with In addition to Accompanied by As well as Correct: Mark and Nina are going out. Mark together with Nina is going out. * Pronouns from the following table always take a singular verb: Everybody Everyone Everything

Anybody Anyone Anything

Somebody Someone Something

Nobody No one Nothing

Either Each Neither Every

Correct: Neither of us has ever seen anything more interesting before. Correct: Everybody knows the difference between these two performances. Correct: Each of the three cats eats much meat. Three cats each eat much meat. Note how the number of the verb varies in the last example. * Adjectives describing a certain quantity, such as Several, Many, Both, Few always take a plural verb. Correct: Few people have ever entered this garden. * With words that indicate portions— percent, fraction, part, majority, some, all, none, remainder, etc.— it is the noun that determines whether to use a singular or plural verb. If the noun is singular, use a singular verb. If the noun is plural, use a plural verb. Correct: Fifty percent of the chocolate has disappeared. Correct: Fifty percent of the chocolates have disappeared. Correct: One-third of the city is unemployed. Correct: One-third of the people are unemployed. * ”A number of” vs. “the number of” A number of books ARE interesting. – Here the expression “a number of” is an idiomatic way of saying “some”. So, read this sentence this way: Some books ARE interesting. The number of books IS increasing. – Here we definitely speak about the number itself, and it is a

170


singular noun, requiring a singular verb. * Subjects that are not nouns or pronouns. An entire clause can serve as a subject of a sentence. When used as a subject, a clause always takes a singular verb. Correct: Whether the economy will improve in the near future is a matter of great concern. Correct: That every summer vacation ends is well-known to all school-children. Infinitives and gerunds can be used as subjects. Remember that they are singular subjects. Correct: To analyze is mental. Correct: Rollerblading is dangerous. * Neither…nor, either…or, not only… but also When two distinct words or phrases are joined by the correlatives either … or, neither … nor, not only … but also, the number of the verb is determined by the second word or phrase in the pair. Correct: Either his parents or he is bringing the pie. Correct: Either he or his parents are bringing the pie. Subject-Verb Agreement practice:

GoGMAT Problem

171


GoGMAT Problem Explanation

Instructions: chose one answer out of five given in each task according to the rules you’ve read in the chapter. You should spend no more than 1,5 minutes per question.

1. According to a study by the American Education Bureau, the average number of calendar days in a school year across the fifty states have increased by nearly twenty five percent since 1950. (A) (B) (C) (D) (E)

the average number of calendar days in a school year across the fifty states have increased across the fifty states, the average number of calendar days in a school year have increased the average number of calendar days, across the fifty states, in a school year have increased the average number of calendar days in a school year across the fifty states has increased the average numbers of calendar days in a school year across the fifty states has increased

2. Information for travelers, such as road maps, hotel directions, or rest area locations, are provided free of charge from the automotive club, long known for its roadside assistance plan.

172


(A) (B) (C) (D) (E)

are provided free of charge from the automotive club, long known for its is provided free of charge from the automotive club, long known for its are provided free of charge from the automotive club, long known for their is provided free of charge from the automotive club, long known for their is to be provided free of charge from the automotive club, long known for their

3. The reasons for the budget cuts, of which there is dozens, will be revealed at tonight’s city council meeting by the mayor and the council members. (A) (B) (C) (D) (E)

The reasons for the budget cuts, of which there is dozens The reason for the budget cuts, of which there is dozens The reasons for the budget cut, of which there is dozens The reason for the budget cuts, of which there are dozens The reasons for the budget cuts, of which there are dozens

4. During the debate in October 1992, each of the candidates – George Bush, Bill Clinton, and Ross Perot – was allowed to ask one question of each of his opponents. (A) (B) (C) (D) (E)

was allowed to ask one question of each of his opponents were allowed to ask one question of each of his opponents was allowed to ask one question of every one of opponents were allowed to ask one question of each of their opponents were allowed one question of each of his opponents

5. According to the Constitution, neither those Senators under 35 years of age nor the representative born outside of the United States are able to run for the position of President. (A) nor the representative born outside of the United States are able to run for the position of President (B) nor the representative born outside of the United States is running for the position of President (C) nor the representative born outside of the United States are running for the position of President (D) nor the representative born outside of the United States is not able to run for the position of President (E) nor the representative born outside of the United States is able to run for the position of President

6. Once each vote and piece of paper are burned, the cardinals in the conclave are given time to reflect before casting their next vote. (A) (B) (C) (D) (E)

Once each vote and piece of paper are burned Once each vote and piece of paper is burned Once each vote and piece of paper is burning Once each vote and pieces of paper are burned Once each votes and piece of paper are burned

173


7. Neither the earthquake or the subsequent fire was able to destroy the spirit of the city dwellers. (A) (B) (C) (D) (E)

8. (A) (B) (C) (D) (E)

or the subsequent fire was nor the subsequent fire were or the subsequent fire were nor the subsequent fire was or the fire that occurred subsequently were

Fame as well as fortune were his goals in life. Fame as well as fortune were his goals Fame as well as fortune was his goals Fame as well as fortune were his goal Fame and fortune were his goals Fame also fortune were his goals

9. The first of a number of receptions and testimonial dinners for the departing school superintendent have been scheduled, with more events still in the planning stage. (A) (B) (C) (D) (E) 10. (A) (B) (C) (D) (E)

have been scheduled, with more events still have been scheduled, and with more events still has been scheduled, and with more events still has been scheduled, with more events still have been scheduled, and there is still more events Her brother along with her parents insist that she remain in school. insist insists are insisting were insisting have insisted

11. On the African continent, the incidence of vitamin deficiencies correlates positively with the level of solar radiation. (A) (B) (C) (D) (E)

12.

deficiencies correlates positively with deficiencies correlate positively with deficiencies, correlate positively with, deficiencies correlate positively to deficiencies correlates positively to

Neither the judge nor I am ready to announce who the winner is.

(A) Neither the judge nor I am ready to announce who the winner is. (B) Neither the judge nor I are ready to announce who the winner is.

174


(C) Neither the judge nor I are ready to announce who is the winner. (D) Neither the judge nor I am ready to announce who is the winner. (E) Neither I or the judge are ready to announce who is the winner. Home Practice 1 1. Excavators at the Indus Valley site of Harappa in eastern Pakistan say the discovery of inscribed shards dating to circa 2800-2600 B.C. indicate their development of a Harappan writing system, the use of inscribed seals impressed into clay for marking ownership, and the standardization of weights for trade or taxation occurred many decades, if not centuries, earlier than was previously believed. (A) indicate their development of a Harappan writing system, the use of (B) indicate that the development of a Harappan writing system, using (C) indicates that their development of a Harappan writing system, using (D) indicates the development of a Harappan writing system, their use of (E) indicates that the development of a Harappan writing system, the use of 2. A surge in new home sales and a drop in weekly unemployment claims suggest that the economy might not be as weak as some analysts previously thought. (A) claims suggest that the economy might not be as weak as some analysts previously thought (B) claims suggests that the economy might not be so weak as some analysts have previously thought (C) claims suggest that the economy might not be as weak as have been previously thought by some analysts (D) claims, suggesting about the economy that it might not be so weak as previously thought by some analysts (E) claims, suggesting the economy might not be as weak as previously thought to be by some analysts 3. Quasars, at billions of light-years from Earth the most distant observable objects in the universe, believed to be the cores of galaxies in an early stage of development. (A) believed to be (B) are believed to be (C) some believe them to be (D) some believe they are (E) it is believed that they are 4.The first trenches that were cut into a 500-acre site at Tell Hamoukar, Syria, have yielded strong evidence for centrally administered complex societies in northern regions of the Middle East that were arising simultaneously with but independently of the more celebrated city-states of southern Mesopotamia, in what is now southern Iraq. (A) that were cut into a 500-acre site at Tell Hamoukar, Syria, have yielded strong evidence for centrally administered complex societies in northern regions of the Middle East that were arising simultaneously with but (B) that were cut into a 500-acre site at Tell Hamoukar, Syria, yields strong evidence that centrally administered complex societies in northern regions of the Middle East were arising simultaneously with but also

175


(C) having been cut into a 500-acre site at Tell Hamoukar, Syria, have yielded strong evidence that centrally administered complex societies in northern regions of the Middle East were arising simultaneously but (D) cut into a 500-acre site at Tell Hamoukar, Syria, yields strong evidence of centrally administered complex societies in northern regions of the Middle East arising simultaneously but also (E) cut into a 500-acre site at Tell Hamoukar, Syria, have yielded strong evidence that centrally administered complex societies in northern regions of the Middle East arose simultaneously with but 5. Fossils of the arm of a sloth found in Puerto Rico in 1991, and dated at 34 million years old, made it the earliest known mammal of the Greater Antilles Islands. (A) sloth found in Puerto Rico in 1991, and dated at 34 million years old, made it the earliest known mammal of (B) sloth, that they found in Puerto Rico in 1991, has been dated at 34 million years old, thus making it the earliest mammal known on (C) sloth that was found in Puerto Rico in 1991, was dated at 34 million years old, making this the earliest known mammal of (D) sloth, found in Puerto Rico in 1991, have been dated at 34 million years old, making the sloth the earliest known mammal on (E) sloth which, found in Puerto Rico in 1991, was dated at 34 million years old, made the sloth the earliest known mammal of 6. Often visible as smog, ozone is formed in the atmosphere from hydrocarbons and nitrogen oxides, two major pollutants emitted by automobiles, react with sunlight. (A) ozone is formed in the atmosphere from (B) ozone is formed in the atmosphere when (C) ozone is formed in the atmosphere, and when (D) ozone, formed in the' atmosphere when (E) ozone, formed in the atmosphere from 7. Cajuns speak a dialect brought to southern Louisiana by the 4,000 Acadians who migrated there in 1755; their language is basically seventeenth-century French to which has been added English, Spanish, and Italian words. (A) to which has been added English, Spanish, and Italian words (B) added to which is English, Spanish, and Italian words (C) to which English, Spanish, and Italian words have been added (D) with English, Spanish, and Italian words having been added to it (E) and, in addition, English, Spanish, and Italian words are added

II.

Modifiers (GoGMAT Session 4)

Modifier is a word or an expression that qualifies, describes, or specifies the meaning of Subject, Verb, or Object. Bookďƒ New book Run ďƒ Run quickly

176


He looked out of the window  After completing his task, he looked out of the window. I saw a beautiful girl  I saw a beautiful girl, walking down the street. Common modifier mistakes: 1.

Modifier refers to wrong subject

Wrong: Sifting the sand of a river bed, gold was discovered by prospectors in California in 1848. Correct: Sifting the sand in a river bed, prospectors discovered gold in California in 1848. Correct but less preferable: Prospectors, sifting the sand in a river bed, discovered gold in California in 1848. Wrong: Engraved, printed, and colored by hand, William Blake could make only a few copies of each of his books of poetry. Correct: William Blake could make only a few copies of each of his books of poetry, which he engraved, printed, and colored by hand. Wrong: Desiring to free his readers from superstition, the theories of Epicurus are expounded in Lucretius’s poem De rerum natura. Correct: Desiring to free his readers from superstition, Lucretius expounded the theories of Epicurus in his poem De rerum natura. 2. Modifier is misplaced: Wrong: That night they sat discussing when the cow might calve in the kitchen. Correct: That night they sat in the kitchen discussing when the cow might calve. Wrong: As a young man, the French novelist Gustave Flaubert traveled in Egypt, which was a fascinating experience. Correct: Traveling in Egypt as a young man was a fascinating experience for the French novelist Gustave Flaubert.

177


Modifier practice:

GoGMAT Problem

178


GoGMAT Problem Explanation

Instructions: chose one answer out of five given in each task according to the rules you’ve read in the chapter. You should spend no more than 1,5 minutes per question. 1. Created by the Pennsylvania Committee of Safety on July 6, 1798, the Pennsylvania Navy served America from the Revolutionary War until the formation of the U.S. Navy in 1975. (A) (B) (C) (D) (E)

the Pennsylvania Navy served America from the Revolutionary War Pennsylvania’s Navy served America from the Revolutionary War the Revolutionary War was served by the Pennsylvania Navy America was served by the Pennsylvania Navy from the Revolutionary War the Pennsylvania Navy served the Revolutionary War for American

2. Excited at the prospect of receiving the pair of Giant Pandas, it was decided by the zookeeper to start building a habitat modeled after the mountains of Tibet. (A) it was decided by the zookeeper to start building a habitat modeled after the mountains of Tibet (B) a habitat modeled after the mountains of Tibet was built by the zookeeper

179


(C) it was decided to start building a habitat modeled after the mountains of Tibet by the zookeeper (D) the zookeeper decided to start building a habitat modeled after the mountains of Tibet (E) and modeled after the mountains of Tibet, the zookeeper decided to start building a habitat

3. Many celebrities, such as Britney Spears, Tom Cruise, and Jessica Simpson, have secretly vacationed on Turtle island in Fiji, an exclusively tropical resort known for its privacy and beauty. (A) have secretly vacationed on Turtle island in Fiji, an exclusively tropical resort known for its privacy and beauty. (B) have secretly vacationed on Turtle island in Fiji, an exclusive, tropical resort known for its privacy and beauty. (C) have vacationed in secret on Turtle island in Fiji, an exclusively tropical resort known for its privacy and beauty. (D) have secretly vacationed on Turtle island in Fiji, a tropically exclusive resort known for its privacy and beauty. (E) have secretly vacationed on Turtle island in Fiji, an exclusively tropical, private, beautiful resort.

4. Having the company of one of his four brothers, 6-year-old Luke would ramble through the bayous along the Mississippi River near his home in Donaldsonville, sporting a Daisy Red Ryder air rifle. (A) (B) (C) (D) (E)

Having the company of one of his four brothers With four brothers, having the company of one Because one of his four brothers accompanied him One of his four brothers being his company Accompanied by one of his four brothers

5. Based on accounts of various researchers, Colstrom scientists have known for decades that for every kind of particle – whether the familiar electrons, neutrons or protons, or the more recently discovered quarks and neutrinos – there exists a particle that is its mirror image, identical in mass and amount of charge, but different in at least one crucial way. (A) (B) (C) (D) (E)

Based on accounts of various researchers Basing it on various researchers’ accounts With accounts of various researchers used for a basis By the accounts of various researchers they used Using accounts of various researchers

6. In addition to having more investors than DSN, the investors in Netfix are more financially sound than that in DSN, with more of them having scientific backgrounds. (A) (B) (C) (D) (E)

180

the investors in Netfix are more financially sound than that in Netfix has investors which are more financially sound than that in the investors in Netfix are more financially sound than they are in Netfix investors are more sound financially than they are in Netfix has investors who are more financially sound than those in


7. On entering the stadium, cheers greeted them as a sign of universal approval of their great achievement. (A) (B) (C) (D) (E)

On entering the stadium, cheers greeted them On entering the stadium, they were greeted by cheers While entering the stadium, cheers greeted I On entering the stadium cheers greeted them On entering the stadium: cheers greeted them

8.

Using it wisely, leisure promotes health, efficiency, and happiness.

(A) (B) (C) (D) (E) 9. (A) (B) (C) (D) (E)

Using it wisely, If used wisely, Having used it wisely, Because it is used wisely, Because of usefulness, Crossing the street, a car almost struck us. Crossing the street, a car almost struck us. A car almost struck us, crossing the street. As we crossed the street, a car almost struck us. A car, crossing the street, almost struck us. Having crossed the street, a car almost struck us.

10. Before starting a program of diet and exercise, a consultation with your physician is advisable. (A) (B) (C) (D) (E)

a consultation with your physician is advisable it is advisable to have a consultation with your physician a physician's consultation is advisable a consultation with your physician is necessary you should consult your physician

11. Familiar with the terrain from previous visits, the explorer's search for the abandoned mine site was a success. (A) (B) (C) (D) (E)

the explorer's search for the abandoned mine site was a success the success of the explorer's search for the abandoned mine site was assured the explorer succeeded in finding the abandoned mine site the search by the explorer for the abandoned mine was successful the explorer in his search for the abandoned mine site was a success

Home Practice 2 1. In 1995 Richard Stallman, a well-known critic of the patent system, testified in Patent Office hearings that, to test the system, a colleague of his had managed to win a patent for one of

181


Kirchhoff's laws, an observation about electric current first made in 1845 and now included in virtually every textbook of elementary physics. (A) (B) (C) (D) (E)

laws, an observation about electric current first made in 1845 and laws, which was an observation about electric current first made in 1845 and it is laws, namely, it was an observation about electric current first made in 1845 and laws, an observation about electric current first made in 1845, it is laws that was an observation about electric current, first made in 1845, and is

2. Seldom more than 40 feet wide and 12 feet deep, but it ran 363 miles across the rugged wilderness of upstate New York, the Erie Canal connected the Hudson River at Albany to the Great Lakes at Buffalo, providing the port of New York City with a direct water link to the heartland of the North American continent. (A) Seldom more than 40 feet wide and 12 feet deep, but it ran 363 miles across the rugged wilderness of upstate New York, the Erie Canal connected (B) Seldom more than 40 feet wide or 12 feet deep but running 363 miles across the rugged wilderness of upstate New York, the Erie Canal connected (C) It was seldom more than 40 feet wide and 12 feet deep, and ran 363 miles across the rugged wilderness of upstate New York, but the Erie Canal, connecting (D) The Erie Canal was seldom more than 40 feet wide or 12 feet deep and it ran 363 miles across the rugged wilderness of upstate New York, which connected (E) The Erie Canal, seldom more than 40 feet wide and 12 feet deep, but running 363 miles across the rugged wilderness of upstate New York, connecting 3. A pioneer journalist. Nellie Bly's exploits included circling the globe faster than Jules Verne's fictional Phileas Fogg. (A) A pioneer journalist, Nellie Bly's exploits included (B) The exploits of Nellie Bly, a pioneer journalist, included (C) Nellie Bly was a pioneer journalist including in her exploits the (D) Included in the pioneer journalist Nellie Bly's exploits are (E) The pioneer journalist's exploits of Nellie Bly included 4. By 1940, the pilot Jacqueline Cochran held seventeen official national and international speed records, and she earned them at a time when aviation was still so new for many of the planes she flew to be of dangerously experimental design. (A) and she earned them at a time when aviation was still so new for many of the planes she flew to be (B) earning them at a time that aviation was still so new for many of the planes she flew to be (C) earning these at a time where aviation was still so new that many of the planes she flew were (D) earned at a time in which aviation was still so new such that many of the planes she flew were (E) earned at a time when aviation was still so new that many of the planes she flew were 5. In 1850, Lucretia Mott published her Discourse on Women, arguing in a treatise for women to have equal political and legal rights and for changes in the married women's property laws. (A) arguing in a treatise for women to have equal political and legal rights (B) arguing in a treatise for equal political and legal rights for women

182


(C) a treatise that advocates women's equal political and legal rights (D) a treatise advocating women's equal political and legal rights (E) a treatise that argued for equal political and legal rights for women 6. In 1713, Alexander Pope began his translation of the Iliad, a work that, taking him seven years until completion, and that literary critic Samuel Johnson, Pope's contemporary, pronounced the greatest translation in any language. (A) his translation of the Iliad, a work that, taking him seven years until completion, and that literary critic Samuel Johnson, Pope's contemporary, pronounced (B) his translation of the Iliad, a work that took him seven years to complete and that literary critic Samuel Johnson, Pope's contemporary, pronounced (C) his translation of the Iliad, a work that had taken seven years to complete and that literary critic Samuel Johnson, Pope's contemporary, pronounced it as (D) translating the Iliad, a work that took seven years until completion and that literary critic Samuel Johnson, Pope's contemporary, pronounced it as (E) translating the Iliad, a work that had taken seven years to complete and literary critic Samuel Johnson, Pope's contemporary, pronounced it 7. A leading figure in the Scottish Enlightenment, Adam Smith's two major books are to democratic capitalism what Marx's Das Kapital is to socialism. (A) Adam Smith's two major books are to democratic capitalism what (B) Adam Smith's two major books are to democratic capitalism like (C) Adam Smith's two major books are to democratic capitalism just as (D) Adam Smith wrote two major books that are to democratic capitalism similar to (E) Adam Smith wrote two major books that are to democratic capitalism what 8. Rivaling the pyramids of Egypt or even the ancient cities of the Maya as an achievement, the army of terra-cotta warriors created to protect Qin Shi Huang, China's first emperor, in his afterlife is more than 2000 years old and took 700.000 artisans more than 36 years to complete. (A) the army of terra-cotta warriors created to protect Qin Shi Huang, China's first emperor, in his afterlife is more than 2,000 years old and took 700,000 artisans more than 36 years to complete (B) Qin Shi Huang, China's first emperor, was protected in his afterlife by an army of terra cotta warriors that was created more than 2,000 years ago by 700,000 artisans who took more than 36 years to complete it (C) it took 700,000 artisans more than 36 years to create an army of terra-cotta warriors more than 2,000 years ago that would protect Qin Shi Huang, China's first emperor, in his afterlife (D) more than 2,000 years ago, 700,000 artisans worked more than 36 years to create an army of terra-cotta warriors to protect Qin Shi Huang, China's first emperor, in his afterlife (E) more than 36 years were needed to complete the army of terra-cotta warriors that 700,000 artisans created 2,000 years ago to protect Qin Shi Huang, China's first emperor, in his afterlife 9. Unlike the original National Museum of Science and Technology in Italy, where the models are encased in glass or operated only by staff members, the Virtual Leonardo Project, an online version of the museum, encourages visitors to "touch" each exhibit, which thereby activates the animated functions of the piece. (A) exhibit, which thereby activates

183


(B) exhibit, in turn an activation of (C) exhibit, and it will activate (D) exhibit and thereby activate (E) exhibit which, as a result, activates 10. Defense attorneys have occasionally argued that their clients' misconduct stemmed from a reaction to something ingested, but in attributing criminal or delinquent behavior to some food allergy, the perpetrators are in effect told that they are not responsible for their actions. (A) in attributing criminal or delinquent behavior to some food allergy, (B) if criminal or delinquent behavior is attributed to an allergy to some food, (C) in attributing behavior that is criminal or delinquent to an allergy to some food, (D) if some food allergy is attributed as the cause of criminal or delinquent behavior, (E) in attributing a food, allergy as the cause of criminal or delinquent behavior,

III.

Parallel Constructions

Parallel constructions appear in a sentence as a series of equivalent elements. Three types of parallels are used: -

Sequences: A, B, and C

-

Comparisons: A is greater than B

-

Correlative conjunctions: Either A, or B; Neither A, nor B; Not only A, but also B. Elements of parallel constructions must conform to 2 criteria:

1) Uniformity *grammatical uniformity: Wrong: The donation might be used to fund the new playground or replacing the old bus. Correct: The donation might be used to fund the new playground or to replace the old bus. Wrong: Symptoms of a severe allergic reaction may include dizziness, hives or rashes, difficulty breathing, and losing consciousness. Correct: Symptoms of a severe allergic reaction may include dizziness, hives or rashes, difficulty breathing, and a loss of consciousness. Wrong: The group is asking all boat owners to dispense fuel only into approved containers, throw away all litter, and to avoid disposing of cigarette butts in the water. Correct: The group is asking all boat owners to dispense fuel only into approved containers, throw away all litter, and avoid disposing of cigarette butts in the water. Correct: The group is asking all boat owners to dispense fuel only into approved containers, to throw away all litter, and to avoid disposing of cigarette butts in the water.

184


Wrong: The Department is considering a new system that would alleviate lines for walk-in customers renewing their licenses and if employees are overwhelmed they would be assisted. Correct: The Department is considering a new system that would alleviate lines for walk-in customers renewing their licenses and assist employees who are overwhelmed. Wrong: The first task to accomplish when writing an application essay is formulating an outline that lists all the things you want to say. Correct: The first task to accomplish when writing an application essay is to formulate an outline that lists all the things you want to say. *functional uniformity: Correct: Doctors agree that their patients should take medication within a strictly monitored regimen rather than at random times during the day. See how the parallel modifiers of action are completely different in construction but perform a similar function in the sentence, and are thus parallel. *logical uniformity: Wrong: Like a black bear I once saw in the Buenos Aires Zoo, the Central Park Zoo polar bear's personality strikes me as being sadly neurotic. Correct: Like that of a black bear I once saw in the Buenos Aires Zoo, the Central Park Zoo polar bear's personality strikes me as being sadly neurotic. Wrong: The article questioned the popularity of Jazz compared to classical music. Correct: The article questioned the popularity of Jazz compared to that of classical music. Wrong: In New Zealand, the average sheep eats almost ten more pounds of grass annually than that of its Australian counterpart. Correct: In New Zealand, the average sheep eats almost ten more pounds of grass annually than its Australian counterpart. 2) Independence Wrong: I always have and always will be true. Correct: I always have been and always will be true. Wrong: I have a deep interest and admiration for the works of this scientist. Correct: I have a deep interest in and admiration for the works of this scientist. Wrong: I either prefer to go this way or that way. Wrong: I prefer to go either this way or to go that way. Correct: I either prefer to go this way or prefer to go that way. Correct: I prefer to go either this way or that way. Note that the second variant is less wordy. Sequence rules: Punctuation:

185


I read books and newspapers. I read books, newspapers, and magazines. Note the comma before the last element of the sequence that consists of 3 or more elements. Articles and particle: Correct: I like to swim, to run, and to dance. Correct: I like to swim, run, and dance. Correct: I like the apples in June, the peaches in July, and the plums in September. Correct: I like the apples in June, peaches in July, and plums in September. All variants are correct. Note that the second variant of each sentence is less wordy. Wrong: I like to swim, run, and to dance. Wrong: I like to swim, to run, and dance. Comparison rules: Like vs. As Wrong: Like Mozart did, Bach loved music. Correct: As Mozart did, Bach loved music. “As” is used to compare verbs. Correct: Like Mozart, Bach loved music. ”Like/Unlike” is used to compare nouns. Confusing situations: Wrong: I know many good painters, like Picasso, Malevych, and Chagall. Correct: I know many good painters, such as Picasso, Malevych, and Chagall. Wrong: Managers sometimes also work like secretaries. Correct: Managers sometimes also work as secretaries. Just as … so (too) This expression is used to draw an analogy between two different situations, i.e. compare two separate clauses. Wrong: Just like electrons spin around nuclei, so the Earth revolves around the Sun. Wrong: Just as electrons spin around nuclei, the Earth revolves around the Sun. Correct: Just as electrons spin around nuclei, so the Earth revolves around the Sun. Incomplete comparison: Singular nouns: Correct: The book by the English author was more interesting than the book by the French author. Correct: The book by the English author was more interesting than that by the French author. Wrong: The book by the English author was more interesting than by the French author. Wrong: The book by the English author was more interesting than that of the French author. Plural nouns: Correct: The apples from Greece are tastier than the apples from Spain. Correct: The apples from Greece are tastier than those from Spain. Wrong: The apples from Greece are tastier than that from Spain.

186


Parallel constructions practice:

GoGMAT Problem

187


GoGMAT Problem Explanation

Instructions: chose one answer out of five given in each task according to the rules you’ve read in the chapter. You should spend no more than 1,5 minutes per question.

1. While a supernova originally causes a star to increase in brilliance, it will eventually cause the star’s light to gradually decline and disappear, as a flashlight fading from weakening batteries. (A) (B) (C) (D) (E)

decline and disappear, as a flashlight fading from weakening batteries decline and disappear, like a flashlight fading from weakening batteries decline or disappear, as a flashlight fading from weakening batteries either decline or disappear, as a flashlight fading from weakening batteries decline and disappear, just as a flashlight fading from weakening batteries

2. Plant cell walls not only function to maintain cell shape, which provides structural and mechanical support for the cell and the plant, and they also function to prevent expansion when water enters the cell. (A) (B) (C) (D) (E)

188

structural and mechanical support for the cell and the plant, and they also structural and mechanical support for the cell and the plant, for they also structural and mechanical support for the cell and the plant, but they also structural or mechanical support for the cell and the plant, and they also structural and mechanical support for the cell and the plant, however they also


3. By the time Robert Clark Young was fourteen years old, he was reading a second language, authoring a novel, and had published newspaper articles. (A) (B) (C) (D) (E)

and had published newspaper articles and published newspaper articles and would publish newspaper articles and publishing newspaper articles and was publishing newspaper articles

4. Doctors Without Borders is a private organization that arranges medical treatment in foreign lands, particularly in nations with an endemic outbreak of disease, states at war, and in povertystricken countries. (A) (B) (C) (D) (E)

states at war, and in poverty-stricken countries states at war, and in countries that are poverty-stricken by visiting states at war, and helping poverty-stricken countries in states at war, and in poverty-stricken countries states at war, and with poverty-stricken countries

5. In a 1893 court case, the Supreme Court ruled that because the tomato is most often served with dinner, it is a vegetable rather than defined as a fruit. (A) (B) (C) (D) (E)

rather than defined as a fruit rather than as a fruit rather than a fruit instead of defined as a fruit instead of defined as a type of fruit

6. Like Mozart’s first concerts, Frederic Chopin began performing at age six, both for private parties and public charity events. (A) (B) (C) (D) (E)

Like Mozart’s first concerts, Frederic Chopin began performing at age six Like Mozart’s young age, Frederic Chopin began performing at age six Like Mozart and his first concerts, Frederic Chopin began performing at age six Like Mozart, Frederic Chopin began performing at age six Like Mozart’s first concerts, Frederic Chopin’s concerts began performing at age six

7. Americans have a difficult time understanding the inability to improve one’s status under a feudal society, but feudalism’s hierarchical system is much more rigid than a democracy. (A) (B) (C) (D) (E)

than than that of than is so of compared to compared to that of

189


8. Unlike homophones, which are words with the same sound and spelling but different meanings, homonyms are words with the same sound but different spellings and meanings. (A) (B) (C) (D) (E)

homonyms are words with the same sound words called homonyms with the same sound the same sound is made by homonyms homonyms, same-sounding words, the sound is the same for homonyms

9. On their return, they not only witnessed the sinking ship but the amazing escape of the passengers. (A) (B) (C) (D) (E)

not only witnessed the sinking ship but the not only witnessed the sinking ship, but the not only witnessed the sinking ship, but also the witnessed not only the sinking ship but also the witnessed the sinking ship and also the

10. Either you transfer the data which was demanded or file a report explaining why you did not submit the overall annual figures. (A) (B) (C) (D) (E)

Either you transfer the data which was demanded You either transfer the data, which was demanded, You either transfer the data which were demanded Either you transfer the data, which was demanded, Either you transfer the data, which were demanded,

11. The students have always had a most sincere interest and admiration for the important work of Professor Jakobsen. (A) (B) (C) (D) (E)

a most sincere interest and admiration for a most sincere interest in and admiration for mostly a sincere interest and admiration for a most sincere interest, and admiration for a most sincere interest and an admiration for

12. The fourteen-hour day not only has been reduced to one of ten hours but also, in some lines of work, to one of eight or even six. (A) (B) (C) (D) (E)

190

The fourteen-hour day not only has been reduced Not only the fourteen-hour day has been reduced Not the fourteen-hour day only has been reduced The fourteen-hour day has not only been reduced The fourteen-hour day has been reduced not only


Home Practice 3 1. New data from United States Forest Service ecologists show that for every dollar spent on controlled small-scale burning, forest thinning, and the training of fire-management personnel, it saves seven dollars that would not be spent on having to extinguish big fires. (A) that for every dollar spent on controlled small-scale burning, forest thinning, and the training of fire-management personnel, it saves seven dollars that would not be spent on having to extinguish (B) that for every dollar spent on controlled small-scale burning, forest thinning, and the training of fire-management personnel, seven dollars are saved that would have been spent on extinguishing (C) that for every dollar spent on controlled small-scale burning, forest thinning, and the training of fire-management personnel saves seven dollars on not having to extinguish (D) for every dollar spent on controlled small-scale burning, forest thinning, and the training of firemanagement personnel, that it saves seven dollars on not having to extinguish (E) for every dollar spent on controlled small-scale burning, forest thinning, and the training of firemanagement personnel, that seven dollars are saved that would not have been spent on extinguishing 2. Dressed as a man and using the name Robert Shurtleff, Deborah Sampson, the first woman to draw a soldier's pension, joined the Continental Army in 1782 at the age of 22, was injured three times, and was discharged in 1783 because she had become too ill to serve. (A) 22, was injured three times, and was discharged in 1783 because she had become (B) 22, was injured three times, while being discharged in 1783 because she had become (C) 22 and was injured three times, and discharged in 1783, being (D) 22, injured three times, and was discharged in 1783 because she was (E) 22, having been injured three times and discharged in 1783, being 3. The nineteenth-century chemist Humphry Davy presented the results of his early experiments in his "Essay on Heat and Light," a critique of all chemistry since Robert Boyle as well as a vision of a new chemistry that Davy hoped to found. (A) a critique of all chemistry since Robert Boyle as well as a vision of a (B) a critique of all chemistry following Robert Boyle and also his envisioning of a (C) a critique of all chemistry after Robert Boyle and envisioning as well (D) critiquing all chemistry from Robert Boyle forward and also a vision of (E) critiquing all the chemistry done since Robert Boyle as well as his own envisioning of 4. When Congress reconvenes, some newly elected members from rural states will try and establish tighter restrictions for the amount of grain farmers are to be allowed to grow and to encourage more aggressive sales of United States farm products overseas. (A) and establish tighter restrictions for the amount of grain farmers are to be allowed to grow and to encourage

191


(B) and establish tighter restrictions on the amount of grain able to be grown by farmers and encouraging (C) establishing tighter restrictions for the amount of grain farmers are allowed to grow and to encourage (D) to establish tighter restrictions on the amount of grain capable of being grown by farmers and encouraging (E) to establish tighter restrictions on the amount of grain farmers will be allowed to grow and to encourage 5. More and more in recent years, cities are stressing the arts as a means to greater economic development and investing millions of dollars in cultural activities, despite strained municipal budgets and fading federal support. (A) to greater economic development and investing (B) to greater development economically and investing (C) of greater economic development and invest (D) of greater development economically and invest (E) for greater economic development and the investment of 6. In 2000, a mere two dozen products accounted for half the increase in spending on prescription drugs, a phenomenon that is explained not just because of more expensive drugs but by the fact that doctors are writing many more prescriptions for higher-cost drugs. (A) a phenomenon that is explained not just because of more expensive drugs but by the fact that doctors are writing (B) a phenomenon that is explained not just by the fact that drugs are becoming more expensive but also by the fact that doctors are writing (C) a phenomenon occurring not just because of drugs that are becoming more expensive but because of doctors having also written (D) which occurred not just because drugs are becoming more expensive but doctors are also writing (E) which occurred not just because of more expensive drugs but because doctors have also written 7. Unlike the buildings in Mesopotamian cities, which were arranged haphazardly, the same basic plan was followed for all cities of the Indus Valley: with houses laid out on a north-south, east-west grid, and houses and walls were built of standard-size bricks. (A) the buildings in Mesopotamian cities, which were arranged haphazardly, the same basic plan was followed for all cities of the Indus Valley: with houses (B) the buildings in Mesopotamian cities, which were haphazard in arrangement, the same basic plan was used in all cities of the Indus Valley: houses were (C) the arrangement of buildings in Mesopotamian cities, which were haphazard, the cities of the Indus Valley all followed the same basic plan: houses (D) Mesopotamian cities, in which buildings were arranged haphazardly, the cities of the Indus Valley all followed the same basic plan: houses were

192


(E) Mesopotamian cities, which had buildings that were arranged haphazardly, the same basic plan was used for all cities in the Indus Valley: houses that were 8. Like the grassy fields and old pastures that the upland sandpiper needs for feeding and nesting when it returns in May after wintering in the Argentine Pampas, the sandpipers vanishing in the northeastern United States is a result of residential and industrial development and of changes in farming practices. (A) the sandpipers vanishing in the northeastern United States is a result of residential and industrial development and of changes in (B) the bird itself is vanishing in the northeastern United States as a result of residential and industrial development and of changes in (C) that the birds themselves are vanishing in the northeastern United States is due to residential and industrial development and changes to (D) in the northeastern United States, sandpipers' vanishing due to residential and industrial development and to changes in (E) in the northeastern United States, the sandpipers' vanishing, a result of residential and industrial development and changing 9. Laos has a land area about the same as Great Britain but only four million in population, where many are members of hill tribes ensconced in the virtually inaccessible mountain valleys of the north. (A) about the same as Great Britain but only four million in population, where many (B) of about the same size as Great Britain is, but in Laos there is a population of only four million, and many (C) that is about the same size as Great Britain's land area, but in Laos with a population of only four million people, many of them (D) comparable to the size of Great Britain, but only four million in population, and many (E) comparable to that of Great Britain but a population of only four million people, many of whom 10. The stars, some of them at tremendous speeds, are in motion just as the planets are, yet being so far away from Earth that their apparent positions in the sky do not change enough for their movement to be observed during a single human lifetime. (A) The stars, some of them at tremendous speeds, are in motion just as the planets are, yet being (B) Like the planets, the stars are in motion, some of them at tremendous speeds, but they are (C) Although like the planets the stars are in motion, some of them at tremendous speeds, yet (D) As the planets, the stars are in motion, some of them at tremendous speeds, but they are (E) The stars are in motion like the planets, some of which at tremendous speeds are in motion but 11. Authoritative parents are more likely than permissive parents to have children who as adolescents are self-confident, high in self-esteem, and responsibly independent. (A) Authoritative parents are more likely than permissive parents to have children who as adolescents are self-confident, high in self-esteem, and responsibly independent.

193


(B) Authoritative parents who are more likely than permissive parents to have adolescent children that are self-confident, high in self-esteem, and responsibly independent. (C) Children of authoritative parents, rather than permissive parents, are the more likely to be selfconfident, have a high self-esteem, and to be responsibly independent as adolescents. (D) Children whose parents are authoritative rather than being permissive, are more likely to have self-confidence, a high self-esteem, and be responsibly independent when they are an adolescent. (E) Rather than permissive parents, the children of authoritative parents are the more likely to have self-confidence, a high self-esteem, and to be responsibly independent as an adolescent. IV.

Verbs (GoGMAT Session 10)

Verb (predicate): a word that expresses an action or a state of being. Verbal: a word that is formed from a verb but is not functioning as a verb. There are three kinds of verbals: participles, gerunds, and infinitives. Important: a verbal cannot substitute a verb in a sentence. A group of words containing a verbal but lacking a verb is not a sentence, but a phrase. 1) Participle: ends in –ing or –ed. It is used as an adjective in a sentence. Let sleeping dogs lie. It is difficult to calm a frightened child. Peering into his microscope, Robert Koch saw the tuberculosis bacilli. 2) Gerund: ends in –ing. It is used as a noun in a sentence. Skiing can be dangerous. Raising a family is a serious task. I was surprised at his acting like a coward. 3) Infinitive: The basic form of a verb, generally preceded by to. It is usually used as a noun, but may be used as an adjective or an adverb. Winston Churchill liked to paint. (Infinitive is used as an object) The will to conquer is crucial. (Infinitive is used as an adjective – it modifies “the will”) Students in imperial China studied the Confucian classics to excel on civil service exams. (Infinitive is used as an adverb – modifies “studied”) Past infinitive: I am happy to be here. I am happy to have been here. I was happy to be there. I was happy to have been there. Note how the form of past infinitive does not depend on the main tense of the sentence. It just shows that the action expressed by the infinitive took place before the main action of the sentence, regardless of its actual tense. Tense table

194


Past SIMPLE + I did begin. I began. (yesterday, 2 minutes do + base verb ago, in 1990, the other (except Future: day, last Friday, will + base verb) recently, if-clause type II) - I did not begin.

Present I do begin. I begin. (always, every‌, never, normally, often, seldom, sometimes, usually, if-clause type I) I do not begin.

Future I will begin. (in a year, next ‌, tomorrow, in one year)

I will not begin.

? Did I begin? PERFECT SIMPLE + I had begun. (already, just, never, had/have/will have + not yet, once, until that past participle day, if-clause type III)

Do I begin? Will I begin? I have begun. I will have begun. (already, ever, just, never, not yet, so far, till now, up to now, recently) I have not begun. I will not have - I had not begun. begun. Have I begun? Will I have begun? ? Had I begun? I am beginning. I will be beginning. CONTINUOUS + I was beginning. (when, while, as long (at the moment, just, (for ‌, the last be + -ing as) just now) couple of hours, all day long) I am not beginning. I will not be - I was not beginning. beginning. Am I beginning? Will I be beginning? ? Was I beginning? I have been beginning. I will have been PERFECT + I had been beginning. (for, since, the whole (all day, for 4 years, beginning. CONTINUOUS day, all day) since 1993, how long?, had/have/will have + the whole week) been + -ing had not been I have not been I will not have been - I beginning. beginning. beginning. ? Had I been beginning? Have I been beginning? Will I have been beginning? A few important notes about tenses: * Continuous forms of tenses are generally not preferable on GMAT. * Perfect tenses are tested more often than non-perfect tenses. * To use Past Perfect tense, the sentence must also use Past Simple tense. Sentences containing only Past Perfect tenses are wrong. * If a sentence describes 2 actions in the past and indicates exact moments when they happened, the earlier action will be described with Past Perfect tense. E.g.: A Canadian scientist discovered an important law in 1999, while a British scientist had discovered the same law in 1888.

195


* “before”-rule. I had finished the work before they came. I finished the work before they came. Both sentences are correct. Though the preferable form is a full Past Perfect tense, the use of 2 Past Simple forms is also possible. Conditional: i. If I am here, I will call on you. Note how a conditional sentence consists of 2 halves: the condition and the result. The condition part never uses any from of Future tense. Condition can be also introduced by the following words: Unless, In case, When, etc. ii. If I were here, I would call on you. Note how, in the 2nd conditional, the verb “to be” uses the form “were” for all persons. iii. If I had been here, I would have called on you. Note how the 3rd conditional can also use an inverse form: Had I been here, I would have called on you. Both forms are used on GMAT, none of them is preferable over the other. Subjunctive mood: There are a number of words and expressions, after which the author may wish to express a requirement in a special form. This form is generally absent from spoken English. Require Demand Order Insist Advocate Mandate Suggest Recommend Propose It is important

 …Bill to do/of Bill to do/for Bill to do A complex object form.

 … THAT Bill DO/ THAT Bill NOT DO/THAT Bill BE ARRESTED/THAT Bill NOT BE ARRESTED A subjunctive mood form. The verb form represents an infinitive without particle. The form does not change in number, person, or tense.

Note that both forms are grammatically correct and that the Subjunctive mood form is preferable over the Complex Object form.

Verbs practice:

GoGMAT Problem

196


197


GoGMAT Problem Explanation

Instructions: chose one answer out of five given in each task according to the rules you’ve read in the chapter. You should spend no more than 1,5 minutes per question. 1. They decided to honor Ms. Wilson, who will be president of the club for ten years next Tuesday. (A) (B) (C) (D) (E)

will be president of the club for ten years next Tuesday shall have been president of the club for ten years next Tuesday next Tuesday will have been president of the club for ten years next Tuesday has been president of the club for ten years had been president of the club for ten years next Tuesday

2. If they would have taken greater care in the disposal of the nuclear waste, the disaster would not have occurred. (A) (B) (C) (D) (E)

198

If they would have taken greater care Unless they took greater care Had they not taken greater care If they had taken greater care If they took greater care


3. France’s longest-ruling monarch, Louis XIV, inherited the throne at the age of four and the country was ruled by him for 72 years. (A) (B) (C) (D) (E)

inherited the throne at the age of four and the country was ruled by him for 72 years had inherited the throne at the age of four and the country was ruled by him for 72 years inherited the throne at the age of four and ruled the country for 72 years inherited the throne at the age of four and the country had been ruled by him for 72 years had inherited the throne at the age of four and he ruled the country for 72 years

4. Scientists believe that the great white shark has evolved from the megalodon, a prehistoric shark measuring over 50 feet in length. (A) (B) (C) (D) (E)

believe that the great white shark has evolved believe that the great white shark evolved believed that the great white shark has evolved are believing that the great white shark has evolved have believed that the great white shark has evolved

5. At the time of Great Depression, the Government was more involved in the regulation of economy than it ever was before, which is why it is often blamed for the economic collapse. (A) (B) (C) (D) (E)

was more involved in the regulation of economy than it ever was before was more involved in the regulation of economy than it ever would have been before having been more involved in the regulation of economy than it ever was before was more involved in the regulation of economy than it ever had been before was to be more involved in the regulation of economy than it ever will have been before

6. Beginning as an NBC radio program in 1937, the soap opera Guiding Light moved to CBS television in 1952 and is currently the longest running drama in broadcast history. (A) (B) (C) (D) (E)

moved to CBS television in 1952 and is had moved to CBS television in 1952 and is had been moved to CBS television in 1952 and is moved to CBS television in 1952 and was moved to CBS television in 1952 and will be

7. Unless they reverse present policies immediately, the world may suffer permanent damage from the unregulated use of pesticides. (A) (B) (C) (D) (E)

Unless they reverse present policies Unless present policies are reversed Unless present policies will be reversed If it will not reverse present policies If present policies will not be reversed

199


8. In 1896, Henri Bequerel found that uranium salts emitted penetrating radiations similar to those which Roentgen produced only a year earlier with a gas discharge tube. (A) (B) (C) (D) (E)

similar to those which Roentgen like those which Roentgen similar to those that Roentgen had similar to them that Roentgen similar to those Roentgen

9. Economic conditions demand that we not only cut wages and prices but also reduce inflationraised tax rates. (A) (B) (C) (D) (E)

10. (A) (B) (C) (D) (E)

that we not only cut wages and prices but also not only cutting wages and prices but also to not only to cut wages and prices but also to not only a cut in wages and prices but also to not only to cut wages and prices but that we also

If she were I, she would have accepted the prize if she had won it. were I, she would have accepted the prize if she had was I, she would have accepted the prize if she would have was I, she would have accepted the prize if she had were I, she would have accepted the prize if she would have were me, she would have accepted the prize if she had

Home Practice 4 1. To develop more accurate population forecasts, demographers have to know a great deal more than now about the social and economic determinants of fertility. (A) have to know a great deal more than now about the social and economic (B) have to know a great deal more than they do now about the social and economical (C) would have to know a great deal more than they do now about the social and economical (D) would have to know a great deal more than they do now about the social and economic (E) would have to know a great deal more than now about the social and economic 2. The report recommended that the hospital should eliminate unneeded beds, expensive services should be consolidated, and use space in other hospitals. (A) should eliminate unneeded beds, expensive services should be consolidated, and use space in other hospitals (B) should eliminate unneeded beds, expensive services should be consolidated, and other hospitals' space be used (C) should eliminate unneeded beds, expensive services should be consolidated, and to use space in other hospitals (D) eliminate unneeded beds, consolidate expensive services, and other hospitals' space used

200


(E) eliminate unneeded beds, consolidate expensive services, and use space in other hospitals 3. Affording strategic proximity to the Strait of Gibraltar, Morocco was also of interest to the French throughout the first half of the twentieth century because they assumed that if they did not hold it, their grip on Algeria was always insecure. (A) if they did not hold it, their grip on Algeria was always insecure (B) without it their grip on Algeria would never be secure (C) their grip on Algeria was not ever secure if they did not hold it (D) without that, they could never be secure about their grip on Algeria (E) never would their grip on Algeria be secure if they did not hold it 4. His studies of ice-polished rocks in his Alpine homeland, far outside the range of present-day glaciers, led Louis Agassiz in 1837 to propose the concept of an age in which great ice sheets had existed in now currently temperate areas. (A) in which great ice sheets had existed in now currently temperate areas (B) in which great ice sheets existed in what are now temperate areas (C) when great ice sheets existed where there were areas now temperate (D) when great ice sheets had existed in current temperate areas (E) when great ice sheets existed in areas now that are temperate 5. A 1972 agreement between Canada and the United States reduced the amount of phosphates that municipalities had been allowed to dump into the Great Lakes. (A) reduced the amount of phosphates that municipalities had been allowed to dump (B) reduced the phosphate amount that municipalities had been dumping (C) reduces the phosphate amount municipalities have been allowed to dump (D) reduced the amount of phosphates that municipalities are allowed to dump (E) reduces the amount of phosphates allowed for dumping by municipalities 6. The results of the company's cost-cutting measures are evident in its profits, which increased 5 percent during the first 3 months of this year after it fell over the last two years. (A) which increased 5 percent during the first 3 months of this year after it fell (B) which had increased 5 percent during the first 3 months of this year after it had fallen (C) which have increased 5 percent during the first 3 months of this year after falling (D) with a 5 percent increase during the first 3 months of this year after falling (E) with a 5 percent increase during the first 3 months of this year after having fallen V.

Pronouns (GoGMAT Session 4)

Pronoun is a part of speech that can substitute or indicate a noun (subject or object) in a sentence. If this noun is present in the sentence, it is called ANTECEDENT. 1) Personal pronouns Subjective case Objective case I Me You You

Possessive case Mine Yours

201


He She It We They Who Which

Him Her It Us Them Whom Which

His Hers Its Ours Theirs Whose Whose

2) Interrogative Pronouns An interrogative pronoun is used to ask questions. The interrogative pronouns are "who," "whom," "which," "what" and the compounds formed with the suffix "ever" ("whoever," "whomever," "whichever," and "whatever"). 3) Relative Pronouns The relative pronouns are "who," "whom," "that," and "which." The compounds "whoever," "whomever," and "whichever" are also relative pronouns. The difference between relative and interrogative pronouns is that interrogative pronouns are those used in questions while relative pronouns are used in assertive sentences. 4) Indefinite Pronoun Indefinite pronouns replace nouns without specifying which noun they replace. Singular: another, anybody, anyone, anything, each, either, everybody, everyone, everything, little, much, neither, nobody, no one, nothing, one, other, somebody, someone, something Plural: both, few, many, others, several Singular or Plural: all, any, more, most, none, some 5) Reflexive Pronoun: The reflexive pronouns are "myself," "yourself," "herself," "himself," "itself," "ourselves," "yourselves," and "themselves." Pronouns must conform to 4 basic rules: 1. To have a specified or identifiable antecedent. This rule is usually violated by 2 personal pronouns: “it” and “they”. “Mysterious they” Correct: They entered the room and turned on the light. Wrong: There is no known cure for certain forms of hepatitis; they hope, though, that a cure will be found soon.

202


Note that in the first sentence, the antecedent is not present but can be easily identified; the pronoun “they” refers to a certain group of people. In the second sentence, “they” does not refer to any special group of people; rather, it represents a spoken variant of an impersonal sentence “It is hoped”. “It” errors Very often, pronoun “it” substitutes not a single noun in a sentence, but a verb or even an entire clause. Wrong: If the partners cannot resolve their problems, the court may have to do it. Correct: If the partners cannot resolve their problems, the court may have to do so. Wrong: The goalkeeper stumbled at the gate, and it decided the game. Correct: The goalkeeper stumbled at the gate, and this mistake decided the game. 2. To stand close to antecedent. In other words, a pronoun always refers to the closest noun having a suitable form. Wrong: Beatrix Potter's stories depict animals in an unsentimental and humorous manner, and the author illustrated them with delicate watercolor paintings. Correct: Beatrix Potter's stories depict animals in an unsentimental and humorous manner, and the author illustrated the books with delicate watercolor paintings. 3. To agree with antecedent Pronoun must agree with antecedent in Number and Gender. Wrong: The Commerce Department, which usually does not make any fiscal announcements until after the budget is ratified, announced that their accounting practices would be overhauled next year. Correct: The Commerce Department, which usually does not make any fiscal announcements until after the budget is ratified, announced that its accounting practices would be overhauled next year. Wrong: Every employee brought their softball mitt to the game. Correct: Every employee brought his or her softball mitt to the game. 4. To use the correct case If a pronoun substitutes the subject of a sentence, it must use the Subjective case: Wrong: John and me went to the movies. Correct: John and I went to the movies. Wrong: John is taller than me. Correct: John is taller than I. Wrong: It is me knocking on the door. Correct: It is I knocking on the door.

203


If a pronoun substitutes an object in a sentence, it must use the Objective case: Wrong: Except for John and I, everyone went to the movies. Correct: Except for John and me, everyone went to the movies. Wrong: There goes a girl who I love. Correct: There goes a girl whom I love. * Objective case vs. Possessive adjective Wrong: I like him reading. Correct: I like his reading. In this sentence, the object is “reading”. Possessive adjective “his” modifies this object. Wrong: I want he to read. Correct: I want him to read. In this sentence, “him to read” is a complex object, so it uses a pronoun in Objective case. * One vs. You “One” and “You” can both be used as indefinite persons. However, it would be a mistake to put them both in 1 sentence. Wrong: Not even your best friend can always be relied on to give one good advice. Correct: Not even your best friend can always be relied on to give you good advice. Correct: Not even one’s best friend can always be relied on to give one good advice.

204


Pronouns practice:

GoGMAT Problem

205


GoGMAT Problem Explanation

Instructions: chose one answer out of five given in each task according to the rules you’ve read in the chapter. You should spend no more than 1,5 minutes per question. 1. The bank has offered so many convenient services, such as checking by phone and online banking, that many of their customers no longer visit the bank itself. (A) (B) (C) (D) (E)

that many of their customers no longer visit the bank itself that each of its customers no longer visit the bank itself that many of their customers no longer visit the bank themselves that many of its customers no longer visit the bank itself that many of its customers no longer visit the bank him or herself

2. Upon hearing of the chairman’s illness, the committee motioned to postpone their next meeting until after he was released from the hospital. (A) (B) (C) (D)

206

to postpone their next meeting until after he was released to postpone its next meeting until after he was released to postpone their next meeting until after he or she was released to postpone their next meeting until after they were released


(E)

to postpone its next meeting until after each was released

3. The company, known for its benevolence, has donated over $100,000 to charity over the last twenty years and will continue to do it as long as it is financially able. (A) (B) (C) (D) (E)

will continue to do it as long as it is will continue to do that as long as it is will continue to do so as long as it is will continue to do it as long as the will continue to do it as long as they are

4. A manatee differs from its relative, the dugong, in both size and shape; the largest difference is the dugong’s tail, which is forked, unlike their paddle-shaped tail. (A) (B) (C) (D) (E)

which is forked, unlike their paddle-shaped tail which the dugong’s is forked, unlike their paddle-shaped tail which being forked, unlike their paddle-shaped tail which is forked, unlike its paddle-shaped tail which is forked, unlike the manatee’s paddle-shaped tail

5. Student admissions to medical school are not accepted solely based on their MCAT scores; other considerations include their undergraduate grade point averages and extracurricular activities. (A) Student admissions to medical school are not accepted solely based on their MCAT scores (B) Students seeking admission to medical school are not accepted solely based on their MCAT scores (C) Student admissions to medical school are not accepted solely based on the schools’ MCAT scores (D) Student admissions to medical school are not accepted solely based on MCAT scores (E) Students seeking admission to medical school are not accepted solely based on its MCAT scores 6. (A) (B) (C) (D) (E)

No one but him could have told them that the thief was I. him could have told them that the thief was I he could have told them that the thief was I he could have told them that the thief was me him could have told them that the thief was me he could have told them the thief was me

7. Everyone of us have understood that without him helping us we would not have succeeded in our program over the past six months. (A) (B) (C) (D) (E) 8. (A) (B) (C) (D) (E)

Everyone of us have understood that without him helping us Everyone of us has understood that without his helping us Everyone of us have understood that without his help Everyone of us has understood that without him helping us Every single one of us have understood that without him helping us We want the teacher to be him who has the best rapport with the students. We want the teacher to be him We want the teacher to be he We want him to be the teacher We desire that the teacher be him We anticipate that the teacher will be him

207


9. With the exception of Frank and I, everyone in the class finished the assignment before the bell rang. (A) (B) (C) (D) (E)

Frank and I, everyone in the class finished Frank and me, everyone in the class finished Frank and me, everyone in the class had finished Frank and I, everyone in the class had finished Frank and me everyone in the class finished

10. When one eats in this restaurant, you often find that the prices are high and that the food is poorly prepared. (A) (B) (C) (D) (E)

When one eats in this restaurant, you often find When you eat in this restaurant, one often finds As you eat in this restaurant, you often find If you eat in this restaurant, you often find When one ate in this restaurant, he often found

11. He interviewed several candidates who he thought had the experience and qualifications the position required. (A) (B) (C) (D) (E)

who he thought whom he thought of whom he thought he thought who which he thought

Home Practice 5 1. The Supreme Court has ruled that public universities can collect student activity fees even with students' objections to particular activities, so long as the groups they give money to will be chosen without regard to their views. (A) with students' objections to particular activities, so long as the groups they give money to will be (B) if they have objections to particular activities and the groups that are given the money are (C) if they object to particular activities, but the groups that the money is given to have to be (D) from students who object to particular activities, so long as the groups given money are (E) though students have an objection to particular activities, but the groups that are given the money be 2.Nobody knows exactly how many languages there are in the world, partly because of the difficulty of distinguishing between a language and the sublanguages or dialects within it, but those who have tried to count typically have found about five thousand. (A) and the sublanguages or dialects within it, but those who have tried to count typically have found (B) and the sublanguages or dialects within them, with those who have tried counting typically finding (C) and the sublanguages or dialects within it, but those who have tried counting it typically find (D) or the sublanguages or dialects within them, but those who tried to count them typically found (E) or the sublanguages or dialects within them, with those who have tried to count typically finding

208


3. Heavy commitment by an executive to a course of action, especially if it has worked well in the past makes it likely to miss signs of incipient trouble or misinterpret them when they do appear. (A) Heavy commitment by an executive to a course of action, especially if it has worked well in the past, makes it likely to miss signs of incipient trouble or misinterpret them when they do appear. (B) An executive who is heavily committed to a course of action, especially one that worked well in the past, makes missing signs of incipient trouble or misinterpreting ones likely when they do appear. (C) An executive who is heavily committed to a course of action is likely to miss or misinterpret signs of incipient trouble when they do appear, especially if it has worked well in the past. (D) Executives’ being heavily committed to a course of action, especially if it has worked well in the past, makes them likely to miss signs of incipient trouble or misinterpreting them when they do appear. (E) Being heavily committed to a course of action, especially one that has worked well in the past, is likely to make an executive miss signs of incipient trouble or misinterpret them when they do appear. 4. A mutual fund having billions of dollars in assets will typically invest that money in hundreds of companies, rarely holding more than one percent of the shares of any particular corporation. (A) companies, rarely holding more than one percent (B) companies, and it is rare to hold at least one percent or more (C) companies and rarely do they hold more than one percent (D) companies, so that they rarely hold more than one percent (E) companies; rarely do they hold one percent or more 5. Because there are provisions of the new maritime code that provide that even tiny islets can be the basis for claims to the fisheries and oil fields of large sea areas, they have already stimulated international disputes over uninhabited islands. (A) Because there are provisions of the new maritime code that provide that even tiny islets can be the basis for claims to the fisheries and oil fields of large sea areas, they have already stimulated (B) Because the new maritime code provides that even tiny islets can be the basis for claims to the fisheries and oil fields of large sea areas, it has already stimulated (C) Even tiny islets can be the basis for claims to the fisheries and oil fields of large sea areas under provisions of the new maritime code, already stimulating (D) Because even tiny islets can be the basis for claims to the fisheries and oil fields of large sea areas under provisions of the new maritime code, this has already stimulated (E) Because even tiny islets can be the basis for claims to the fisheries and oilfields of large sea areas under provisions of the new maritime code, which is already stimulating VI.

Word Pairs and Preferable answers

as…as

The movie was as long as it was boring.

more…than less…than greater….than

The workshop was more thrilling than anything I'd ever done.

Not only…but also She was not only exhausted but also famished.

209


Not….but

She was not exhausted but simply bored.

so…that so… as to

The apartment was so expensive that no self-supporting student could afford it. The apartment was so expensive as to be unaffordable to any self-supporting student.

Just as…so (too)

Just as it is the duty of employees to contribute to the well-being of the company, so (too) it is the duty of the company to contribute to the well-being of its employees.

neither…nor either…or

Neither a borrower nor a lender be.

A few rules about preferable answers: IF vs. WHETHER Use “If” only with Conditionals (see “Verbs”). In all other cases, use “Whether”. Incorrect: Her client did not tell her if he had sent his payment yet. Correct: Her client did not tell her whether he had sent his payment yet. BEING “Being” is usually wrong on SC. Wrong: Being tired, I did not walk the dog. Wrong: Having been tired, I did not walk the dog. Correct: Tired, I did not walk the dog. See how 2 wrong variants use different forms of a meaningless, redundant “being”, and how the meaning of the sentence is not compromised without this word. Note that if the form “being” is required grammatically, it cannot be crossed out. Correct: I enjoy being here. BECAUSE vs. ON ACCOUNT OF: “Because” is preferred over “on account of”. The former can introduce a subordinate clause and is less wordy. Wrong: Golden crab is not fished, on account of living… Correct: Golden crab is not fished, because it lives… IN THAT vs. BECAUSE: These expressions slightly differ in meanings: Correct: He reminds me of Brad Pitt because he is handsome. Correct: He reminds me of Brad Pitt in that he is handsome. Neither of the meanings is more correct than or preferable over the other, so try to keep the meaning of “A”. DUE TO

210


“Due to” means "caused by" It should only be used if it can be substituted with "caused by". It does not mean the same thing as "because of." Wrong: The game was postponed due to rain. Correct: The game was postponed because of the rain. Correct: The postponement of the game was due to the rain. USUAL vs. IS USUAL: When something is compared to a subgroup to which it belongs, is usual should be used. When something is compared to itself, usual is fine Correct: He is faster than is usual for any human being. Correct: He is faster than usual today. Wrong: A Mercedes is more expensive than usual for a car. Correct: A Mercedes is more expensive than is usual for a car. NOT/BUT vs. RATHER THAN These expressions differ in meanings and cannot be substituted with one another. Example: Pucci is not a dog but a cat. I want a cat rather than a dog. SO … THAT vs. SO… AS TO vs. ENOUGH TO “So… that” and “so… as to” have equal meaning: Correct: Her debts are so extreme as to threaten the future of the company. Also correct: Her debts are so extreme that they threaten the future of the company. If both are among the answers, choose “so… that” – it is preferable. Another expression with a similar but different meaning is often used to distract you: “enough to”. As it does not have the same meaning as “so… that” and “so… as to”, it cannot substitute them. Very often you will see hybrids of these 3 expressions; these hybrids are always wrong. Wrong: Her debts were extreme enough so that they threatened the future of the company. TO vs. SO AS TO vs. IN ORDER TO Wordy expressions, such as “in order to” and “so as to”, are not preferable on SC. Wrong: I am reading the book in order to pass the exam. Wrong: I am reading the book so as to pass the exam. Correct: I am reading the book to pass the exam. HOWEVER + adjective/adverb “However” with any adjective or adverb means “no matter”. Correct: However much you complain, I will not change my mind. Meaning: No matter how much you complain, I will not change my mind. Correct: However beautiful she is, she will not win the contest. Meaning: No matter how beautiful she is, she will not win the contest. PRACTICE vs. PRACTISE:

211


Verb To practise To advise To counsel

Noun A practice Advice A council

ECONOMIC vs. ECONOMICAL: Economic means "having to do with the economy or the study of economics." Economical means "careful or prudent in managing finances, money-saving." A tip to remember: “Political BUT Economic” COMPARE TO vs. COMPARE WITH: “Compare to” is used to draw an analogy between 2 different situations: Correct: He compared her to a summer day. Correct: Scientists compare the human brain to a computer. “Compare with” is used in a more common situation of a comparison between 2 objects: Correct: The police compared the forged signature with the original. VII.

Quantity words

2 items Between More Better Less

more than 2 items among most best least

Non-countable words Less Amount, quantity Much

Countable words fewer number many

212


Quantity words practice:

GoGMAT Problem

213


GoGMAT Problem Explanation

Instructions: chose one answer out of five given in each task according to the rules you’ve read in the chapter. You should spend no more than 1,5 minutes per question. 1. Of the two candidates for this government position, Joann Harald is the most qualified because of her experience in the field. (A) (B) (C) (D) (E)

2. (A) (B) (C) (D) (E)

214

most qualified because of most qualified due to more qualified due to more qualified because of most qualified as a result of

After viewing both movies, John agreed that the first one was the best of the two. John agreed that the first one was the best of the two John agreed that the first was the best of the two John agreed that the first one was the better of the two John agreed that of the two the better one was the first John agreed that the best of the two was the first


Redundancy practice:

GoGMAT Problem

GoGMAT Problem Explanation

215


Instructions: chose one answer out of five given in each task. You should look for the answer that uses the fewest number of words to give the same meaning as in “A� answer. Avoid answers using unnecessary wording. You should spend no more than 1,5 minutes per question. 1. From the moment he took public office, his actions have been loaded with significance and filled with worth. (A) (B) (C) (D) (E)

2. (A) (B) (C) (D) (E)

3. (A) (B) (C) (D) (E)

4. (A) (B) (C) (D) (E)

5. (A) (B) (C) (D) (E)

216

been loaded with significance and filled with worth been significant and worthwhile become loaded with significance and worth to be loaded with significance and filled with worth been actions of significance and worth

Completing the physical examination, the tonsils were found to be diseased. Completing the physical examination, the tonsils were found to be diseased. Having completed the physical examination, the tonsils were found to be diseased. When the physical examination was completed, the tonsils were found to be diseased. The physical examination completed, the tonsils were found to be diseased. The physical examination found that the tonsils were diseased.

If we cooperate together by dividing up the work, we shall be able to finish it quickly. If we cooperate together by dividing up the work If we cooperate by dividing up the work If we cooperate together by dividing the work If we cooperate by dividing up the work together If we cooperate by dividing the work

My grandmother is the most remarkable person of all the persons I have ever met. My grandmother is the most remarkable person of all the persons I have ever met. Of all the persons I have ever met, my grand mother is the most remarkable person. Of all the persons I have ever met, the most remarkable person is my grandmother. Of all the persons I have ever met, the most remarkable is my grandmother. My grandmother, of all the persons I have ever met, is the most remarkable.

It was decided by us that the emphasis would be placed on the results that might be attained. It was decided by us that the emphasis would be placed on the results that might be attained. We decided that the emphasis would be placed on the results that might be attained. We decided to emphasize the results that might be attained. We decided to emphasize the results we might attain. It was decided that we would place emphasis on the results that might be attained.


6. (A) (B) (C) (D) (E)

May I venture to say that I think this performance is the most superior I have ever heard. May I venture to say that I think this performance is the most superior May I venture to say that this performance is the most superior May I say that this performance is the most superior I think this performance is superior to any This performance is the most superior of any

HOME PRACTICE ANSWER KEY

HP/Q s# HP1 HP2 HP3 HP4 HP5

1 E A B D D

2 A B A E A

3 B B A B E

4 E E A B A

5 D E E D B

6 B B A C

7 C E B

8

9

10

11

12

A D

D B

B E

B

A

217


SENTENCE CORRECTION BANK (BG) Set 1

1. In her candid autobiography, the author discusses her early years, her desire to become an actress and how she made her debut on the stage. (A) (B) (C) (D) (E)

to become an actress, and how she made that she become an actress, and how she made to become an actress, and that she become an actress, and that she become an actress and that she make

2. The possibility of expropriation was believed to be unlikely in the near future due to the lack of mining technology and capital available in this small South American country. (A) (B) (C) (D) (E)

due to the lack of mining technology and because of the lack of mining technology and because there was no mining technology and because of the lack of mining technology and there was no due to the lack of mining technology and there was no

3. The doctrine applies in Canada, where there is a federal law and a provincial law that are each valid and consistent. (A) (B) (C) (D) (E)

is a federal law and a provincial law that are each valid and are a federal law and a provincial law that are each valid and are a federal law and a provincial law both of which are each valid and is a federal law and a provincial law both of which are each valid and are a federal law and a provincial law that are each valid or

4. Former Postal Service employees who believe they may be affected by this settlement should contact their last place of USPS employment, the department advised. (A) (B) (C) (D) (E)

218

may be affected may be effected will have been affected will be effected will have been effected


5. Blake is among the very few individuals who critics regard as genuinely significant in the history of both art and literature. (A) (B) (C) (D) (E)

who critics regard as genuinely significant in the history of both whom critics regard as genuinely significant in the history of both whom critics regard as genuinely significant both in the history of who critics regard as genuinely significant both in the history of who is regarded by critics as genuinely significant in the history of both

6. Many scientists are alarmed over the interest in such pseudo-scientific topics as ESP, flying saucers, and the occult, fearing that this interest may herald a new dark age of gullibility, ignorance, and thinking in superstitious ways. (A) this interest may herald a new dark age of gullibility, ignorance, and thinking in superstitious ways (B) it may herald a new dark age of gullibility, ignorance, and thinking in superstitious ways (C) it may herald a new dark age of gullibility, ignorance, and superstition (D) this interest may herald a new dark age of gullibility, ignorance, and superstition (E) they may herald a new dark age of gullibility, ignorance, and superstition

7. Having broken with Freud, Jung's later writings nevertheless bore signs of the continued influence of Freudian doctrine and theories. (A) (B) (C) (D) (E)

Having broken with Freud, Jung's later writings nevertheless bore signs of the continued Since breaking with Freud, Jung's later writings nevertheless bore signs of the continued Although he had broken with Freud, in his later writings Jung nevertheless showed signs of the continued Having broken with Freud, Jung's later writings nevertheless bore signs of the continual Having broken with Freud, later writings by Jung nevertheless bore signs of the continued

8. That Giotto's paintings are significant in the history of the early Renaissance is undeniable, but Giotto cannot scarcely be considered the equal of such masters as Leonardo and Raphael. (A) (B) (C) (D) (E)

cannot scarcely be considered can scarcely be considered cannot hardly be considered cannot scarcely be considered to be isn't hardly to be considered

9. Although the theory of continental drift was not widely accepted until the mid-twentieth century, the basic concept had been described as early as 1620.

219


(A) (B) (C) (D) (E)

was not widely accepted until the mid-twentieth century, the basic concept had been was not widely accepted until the mid-twentieth century, the basic concept was was not widely accepted until the mid-twentieth century, the basic concept has been had not been widely accepted until the midtwentieth century, the basic concept has been had not been widely accepted until the midtwentieth century, the basic concept was

10. Fear of future nationalistic feelings and a conviction that natural resource endowments should be exploited for the welfare of the residents of the country, rather than for private profit, are shared by all managers of extractive industries there. (A) (B) (C) (D) (E)

rather than for private profit, are rather than for private profit, is irregardless of private profit, are as opposed to private profit, is and not necessarily for private profit, is

11. The lieutenant reminded Company B that the only information to be given to the enemy if captured was each individual's name, rank, and serial number. (A) (B) (C) (D) (E)

the only information to be given to the enemy if captured was each individual's name, rank, and serial number the only information to be given to the enemy if they were captured was each individual's name, rank, and serial number the only information to be given to the enemy if captured were each individual's name, rank, and serial number if captured, the only information to be given to the enemy was each individual's name, rank, and what his serial number was if they were captured, the only information to be given to the enemy was each individual's name, rank, and serial number

12. Writing a beautiful sonnet is as much an achievement as to finish a 400-page novel. (A) (B) (C) (D) (E)

to finish it is to finish finishing if you finished to have finished

13. Anyone interested in Web site design can find a job in contemporary industry if you learn the basic coding languages, such as HTML and Java. (A)

220

if you learn


(B) (C) (D) (E)

if you will learn if he would learn by the study of by studying

14. During the gasoline shortage of the 1970s caused by the actions of the OPEC nations, the number of accidents on our highways decreased markedly. (A) (B) (C) (D) (E)

the number of accidents on our highways decreased markedly the amount of accidents on our highways decreased markedly there were less accidents on our highways there were a fewer amount of accidents on our highways they found there were many fewer accidents on our highways

15. I have studied the works of George Bernard Shaw not only for their plots but also because they are very witty. (A) (B) (C) (D) (E)

also because they are very witty because they are also very witty for their wit also because they are very witty also also for their wit

16. The noise at the airport was deafening, which made conversation difficult if not impossible. (A) (B) (C) (D) (E)

The noise at the airport was deafening, which made conversation The noise at the airport was deafening, and it made conversation The deafening noise at the airport made conversation The airport noise was deafening, which made conversation The noise at the airport was deafening, conversation being

17. Equestrian enthusiasts predict that the alleged abuse of anabolic steroids among horse trainers would subside as long as the testing of the animals is more random and more rigorously enforced.

(A) (B) (C) (D) (E)

would subside as long as the testing of the animals is more random and would subside if the testing of the random animals were will have subsided when testing of the animals is more random and will subside if random testing of the animals were will subside if the random testing of the animals is

221


18. (OG) A representative of the Internal Revenue Service usually finds most people are willing to cooperate during an audit – yet they become agitated, defensive, and suspect computer error.

(A) most people are willing to cooperate during an audit, yet they become agitated, defensive, and suspect (B) most people to be willingly cooperative during an audit, and they are also agitated, defensive, and they suspect (C) that most people are willing to cooperate during an audit, yet they become agitated, defensive, and suspicious of (D) that people are mostly willing to cooperate during an audit, and they become agitated, defensive, and suspicious of (E) that most people are willingly cooperative during an audit, yet they are becoming agitated, defensive, and suspect

19. (OG) The Center for Public Integrity has discovered that drug companies obtain people's health records through the Internet either to contact them individually and suggest alternative forms of treating various illnesses or the estimation of the market of each new drug they produce. (A) (B) (C) (D) (E)

the estimation of to estimate for estimating they want to estimate it wants to estimate

20. Gianlorenzo Bernini should be judged not by the degree to which his sculptures and architecture are admired throughout the world, but by his Bacchanalian lifestyle, his notorious temper, and his scathing jealousy of his counterparts. (A) (B) (C) (D) (E)

be judged not by the degree to which his sculptures and architecture are admired throughout the world, but by not be judged by the degree of admiration the world has for his sculptures and architecture, and instead by be judged to the degree that his sculptures and architecture are admired throughout the world, and not by not be judged by the degree to which the world admires his sculptures and architecture, but instead be judged to the degree of admiration which the world has for his sculptures and architecture, not by

(K) Set 2

222


1. Recent surveys indicate that, contrary to popular belief, total abstinence from alcohol does not correlate as strongly with good health as does moderate drinking. (A) (B) (C) (D) (E)

as strongly with good health as does moderate drinking strongly with good health, like moderate drinking does as strongly with good health as does moderately drinking as strongly with good health as with moderate drinking as strongly with good health as moderate drinking

2. Despite the platform of the Republican Party supporting the measure, they keep voting against campaign finance reform in Congress. (A) (B) (C) (D) (E)

the platform of the Republican Party supporting the measure, they keep voting the Republican Party's platform supporting the measure, they keep voting the Republicans' platform supporting the measure, it keeps voting the Republican Party's platform supporting the measure, party members keep voting the supporting measure of the platform of the Republican Party, they keep voting

3. The creation of an independent treasury, securing lower tariffs, and purchasing the Oregon Territory, all credited to the presidency of James Knox Polk, are among the significant accomplishments that persuade historians to rank this former governor of Tennessee well as a U.S. President. (A) (B) (C) (D) (E)

The creation of an independent treasury, securing lower tariffs, and purchasing The creation of an independent treasury, securing lower tariffs as well as purchasing The creation of an independent treasury, the establishment of lower tariffs, and the purchase of Creating an independent treasury, securing lower tariffs, and purchasing Creating an independent treasury, the securing of lower tariffs, and the purchasing of

4. In many coastal New England towns, the fisherman still operates as they have for generations, displaying and selling their catch dockside at the end of each day. (A) (B) (C) (D) (E)

the fisherman still operates as they have the fisherman still operates as was done fishermen still operate as they have done the fisherman still operates as he has fishermen still operate as they had

5. The average salary of new jobs is expected to rise in the near future as jobs become available in high-paying industries. (A)

is expected to rise in the near future as jobs become available in high-paying industries

223


(B) (C) (D) (E)

are expected to rise in the near future as jobs become available in high-paying industries are expected to rise in the near future through jobs becoming available in high-paying industries would be expected to rise in the near future through jobs becoming available in high-paying industries will rise in the near future because jobs in high-paying industries are expected to become available

6. Many climatologists now suspect that the effects of global warming will include unusual temperature fluctuations throughout the northern and southern hemispheres that far exceeds what scientists at the Kyoto Summit were predicting only a few years ago. (A) (B) (C) (D) (E)

far exceeds what scientists at the Kyoto Summit were predicting only a few years ago far exceeds those predicted by scientists at the Kyoto Summit only a few years ago exceeds by far that which scientists at the Kyoto Summit had predicted only a few years ago exceed by far those that scientists at the Kyoto Summit had predicted only a few years ago far exceed what scientists at the Kyoto Summit were predicting only a few years ago

7. Several stock market analysts now report that the plummeting values of many high-tech stocks have fallen so as to make them once again attractive to investors. (A) (B) (C) (D) (E)

values of many high-tech stocks have fallen so as to make high-tech stock values have fallen, making descent of many high-tech stock values have made values of many high-tech stocks are making values of many high-tech stocks have fallen, which has made

8. Unlike its fellow Baltic nations. Latvia and Lithuania, the economy of Estonia grew at an astonishing rate in the late 1990s, and at the end of the decade it was placed on the fast track to join the European Union. (A) (B) (C) (D) (E)

224

its fellow Baltic nations, Latvia and Lithuania, the economy of Estonia grew at an astonishing rate in the late 1990s, and at the end of the decade it was placed on the fast track to join its fellow Baltic nations, Latvia and Lithuania, Estonia's economy grew at an astonishing rate in the late 1990s, and at the end of the decade was placed on the fast track to join its fellow Baltic nations, Latvia and Lithuania, Estonia's economy grew at an astonishing rate in the late 1990s, and at the end of the decade they were placed on the fast track to join Latvia and Lithuania, its fellow Baltic nations, the economy of Estonia grew at an astonishing rate in the late 1990s, and at the end of the decade was placed on the fast track to join Latvia and Lithuania, its fellow Baltic nations, Estonia possessed an economy that grew at an astonishing rate in the late 1990s, and at the end of the decade the country was placed on the fast track to join


9. As did many other newer American cities, Atlanta doubled in size in only its first ten years of existence. (A) (B) (C) (D) (E)

As did As have Like Just like As with

10. (BG) Accredited travel agents were not required to provide advice pertaining to hotels and entertainment when they were organizing travel packages, but many do so anyway in an attempt to secure repeat business. (A) (B) (C) (D) (E)

were not required to provide advice pertaining to hotels and entertainment when they were organizing travel packages, but many do so are not required to provide advice pertaining to hotels and entertainment when they organize travel packages, but many do so were not required for providing hotel and entertainment advice when they organized travel packages, but many do it were not required that they provide advice pertaining to hotels and entertainment when they organized travel packages, but many had been doing so had not had the requirement for them to provide advice pertaining to hotels and entertainment when they organize travel packages, but many do it

11. Tests show that catfish from Lake Apopka are safe to eat, even though they contain almost twice as much of the pesticide DDT this year than they did last year. (A) (B) (C) (D) (E)

than they did more than they did as they did than they had contained than they contained

12. Inflation in the United States has not and, we hope, never will reach a rate of 20 percent a year. (A) (B) (C) (D) (E)

and, we hope, never will reach reached and, we hope, never will and hopefully never will reach reached and, we hope, never will reach reached and hopefully never will

225


13. King James I of England tried unsuccessfully to merge the legislature of Scotland—his original kingdom—with England. (A) (B) (C) (D) (E)

with England and England with that of England and England`s and England`s legislature

14. One student at the school was not yet fifteen, yet he was already a master in both chess and in bridge. (A) (B) (C) (D) (E)

master in both chess and in bridge master of both chess and in bridge master of both chess and of bridge master both chess and bridge master of both chess and bridge

15. It has been demonstrated that individuals who participated in competitive sports when they are in college tend towards remaining physically active in their later years. (A) (B) (C) (D) (E)

when they are in college tend towards remaining when they were in college are likely to be when they were in college were apt to be when they are in college are apt to be when they were in college were liable to be

16. Unlike the Puritan ethic, extolling hard work as the supreme virtue, many modern psychologists focus on the goals of inner peace and self-discovery. (A) (B) (C) (D) (E)

226

Unlike the Puritan ethic, extolling hard work as the supreme virtue, many modern psychologists focus on the goals of inner peace and self-discovery Unlike believers in the Puritan ethic, which extols as the supreme virtue hard work, the goals of inner peace and self-discovery are focused on by many modern psychologists Unlike the Puritan ethic, which extols hard work as the supreme virtue, the focus of the doctrines of many modern psychologists are the goals of inner peace and self-discovery Unlike the Puritan ethic, which extols hard work as the supreme virtue, the doctrines of many modern psychologists focus on the goals of inner peace and self-discovery Unlike those of the Puritans, who extol hard work as the supreme virtue, many modern psychologists focus on the goals of inner peace and self-discovery


17. The customer will not be responsible for transactions made with lost or stolen credit cards after issuing replacement cards. (A) (B) (C) (D) (E)

The customer will not be responsible for transactions made with lost or stolen credit cards after issuing replacement cards No customer will be responsible for transactions made with lost or stolen credit cards after they are issued replacement cards The customer will not be responsible for transactions made with lost or stolen credit cards after replacement cards have been issued Responsibility for transactions made with lost or stolen credit cards will not be the customer's after he is issued replacement cards The customer will not be responsible for transactions made with lost or stolen credit cards after such time as they will have been issued replacement cards

18. Despite that they were able to calculate faster than ever before possible, the earliest computers, built with tubes instead of transistors, were too bulky, expensive, and unreliable to be useful to businesses. (A) (B) (C) (D) (E)

Despite that they were able to calculate faster than ever before possible Even though it could calculate faster than previously possible Although faster in its calculating than ever before possible Despite their calculations being faster than ever before possible Despite their unprecedented speed of calculation

19. As genetics researchers achieve greater success in their attempt to map the exact locations and functions of human genes, the close connection between genes and birth defects has become increasingly clear. (A) (B) (C) (D) (E)

close connection between genes and birth defects has become closeness of the connection between genes and birth defects has become close connection genes have with birth defects has been becoming close connection between genes and birth defects has been becoming close connection between genes and birth defects becomes

20. The three largest American airlines stunned the financial world by announcing a full-scale merger of their business, which created an alliance controlling over 60 percent of all domestic air traffic. (A) (B) (C) (D) (E)

business, which created an alliance controlling businesses, creating an alliance that would control businesses that created a controlling alliance of business, and this alliance controlled business that created an alliance that would control

227


(OG) Set 3 1. The Glass House Mountains in Queensland, Australia, were sighted in 1770 by the English navigator Captain James Cook, by whom they were named supposedly because its sheer wet rocks glistened like glass. (A) (B) (C) (D) (E)

by whom they were named supposedly because its by whom they were named supposedly and their naming them supposedly since their who so named them supposedly because their who so named it since supposedly their

2. Although a surge in retail sales have raised hopes that there is a recovery finally under way, many economists say that without a large amount of spending the recovery might not last. (A) (B) (C) (D) (E)

have raised hopes that there is a recovery finally raised hopes for there being a recovery finally had raised hopes for a recovery finally being has raised hopes that a recovery is finally raised hopes for a recovery finally

3. Of all the vast tides of migration that have swept through history, maybe none is more concentrated as the wave that brought 12 million immigrants onto American shores in little more than three decades. (A) (B) (C) (D) (E)

maybe none is more concentrated as it may be that none is more concentrated as perhaps it is none that is more concentrated than maybe it is none that was more concentrated than perhaps none was more concentrated than

4. Diabetes, together with its serious complications, ranks as the nation's third leading cause of death, surpassed only by heart disease and cancer. (A) (B) (C) (D) (E)

228

ranks as the nation's third leading cause of death, surpassed only rank as the nation's third leading cause of death, only surpassed has the rank of the nation's third leading cause of death, only surpassed are the nation's third leading causes of death, surpassed only have been ranked as the nation's third leading causes of death, only surpassed


5. In late 1997, the chambers inside the pyramid of the Pharaoh Menkaure at Giza were closed to visitors for cleaning and repair due to moisture exhaled by tourists, which raised its humidity to such levels so that salt from the stone was crystallizing and fungus was growing on the walls. (A) (B) (C) (D) (E)

due to moisture exhaled by tourists, which raised its humidity to such levels so that salt from the stone was crystallizing due to moisture that tourists had exhaled, thereby raising its humidity to such levels that salt from the stone would crystallize because tourists were exhaling moisture, which had raised the humidity within them to levels such that salt from the stone would crystallize because of moisture that was exhaled by tourists raising the humidity within them to levels so high as to make the salt from the stone crystallize because moisture exhaled by tourists had raised the humidity within them to such levels that salt from the stone was crystallizing

6. As its sales of computer products have surpassed those of measuring instruments, the company has become increasingly willing to compete for the mass market sales they would in the past have conceded to rivals. (A) (B) (C) (D) (E)

they would in the past have conceded to rivals they would have conceded previously to their rivals that in the past would have been conceded previously to rivals it previously would have conceded to rivals in the past it would in the past have conceded to rivals

7. The widely accepted big bang theory holds that the universe began in an explosive instant ten to twenty billion years ago and has been expanding ever since.

(A) (B) (C) (D) (E)

that the universe began in an explosive instant ten to twenty billion years ago and has been expanding that the universe had begun in an explosive instant ten to twenty billion years ago and had been expanding that the beginning of the universe was an explosive instant ten to twenty billion years ago that has expanded the beginning of the universe to have been an explosive instant ten to twenty billion years ago that is expanding the universe to have begun in an explosive instant ten to twenty billion years ago and has been expanding

8. Like the idolization accorded the Brontes and Brownings, James Joyce and Virginia Woolf are often subjected to the kind of veneration that blurs the distinction between the artist and the human being.

229


(A) (B) (C) (D) (E)

Like the idolization accorded the Brontes and Brownings, As the Brontes' and Brownings' idolization, Like that accorded to the Brontes and Brownings, As it is of the Brontes and Brownings, Like the Brontes and Brownings,

9. Carnivorous mammals can endure what would otherwise be lethal levels of body heat because they have a heat-exchange network which kept the brain from getting too hot. (A) (B) (C) (D) (E)

which kept that keeps which has kept that has been keeping having kept

10. There are several ways to build solid walls using just mud or clay, but the most extensively used method has been the forming of bricks out of mud or clay, and, after some preliminary air drying or sun drying, they are laid in the wall in mud mortar.

(A) (B) (C) (D) (E)

the forming of bricks out of mud or clay, and, after some preliminary air drying or sun drying, they are laid forming the mud or clay into bricks, and, after some preliminary air drying or sun drying, to lay them having bricks formed from mud or clay, and, after some preliminary air drying or sun drying, they were laid to form the mud or clay into bricks, and, after some preliminary air drying or sun drying, to lay them that bricks were formed from mud or clay, which, after some preliminary air drying or sun drying, were laid

11. Rising inventories, when unaccompanied correspondingly by increases in sales, can lead to production cutbacks that would hamper economic growth.

(A) when unaccompanied correspondingly by increases in sales, can lead (B) when not accompanied by corresponding increases in sales, possibly leads (C) when they were unaccompanied by corresponding sales increases, can lead (D) if not accompanied by correspondingly increased sales, possibly leads (E) if not accompanied by corresponding increases in sales, can lead

230


12. Warning that computers in the United States are not secure, the National Academy of Sciences has urged the nation to revamp computer security procedures, institute new emergency response teams, creating a special nongovernment organization to take charge of computer security planning.

(A) (B) (C) (D) (E)

creating a special nongovernment organization to take creating a special nongovernment organization that takes creating a special nongovernment organization for taking and create a special nongovernment organization for taking and create a special nongovernment organization to take

13. Retail sales rose 0.8 of 1 percent in August, intensifying expectations that personal spending in the July-September quarter more than doubled that of the 1.4 percent growth rate in personal spending for the previous quarter.

(A) (B) (C) (D) (E)

that personal spending in the July-September quarter more than doubled that of that personal spending in the July-September quarter would more than double of personal spending in the July-September quarter, that it more than doubled of personal spending in the July-September quarter more than doubling that of of personal spending in the July-September quarter, that it would more than double that of

14. Plants are more efficient at acquiring carbon than are fungi, in the form of carbon dioxide, and converting it to energy-rich sugars.

(A) (B) (C) (D) (E)

Plants are more efficient at acquiring carbon than are fungi, Plants are more efficient at acquiring carbon than fungi, Plants are more efficient than fungi at acquiring carbon, Plants, more efficient than fungi at acquiring carbon, Plants acquire carbon more efficiently than fungi,

15. As contrasted with the honeybee, the yellow jacket can sting repeatedly without dying and carries a potent venom that can cause intense pain.

(A)

As contrasted with the honeybee,

231


(B) In contrast to the honeybee's, (C) Unlike the sting of the honeybee, (D) Unlike that of the honeybee, (E) Unlike the honeybee,

16. Neuroscientists. having amassed a wealth of knowledge over the past twenty years about the brain and its development from birth to adulthood, are now drawing solid conclusions about how the human brain grows and how babies acquire language.

(A) (B) (C) (D) (E)

Neuroscientists, having amassed a wealth of knowledge over the past twenty years about the brain and its development from birth to adulthood, are Neuroscientists, having amassed a wealth of knowledge about the brain and its development from birth to adulthood over the past twenty years, and are Neuroscientists amassing a wealth of knowledge about the brain and its development from birth to adulthood over the past twenty years, and are Neuroscientists have amassed a wealth of knowledge over the past twenty years about the brain and its development from birth to adulthood, Neuroscientists have amassed, over the past twenty years, a wealth of knowledge about the brain and its development from birth to adulthood,

17. None of the attempts to specify the causes of crime explains why most of the people exposed to the alleged causes do not commit crimes and, conversely, why so many of those not so exposed have.

(A) (B) (C) (D) (E)

have has shall do could

18. In a previous design, the weight of the discus used in track competition is concentrated in a metal center, but now it is lined with lead around the perimeter, thereby improving stability in flight and resulting in longer throws.

(A) (B)

232

In a previous design, the weight of the discus used in track competition is concentrated in a metal center, but now it is According to a previous design, the weight of the discus used in track competition was concentrated in a metal center, but now it is


(C) (D) (E)

Once designed with its weight concentrated in a metal center, the discus used in track competition is now The discus used in track competition, once designed with its weight concentrated in a metal center, but now The discus used in track competition was once designed having its weight concentrated in a metal center and now

19. Based on accounts of various ancient writers, scholars have painted a sketchy picture of the activities of an all-female cult that, perhaps as early as the sixth century B.C., worshipped a goddess known in Latin as Bona Dea, "the good goddess."

(A) (B) (C) (D) (E)

Based on accounts of various ancient writers, Basing it on various ancient writers' accounts, With accounts of various ancient writers used for a basis, By the accounts of various ancient writers they used, Using accounts of various ancient writers,

20. The price of a bushel of corn has fallen so drastically that some farmers have found it to be more cost-effective to destroy their crops as to make the effort to get them to market.

(A) have found it to be more cost-effective to destroy their crops as to make (B) found that the destruction of their crops is more cost-effective than making (C) find the destruction of their crops as more cost-effective than making (D) find it more cost-effective to destroy their crops than to make (E) are finding that the destruction of their crops is more cost-effective than the making of

(K) Set 4

1. That young girls score as well if not better than young boys on standardized tests disprove one prevalent male-supremacy myth.

(A) (B) (C) (D) (E)

That young girls score as well if not better than young boys on standardized tests disprove That young girls score as well if not better than young boys on standardized tests disproves That young girls score as well as if not better than young boys on standardized tests disproves That young girls score so well as if not better than young boys on standardized tests disprove The fact of young girls' scoring as well if not better than young boys on standardized tests

233


disprove

2. During her testimony, the witness referred to facts about the defendant that had been ruled inadmissible as evidence, so the judge ordered that her remarks were stricken from the record.

(A) (B) (C) (D) (E)

were stricken from the record are stricken from the record be stricken from the record are not recorded will not be recorded

3. To restore fishing grounds damaged by pollution, marine engineers can create an artificial reef by towing old barges to an offshore location and sinking to the sandy bottom.

(A) (B) (C) (D) (E)

and sinking and sinking them and sinking it where it sinks having sunk them

4. Popular child psychologists have advocated that parents discipline male children similarly to the fashion in which they discipline daughters.

(A) (B) (C) (D) (E)

similarly to the fashion in which they discipline in the same manner that they would use with like they would handle as they discipline as they would

5. In the Middle Ages philosophers were so devoted to Aristotle that they neglected the evidence of their own senses and accepted whatever he wrote.

(A) (B)

234

were so devoted to Aristotle that they neglected were so devoted to Aristotle as to neglect


(C) (D) (E)

were devoted to Aristotle to such a degree that they were to neglect were so devoted to Aristotle that they had to neglect were as devoted to Aristotle as to neglect

6. To rely on anecdotal evidence of social phenomena is ignoring decades of increasing sophistication in the use of statistics in sociology.

(A) (B) (C) (D) (E)

To rely on anecdotal evidence of social phenomena is ignoring To rely on anecdotal evidence of social phenomena is to ignore To rely on anecdotes for demonstration of social phenomena is ignoring Relying on anecdotal evidence of social phenomena is to ignore Relying on anecdotal evidence of social phenomena amounts to the ignoring of

7. The team streamed into the locker room, donned their uniforms, and, before commencing its first practice, they joined in a brief strategy session.

(A) (B) (C) (D) (E)

and, before commencing its first practice, they joined in a brief strategy session and its first practice was preceded by a brief strategy session and a brief strategy session preceded its first practice and, before commencing their first practice, they joined in a brief strategy session and, before commencing their first practice, joined in a brief strategy session

8. Records of the first 736 British convicts deported to Australia reveal convictions for crimes against property in all cases and they ranged from highway robbery to forgery.

(A) (B) (C) (D) (E)

convictions for crimes against property in all cases and they ranged convictions in all cases were for crimes against property and ranging the ranging of convictions for crimes against property in all cases that all were convicted of crimes against property ranging that all of them had convictions for crimes that were against property; the range was

9. A turning point in U.S. labor history was reached when most of the nation's air traffic controllers supported their union's strike and ignored President Reagan's demand that they go back to work.

235


(A) (B) (C) (D) (E)

that they go back to work that these people go back to work for them to go back to work that they would go back to work they should go back to work

10.

The symphonies of Mahler are characterized by a much freer use of dissonance than Haydn.

(A) (B) (C) (D) (E)

a much freer use of dissonance than Haydn a much freer usage than Haydn of dissonance a much freer use of dissonance than are those of Haydn a usage much freer than the dissonance of Haydn a use of dissonance that is much freer than Haydn

11.

To be a leading producer in the computer industry, a company must be international, achieve a turnover that makes efficient large-scale production possible, and secure information about technical advances.

(A) (B) (C) (D) (E)

and secure information about technical advances and securing information about technical advances while secure information about technical advances and secure information of how to technically advance and secure information in regards to technical advances

12.

Of the people who brought about the Reformation, the religious revolution that grew out of objections to the doctrines of the medieval church, Martin Luther is the best known of them.

(A) (B) (C) (D) (E)

Martin Luther is the best known of them the best known is Martin Luther the better known of them was Martin Luther Martin Luther is better known the best known one was Martin Luther

13.

The bylaws of a corporation specify how the directors of the corporation are to be elected, whether the existing stockholders will have first right to buy any new stock issued by the firm, as well as duties of management committees.

236


(A) (B) (C) (D) (E)

as well as duties of management committees as well as the duties that the management committees have and that the management committees have duties and the management committees have duties too and what the duties of management committees will be

14.

Although it is conceivable that certain electronic devices implanted in the brain may someday correct blindness caused by nerve damage, there is now no clear evidence of their ability to do it.

(A) (B) (C) (D) (E)

of their ability to do it of their doing that that they can do so that they might one day be able to do it to do so

15.

Setting up a corporation requires more paper-work and legal provision than to establish either a proprietorship or a partnership.

(A) (B) (C) (D) (E)

than to establish either a proprietorship or a partnership than what it requires to establish either a proprietorship or a partnership than to either establish a proprietorship or a partnership than establishing either a proprietorship or a partnership than establishing proprietorships and partnerships

16.

The Minnesota Multiphasic Personality Inventory, a test developed in the 1930s to assess character, attitudes, and behavior, is finally being revised, after many attacks recently as dated and culturally biased.

(A) (B) (C) (D) (E)

is finally being revised, after many attacks recently as dated and culturally biased has recently been attacked as dated and culturally biased and is finally being revised is finally under revision for receiving much recent attack as dated and culturally biased was finally revised recently because it is being attacked as dated and culturally biased is finally to be revised after recent attacks as dated and culturally biased

237


17.

In the Champagne region of France, wine growers still harvest grapes as they have done

for centuries, by carefully handpicking their crop and storing their precious fruit in specially aged casks.

(A) (B) (C) (D) (E)

wine growers still harvest grapes as they have done the wine grower still harvests grapes as he has wine growers still harvest grapes as was done wine growers still harvest grapes as has been done wine growers still harvest grapes as they did

18.

In an unusual move for a Republican, the president's directive established price controls.

(A) (B) (C) (D) (E)

the president's directive established price controls the president's directive was to establish price controls the president was successful in the establishment of controls on prices the president had instituted controls on prices the president established price controls

19.

Otto Wichterle, the Czech inventor who created the first soft contact lens, was not like most successful inventors due to his making a fortune from human vanity instead of being inspired by necessity.

(A)

Otto Wichterle, the Czech inventor who created the first soft contact lens, was not like most successful inventors due to his making a fortune from human vanity instead of being inspired by necessity. The Czech inventor Otto Wichterle, who was not like other inventors' developments that were inspired by necessity, made his fortune from human vanity instead by creating the first soft contact lens. Unlike most successful inventors, whose developments were inspired by necessity, Otto Wichterle, the Czech inventor who created the first soft contact lens, made a fortune from human vanity. The first soft contact lens, which was created by Czech inventor Otto Wichterle, who made a fortune from human vanity, was unlike most successful inventors whose developments were inspired by necessity. The developments of most successful inventors, which had been inspired by necessity, were unlike the first soft contact lens that was created by Czech inventor Otto Wichterle, who made a fortune from human vanity.

(B)

(C)

(D)

(E)

238


20.

The government's attempts to store chemical weapons in a rural community in Oregon, a state with a decidedly environ mentalist history, have encountered massive political resistance from Oregon's state legislature.

(A) (B) (C) (D) (E)

have encountered massive political resistance has encountered massive resistance politically have politically encountered massive resistance has encountered massive political resistance had encountered politically massive resistance

(K) Set 5 1.

The chairman of the board of directors have decided that this year's profits will be used for research and development, rather than for a shareholder dividend.

(A) (B) (C) (D) (E)

have decided that this year's profits will be used has decided that this year's profits will be used have decided on using this year's profits has decided on using this year's profits decided this year's profits will have been used

2.

Like most religions, the teachings of Sikhism are codified in a book.

(A) (B) (C) (D) (E)

Like most religions, the teachings of Sikhism are codified in Like most other religions, the teachings of Sikhism are codified in Sikhism's teachings, like those of most other religions, are codified in For their codification, like the teachings of most religions, Sikhism has Like those of most religions, Sikhism has codified its teachings in

3.

Not only do deep-sea divers risk nitrogen narcosis, often called "raptures of the deep," if they descend below 200 feet, but they also fall prey to decompression sickness, commonly known as "the bends," if they ascend too quickly.

(A)

Not only do deep-sea divers risk nitrogen narcosis, often called "raptures of the deep," if they descend below 200 feet, but Deep-sea divers risk nitrogen narcosis, often called "raptures of the deep," if they descend below 200 feet, but Nitrogen narcosis, often called "raptures of the deep," is risked by deep-sea divers if they descend below 200 feet, in addition The descending of deep-sea divers to below 200 feet causes them risking nitrogen narcosis, often called "raptures of the deep," and Not only does a deep-sea diver risk nitrogen narcosis, often called "raptures of the deep," if they descend below 200 feet, but

(B) (C) (D) (E)

239


4.

A 32 percent wage hike, working four days a week, and above all a higher standard of factory safety is a demand that management must meet if it wishes to avoid a crippling strike.

(A)

A 32 percent wage hike, working four days a week, and above all a higher standard of factory safety is a demand A 32 percent wage hike, a four-day work-week, and above all a higher standard of factory safety is a demand A 32 percent wage hike, a four-day work-week, and above all a higher standard of safety in the factory are demands A 32 percent wage hike, working four days a week, and above all a higher standard of factory safety are demands Hiking wages 32 percent, working four days a week, and above all heighten the standard of factory safety are demands

(B) (C) (D) (E)

5.

Although new farm subsidy legislation is presently being considered in the House of Representatives, significant revision is to be expected if it is to be passed.

(A) (B) (C) (D) (E)

significant revision is to be expected if it is to be passed they do not expect it passing without significant revision they do not expect it passing without it being significantly revised it is not expected to pass without it being significantly revised it is not expected to pass without significant revision

6.

At a recent conference, the Transit Authority has stated that the provisions in the Clean Air Act fail to promote mass transit as an alternative to private transportation.

(A) (B) (C) (D) (E)

has stated that the provisions in the Clean Air Act fail stated that the provisions of the Clean Air Act fail has stated that the provisions of the Clean Air Act will fail stated that the provisions in the Clean Air Act are a failure has stated the provisions in the Clean Air Act failed

7.

A revolution has taken place in medical science as a result of the introduction of new methods of surgically implanting artificial and human organs.

(A) (B) (C) (D) (E)

the introduction of new methods of surgically implanting artificial and human organs the introduction of new surgical implantation methods of human and artificial organs the surgical introduction of new artificial and human organ implantation methods the introduction of implantation methods of new artificial and human organs the introduction of methods of surgically implanting new artificial and human organs

240


8.

Added to worries about budget cuts and cost of living increases, the administrators of public health care facilities must contend with the possibility of a strike by employees.

(A) (B) (C) (D) (E)

Added to worries about budget cuts and cost of living increases Added to budget cuts and increases in the cost of living In addition to worry about cuts in the budget and as the cost of living increases Added to their worrying about budget cuts and cost of living increases In addition to worrying about budget cuts and increases in the cost of living

9.

Renaissance scientist Copernicus found that his rejection of the Ptolemaic system placed him at odds not only with the Church and the scientific community of his day, but also with ship captains, who navigated according to a geocentric universe.

(A)

not only with the Church and the scientific community of his day, but also with ship captains, who navigated according to a geocentric universe opposing the Church and the scientific community of his day, and also the ship captains, who navigated according to a geocentric universe not only in opposition to the Church and the scientific community of his day, but with ship captains, who navigated according to a geocentric universe not only with the Church and the scientific community of his day, but with ship captains, who calculated their navigations on a geocentric basis opposing not only churchmen and scientists, but the ship captains, who navigated according to a geocentric universe

(B) (C) (D) (E)

10.

Despite them attempting to do it, doctors have not yet found a cure for the common cold.

(A) (B) (C) (D) (E)

them attempting to do it their attempts to do so them attempting to do so the fact that they have attempted to do it their attempts to do it

11.

The United Automobile Workers returned to the A.F.L.-C.I.O. because of the wish of both groups to strengthen labor's role not only in politics but also industry.

(A) (B) (C) (D) (E)

of the wish of both groups to strengthen labor's role not only in politics but also industry both groups wished to strengthen labor's role in politics as well as in industry the strengthening of labor's role in politics as well as industry was wished for by both groups it was wished by both groups to strengthen labor's role in politics and industry as well both groups wished to strengthen labor's role in both of the two areas of politics and industry

241


12.

Unlike the people whom settled the Western states after the passage of the Homestead Act of 1862, the hope of the prospectors and adventurers who came to California during the gold rush was to get rich quickly.

(A)

the people whom settled the Western states after the passage of the Homestead Act of 1862, the hope of the prospectors and adventurers who came to California during the gold rush was the people who settled the Western states after the passage of the Homestead Act of 1862, the hope of the prospectors and adventurers who came to California during the gold rush was that of the people who settled the Western states after the passage of the Homestead Act of 1862, the prospectors and adventurers who came to California during the gold rush hoped that of the people whom settled the Western states after the passage of the Homestead Act of 1862, the hope of the prospectors and adventurers who came to California during the gold rush was the people who settled the Western states after the passage of the Homestead Act of 1862, the prospectors and adventurers who came to California during the gold rush hoped

(B) (C) (D)

(E)

13.

Before George Eliot became the popular and respected novelist known as George Eliot, she was an anonymous translator and essayist of formidably far-ranging scholarship.

(A)

Before George Eliot became the popular and respected novelist known as George Eliot, she was Before she had been the popular and respected novelist, George Eliot, she was George Eliot has been the popular and respected novelist, George Eliot, after such time as she was Before George Eliot became the popular and respected novelist, George Eliot, she was George Eliot, before she was the popular and respected novelist, George Eliot, had been

(B) (C) (D) (E)

14.

Just like Congress is the legislative branch of the Federal government of the United States, so Parliament is the legislative body of the United Kingdom of Great Britain and Northern Ireland.

(A) (B) (C)

Just like Congress is the legislative branch of the Federal government of the United States, so As Congress is the legislative branch of the Federal government of the United States, As Congress is the legislative branch of the Federal government of the United States, in the same way Just as Congress is the legislative branch of the Federal government of the United States, so Just as the Federal government of the United States' legislative branch is Congress,

(D) (E)

15.

The reason Frances Willard founded the Women's Christian Temperance Union was because she believed that national prohibition of alcohol will empty the poorhouses, jails, and asylums of the United States.

(A)

The reason Frances Willard founded the Women's Christian Temperance Union was because she believed that national prohibition of alcohol will empty the poorhouses, jails, and asylums

242


(B)

(C)

(D)

(E)

of the United States Frances Willard founded the Women's Christian Temperance Union, the reason being that she believed that national prohibition of alcohol will empty the poorhouses, jails, and asylums of the United States The reason Frances Willard founded the Women's Christian Temperance Union was she believed that national prohibition of alcohol will empty the poorhouses, jails, and asylums of the United States Because she believed that national prohibition of alcohol would empty the poorhouses, jails, and asylums of the United States, so Frances Willard founded the Women's Christian Temperance Union Frances Willard founded the Women's Christian Temperance Union because she believed that national prohibition of alcohol would empty the poorhouses, jails, and asylums of the United States

16.

The combination of technical expertise, commercial enterprise and that the government backs them judiciously should ensure Italian firms continuing to innovate and gain competitively in world markets.

(A) (B)

that the government backs them judiciously should ensure Italian firms continuing the government backing them judiciously should ensure Italian firms of being about to continue judicious government backing should ensure that Italian firms will continue the government's judicious backing should ensure Italian firms that they will continue the government to back them judiciously should ensure Italian firms of continuing

(C) (D) (E)

17.

The sloth, which is a South American mammal related to armadillos and anteaters, live in tropical forests where they travel through the trees upside down.

(A)

which is a South American mammal related to armadillos and anteaters, live in tropical forests where they travel a South American mammal related to armadillos and anteaters, live in tropical forests where they travel a South American mammal related to the armadillo and the anteater, lives in tropical forests where it travels a South American mammal related to the armadillo and the anteater, lives in tropical forests where they travel a South American mammal in relation to the armadillo and the anteater, lives in tropical forests where it travels

(B) (C) (D) (E)

18.

Early derisive reactions from art critics and established painters did not discourage the Primitivist painter Henri Rousseau to exhibit repeatedly, despite him completely lacking formal training and starting late as a professional artist.

(A)

did not discourage the Primitivist painter Henri Rousseau to exhibit repeatedly, despite him

243


(B) (C) (D) (E)

completely lacking formal training and starting late did not discourage the Primitivist painter Henri Rousseau exhibiting repeatedly, completely lacking formal training though he was, and starting late did not discourage the Primitivist painter Henri Rousseau to exhibit repeatedly, despite their complete lack of formal training and late start did not discourage the Primitivist painter Henri Rousseau in exhibiting repeatedly, despite his complete lack of formal training and late start did not discourage the Primitivist painter Henri Rousseau from exhibiting repeatedly, despite his complete lack of formal training and late start

19.

The poll taxes enacted in the Southern states between 1889 and 1910 disenfranchised many citizens, since payment of the tax was a prerequisite for voting.

(A) (B) (C) (D) (E)

and 1910 disenfranchised many citizens, since to 1910 disenfranchised many citizens, since and 1910 have disenfranchised many citizens since to 1910 has been disenfranchising many citizens because and 1910 had the effect of disenfranchising many citizens, inasmuch as

20.

With a population equal to Kansas in an area one three-hundredth its size. Singapore is the most densely populated nation in the world, averaging almost ten thousand people per square mile.

(A)

With a population equal to Kansas in an area one three-hundredth its size, Singapore is the most densely populated nation in the world, averaging With a population equal to that of Kansas in an area one three-hundredth its size, Singapore is the most densely populated nation in the world, averaging With Kansas' population in one three-hundredth of its area, the densest population in the world is that of Singapore, with an average of Singapore has the same population as Kansas but only one three-hundredth of its area, and makes it the most densely populated nation in the world, averaging Singapore is the most densely populated nation in the world, with Kansas' population in one three-hundredth of its area, or an average of

(B) (C) (D) (E)

(K) Set 6 1.

Unlike the Pulitzer, which is given for a specific work, an author receives the Nobel Prize for a lifetime's achievement in literature.

(A) (B) (C) (D) (E)

an author receives the Nobel Prize for a lifetime's achievement in literature an author's lifetime achievement in literature receives the Nobel Prize the Nobel Prize is awarded to an author's lifetimes achievement in literature the Nobel Prize is awarded for a lifetime of achievement in literature the Nobel Prize is awarded for a lifetime of achievement in literature by an author

244


2.

Medical researchers, who have identified a genetic abnormality in parents of children with Down's syndrome, which they believe, instead of the age of the mother, may be the cause of this congenital birth defect.

(A) (B) (C) (D) (E)

which they believe, instead of the age of the mother, may be which, they believe, may be more important than the age of the mother as believe this abnormality and not the age of the mother as being believe that this abnormality, other than how old the mother is, may be believe that this abnormality, rather than the age of the mother, may be

3.

Unable to walk without assistance, radio provided Franklin Roosevelt with a medium for conveying a message of vigor and confidence that would have been belied by the visual image presented on television.

(A) (B) (C) (D) (E)

radio provided Franklin Roosevelt with a medium for conveying Franklin Roosevelt conveyed in radio Franklin Roosevelt used radio to convey radio allowed Franklin Roosevelt to convey through radio it was possible

4.

Modern inventions such as the pneumatic drill may seem to have improved the work conditions of manual laborers, but in fact, have increased the incidence of health hazards such as silicosis because it has aggravated the dust hazard involved in excavation and demolition.

(A)

have increased the incidence of health hazards such as silicosis because it has aggravated the dust hazard has increased the incidence of health hazards such as silicosis because it has aggravated the dust hazard have increased the incidence of health hazards such as silicosis because they have aggravated the dust hazard increased the incidence of health hazards such as silicosis because they aggravated the dust hazard increased the incidence of health hazards such as silicosis because of aggravating the dust hazard

(B) (C) (D) (E)

5.

The Federal Reserve Board assumes primary responsibility of the regulation for this nation's commercial banks and savings institutions.

(A) (B) (C) (D)

Board assumes primary responsibility of the regulation for Board's assumption of primary responsibility for the regulation of Board's assuming primary responsibility to regulate Board assumes primary responsibility for the regulation of

245


(E)

Board regulates primary responsibility for

6.

The ground swell of public opinion made it inevitable that the Senate would approve the president's energy proposals.

(A) (B) (C) (D) (E)

it inevitable that the Senate would approve it inevitable that the Senate had approved it inevitable of the Senate to approve inevitable the approval of the Senate of the approval of the Senate inevitable of

7.

Though caterpillars transform large amounts of plant matter into animal tissue and wastes, and therefore hold an important place in the food chain, they are so voracious that they have become an economic threat to farmers whose crops they eat.

(A) (B) (C) (D) (E)

they are so voracious that they have become an economic they are of such voracity, they have become an economical so voracious are they as to become an economical such is their voracity, they become an economic there is so much voracity that it has become an economical

8.

The nations with nuclear capabilities have restrained the proliferation of nuclear weapons less by reaching military understandings than by the refusal to sell associated technologies.

(A) (B) (C) (D) (E)

by the refusal to sell not selling the refusal to sell refusing to sell by refusing to sell

9.

During and immediately after the California gold rush, the way for a merchant to generate the most profit was to move a limited amount of scarce goods to San Francisco as quickly as possible, rather than to carry larger loads more slowly, determining the design of the chipper ship.

(A) (B) (C) (D) (E)

to carry larger loads more slowly, determining to carry larger loads more slowly, a situation that determined carry larger loads more slowly, which determined slowly carry larger loads which determined carrying larger loads more slowly, and this was a situation in determining

246


10.

The analyst suggested that traders pay more attention to a broader measure of the money supply, known as M2, but still not to ignore standard indicators.

(A) (B) (C) (D) (E)

but still not to ignore and not to ignore any longer but that they still not ignore and not that they continue to ignore but that they are still not ignoring

11.

In the conflict between the Israelis and the Palestinians, the refusal of each side to acknowledge each other as a legitimate national movement is closer to the heart of the problem than is any other issue.

(A)

the refusal of each side to acknowledge each other as a legitimate national movement is closer to the heart of the problem than that the refusal of each side to acknowledge another as a legitimate national movement is closer to the heart of the problem as the refusal of each side to acknowledge another as a legitimate national movement is closer to the heart of the problem than that the refusal of each side to acknowledge another as a legitimate national movement is closer to the heart of the problem than the refusal of each side to acknowledge each other as a legitimate national movement is closer to the heart of the problem as

(B) (C) (D) (E)

12.

Agencies studying discrimination in housing have experimentally proved that minority clients are often discouraged as prospective buyers of residential real estate and the antidiscrimination legislation of recent decades were only mitigating, rather than abolishing, inequity in housing practices.

(A)

the antidiscrimination legislation of recent decades were only mitigating, rather than abolishing, inequity in housing practices in recent decades, the antidiscrimination legislation only mitigated, rather than abolishing, inequity in housing practices that antidiscrimination legislation of recent decades has only mitigated, rather than abolished, inequity in housing practices that, in recent decades, antidiscrimination legislation has only mitigated, rather than abolishing, housing practices' inequity that recent decades' antidiscrimination legislation only were mitigating, rather than abolishing, housing practices' inequity

(B) (C) (D) (E)

13.

Certain gerontologists have reported that the more older people continue to challenge their brains with reading, writing, and other thought-provoking exercises, their cognitive functions are less likely to diminish.

247


(A) (B) (C) (D) (E)

their cognitive functions are less likely to diminish the less likely are their cognitive functions to be diminished the less are they likely to have diminished cognitive function the less likely their cognitive functions will diminish they are less likely to have diminished cognitive function

14.

Archaeologists believe that the Dead Sea Scrolls, discovered in Khirbet Qumran between 1947 and 1967 and estimated at almost 2000 years old, provided rare insight into first century C. E. religious communities.

(A) (B) (C) (D) (E)

and estimated at almost 2000 years old, provided rare insight and estimated at almost 2000 years old, provide rare insight and estimated to be almost 2000 years old, to provide rare insight and estimated to be almost 2000 years old, provide rare insight and they estimate them to be almost 2000 years old, providing rare insight

15.

Proponents of the theory of spontaneous generation argued that simple living organisms sprang to life not through a recognizable reproductive process but came to life independently from non-living matter.

(A) (B) (C) (D) (E)

sprang to life not through a recognizable reproductive process but came to life did not spring to life through a recognizable reproductive process and came to life did not spring to life through a recognizable reproductive process but came to life sprang to life not through a recognizable reproductive process but did not spring to life through a recognizable system of reproduction but had come to life

16.

A new initiative requires that cable companies report to their new customers all the costs, including taxes, that they will be expected to pay.

(A) (B) (C) (D) (E)

to their new customers all the costs, including taxes, that they will be expected to pay to their new customers all the costs, including taxes, that the customers will be expected to pay to its new customers all the costs, including taxes, that they will have to pay to the companies' new customers all the costs, including taxes, that the companies have paid to their new customers all the costs, including taxes, that its customers will be expected to pay

17.

Imperceptible to the eye or the ear, carbon monoxide is formed from materials containing carbon, or carbonaceous materials, are only partially combusted.

(A) (B) (C) (D)

carbon monoxide is formed from carbon monoxide is formed by the formation of carbon monoxide is when carbon monoxide is formed when

248


(E)

carbon monoxide forms when its

18.

Galileo's theory that ours was indeed a solar system, in which Earth and other planets revolve around the sun, reinforced those of Copernicus.

(A)

ours was indeed a solar system, in which Earth and other planets revolve around the sun, reinforced those of Copernicus ours was indeed a solar system, in which Earth and other planets revolve around the sun, reinforced that of Copernicus our solar system was indeed solar for Earth and the other planets revolve around the sun, reinforced Copernicus' our system was indeed solar, in which Earth and other planets revolved around the sun, reinforced those of Copernicus Earth and the other planets revolve around the sun in our solar system reinforces that of Copernicus

(B) (C) (D) (E)

19.

Having lost his sight to sustained eyestrain, John Milton nevertheless composed Paradise Lost, considered by many to be the greatest English epic.

(A) (B) (C) (D) (E)

Having lost his sight to sustained eyestrain With his sight lost to sustained eyestrain Blinded by sustained eyestrain Having been blinded by excessive eyestrain Blinded with sustained eyestrain

20.

One benefit of learning to speak a language while learning to read it rather than separately is that the student can more effectively apply his or her knowledge in social settings.

(A) (B) (C) (D) (E)

rather than separately rather than independently instead of separately instead of as separate processes rather than in a separate process

(K) Set 7 1.

The increase in the number and scope of investigations into monopolistic business practices, in addition to the expansion of definitions of such practices, have resulted in more antimonopoly litigation presently than ever before.

(A) (B)

have resulted in more antimonopoly litigation presently than ever before has resulted in greater antimonopoly litigation presently than previously

249


(C) (D) (E)

has resulted in more antimonopoly litigation at present than was seen ever before have resulted in more antimonopoly litigation at present than ever before has resulted in more antimonopoly litigation at present than ever before

2.

Carthaginians are still commonly credited as the ones who salted Roman fields during the Punic Wars despite the existence of credible evidence to the contrary.

(A) (B) (C) (D) (E)

as the ones who salted as the salters of for salting with having salted with the salting of

3.

Charles Dickens viewed inadequate sanitation, as many Victorian authors, as a pressing and dangerous problem in London.

(A) (B) (C) (D) (E)

Charles Dickens viewed inadequate sanitation, as many Victorian authors, as Charles Dickens, like many Victorian authors, viewed inadequate sanitation as Many Victorian authors, like Charles Dickens, viewed inadequate sanitation as Inadequate sanitation was viewed by Charles Dickens, like many Victorian authors, as Charles Dickens was like many Victorian authors in that he viewed inadequate sanitation as

4.

One critic wrote that, if the best-selling guide to finding a husband is right, the idea of the necessity of honesty and trust in a relationship is fundamentally wrong.

(A) (B) (C) (D) (E)

the idea of the necessity of honesty and trust in a relationship the idea that honesty and trust are necessary in a relationship honesty and trust are not necessary in a relationship the idea that honesty and trust is necessary to a relationship the necessity of honest and trust to a relationship

5. The average income for a person with a bachelor of arts degree is several thousand dollars per year higher than somebody with a high school diploma only. (A) (B) (C) (D) (E)

250

than somebody with a high school diploma only than that for somebody with only a high school diploma than for somebody with a high school diploma only than it is for somebody with a high school diploma only that that of somebody having only a high school diploma


6. St. John’s, Newfoundland, lies on the same latitude as Paris, France, but in spring St. John’s residents are less likely to be sitting at outdoor cafes than to be bracing themselves against arctic chills, shoveling snow, or seeking shelter from a raging northeast storm. (A) (B) (C) (D) (E)

residents are less likely to be sitting at outdoor cafes than to be bracing themselves against arctic chills, shoveling snow, or seeking residents are less likely to sit at outdoor cafes, and more to brace themselves against arctic chills, shovel snow, or be seeking residents are less likely to be sitting at outdoor cafes, and more likely to be bracing themselves against arctic chills, shoveling snow, or to be seeking residents, instead of their sitting at outdoor cafes, they are more likely to brace themselves against arctic chills, shovel snow, or seek residents, instead of sitting at outdoor cafes, are more likely to brace themselves against arctic chills, shovel snow, or to be seeking

7. The black hole has entered the popular imagination as an object too massive that neither light nor matter can escape its gravitational pull. (A) (B) (C) (D) (E)

too massive that neither light nor matter can escape its too massive for either allowing light or matter to escape its massive enough that either light or matter cannot escape their so massive that neither light nor matter could escape their so massive that neither light nor matter can escape its

8. After crude oil, natural gas is the United States second biggest fuel source and supplied almost exclusively from reserves in North America. (A) (B) (C) (D) (E)

9.

(A)

After crude oil, natural gas is the United States second biggest fuel source and supplied almost exclusively from reserves in North America. Natural gas, after crude oil the United States second biggest fuel source, supplied almost exclusively from reserves in North America. Being supplied almost exclusively from reserves in North America, natural gas, the United States second biggest fuel source after crude oil. Natural gas, the United States’ second biggest fuel source after crude oil, is supplied almost exclusively from reserves in North America. Natural gas is supplied almost exclusively from reserves in North America, being the United States’ second biggest fuel source after crude oil.

The airline industry is cutting its lowest discount fares more widely, more substantially, and earlier this year than it normally does at the end of the summer, a time during which travel usually decreases and the industry uses some lower fares for the attraction of passengers. during which travel usually decreases and the industry uses some lower fares for the attraction of

251


(B) (C) (D) (E)

during which travel usually decreases and therefore the industry will use some lower fares for the attraction of in which travel usually decreases and in which the industry therefore uses some lower fares attracting when travel usually decreases and the industry uses some lower fares to attract when travel usually decreases and therefore the industry will use lower fares for the attraction of

10. The aristocratic values expressed in the writings of Marguerite Yourcenar place her within the French classical tradition, as does her passionate interest in history, particularly Roman history. (A) (B) (C) (D) (E)

as does so do as do so is the case with similarly, does

11. Selling several hundred thousand copies in six months, the publication of “Maple Leaf Rag” in 1899 was an instant hit, helping to establish Scott Joplin as the preeminent ragtime composer. (A) Selling several hundred thousand copies in six months, the publication of “Maple Leaf Rag” in 1899 was an instant hit, helping to establish Scott Joplin as the preeminent ragtime composer. (B) The publication in 1899 of “Maple Leaf Rag” was an instant hit: in six months they sold several hundred thousand copies and it helped establish Scott Joplin as the preeminent ragtime composer. (C) Helping to establish Scott Joplin as the preeminent ragtime composer was the publication of “Maple Leaf Rag” in 1899, which was an instant hit: it sold several hundred thousand copies in six months. (D) “Maple Leaf Rag” was an instant hit: it helped establish Scott Joplin as the preeminent ragtime composer, published in 1899 and selling several hundred thousand copies in six months. (E) Published in 1899, “Maple Leaf Rag” was an instant hit, selling several hundred thousand copies in six months: it helped establish Scott Joplin as the preeminent ragtime composer.

1. Because paper of all kinds is the biggest single component of municipal trash, many municipalities have tried recycling to reduce the cost of trash disposal. (A) Because paper of all kinds is the biggest single component of municipal trash, municipalities have tried recycling to reduce the cost of trash disposal. (B) Because paper of all kinds is the biggest single component in municipal trash, municipalities tried to recycle so that the cost of trash disposal is reduced. (C) Because paper of all kinds are the biggest single components in municipal trash, municipalities have tried to recycle to reduce the cost of trash disposal. (D) All kinds of paper are the biggest single components of municipal trash, and so municipalities have tried recycling to reduce the cost of trash disposal.

252

many many many many


(E)

All kinds of paper is the biggest single component of municipal trash, so many municipalities have tried to recycle so that the cost of trash disposal could be reduced.

12. As rare as something becomes, be it a baseball card or a musical recording or a postage stamp, the more avidly it is sought by collectors . (A) (B) (C) (D) (E)

As rare as something becomes, be it As rare as something becomes, whether it is As something becomes rarer and rarer, like The rarer something becomes, like The rarer something becomes, whether it is

13. Psychologists now contend that the way adults think and feel are determined as much by their peers in early childhood than by their parents. (A) (B) (C) (D) (E)

are determined as much by their peers in early childhood than by their are determined as much by peers in early childhood as do their is determined as much by their early childhood peers as by their have been determined by childhood peers as much as their was determined as much by one’s peers in childhood as by one’s

14. Migraine, the most debilitating common form of headache, afflicts perhaps 18 million Americans, who collectively lose 64 million workdays a year, and they cost the nation $50 billion in medical expenses and lost work time. (A) (B) (C) (D) (E)

year, and they cost the nation $50 billion in medical expenses and lost year and thus cost the nation $50 billion in medical expenses and lost year, so as to cost the nation $50 billion in medical expenses and lost year that costs the nation $50 billion in lost medical expenses and year, which thus cost the nation $50 billion in lost medical expenses and

15. Like many others of his generation of Native American leaders, Joseph Brant lived in two worlds; born into an Iroquois community and instructed in traditional Iroquois ways, he also received an education from English-speaking teachers. (A) (B) (C) (D)

Like many others of his generation of Native American leaders, Joseph Brant lived in two worlds; Like many others of his generation of Native American leaders, living in two worlds, Joseph Brant was Like many another of his generation of Native American leaders, Joseph Brant, living in two worlds, was As with many others of his generation of Native American leaders, living in two worlds,

253


(E)

Joseph Brant was As with many another of his generation of Native American leaders, Joseph Brant lived in two worlds;

16. Sales of United States manufactured goods to non-industrialized countries rose to $167 billion in 1992, which is 14 percent more than the previous year and largely offsets weak demand from Europe and Japan. (A) (B) (C) (D) (E)

which is 14 percent more than the previous year which is 14 percent higher than it was the previous year 14 percent higher than the previous year’s figure an amount that is 14 percent more than the previous year was an amount that is 14 percent higher than the previous year’s figure

17. All-terrain vehicles have allowed vacationers to reach many previously inaccessible areas, but they have also been blamed for causing hundreds of deaths, injury to thousands, and seriously damaging the nation’s recreational areas. (A) (B) (C) (D) (E)

deaths, injury to thousands, and seriously damaging deaths and injuring thousands, and serious damage to deaths, thousands who are injured, as well as seriously damaging deaths and thousands of injuries, as well as doing serious damage to deaths, thousands are injured, and they do serious damage to

18. Foreign investors, because of their growing confidence in their capability for making profitable investments in the United States, have been led to move from passive involvement in commercial real estate partnerships to active development of their own increasingly ambitious projects. (A) (B) (C) (D) (E)

254

Foreign investors, because of their growing confidence in their capability for making profitable investments in the United States, have been led Foreign investors, growing confident about their capability for making profitable investments in the United States, has led them Growing confidence in their ability to make profitable investments in the United States has led foreign investors Growing confidence in their ability for making profitable investments in the United States have led foreign investors Growing confident about their capabilities for making profitable investments in the United States, foreign investors have been led


19. The most prominent result of Professor Winick's archaeological research has been discovering that a pharaoh who had ruled in the last days of Egypt was buried with fewer artifacts than their earlier counterparts. (A) (B) (C) (D) (E)

discovering that a pharaoh who had ruled in the last days of Egypt was buried with fewer artifacts than their the discovery that pharaohs who had ruled in the last days of Egypt were not buried with as many artifacts as their to discover that pharaohs, which ruled in the last days of Egypt, was buried with fewer artifacts than their the discovery that pharaohs who ruled in the last days of Egypt were buried with fewer artifacts than were their to discover that a pharaoh who ruled in the last days of Egypt was buried with fewer artifacts than were his

(OG) Set 8 1. Emily Dickinson's letters to Susan Huntington Dickinson were written over a period beginning a few years before Susan's marriage to Emily's brother and ending shortly before Emily's death in 1886, outnumbering her letters to anyone else. (A) (B) (C) (D) (E)

Dickinson were written over a period beginning a few years before Susan's marriage to Emily's brother and ending shortly before Emily's death in 1886, outnumbering Dickinson were written over a period that begins a few years before Susan's marriage to Emily's brother and ended shortly before Emily's death in 1886, outnumber Dickinson, written over a period beginning a few years before Susan's marriage to Emily's brother and that ends shortly before Emily's death in 1886 and outnumbering Dickinson, which were written over a period beginning a few years before Susan's marriage to Emily's brother, ending shortly before Emily's death in 1886, and outnumbering Dickinson, which were written over a period beginning a few years before Susan's marriage to Emily's brother and ending shortly before Emily's death in 1886, outnumber

2. Paleontologists believe that fragments of a primate jawbone unearthed in Burma and estimated at 40 to 44 million years old provide evidence of a crucial step along the evolutionary path that led to human beings. (A) (B) (C) (D) (E)

at 40 to 44 million years old provide evidence of as being 40 to 44 million years old provides evidence of that it is 40 to 44 million years old provides evidence of what was to be 40 to 44 million years old provide evidence of as 40 to 44 million years old provides evidence of what was

255


3. Building on civilizations that preceded them in coastal Peru, the Mochica developed their own elaborate society, based on cultivating such crops like corn and beans, the harvesting of fish and shellfish, and exploiting other wild and domestic resources. (A) (B) (C) (D) (E)

based on cultivating such crops like corn and beans, the harvesting of fish and shellfish, and exploiting based on the cultivation of such crops as corn and beans, the harvesting of fish and seafood, and the exploitation of and basing it on the cultivation of crops like corn and beans, harvesting fish and seafood, and the exploiting of and they based it on their cultivation of crops such as corn and beans, the harvest of fish and seafood, and exploiting and they based it on their cultivating such crops like corn and beans, their harvest of fish and shellfish, and they exploited

4. The end of the eighteenth century saw the emergence of prize-stock breeding, with individual bulls and cows receiving awards, fetching unprecedented prices, and excited enormous interest whenever they were put on show. (A) (B) (C) (D) (E)

excited it excited exciting would excite it had excited

5. Unlike the conviction held by many of her colleagues that genes were relatively simple and static, Barbara McClintock adhered to her own more complicated ideas about how genes might operate, and in 1983, at the age of 81, was awarded a Nobel Prize for her discovery that the genes in corn are capable of moving from one chromosomal site to another. (A) (B) (C) (D) (E)

Unlike the conviction held by many of her colleagues that genes were Although many of her colleagues were of the conviction of genes being Contrary to many of her colleagues being convinced that genes were Even though many of her colleagues were convinced that genes were Even with many of her colleagues convinced of genes being

6. Because an oversupply of computer chips has sent prices plunging, the manufacturer has announced that it will cut production by closing its factories for two days a month. (A) (B) (C) (D) (E)

256

Because an oversupply of computer chips has sent prices plunging, Because of plunging prices for computer chips, which is due to an oversupply, Because computer chip prices have been sent plunging, which resulted from an oversupply, Due to plunging computer chip prices from an oversupply, Due to an oversupply, with the result that computer chip prices have been sent plunging,


7. Beyond the immediate cash flow crisis that the museum faces, its survival depends on if it can broaden its membership and leave its cramped quarters for a site where it can store and exhibit its more than 12,000 artifacts. (A) (B) (C) (D) (E)

if it can broaden its membership and leave whether it can broaden its membership and leave whether or not it has the capability to broaden its membership and can leave its ability for broadening its membership and leaving the ability for it to broaden its membership and leave

8. Along with the drop in producer prices announced yesterday, the strong retail sales figures released today seem like it is indicative that the economy, although growing slowly, is not nearing a recession. (A) (B) (C) (D) (E)

like it is indicative that as if to indicate to indicate that indicative of like an indication of

9. Dressed as a man and using the name Robert Shurtleff, Deborah Sampson, the first woman to draw a soldier's pension, joined the Continental Army in 1782 at the age of 22, was injured three times, and was discharged in 1783 because she had become too ill to serve. (A) (B) (C) (D) (E)

22, was injured three times, and was discharged in 1783 because she had become 22, was injured three times, while being discharged in 1783 because she had become 22 and was injured three times, and discharged in 1783, being 22, injured three times, and was discharged in 1783 because she was 22, having been injured three times and discharged in 1783, being

10. Although schistosomiasis is not often fatal, it is so debilitating that it has become an economic drain on many developing countries. (A) (B) (C) (D) (E)

it is so debilitating that it has become an economic it is of such debilitation, it has become an economical so debilitating is it as to become an economic such is its debilitation, it becomes an economical there is so much debilitation that it has become an economical

257


11. In 1850, Lucretia Mott published her Discourse on Women, arguing in a treatise for women to have equal political and legal rights and for changes in the married women's property laws. (A) (B) (C) (D)

arguing in a treatise for women to have equal political and legal rights arguing in a treatise for equal political and legal rights for women a treatise that advocates women's equal political and legal rights a treatise advocating women's equal political and legal rights (E) a treatise that argued for equal political and legal rights for women

12. In 1527 King Henry VIII sought to have his marriage to Queen Catherine annulled so as to marry Anne Boleyn (A) (B) (C) (D)

so as to marry and so could be married to to be married to so that he could marry (E) in order that he would marry

13. Dr. Tonegawa won the Nobel Prize for discovering how the body can constantly change its genes to fashion a seeming unlimited number of antibodies, each specifically targeted at an invading microbe or foreign substance. (A) (B) (C) (D) (E)

seeming unlimited number of antibodies, each specifically targeted at seeming unlimited number of antibodies, each targeted specifically to seeming unlimited number of antibodies, all specifically targeted at seemingly unlimited number of antibodies, all of them targeted specifically to seemingly unlimited number of antibodies, each targeted specifically at

14. To develop more accurate population forecasts, demographers have to know a great deal more than now about the social and economic determinants of fertility. (A) (B) (C) (D) (E)

have to know a great deal more than now about the social and economic have to know a great deal more than they do now about the social and economical would have to know a great deal more than they do now about the social and economical would have to know a great deal more than they do now about the social and economic would have to know a great deal more than now about the social and economic

15. Scientists have recently discovered what could be the largest and oldest living organism on Earth, a giant fungus that is an interwoven filigree of mushrooms and rootlike tentacles spawned by a single fertilized spore some 10,000 years ago and extending for more than 30 acres in the soil of a Michigan forest.

258


(A) (B) (C) (D) (E)

extending extends extended it extended is extending

16. Laos has a land area about the same as Great Britain but only four million in population, where many are members of hill tribes ensconced in the virtually inaccessible mountain valleys of the north. (A) about the same as Great Britain but only four million in population, where many (B) of about the same size as Great Britain is, but in Laos there is a population of only four million, and many (C) that is about the same size as Great Britain's land area, but in Laos with a population of only four million people, many of them (D) comparable to the size of Great Britain, but only four million in population, and many (E) comparable to that of Great Britain but a population of only four million people, many of whom

17. The plot of The Bostonians centers on the rivalry between Olive Chancellor, an active feminist, with her charming and cynical cousin, Basil Ransom, when they find themselves drawn to the same radiant young woman whose talent for public speaking has won her an ardent following. (A) rivalry between Olive Chancellor, an active feminist, with her charming and cynical cousin, Basil Ransom, (B) rivals Olive Chancellor, an active feminist, against her charming and cynical cousin, Basil Ransom, (C) rivalry that develops between Olive Chancellor, an active feminist, and Basil Ransom, her charming and cynical cousin, (D) developing rivalry between Olive Chancellor, an active feminist, with Basil Ransom, her charming and cynical cousin, (E) active feminist, Olive Chancellor, and the rivalry with her charming and cynical cousin Basil Ransom,

18. Quasars, at billions of light-years from Earth the most distant observable objects in the universe, believed to be the cores of galaxies in an early stage of development. (A) (B) (C) (D) (E)

believed to be are believed to be some believe them to be some believe they are it is believed that they are

259


19. In ancient Thailand, much of the local artisans' creative energy was expended for the creation of Buddha images and when they constructed and decorated the temples that enshrined them. (A) much of the local artisans' creative energy was expended for the creation of Buddha images and when they constructed and decorated the temples that enshrined them (B) much of the local artisans' creative energy was expended on the creation of Buddha images and on construction and decoration of the temples in which they were enshrined (C) much of the local artisans' creative energy was expended on the creation of Buddha images as well as constructing and decoration of the temples in which they were enshrined (D) creating images of Buddha accounted for much of the local artisans' creative energy, and also constructing and decorating the temples enshrining them (E) the creation of Buddha images accounted for much of the local artisans' creative energy as well as construction and decoration of the temples that enshrined them

20. Five fledgling sea eagles left their nests in western Scotland this summer, bringing to 34 the number of wild birds successfully raised since transplants from Norway began in 1975. (A) (B) (C) (D) (E)

bringing and brings and it brings and it brought and brought

(OG) Set 9 1. The company is negotiating to sell its profitable credit card subsidiary, which it plans to use money from to acquire some of the mortgage-servicing operations that are being sold by troubled savings institutions. (A) (B) (C) (D) (E)

subsidiary, which it plans to use money from subsidiary, from which it plans to use money subsidiary, and it plans the use of money from that subsidiary and plans to use money from that sale subsidiary and plans the use of money from that sale

2. In the 1980’s the rate of increase of the minority population of the United States was nearly twice as fast as the 1970’s. (A) (B) (C) (D) (E)

260

twice as fast as twice as fast as it was in twice what it was in two times faster than that of two times greater than


3. The figure of the jaguar, being a recurring symbol within Olmec art, is prominent among the hieroglyphics inscribed on a monument that was discovered in the Mexican state of Veracruz. (A) (B) (C) (D) (E)

being a recurring symbol within a symbol having recurred within a recurring symbol in having been a symbol that recurred in recurring as it is, a symbol in

4. As the etched lines on computer memory chips have become thinner and the chips’ circuits more complex, both the power of the chips and the electronic devices they drive have vastly increased. (A) the chips’ circuits more complex, both the power of the chips and the electronic devices they drive have (B) the chips’ circuits more complex, the power of both the chips and the electronic devices they drive has (C) the chips’ circuits are more complex, both the power of the chips and the electronic devices they drive has (D) their circuits are more complex, the power of both the chips and the electronic devices they drive have (E) their circuits more complex, both the power of the chips and the electronic devices they drive have

5. Most energy analysts now agree that the costs of building and maintaining nuclear reactors are too high for nuclear power to likely prove cheaper than coal or oil in the long run. (A) (B) (C) (D) (E)

too high for nuclear power to likely high enough for nuclear power to be unlikely to high enough that it is unlikely nuclear power will so high that nuclear power is unlikely to so high as to be unlikely that nuclear power will

6. New Jersey’s is one of the five highest number of reported cases of Lyme disease in the United States. (A) (B) (C) (D) (E)

New Jersey’s is one of the five highest number of reported New Jersey’s is one of the five highest numbers in reporting New Jersey has a report of one of the five highest numbers of New Jersey has one of the five highest numbers of reported New Jersey reports one of the five highest number of

261


7. According to a ruling by the state supreme court, the owner of polluted land is liable for the cleanup of the property even if the owner did not have the responsibility that pollution occurred before the title changed hands. (A) (B) (C) (D) (E)

the owner did not have the responsibility that pollution the owner is not responsible for pollution that it was not the owner’s responsibility that pollution would have the responsibility of the owner is not that pollution the responsibility was not the owner’s that pollution would have

8. Scientists have suggested that once every 10 million years or so a truly colossal object from space cuts through the atmosphere and slams into Earth, sending up a global pall of dust that blots out the Sun, alters the climate, and changes the course of evolution by killing off many plant and animal species. (A) (B) (C) (D) (E)

sending up a global pall of dust that blots thus sending up a global pall of dust to blot thereby sending up a global pall of dust to blot and that sends up a global pall of dust, blotting which sends up a global pall of dust, blots

9. By the mid-seventeenth century, Amsterdam had built a new town hall so large that only St. Peter’s in Rome, the Escorial in Spain, and the Palazza Ducale in Venice could rival it for scale or magnificence. (A) (B) (C) (D) (E)

could rival it for were the rivals of it in their were its rival as to could be its rivals in their were rivaling its

10. The Audubon Society and other conservation groups, concerned over what they have perceived to be the serious threatening of the environment as posed by the policies of the government, are preparing for a major political effort. (A) (B) (C) (D) (E)

262

have perceived to be the serious threatening of the environment as perceived as the serious threat to the environment as perceive being the serious threat to the environment are perceiving as the serious threatening of the environment perceive as the serious threat to the environment


11. At a time when it was unusual to do it, Dorothy Sterling wrote about such major figures of Black history as Harriet Tubman and W. E. B. Du. Bois. (A) (B) (C) (D) (E)

it was unusual to do it it was unusual to do so doing that was unusual that was an unusual thing to be doing it was not usual to do

12. Houseflies that hatch in summer live only about three weeks, but those that emerge in the cooler days of fall often live longer than six months. (A) (B) (C) (D) (E)

weeks, but those that emerge in the cooler days of fall often live weeks, but those that emerge in the cooler days of fall often live as long or weeks, which is different from those that emerge in the cooler days of fall and often live weeks; then those that emerge in the cooler days of fall often live as long as or weeks; this is different from those that emerge in the cooler days of fall, who often live

13. The auto industry has experienced one of its most significant trends in the last 50 years, which is the migration of motorists from passenger cars to minivans, sport utility vehicles, and pickups. (A) (B) (C) (D) (E)

The auto industry has experienced one of its most significant trends in the last 50 years, which is Of the trends the auto industry experienced in the last 50 years has been one of the most significant. In the last 50 years, one of the most significant trends that the auto industry has been experiencing has been One of the most significant trends that the auto industry has experienced in the last 50 years is In the last 50 years, the auto industry experienced one of the most significant trends that it has had, that of

14. The animosity between those who regulate and those who are regulated, never more pronounced than in recent debates over environmentalism and pollution control. (A) (B) (C) (D) (E)

The animosity between those who regulate and those who are regulated, never The animosity between those who regulate and those who are regulated, never being The animosity between those who regulate and those who are regulated has never been Between those who regulate and those who are regulated, such animosity was never Between those who regulate and between those regulated, such animosity has never been

15. Some psychiatric studies indicate that among distinguished artists the rates of manic depression and major depression are ten to thirteen times as prevalent as in the population at large.

263


(A) (B) (C) (D) (E)

the rates of manic depression and major depression are ten to thirteen times as prevalent as in the rates of manic depression and major depression are ten to thirteen times more prevalent than in the rates of manic depression and major depression are ten to thirteen times more prevalent when compared to manic depression and major depression are ten to thirteen times as prevalent when compared to manic depression and major depression are ten to thirteen times more prevalent than in

16. Until quite recently, American presidents lived in a world in which the public and private realms of their lives were largely separate, and the press cooperated in maintaining the distinction, and Americans judged national leaders without receiving, or expecting, intimate information about them. (A) (B) (C) (D) (E)

and the press cooperated in maintaining the distinction, and where the press cooperated in maintaining the distinction, and where for the press cooperated to maintain the distinction and the press cooperated to maintain the distinction, for in which the press cooperated in maintaining the distinction, and in which

17. Intended primarily to stimulate family summer travel, the new airfare, which allows both an adult and a child to fly for the price of one ticket, and also shortens the advance-purchase requirement for family travel to a minimum of seven days rather than fourteen. (A) (B) (C) (D) (E)

and also shortens the advance-purchase requirement for family travel to a minimum of seven days rather than and also lessens the advance-purchase requirement for family travel to a seven-day minimum from also shortens the advance-purchase requirement for family travel to a minimum of seven days rather than that of also lessens the advance-purchase requirement for family travel to a seven-day minimum from also shortens the advance-purchase requirement for family travel to a minimum of seven days rather than

18. People have discovered the principles of solar energy whenever fuel becomes scarce and expensive but will forget them every time a new source of cheap energy is developed. (A) (B) (C) (D) (E)

264

have discovered the principles of solar energy whenever fuel becomes scarce and expensive but will forget have discovered the principles of solar energy whenever fuel has become scarce and expensive but they forget discovered the principles of solar energy every time fuel becomes scarce and expensive, forgetting discover the principles of solar energy every time fuel became scarce and expensive, but they forget discover the principles of solar energy whenever fuel becomes scarce and expensive but forget


19. New techniques in thermal-scanning photography, a process that records radiation from surface areas, makes it possible to study the effects of calefaction, or warming, of a river in greater detail than ever before. (A) (B) (C) (D) (E)

makes it possible to study the effects of calefaction, or warming, of a river in greater detail than ever before make it possible to study, in greater detail, the effects of calefaction, or warming, of a river than ever before have made it possible to study in greater detail than ever before the effects of calefaction, or warming, of a river make possible the study of the effects of calefaction, or warming, of a river in greater detail than it ever was before has made it more possible than ever before to study in greater detail the effects of calefaction, or warming, of a river

20. Any real estate professional will tell you that the value of a parcel of land is most directly affected by the extent of its development and how close it is to a major business center. (A) (B) (C) (D) (E)

the extent of its development whether it has been developed extensively how extensively it has developed the extent to which it has developed how extensively it has been developed

(K) SET 10 1. Bob Wilber became Sidney Bechet’s student and protege when he was nineteen and, for a few years in the 1940’s, came as close to being a carbon copy of the jazz virtuoso in performance as anyone has ever come. (A) (B) (C) (D) (E)

as anyone has ever come as anyone ever had been as anyone ever had done that anyone ever did that anyone ever came

2. Despite its attractiveness, investing abroad can still pose big risks, ranging from the potential for political instability in some countries to the shortage of regulations to protect investors and a serious lack of information about investments in others. (A)

to the shortage of regulations to protect investors and a serious lack of information about investments in others

265


(B) (C) (D) (E)

to the shortage of regulations to protect investors and in others a serious lack of information about investments and the shortage of regulations to protect investors and a serious lack of information about investments in others and the shortage of regulations to protect investors to a serious lack of information about investments in others to the shortage of regulations to protect investors in others and a serious lack of information about investments

3. That the new managing editor rose from the publication’s “soft” news sections to a leadership position is more of a landmark in the industry than her being a woman. (A) (B) (C) (D) (E)

her being a woman being a woman is her womanhood that she was a woman that she is a woman

4. In the initial planning stages, the condominium corporation took into account only the concerns of its prospective clients, not those of surrounding homeowners. (A) (B) (C) (D) (E)

the condominium corporation took into account only the concerns of its prospective clients the condominium corporation has only taken into account the concerns of their prospective clients the condominium corporation only took their prospective clients’ concerns into account the concerns of its prospective clients only were taken into account by the condominium corporation prospective clients had their concerns only taken into account by the condominium corporation

5. Unlike other arachnids, which have their nerve cells evenly distributed along their bodies, the scorpion’s nerve cells are clustered in its head, like a mammal’s. (A) (B) (C) (D) (E)

bodies, the scorpion’s nerve cells are clustered in its head, like a mammal’s bodies, the scorpion’s head had a cluster of nerve cells, as a mammal does body, the scorpion has a cluster of nerve cells in its head, as a mammal does body, nerve cells are clustered in the scorpion’s head, like a mammal’s body, a cluster of nerve cells is in the scorpion’s head, like a mammal’s

6. Of all the wild animals in their area, none was more useful to the Delaware tribes than the Virginia white-tailed deer: it was a source of meat, and its hide was used for clothing, its antlers and bones for tools, and its sinews and gut for bindings and glue.

266


(A) (B) (C) (D) (E)

deer: it was a source of meat, and its hide was used for clothing, its antlers and bones for tools, and its sinews and gut deer: it was a source of meat, and its hide used for clothing, with its antlers and bones for tools, and its sinews and gut used deer: which was a source of meat, with its hide used for clothing, antlers and bones for tools, as well as its sinews and gut used deer: which, as well as being a source of meat, its hide was used for clothing, its antlers and bones for tools, and its sinews and gut were deer: with, as well as being a source of meat, its hide used for clothing, its antlers and bones for tools, and its sinews and gut

7. Unlike most warbler species, the male and female blue-winged warbler are very difficult to tell apart. (A) (B) (C) (D) (E)

Unlike most warbler species, the male and female blue-winged warbler are very difficult to tell apart. Unlike most warbler species, the gender of the blue-winged warbler is very difficult to distinguish. Unlike those in most warbler species, the male and female blue-winged warblers are very difficult to distinguish. It is very difficult, unlike in most warbler species, to tell the male and female blue-winged warbler apart. Blue-winged warblers are unlike most species of warbler in that it is very difficult to tell the male and female apart.

8. In addition to providing more course offerings than Willow High School, the teachers at Menio High School are better trained than those at Willow, having received more information on instructing a multilingual and culturally diverse student body. (A) (B) (C) (D) (E)

the teachers at Menio High School are better trained than those at Menio High School has teachers who are better trained than those at Menio High School teachers are better trained than they are at the teachers at Menio High School are better in training than those at Menio High School has teachers who are better trained than at

9. In 1905, The House of Mirth, Edith Wharton's novel about the blighted aspirations of Lily Bart was published by Scribner's and it was a reputable press in the early twentieth century. (A) (B) (C) (D) (E)

Lily Bart was published by Scribner's and it was Lily Bart, published by Scribner's, and was Lily Bart was published by Scribner's, being Lily Bart, which was published by Scribner's, was Lily Bart, was published by Scribner's,

267


10. Dreading another trip to the cat clinic, her veterinarian was persuaded to treat her cat at her home, a rather combative two-vear-old black male. (A) (B) (C) (D) (E)

her veterinarian was persuaded to treat her cat at her home, a rather combative two-year-old black male she persuaded her veterinarian to treat her cat at her home, the cat being a rather combative two-year-old black male Jessica persuaded her veterinarian to treat her cat, a rather combative two-year-old black male, at her home Jessica persuaded her veterinarian that her rather combative two-year-old black male cat would best be treated at her home she persuaded her veterinarian that her home should be the place that her rather combative twoyear-old black male cat was treated at

11. However much parents in Johannson's district may agree that the instruction of moral values should take place in the elementary school classroom, it is difficult for them to arrive at consensus concerning what those values are and how they should be taught. (A) (B) (C) (D) (E)

However much parents in Johannson's district may agree that Despite the fact that parents in Johannson's district may agree that There is general agreement among the parents in Johannson's district that Although the parents in Johannson's district agree for Even though the parents in Johannson's district agree

12. The bylaws of a corporation specify how the directors of the corporation are to be elected, whether the existing stockholders will have first right to buy any new stock issued by the firm, as well as duties of management committees. (A) (B) (C) (D) (E)

as well as duties of management committees as well as the duties that the management committees have and that the management committees have duties outlining the duties of management committees and what the duties of management committees will be

13. There is no agreement at present on what function, if any, certain herbs like gingko biloba have in slowing the deterioration or improving memory. (A) (B) (C) (D) (E)

268

in slowing the deterioration or improving in the improvement or slowing of the deterioration of in slowed or improved deterioration of in the improvement to or the slowing of the deterioration of in slowing the deterioration of or improving


14. Often credited as the founder of English literature, Geoffrey Chaucer both contributed to a growing canon of English literature, extended the capabilities of the language, and using it to invent a new poetic meter. (A) (B) (C) (D) (E)

literature, extended the capabilities of the language, and using it literature, extended the capabilities of the language, and used it literature and extended the capabilities of the language, using it literature, extending the capabilities of the language and using it literature and, extending the capabilities of the language, using it

15. Unlike German shepherds or Doberman pinchers, there is an unwillingness on the part of many people to believe that pit bulls might be fully domesticated. (A) (B) (C) (D) (E)

Unlike German shepherds or Doberman pinchers, there is an unwillingness on the part of many people to believe that pit bulls might be fully domesticated. Many people, willing to believe that German shepherds and Doberman pinchers might be fully domesticated, are unwilling to believe the same of pit bulls. Unlike German shepherds or Doberman pinchers, pit bulls bring out an unwillingness in many people to believe that they might be fully domesticated. Many people are unwilling to believe that pit bulls might be fully domesticated even while they are willing to believe that German shepherds and Doberman pinchers might be. Unlike German shepherds and Doberman pinchers, which many people are willing to believe can be fully domesticated, such belief does not extend to pit bulls.

16. Early medieval monasteries, while clearly less accessible to outsiders, often served as repositories for texts like modern libraries. (A) (B) (C) (D) (E)

while clearly less accessible to outsiders, often served as repositories for texts like modern libraries like modern libraries, often served as text repositories, though they were clearly less accessible to outsiders while clearly less accessible to outsiders, often served as repositories for texts as does the modern library like modern libraries, while clearly less accessible to outsiders, often served as repositories tor texts while clearly less accessible to outsiders, acted like modern libraries act by serving as repositories for texts

17. When presented with only circumstantial evidence, a juror tends to decide a case according to his or her gut instinct, which are not formally sanctioned or prohibited means of reaching a verdict. (A)

which are not formally

269


(B) (C) (D) (E)

which are not a formally which is not a formally which is formally not a which is not formally

18. Recent studies have shown that there are now one teacher for every thirty-one students in California's elementary schools, not nearly as many than there were even five years ago.

(A) (B) (C) (D) (E)

there are now one teacher for every thirty-one students in California's elementary schools, not nearly as many than there were there are now one teacher for every thirty-one students in California's elementary schools, not nearly as many as there were there is now one teacher for every thirty-one students in California's elementary schools, not nearly as many than there was there is now one teacher for every thirty-one students in California's elementary schools, but that ratio is much lower than there is now one teacher for every thirty-one students in California's elementary schools, not nearly as many as there were

19. Although the government's expenditures on law suits involving tobacco companies amounts to a sum dramatically lower than that spent by tobacco companies, many believe that the government should allocate no more funds to a battle they perceive as pointless. (A) (B) (C) (D) (E)

expenditures on law suits involving tobacco companies amounts to a sum dramatically lower than expenditures on law suits involving tobacco companies amount to a sum dramatically less than expenditures on law suits involving tobacco companies amount to a sum dramatically lower than law suit expenditures regarding tobacco companies amount to a dramatically lower sum than law suit expenditures against tobacco companies amounts to a sum dramatically lower than

20. (K) Proponents of affirmative action, including most university presidents, need only cite declining minority enrollment in universities in California and Texas, the two most populous states, to support their cause. (A)

(B)

(C)

270

Proponents of affirmative action, including most university presidents, need only cite declining minority enrollment in universities in California and Texas, the two most populous states, to support their cause. Most university presidents who are proponents of affirmative action need to support their cause by only citing that declining minorities are enrolling in universities in the two most populous states of California and Texas. In order for proponents of affirmative action, which include most university presidents, to


(D)

(E)

support its cause, they need only to cite the decline in minority enrollment in universities in California and Texas, the two most populous states. Minority enrollment in universities in California and Texas, the two most populous states, are declining, and proponents of affirmative action, including most university presidents, only need to cite this fact to support their cause. University presidents, including those in California and Texas, the two most populous states where declining minorities are enrolling in universities, should cite these facts and support their cause as proponents of affirmative action.

(K) Set 11

1. The symptoms of the participants in a recent decongestant effectiveness study, including a required "stuffy nose�, parallel that of the multitudes of Americans with the common cold each winter. (A) (B) (C) (D) (E)

parallel that of the multitudes of Americans with parallel those of the multitudes of Americans afflicted with parallel the multitudes of Americans afflicted with parallels those of the multitudes of Americans afflicted from parallels that of the multitudes of Americans afflicted from

2. It was anomalies in the orbit of Uranus that led to the discovery of Neptune. (A) (B) (C) (D) (E)

It was anomalies in the orbit of Uranus that It was the orbit of Uranus showing anomalies that They were anomalies in the orbit of Uranus that The orbit of Uranus being anomalous was what The anomalies in the orbit of Uranus was what

3. After the company recalled one of its best-selling products, it would be forced to accept a one billion dollar loss, an expense that would have forced its main competitor to declare bankruptcy if it had suffered such a loss. (A)

(B) (C) (D)

(E)

After the company recalled one of its best-selling products, it would be forced to accept a one billion dollar loss, an expense that would have forced its main competitor to declare bankruptcy if it had suffered such a loss. As the company recalled one of its best-selling products, it accepted a one billion dollar loss; forcing its main competitor to declare bankruptcy if it had suffered such a loss. After the company recalled one of its best-selling products, its one billion dollar loss would have to be accepted; such a loss would have forced its main competitor to declare bankruptcy. After the company recalled one of its best-selling products, it was forced to accept a one billion dollar loss; its main competitor would have been forced to declare bankruptcy if it had suffered such a loss. When the company recalled one of its best-selling products, a one billion dollar loss it was

271


forced to accept, an expense that would have forced its main competitor into bankruptcy

4. Among the emotions on display in the negotiating room were anger for repeatedly raising the issue over and over again and preventing the raw wounds from earlier battles from ever beginning to heal. (A) (B) (C) (D) (E)

were anger for repeatedly raising the issue over and over again and preventing the raw wounds from earlier battles from ever beginning to heal was anger for repeatedly raising the issue and preventing the raw wounds from earlier battles from ever beginning to heal were anger over repeatedly raising the issue and preventing the raw wounds from earlier battles to begin healing was anger about the issue, which was raised over and over, and preventing the wounds from earlier battles, still raw, to begin healing were anger about the issue, which was raised repeatedly, and preventing the raw wounds from earlier battles to begin to heal

5. In 1922, when Truman was almost forty years old, he was living in his mother-in-law’s house, watching the haberdashery store he opened three years earlier go bankrupt, and he faced a future with no visible prospects. (A) (B) (C) (D) (E)

opened three years earlier go bankrupt, and he faced opened three years earlier go bankrupt and faced had opened three years earlier go bankrupt, and he was facing had opened three years earlier go bankrupt, and facing was opening three years earlier going bankrupt, and facing

6. In 1978 only half the women granted child support by a court received the amount awarded; at least as much as a million and more others had not any support agreements whatsoever. (A) (B) (C) (D) (E)

at least as much as a million and more others had not any at least as much as more than a million others had no more than a million others had not any more than a million others had no there was at least a million or more others without any

7. (OG) The development of a new jumbo rocket that is expected to carry the United States into its next phase of space exploration will be able to deliver a heavier load of instruments into orbit than the space shuttle and at a lower cost. (A)

272

The development of a new jumbo rocket that is expected to carry the United States into its next phase of space exploration will be able to deliver a heavier load of instruments into orbit than


(B)

(C)

(D)

(E)

the space shuttle and at a lower cost. The development of a new jumbo rocket is expected to carry the United States into its next phase of space exploration and be able to deliver a heavier load of instruments into orbit at a lower cost than the space shuttle. The new development of a jumbo rocket, which is expected to carry the United States into its next phase of space exploration, will be able to deliver a heavier load of instruments into orbit at a lower cost than the space shuttle. A newly developed jumbo rocket, which is expected to carry the United States into its next phase of space exploration, will be able to deliver a heavier load of instruments into orbit than the space shuttle can, and at a lower cost. A newly developed jumbo rocket, which is expected to carry the United States into its next phase of space exploration, will be able to deliver a heavier load of instruments into orbit than the space shuttle and to cost less.

8. The distribution of mass within the core of the Earth, like the mantle that surrounds the core, has been deduced from the orbital behavior of the Earth and the motions of satellites controlled by the gravity of the Earth. (A) (B) (C) (D) (E)

the mantle that surrounds the core that within the mantle surrounding the core that of the mantle surrounding the core the mantle the core surrounds the distribution of mass within the mantle that surrounds the core

9. The domesticated camel, which some scholars date around the twelfth century B.C., was the key to the development of the spice trade in the ancient world. (A) (B) (C) (D) (E)

The domesticated camel, which some scholars date The domesticated camel, which some scholars having thought to occur Domesticating the camel, dated by some scholars at The domestication of the camel, thought by some scholars to have occurred The camel’s domestication, dated by some scholars to have been

10. (PR) The economic forces which may affect the new public offering of stock include sudden downturns in the market, hedging and other investor strategies for preventing losses, loosening the interest rates in Washington, and fearing that the company may still be undercapitalized. (A) (B) (C) (D)

loosening the interest rates in Washington, and fearing that the company may still be undercapitalized loosening the interest rates in Washington, and a fear of the company still being undercapitalized a loosening of the interest rates in Washington, and fearing that the company may still be undercapitalized a loosening of the interest rates in Washington, and a fear of the still undercapitalized company

273


(E)

a loosening of the interest rates in Washington, and a fear that the company may still be undercapitalized

11. The Emperor Augustus, it appears, commissioned an idealized sculptured portrait, the features of which are so unrealistic as to constitute what one scholar calls an “artificial face”. (A) (B) (C) (D) (E)

so unrealistic as to constitute so unrealistic they constituted so unrealistic that they have constituted unrealistic enough so that they constitute unrealistic enough so as to constitute

12. There are more than forty newspapers published in the cities of Kerala, a state on the Malabar Coast, which reflects the fact that Keralans are by far India’s most literate citizens. (A) (B) (C) (D) (E)

which reflects and that number reflects which reflect that number reflects that reflects

13. There is growing demand in the state for “initiative and referendum”, a procedure that allows voters to propose and pass laws, as well as to repeal them. (A) (B) (C) (D) (E)

allows voters to propose and pass laws, as well as to repeal them allows voters to propose, pass, and to repeal laws allows voters to propose, to pass, and repeal laws will allow the voter to propose, pass, as well as to repeal laws will allow laws to be proposed, passed, as well as repealed by voters

14. There is no consensus on what role, if any, is played by acid rain in slowing the growth or damaging forests in the eastern United States. (A) (B) (C) (D) (E)

274

slowing the growth or damaging the damage or the slowing of the growth of the damage to or the slowness of the growth of damaged or slowed growth of damaging or slowing the growth of


15. There is substantial evidence that certain forms of solar energy either now or within a few years will be economically competitive with conventional sources of heat and power. (A) (B) (C) (D) (E)

either now or within a few years will be economically competitive with conventional sources of heat and power will either be economically competitive with conventional sources of heat and power within a few years or are so now will be economically competitive with conventional sources of heat and power either now or within a few years either are now economically competitive with conventional sources of heat and power or will be so within a few years are either now or will be within a few years economically competitive with conventional sources of heat and power

16. (OG) The Olympic Games helped to keep peace among the pugnacious states of the Greek world in that a sacred truce was proclaimed during the festival’s month. (A) (B) (C) (D) (E)

world in that a sacred truce was proclaimed during the festival’s month world, proclaiming a sacred truce during the festival’s month world when they proclaimed a sacred truce for the festival month world, for a sacred truce was proclaimed during the month of the festival world by proclamation of a sacred truce that was for the month of the festival

17. Byron possessed powers of observation that would have made him a great anthropologist and that makes his letters as a group the rival of the best novels of the time. (A) (B) (C) (D) (E)

makes his letters as a group the rival of makes his letters as a group one to rival makes his letters a group rivaling make his letters as a group the rival of make his letters a group which is the rival of

18. Corporations will soon be required to report to the government whether they have the necessary reserves to pay the pension benefits earned by their employees and that the information be published in annual reports to shareholders. (A) (B) (C) (D) (E)

earned by their employees and that the information be published that their employees earned and that the information be published that was earned by their employees with the information being published earned by their employees, information that must also be published earned by their employees and published the information

275


19. During the early years of European settlement on a continent that was viewed as “wilderness” by the newcomers, Native Americans, intimately knowing the ecology of the land, were a help in the rescuing of many Pilgrims and pioneers from hardship, or even death. (A) (B) (C) (D) (E)

Native Americans, intimately knowing the ecology of the land, were a help in the rescuing of Native Americans knew the ecology and the land intimately and this enabled them to help in the rescue of Native Americans, with their intimate knowledge of the ecology of the land, helped to rescue having intimate knowledge of the ecology of the land, Native Americans helped the rescue of knowing intimately the ecology of the land, Native Americans helped to rescue

20. Cattle were domesticated both for the uses made of the animal—food and leather—but also for the labor the animal could provide. (A) (B) (C) (D) (E)

but also for and for or for but also and also

(OG) Set 12

1. In 1713, Alexander Pope began his translation of the Iliad, a work that, taking him seven years until completion, and that literary critic Samuel Johnson, Pope's contemporary, pronounced the greatest translation in any language. (A) (B) (C) (D) (E)

his translation of the Iliad, a work that, taking him seven years until completion, and that literary critic Samuel Johnson, Pope's contemporary, pronounced his translation of the Iliad, a work that took him seven years to complete and that literary critic Samuel Johnson, Pope's contemporary, pronounced his translation of the Iliad, a work that had taken seven years to complete and that literary critic Samuel Johnson, Pope's contemporary, pronounced it as translating the Iliad, a work that took seven years until completion and that literary critic Samuel Johnson, Pope's contemporary, pronounced it as translating the Iliad, a work that had taken seven years to complete and literary critic Samuel Johnson, Pope's contemporary, pronounced it

2. The automotive conveyor-belt system, which Henry Ford modeled after an assembly-line technique introduced by Ransom Olds, reduced from a day and a half to 93 minutes the required time of assembling a Model T. (A) (B) (C)

276

from a day and a half to 93 minutes the required time of assembling a Model T the time being required to assemble a Model T, from a day and a half down to 93 minutes the time being required to assemble a Model T, a day and a half to 93 minutes


(D) (E)

the time required to assemble a Model T from a day and a half to 93 minutes from a day and a half to 93 minutes, the time required for the assembling of a Model T

3. According to some analysts, the gains in the stock market reflect growing confidence that the economy will avoid the recession that many had feared earlier in the year and instead come in for a "soft landing," followed by a gradual increase in business activity. (A) (B) (C) (D) (E)

that the economy will avoid the recession that many had feared earlier in the year and instead come in the economy to avoid the recession, what many feared earlier in the year, rather to come in the economy's ability to avoid the recession, something earlier in the year many had feared, and instead to come in the economy to avoid the recession many were fearing earlier in the year, and rather to come that the economy will avoid the recession that was feared earlier this year by many, with it instead coming

4. A new study suggests that the conversational pace of everyday life may be so brisk it hampers the ability of some children for distinguishing discrete sounds and words and, the result is. to make sense of speech. (A) (B) (C) (D) (E)

it hampers the ability of some children for distinguishing discrete sounds and words and, the result is, to make that it hampers the ability of some children to distinguish discrete sounds and words and, as a result, to make that it hampers the ability of some children to distinguish discrete sounds and words and, the result of this, they are unable to make that it hampers the ability of some children to distinguish discrete sounds and words, and results in not making as to hamper the ability of some children for distinguishing discrete sounds and words, resulting in being unable to make

5. To Josephine Baker, Paris was her home long before it was fashionable to be an expatriate, and she remained in France during the Second World War as a performer and an intelligence agent for the Resistance. (A) (B) (C) (D) (E)

To Josephine Baker, Paris was her home long before it was fashionable to be an expatriate, For Josephine Baker, long before it was fashionable to be an expatriate, Paris was her home, Josephine Baker made Paris her home long before to be an expatriate was fashionable, Long before it was fashionable to be an expatriate, Josephine Baker made Paris her home, Long before it was fashionable being an expatriate, Paris was home to Josephine Baker,

277


6. The nineteenth-century chemist Humphry Davy presented the results of his early experiments in his "Essay on Heat and Light," a critique of all chemistry since Robert Boyle as well as a vision of a new chemistry that Davy hoped to found. (A) (B) (C) (D) (E)

a critique of all chemistry since Robert Boyle as well as a vision of a a critique of all chemistry following Robert Boyle and also his envisioning of a a critique of all chemistry after Robert Boyle and envisioning as well critiquing all chemistry from Robert Boyle forward and also a vision of critiquing all the chemistry done since Robert Boyle as well as his own envisioning of

7. The report recommended that the hospital should eliminate unneeded beds, expensive services should be consolidated, and use space in other hospitals. (A) (B) (C) (D) (E)

should eliminate unneeded beds, expensive services should be consolidated, and use space in other hospitals should eliminate unneeded beds, expensive services should be consolidated, and other hospitals' space be used should eliminate unneeded beds, expensive services should be consolidated, and to use space in other hospitals eliminate unneeded beds, consolidate expensive services, and other hospitals' space used eliminate unneeded beds, consolidate expensive services, and use space in other hospitals

8. Many house builders offer rent-to-buy programs that enable a family with insufficient savings for a conventional down payment to be able to move into new housing and to apply part of the rent to a purchase later. (A) (B) (C) (D) (E)

programs that enable a family with insufficient savings for a conventional down payment to be able to move into new housing and to apply programs that enable a family with insufficient savings for a conventional down payment to move into new housing and to apply programs; that enables a family with insufficient savings for a conventional down payment to move into new housing, to apply programs, which enables a family with insufficient savings for a conventional down payment to move into new housing, applying programs, which enable a family with insufficient savings for a conventional down payment to be able to move into new housing, applying

9. Many of the earliest known images of Hindu deities in India date from the time of the Kushan Empire, fashioned either from the spotted sandstone of Mathura or Gandharan grey schist. (A) (B) (C) (D)

278

Empire, fashioned either from the spotted sandstone of Mathura or Empire, fashioned from either the spotted sandstone of Mathura or from Empire, either fashioned from the spotted sandstone of Mathura or Empire and either fashioned from the spotted sandstone of Mathura or from


(E)

Empire and were fashioned either from the spotted sandstone of Mathura or from

10. That educators have not anticipated the impact of microcomputer technology can hardly be said that it is their fault: Alvin Toffler, one of the most prominent students of the future, did not even mention microcomputers in Future Shock, published in 1970. (A) (B) (C) (D) (E)

That educators have not anticipated the impact of microcomputer technology can hardly be said that it is their fault That educators have not anticipated the impact of microcomputer technology can hardly be said to be at fault It can hardly be said that it is the fault of educators who have not anticipated the impact of microcomputer technology It can hardly be said that educators are at fault for not anticipating the impact of microcomputer technology The fact that educators are at fault for not anticipating the impact of microcomputer technology can hardly be said

11. While all states face similar industrial waste problems, the predominating industries and the regulatory environment of the states obviously determines the types and amounts of waste produced, as well as the cost of disposal. (A) (B) (C) (D) (E)

all states face similar industrial waste problems the predominating industries and the regulatory environment of the states obviously determines each state faces a similar industrial waste problem, their predominant industries and regulatory environment obviously determine all states face a similar industrial waste problem; their predominating industries and regulatory environment obviously determines each state faces similar industrial waste problems, the predominant industries and the regulatory environment of each state obviously determines all states face similar industrial waste problems, the predominant industries and the regulatory environment of each state obviously determine

12. Rivaling the pyramids of Egypt or even the ancient cities of the Maya as an achievement, the army of terra-cotta warriors created to protect Oin Shi Huang, China's first emperor, in his afterlife is more than 2,000 years old and took 700.000 artisans more than 36 years to complete. (A) (B)

(C) (D)

the army of terra-cotta warriors created to protect Qin Shi Huang, China's first emperor, in his afterlife is more than 2,000 years old and took 700,000 artisans more than 36 years to complete Qin Shi Huang, China's first emperor, was protected in his afterlife by an army of terracotta warriors that was created more than 2,000 years ago by 700,000 artisans who took more than 36 years to complete it it took 700,000 artisans more than 36 years to create an army of terra-cotta warriors more than 2,000 years ago that would protect Qin Shi Huang, China's first emperor, in his afterlife more than 2,000 years ago, 700,000 artisans worked more than 36 years to create an army of

279


(E)

terra-cotta warriors to protect Qin Shi Huang, China's first emperor, in his afterlife more than 36 years were needed to complete the army of terra-cotta warriors that 700,000 artisans created 2,000 years ago to protect Qin Shi Huang, China's first emperor, in his afterlife

13. When Congress reconvenes, some newly elected members from rural states will try and establish tighter restrictions for the amount of grain farmers are to be allowed to grow and to encourage more aggressive sales of United States farm products overseas. (A) (B) (C) (D) (E)

and establish tighter restrictions for the amount of grain farmers are to be allowed to grow and to encourage and establish tighter restrictions on the amount of grain able to be grown by farmers and encouraging establishing tighter restrictions for the amount of grain farmers are allowed to grow and to encourage to establish tighter restrictions on the amount of grain capable of being grown by farmers and encouraging to establish tighter restrictions on the amount of grain farmers will be allowed to grow and to encourage

14. Doctors generally agree that such factors as cigarette smoking, eating rich foods high in fats, and alcohol consumption not only do damage by themselves but also aggravate genetic predispositions toward certain diseases. (A) (B) (C) (D) (E)

not only do damage by themselves but also aggravate do damage by themselves but also are aggravating to are damaging by themselves but also are aggravating not only do damage by themselves, they are also aggravating to are doing damage by themselves, and they are also aggravating

15. In a plan to stop the erosion of East Coast beaches, the Army Corps of Engineers proposed building parallel to shore a breakwater of rocks that would rise six feet above the waterline and act as a buffer, so that it absorbs the energy of crashing waves and protecting the beaches. (A) (B) (C) (D) (E)

280

act as a buffer, so that it absorbs act like a buffer so as to absorb act as a buffer, absorbing acting as a buffer, absorbing acting like a buffer, absorb


16. The 32 species that make up the dolphin family are closely related to whales and in fact include the animal known as the killer whale, which can grow to be 30 feet long and is famous for its aggressive hunting pods. (A) (B) (C) (D) (E)

include the animal known as the killer whale, which can grow to be 30 feet long and is include the animal known as the killer whale, growing as big as 30 feet long and include the animal known as the killer whale, growing up to 30 feet long and being includes the animal known as the killer whale, which can grow as big as 30 feet long and is includes the animal known as the killer whale, which can grow to be 30 feet long and it is

17. Once they had seen the report from the medical examiner, the investigators did not doubt whether the body recovered from the river was the man who had attempted to escape from the state prison. (A) (B) (C) (D) (E)

did not doubt whether the body recovered from the river was have no doubt whether the body recovered from the river was had not doubted that the body recovered from the river was have no doubt whether the body recovered from the river was that of had no doubt that the body recovered from the river was that of

18. More and more in recent years, cities are stressing the arts as a means to greater economic development and investing millions of dollars in cultural activities, despite strained municipal budgets and fading federal support. (A) (B) (C) (D) (E)

to greater economic development and investing to greater development economically and investing of greater economic development and invest of greater development economically and invest for greater economic development and the investment of

19. Combining enormous physical strength with higher intelligence, the Neanderthals appear as equipped for facing any obstacle the environment could put in their path, but their relatively sudden disappearance during the Paleolithic era indicates that an inability to adapt to some environmental change led to their extinction. (A) (B) (C) (D) (E)

appear as equipped for facing any obstacle the environment could put in their path, appear to have been equipped to face any obstacle the environment could put in their path, appear as equipped to face any obstacle the environment could put in their paths, appeared as equipped to face any obstacle the environment could put in their paths, appeared to have been equipped for facing any obstacle the environment could put in their path,

281


20. A proposal has been made to trim the horns from rhinoceroses to discourage poachers; the question is whether tourists will continue to visit game parks and see rhinoceroses after their horns are trimmed. (A) (B) (C) (D) (E)

whether tourists will continue to visit game parks and see rhinoceroses after their horns are whether tourists will continue to visit game parks to see one once their horns are whether tourists will continue to visit game parks to see rhinoceroses once the animals' horns have been if tourists will continue to visit game parks and see rhinoceroses once the animals' horns are if tourists will continue to visit game parks to see one after the animals' horns have been

(OG) Set 13

1. During the Great Depression, industrial output fell by nearly fifty percent from its peak in 1929 down to its nadir in 1933. (A) (B) (C) (D) (E)

During the Great Depression, industrial output fell by nearly fifty percent from its peak in 1929 down to its nadir in 1933. During the Great Depression, industrial output fell by nearly fifty percent from its peak in 1929 to its nadir in 1933. At the time of the Great Depression, industrial output fell by almost fifty percent from its 1929 peak down to its 1933 nadir. At the time of the Great Depression, industrial output fell from its peak in 1929, by nearly fifty percent, to it nadir in 1933. During the Great Depression, industrial output fell from its peak in 1929 to its nadir in 1933 by nearly fifty percent.

2. During the recession of 1973, home mortgage foreclosures resulted in tens of thousands of Americans being evicted from homes that they can no longer afford monthly payments. (A) (B) (C) (D) (E)

that they can that they could on which they can because they can for which they could

3. Even their most ardent champions concede that no less than a technical or scientific breakthrough is necessary before solar cells can meet the goal of providing one percent of the nation’s energy needs. (A) (B) (C) (D)

282

that no less than a technical or scientific breakthrough is necessary that nothing other than a technical or scientific breakthrough is needed that a technical or scientific breakthrough is necessary the necessity for an occurrence of a technical or scientific breakthrough


(E)

the necessity for a technical or scientific breakthrough occurring

4. Even though its per capita food supply hardly increased during two decades, stringent rationing and planned distribution have allowed the People’s Republic of China to ensure nutritional levels of 2,000 calories per person per day for its population. (A) (B) (C) (D) (E)

Even though its per capita food supply hardly increased during Even though its per capita food supply has hardly increased in Despite its per capita food supply hardly increasing over Despite there being hardly any increase in its per capita food supply during Although there is hardly any increase in per capita food supply for

5. Framed by traitorous colleagues, Alfred Dreyfus was imprisoned for twelve years before there was exoneration and his freedom. (A) (B) (C) (D) (E)

there was exoneration and his freedom he was to be exonerated with freedom being exonerated and freed exoneration and his freedom being freed, having been exonerated

6. From 1965 on, Yugoslavia’s standard of living has soared, but unemployment and prices too. (A) (B) (C) (D) (E)

but unemployment and prices too and also unemployment and prices but so have unemployment and prices and so also unemployment and prices but so did unemployment and prices

7. From the bark of the paper birch tree the Menomini crafted a canoe about twenty feet long and two feet wide, with small ribs and rails of cedar, which could carry four persons or eight hundred pounds of baggage so light that a person could easily portage it around impeding rapids. (A) (B) (C) (D) (E)

baggage so light baggage being so light baggage, yet being so light baggage, and so light baggage yet was so light

283


8. Having the right hand and arm being crippled by a sniper’s bullet during the First World War, Horace Pippin, a Black American painter, worked by holding the brush in his right hand and guiding its movements with his left. (A) (B) (C) (D) (E)

Having the right hand and arm being crippled by a sniper’s bullet during the First World War In spite of his right hand and arm being crippled by a sniper’s bullet during the First World War Because there had been a sniper’s bullet during the First World War that crippled his right hand and arm The right hand and arm being crippled by a sniper’s bullet during the First World War His right hand and arm crippled by a sniper’s bullet during the First World War

9. His studies of ice-polished rocks in his Alpine homeland, far outside the range of present-day glaciers, led Louis Agassiz in 1837 to propose the concept of an age in which great ice sheets had existed in now currently temperate areas. (A) (B) (C) (D) (E)

in which great ice sheets had existed in now currently temperate areas in which great ice sheets existed in what are now temperate areas when great ice sheets existed where there were areas now temperate when great ice sheets had existed in current temperate areas when great ice sheets existed in areas now that are temperate

10. It takes a deft balance between savings discipline, investment knowledge, risk taking, luck, and time to raise a million dollars through investments. (A) (B) (C) (D) (E)

It takes a deft balance between Deft balancing is needed between Deftly balanced, it takes It takes a deft balance of A deft balance is what one needs among

11. It was because she was plagued by injuries that tennis celebrity Tracy Austin, who won the Italian and U.S. Opens at age 16 in 1979, sat out the years between 1984 to 1988, occasionally working as a television commentator. (A) (B) (C) (D) (E)

284

It was because she was plagued by injuries that tennis celebrity Tracy Austin, who won the Italian and U.S. Opens at age 16 in 1979, sat out the years between After winning the Italian and U.S. Opens in 1979 at the age of 16, injuries plagued tennis celebrity Tracy Austin and forced her to sit out from Tracy Austin, the tennis celebrity who, plagued by injuries, won the Italian and U.S. Opens at age 16 in 1979, sat out from Plagued by injuries, tennis celebrity Tracy Austin, who won the Italian and U.S. Opens at age 16 in 1979, sat out the years from Tennis celebrity Tracy Austin, who won the Italian and U.S. Opens at age 16 in 1979, was


plagued by injuries when she sat out between

12. It was the loss of revenue from declines in tourism that in 1935 led the Saudi authorities’ granting a concession for oil exploration to the company that would later be known by the name of Aramco. (A) (B) (C) (D) (E)

authorities’ granting a concession for oil exploration to the company that would later be known by the name of authorities’ granting a concession for oil exploration to the company later to be known as named authorities granting a concession for oil exploration to the company that would later be known by the name of authorities to grant a concession for oil exploration to the company that later will be known as being authorities to grant a concession for oil exploration to the company later to be known as

13. Beatrix Potter, in her book illustrations, carefully coordinating them with her narratives, capitalized on her keen observation and love of the natural world. (A) (B) (C) (D) (E)

Beatrix Potter, in her book illustrations, carefully coordinating them with her narratives, In her book illustrations, carefully coordinating them with her narratives, Beatrix Potter In her book illustrations, which she carefully coordinated with her narratives, Beatrix Potter Carefully coordinated with her narratives, Beatrix Potter, in her book illustrations Beatrix Potter, in her book illustrations, carefully coordinated them with her narratives and

14. Because Halley’s comet changes orbit slightly during the seventy-six-year interval between passing close to Earth, it may veer onto a collision course with a planet sometime in the distant future. (A) (B) (C) (D) (E)

between passing of passing between its passes of its passes as it passes

15. Because the Earth’s crust is more solid there and thus better able to transmit shock waves, an earthquake of a given magnitude typically devastates an area 100 times greater in the eastern United States than it does in the West. (A) (B)

of a given magnitude typically devastates an area 100 times greater in the eastern United States than it does in the West of a given magnitude will typically devastate 100 times the area if it occurs in the eastern

285


(C) (D) (E)

United States instead of the West will typically devastate 100 times the area in the eastern United States than one of the comparable magnitude occurring in the West in the eastern United States will typically devastate an area 100 times greater than will a quake of comparable magnitude occurring in the West that occurs in the eastern United States will typically devastate 100 times more area than if it occurred with comparable magnitude in the West

16. (PR) Because the enemy’s new ship is the quietest and it is therefore the most elusive submarine, it is being increasingly viewed by the military as a threat to security. (A) (B) (C) (D) (E)

and it is therefore the most elusive submarine, it is being increasingly viewed it is therefore the most elusive of submarines, and it has increased the view and therefore the most elusive submarine, it is being increasingly viewed and therefore it is the most elusive of submarines, there is an increasing view therefore being the most elusive of submarines, it is increasingly viewed

17. Being a United States citizen since 1988 and born in Calcutta in 1940, author Bharati Mukherjee has lived in England and Canada, and first came to the United States in 1961 to study at the Iowa Writers’ Workshop. (A) (B) (C) (D) (E)

Being a United States citizen since 1988 and born in Calcutta in 1940, author Bharati Mukherjee has Having been a United States citizen since 1988, she was born in Calcutta in 1940; author Bharati Mukherjee Born in Calcutta in 1940, author Bharati Mukherjee became a United States citizen in 1988; she has Being born in Calcutta in 1940 and having been a United States citizen since 1988, author Bharati Mukherjee Having been born in Calcutta in 1940 and being a United States citizen since 1988, author Bharati Mukherjee

18. Believed to originate from a small area on their foreheads, elephants emit low-frequency sounds that may be used as a secret language to communicate with other members of the herd. (A) (B) (C) (D) (E)

286

Believed to originate from a small area on their foreheads, elephants emit low-frequency sounds that may be used Elephants emit low-frequency sounds that are believed to originate from a small area on their foreheads, and they may use this Elephants emit low-frequency sounds, believed to originate from a small area on their foreheads, that they may use Originating, it is believed, from a small area on their foreheads; elephants emit low-frequency sounds they may use Originating, it is believed, from a small area on their foreheads, low-frequency sounds are


emitted by elephants that may be used

19. Besides yielding such psychological rewards as relief from stress, deep relaxation, if practiced regularly, can strengthen the immune system and produce a host of other physiological benefits. (A) (B) (C) (D) (E)

deep relaxation, if practiced regularly, can strengthen the immune system one can strengthen the immune system through deep relaxation, if it is practiced regularly the immune system can be strengthened as a result of deep relaxation if practiced regularly when deep relaxation is practiced regularly, the immune system can be strengthened when practiced regularly, the results of deep relaxation can be to strengthen the immune system

20. Whereas in mammals the tiny tubes that convey nutrients to bone cells are arrayed in parallel lines, in birds the tubes form a random pattern. (A) (B) (C) (D) (E)

Whereas in mammals the tiny tubes that convey nutrients to bone cells are arrayed in parallel lines, in birds the tubes Whereas the tiny tubes for the conveying of nutrients to bone cells are arrayed in mammals in parallel lines, birds have tubes that Unlike mammals, where the tiny tubes for conveying nutrients to bone cells are arrayed in parallel lines, birds’ tubes Unlike mammals, in whom the tiny tubes that convey nutrients to bone cells are arrayed in parallel lines, the tubes in birds Unlike the tiny tubes that convey nutrients to bone cells, which in mammals are arrayed in parallel lines, in birds the tubes

(OG) Set 14 1. Retailers reported moderate gains in their November sales, as much because of their sales of a year earlier being so bad as that shoppers were getting a head start on buying their holiday gifts. (A) (B) (C) (D) (E)

of their sales of a year earlier being so bad as that of their sales a year earlier having been as bad as because of their sales a year earlier being as bad as because their sales a year earlier had been so bad as because their sales of a year earlier were as bad as that

2. Fossils of the arm of a sloth found in Puerto Rico in 1991, and dated at 34 million years old, made it the earliest known mammal of the Greater Antilles Islands. (A) (B)

sloth found in Puerto Rico in 1991, and dated at 34 million years old, made it the earliest known mammal of sloth, that they found in Puerto Rico in 1991, has been dated at 34 million years old, thus

287


(C) (D) (E)

making it the earliest mammal known on sloth that was found in Puerto Rico in 1991, was dated at 34 million years old, making this the earliest known mammal of sloth, found in Puerto Rico in 1991, have been dated at 34 million years old, making the sloth the earliest known mammal on sloth which, found in Puerto Rico in 1991, was dated at 34 million years old, made the sloth the earliest known mammal of

3. Defense attorneys have occasionally argued that their clients' misconduct stemmed from a reaction to something ingested, but in attributing criminal or delinquent behavior to some food allergy, the perpetrators are in effect told that they are not responsible for their actions. (A) (B) (C) (D) (E)

in attributing criminal or delinquent behavior to some food allergy, if criminal or delinquent behavior is attributed to an allergy to some food, in attributing behavior that is criminal or delinquent to an allergy to some food, if some food allergy is attributed as the cause of criminal or delinquent behavior, in attributing a food allergy as the cause of criminal or delinquent behavior,

4. A report by the American Academy for the Advancement of Science has concluded that much of the currently uncontrolled dioxins to which North Americans are exposed comes from the incineration of wastes. (A) (B) (C) (D) (E)

much of the currently uncontrolled dioxins to which North Americans are exposed comes much of the currently uncontrolled dioxins that North Americans are exposed to come much of the dioxins that are currently uncontrolled and that North Americans are exposed to comes many of the dioxins that are currently uncontrolled and North Americans are exposed to come many of the currently uncontrolled dioxins to which North Americans are exposed come

5. Recently physicians have determined that stomach ulcers are not caused by stress, alcohol, or rich foods, but a bacterium that dwells in the mucous lining of the stomach. (A) (B) (C) (D) (E)

not caused by stress, alcohol, or rich foods, but not caused by stress, alcohol, or rich foods, but are by caused not by stress, alcohol, or rich foods, but by caused not by stress, alcohol, and rich foods, but caused not by stress, alcohol, and rich foods, but are by

6. According to a recent poll, owning and living in a freestanding house on its own land is still a goal of a majority of young adults, like that of earlier generations. (A)

288

like that of earlier generations


(B) (C) (D) (E)

as that for earlier generations just as earlier generations did as have earlier generations as it was of earlier generations

7. In 2000, a mere two dozen products accounted for half the increase in spending on prescription drugs, a phenomenon that is explained not just because of more expensive drugs but by the fact that doctors an writing many more prescriptions for higher-cost drugs. (A) (B) (C) (D) (E)

a phenomenon that is explained not just because of more expensive drugs but by the fact that doctors are writing a phenomenon that is explained not just by the fact that drugs are becoming more expensive but also by the fact that doctors are writing a phenomenon occurring not just because of drugs that are becoming more expensive but because of doctors having also written which occurred not just because drugs are becoming more expensive but doctors are also writing which occurred not just because of more expensive drugs but because doctors have also written

8. Often visible as smog, ozone is formed in the atmosphere from hydrocarbons and nitrogen oxides, two major pollutants emitted by automobiles, react with sunlight. (A) (B) (C) (D) (E)

ozone is formed in the atmosphere from ozone is formed in the atmosphere when ozone is formed in the atmosphere, and when ozone, formed in the atmosphere when ozone, formed in the atmosphere from

9. Salt deposits and moisture threaten to destroy the Mohenjo-Daro excavation in Pakistan, the site of an ancient civilization that flourished at the same time as the civilizations in the Nile Delta and the river valleys of the Tigris and Euphrates. (A) (B) (C) (D) (E)

that flourished at the same time as the civilizations that had flourished at the same time as had the civilizations that flourished at the same time those had flourishing at the same time as those did flourishing at the same time as those were

10. Dirt roads may evoke the bucolic simplicity of another century, but financially strained townships point out that dirt roads cost twice as much as maintaining paved roads. (A)

dirt roads cost twice as much as maintaining paved roads

289


(B) (C) (D) (E)

dirt roads cost twice as much to maintain as paved roads do maintaining dirt roads costs twice as much as paved roads do maintaining dirt roads costs twice as much as it does for paved roads to maintain dirt roads costs twice as much as for paved roads

11. Although early soap operas were first aired on evening radio in the 1920s, they had moved to the daytime hours of the 1930s when the evening schedule became crowded with comedians and variety shows. (A) were first aired on evening radio in the 1920s, they had moved to the daytime hours of the 1930s (B) were first aired on evening radio in the 1920s, they were moved to the daytime hours in the 1930s (C) were aired first on evening radio in the 1920s, moving to the daytime hours in the 1930s (D) were aired first in the evening on 1920s radio, they moved to the daytime hours of the 1930s (E) aired on evening radio first in the 1920s, they were moved to the 1930s in the daytime hours

12. Nobody knows exactly how many languages there are in the world, partly because of the difficulty of distinguishing between a language and the sublanguages or dialects within it, but those who have tried to count typically have found about five thousand. (A) and the sublanguages or dialects within it, but those who have tried to count typically have found (B) and the sublanguages or dialects within them, with those who have tried counting typically finding (C) and the sublanguages or dialects within it, but those who have tried counting it typically find (D) or the sublanguages or dialects within them, but those who tried to count them typically found (E) or the sublanguages or dialects within them, with those who have tried to count typically finding

13. The energy source on Voyager 2 is not a nuclear reactor, in which atoms are actively broken apart; rather a kind of nuclear battery that uses natural radioactive decay to produce power. (A) (B) (C) (D) (E)

apart; rather apart, but rather apart, but rather that of apart, but that of apart; it is that of

14. Heating-oil prices are expected to be higher this year than last because refiners are paying about $5 a barrel more for crude oil than they were last year.

290


(A) Heating-oil prices are expected to be higher this year than last because refiners are paying about $5 a barrel more for crude oil than they were (B) Heating-oil prices are expected to rise higher this year over last because refiners pay about $5 a barrel for crude oil more than they did (C) Expectations are for heating-oil prices to be higher this year than last year's because refiners are paying about $5 a barrel for crude oil more than they did (D) It is the expectation that heating-oil prices will he higher for this year over last because refiners are paying about $5 a barrel more for crude oil now than what they were (E) It is expected that heating-oil prices will rise higher this year than last year's because refiners pay about $5 a barrel for crude oil more than they did

15. The recent surge in the number of airplane flights has clogged the nation's air-traffic control system, to lead to 55 percent more delays at airports, and prompts fears among some officials that safety is being compromised. (A) (B) (C) (D) (E)

to lead to 55 percent more delays at airports, and prompts leading to 55 percent more delay at airports and prompting to lead to a 55 percent increase in delay at airports and prompt to lead to an increase of 55 percent in delays at airports, and prompted leading to a 55 percent increase in delays at airports and prompting

16. One of the primary distinctions between our intelligence with that of other primates may lay not so much in any specific skill but in our ability to extend knowledge gained in one context to new and different ones. (A) between our intelligence with that of other primates may lay not so much in any specific skill but (B) between our intelligence with that of other primates may lie not so much in any specific skill but instead (C) between our intelligence and that of other primates may lie not so much in any specific skill as (D) our intelligence has from that of other primates may lie not in any specific skill as (E) of our intelligence to that of other primates may lay not in any specific skill but

17. Unlike Schoenberg's 12-tone system that dominated the music of the postwar period, Bartok founded no school and left behind only a handful of disciples. (A) (B) (C) (D) (E)

Schoenberg's 12-tone system that dominated Schoenberg and his 12-tone system which dominated Schoenberg, whose 12-tone system dominated the 12-tone system of Schoenberg that has dominated Schoenberg and the 12-tone system, dominating

291


18. Even though Clovis points, spear points with longitudinal grooves chipped onto their faces, have been found all over North America, they are named for the New Mexico site where they were first discovered in 1932. (A) Even though Clovis points, spear points with longitudinal grooves chipped onto their faces, have been found all over North America, they are named for the New Mexico site where they were first discovered in 1932. (B) Although named for the New Mexico site where first discovered in 1932, Clovis points are spear points of longitudinal grooves chipped onto their faces and have been found all over North America. (C) Named for the New Mexico site where they have been first discovered in 1932, Clovis points, spear points of longitudinal grooves chipped onto the faces, have been found all over North America. (D) Spear points with longitudinal grooves that are chipped onto the faces, Clovis points, even though named for the New Mexico site where first discovered in 1932, but were found all over North America. (E) While Clovis points are spear points whose faces have longitudinal grooves chipped into them, they have been found all over North America, and named for the New Mexico site where they have been first discovered in 1932.

19. Ranked as one of the most important of Europe's young playwrights, Franz Xaver Kroetz has written 40 plays; his works—translated into more than 30 languages—are produced more often than any contemporary German dramatist. (A) (B) (C) (D) (E)

than any than any other than are any than those of any other as are those of any

20. The stars, some of them at tremendous speeds, are in motion just as the planets are, yet being so far away from Earth that their apparent positions in the sky do not change enough for their movement to be observed during a single human lifetime. (A) (B) (C) (D) (E)

292

The stars, some of them at tremendous speeds, are in motion just as the planets are, yet being Like the planets, the stars are in motion, some of them at tremendous speeds, but they are Although like the planets the stars are in motion, some of them at tremendous speeds, yet As the planets, the stars are in motion, some of them at tremendous speeds, but they are The stars are in motion like the planets, some of which at tremendous speeds are in motion but


Sentence Correction ANSWER KEY Qs#/Set

Set 1 C

Set 2 A

Set 3 D

Set 4 C

Set 5 B

Set 6 D

Set 7 E

Set 8 E

Set 9 D

Set 10 A

Set 11 B

Set 12 B

Set 13 B

Set 14 D

B

D

D

C

C

E

D

D

C

A

A

D

E

D

B

C

E

B

A

C

B

B

C

E

D

A

C

B

A

C

A

D

C

C

B

C

B

A

B

B

B

E

B

A

E

A

E

D

B

D

D

C

D

D

C

C

D

E

E

B

B

A

A

A

D

A

D

A

C

E

C

D

A

E

A

A

E

B

B

E

D

E

E

B

B

E

E

D

E

E

D

C

A

B

B

B

E

B

A

C

B

A

A

B

D

A

A

E

D

E

B

A

A

B

D

C

B

C

A

A

E

C

E

D

D

B

E

C

E

A

B

A

E

E

B

A

A

E

D

B

C

D

E

B

E

C

A

D

A

E

B

A

E

A

E

C

B

E

A

D

E

E

D

E

A

E

C

B

A

E

C

C

D

D

C

D

C

C

E

A

C

A

E

B

E

D

E

D

B

A

E

B

D

C

D

E

C

D

A

B

C

B

A

E

E

A

C

A

C

C

E

C

D

A

C

D

E

C

E

C

D

E

C

C

C

E

C

E

E

B

D

B

E

E

D

A

C

A

B

E

E

C

A

C

C

B

C

C

C

B

A

D

A

B

D

A

B

E

D

A

E

A

B

C

A

B

1 2 3 4 5 6 7 8 9 10 11 12 13 14 15 16 17 18 19 20

293


Part IV Analytical Writing Assessment Assignments Analysis of an Argument Time 30 minutes Score Your essay will be graded on a scale from a low of 0 to a high of 6. Your essay will be graded by a human grader as well as a computerized essay grader (the "e-rater"). If the human and the e-rater agree on a score – that will be the grade your essay will receive. If they disagree by more than one point, a second human will grade the essay to resolve any differences. What is important? 1. Structure 2. Language: grammar and style Tips • Before you begin to write, outline your essay. Good organization is always counted, and now it is more important than ever. • Since your essay will be compared against other essays, think about the points that the best essays make. • The length of your essay is not a factor; the computer does not count the number of words in your response. • Use transitional words and phrases like “first, therefore, since, for example”, so that the computer can recognize structured arguments. • Avoid spelling and grammar errors. Although the e-rater does not grade spelling per se, if it cannot understand you or thinks you used the wrong words, it could give you a lower score. The task is always to WEAKEN a given argument (to point at insufficient evidences, questionable assumptions, and inaccurate conclusions). No personal opinions or any references to out-of-the-argument-scope information are welcome. Paragraph 1 – INTRODUCTION Idea 1 – General statement (e.g. “The argument about … seems logically inconsistent and can not be correctly evaluated”) Idea 2 – Thesis (List of all logical weaknesses of the argument – 3 points are ideal, 2 points are worse but OK) Paragraph 2 – BODY PARAGRAPH 1 (wholly devoted to logical weakness #1) Idea 1 – WHAT is the weak point of the argument? Idea 2 – WHY is it a weak point? Idea 3 – Request for additional information (What should be added to the argument – additional evidence, verified assumptions – to make it logically sound?) Paragraph 3 – BODY PARAGRAPH 2 (wholly devoted to logical weakness #2) *the same contents as in BP #1* Paragraph 4 – BODY PARAGRAPH 3 (wholly devoted to logical weakness #3)

294


*the same contents as in BP #1* Paragraph 5 – CONCLUSION Idea 1 – Paraphrased Thesis (list of logical weaknesses) Idea 2 – Conclusion statement (Therefore, without the additional information mentioned, the conclusion of the argument cannot be justified). Basic principles 1. Your control of language is important. 2. It is better to keep things simple. 3. Minor grammatical flaws will not torpedo your score. 4. Keep sight of your goal.

Strategy Step 1: Digest the issue/argument (2 minutes) Step 2: Select the points you will make (5 minutes) Step 3: Organize (1 minute) Step 4: Write/type your essay (20 minutes) Step 5: Proofread your work (2 minutes)

295


LANGUAGE Principle 1: Avoid Wordiness Do not use several words when one will do. Wordy phrases are like junk food: They add only fat, not muscle. Many people make the mistake of writing phrases such as “at the present time” or “at this point in time” instead of the simpler “now”, or “take into consideration” instead of “consider”, in an attempt to make their prose seem more scholarly or more formal. It does not work. Instead, their prose ends up seeming inflated and pretentious. Do not waste your words or your time. Wordy: I am of the opinion that the aforementioned managers should be advised that they will be evaluated with regard to the utilization of responsive organizational software for the purpose of devising a responsive network of customers. Concise: We should tell the managers that we will evaluate their use of flexible computerized data-bases to develop a customer network. Principle 2: Do Not Be Redundant Redundancy means that the writer needlessly repeats an idea. It is redundant to speak of "a beginner lacking experience." The word beginner implies lack of experience by itself. You can eliminate redundant words or phrases without changing the meaning of the sentence. Watch out for words that add nothing to the sense of the sentence. Here are some common redundancies: Redundant refer back few in number small-sized grouped together end result

Concise refer few small grouped result

Redundancy often results from carelessness, but you can easily eliminate redundant elements when proofreading. Principle 3: Avoid Needless Qualification Since the object of your essay is to convince your reader, you will want to adopt a reasonable tone. There will likely be no single, clear-cut "answer" to the essay topic, so do not overstate your case. Occasional use of such qualifiers as “fairly, rather, somewhat, relatively”, and of such expressions as “seems to be, a little, a certain amount” will let the reader know you are reasonable, but overusing such modifiers weakens your argument. Excessive qualification makes you sound hesitant. Like wordy phrases, qualifiers can add bulk without adding substance. Wordy:This rather serious breach of etiquette may possibly shake the very foundations of the corporate world. Concise: This serious breach of etiquette may shake the foundations of the corporate world. Wrong more unique the very worst

296

Correct unique the worst


completely full

full

Principle 4: Do Not Write Sentences Just to Fill Up Space This principle suggests several things: • Do not write a sentence that gets you nowhere. • Do not ask a question only to answer it • Do not merely copy the essay's directions. • Do not write a whole sentence only to announce that you are changing the subject. Wordy: Which idea of the author's is more in line with what I believe? This is a very interesting.... Concise: The author's beliefs are similar to mine. Principle 5: Avoid Needless Self-Reference Avoid such unnecessary phrases as "I believe," "I feel," and "in my opinion." There is no need to remind your reader that what you are writing is your opinion. Weak: I am of the opinion that air pollution is a more serious problem than the government has led us to believe. Forceful: Air pollution is a more serious problem than the government has led us to believe. Self-reference is another form of qualifying what you say—a very obvious form. One or two selfreferences in an essay might be appropriate, just as the use of qualifiers like probably and perhaps can be effective if you practice using them sparingly. Practice is the only sure way to improve your writing. Principle 6: Use the Active Voice Using the passive voice is a way to avoid accountability. Put verbs in the active voice whenever possible. In the active voice, the subject performs the action (e.g., we write essays). In the passive voice, the subject is the receiver of the action and the performer of the action is often only implied (e.g., essays are written). You should avoid the passive voice EXCEPT in the following cases: • When you do not know who performed the action: The letter was opened before I received it. (For example, see the last sentence of the above paragraph.) • When you prefer not to refer directly to the person who performs the action: An error has been made in computing this data.

Passive: The estimate of this year's tax revenues was prepared by the General Accounting Office. Active: The General Accounting Office prepared the estimate of this year's tax revenues. Principle 7: Avoid Weak Openings Try not to begin a sentence with “there is, there are, it is”. These roundabout expressions usually indicate that you are trying to distance yourself from the position you are taking.

297


Principle 8: Avoid Needlessly Vague Language Do not just ramble on when you're writing your GMAT essays. Choose specific, descriptive words. Vague language weakens your writing because it forces the reader to guess what you mean instead of concentrating fully on your ideas and style. The essay topics you're given are not going to be obscure. You will be able to come up with specific examples and concrete information about the topics. Your argument will be more forceful if you stick to this information. Weak: Brown is highly educated. Forceful: Brown has a master's degree in business administration. Principle 9: Avoid Slang and Colloquialisms Conversational speech is filled with slang and colloquial expressions. However, you should avoid slang on the GMAT Analytical Writing Assessment. Slang terms and colloquialisms can be confusing to the reader, since these expressions are not universally understood. Even worse, such informal writing may give readers the impression that you are poorly educated or arrogant. Inappropriate: He is really into gardening. Correct: He enjoys gardening. Inappropriate: She plays a wicked game of tennis. Correct: She excels in tennis. Principle 10: Use Commas Correctly When using the comma, follow these rules: A. Use commas to separate items in a series. If more than two items are listed in a series, they should be separated by commas; the final comma—the one that precedes the word and—is optional. Never use a comma AFTER the word and. Correct: My recipe for buttermilk biscuits contains flour, baking soda, salt, shortening, and buttermilk. Correct: My recipe for chocolate cake contains flour, baking soda, sugar, eggs, milk and chocolate. B. Do not place commas before the first element of a series or after the last element. Wrong: My investment adviser recommended that I construct a portfolio of, stocks, bonds, commodities futures, and precious metals. Wrong: The elephants, tigers, and dancing bears, were the highlights of the circus. C. Use commas to separate two or more adjectives before a noun; do not use a comma after the last adjective in the series. Wrong: I can't believe you sat through that long, dull, uninspired, movie three times. Correct: I can't believe you sat through that long, dull, uninspired movie three times. D. Use commas to set off parenthetical clauses and phrases. (A parenthetical expression is one that is not necessary to the main idea of the sentence.)

298


Correct: Gordon, who is a writer by profession, bakes an excellent cheesecake. The main idea is that Gordon bakes an excellent cheesecake. The intervening clause merely serves to identify Gordon; thus it should be set off with commas. Correct: The newspaper that has the most insipid editorials is the Daily Times. Correct: The newspaper, which has the most insipid editorials of any I have read, won numerous awards last week. In the first of these examples the clause beginning with that defines which paper the author is discussing. In the second example, the main point is that the newspaper won numerous awards, and the intervening clause beginning with which identifies the paper. E. Use commas after introductory participial or prepositional phrases. Correct: Having watered his petunias every day during the drought, Harold was very disappointed when his garden was destroyed by insects. Correct: After the banquet, Harold and Martha went dancing. F. Use commas to separate independent clauses (clauses which could stand alone as complete sentences) connected by coordinate conjunctions such as “and, but, not, yet�, etc. Correct: Susan's old car has been belching blue smoke from the tailpipe for two weeks, but it has not broken down yet. Note: Make sure the comma separates two independent clauses, joined by a conjunction. It is incorrect to use a comma to separate the two parts of a compound verb. Wrong: Barbara went to the grocery store, and bought two quarts of milk.

299


Examples of essays Response 1: “Your vacation resort can ill afford not to publisize its offerings on Travelshack.com. Our readership has more than doubled in the past year alone and is dominated by savvy consumers with large disposable incomes. Witness the experience of Snowbert Ski Lodge. Since it began advertising on our website a year ago, it is regularly booked to capacity and its annual profits have increased more than threefold from the previous year.” The argument is illogical because it fails to present the information necessary to take a decision in favor of Travelshack.com. Besides, it provides rather misleading information, which can result in wrong conclusions. First of all, we know nothing about Travelshack. The website of this company may be visited by a limited number of people. Even if the number of visitors doubled there still would be the possibility that those visitors are occasional or represent a minority of potential clients of the resort business. It is possible that existing visitors are not interested in a certain type of resort. Therefore we cannot say that there will be enough visitors interested in advertisement of the certain resort. Secondly, the argument does not include any recent information. All facts are connected to some “previous” or “last” periods of time, giving no clue about Travelshack today. The argument states that some Snowbert Ski Lodge resort has started to advertise on Travelshack.com last year, but it provides no evidence that this resort still uses services of Travelshack. Moreover, the argument omits the information about the role of advertisement, which was published on Travelshack.com by Snowbert Ski Lodge, in a success of Snowbert. This advertisement might be a small part of a large advertisement campaign done by Snowbert, therefore its value and efficiency cannot be adequately measured. If it was mentioned in the argument that Snowbert used Travelshack as the sole advertisement company we would also agree to use Travelshack for advertising. Counterarguments stated above incline us to get more recent and specific information about Travelshack and its clients in order to make a final decision. If the necessary information is absent the decision cannot be made in favor of Travelshack as an appropriate company to advertise the resort. Response 2: The following appeared in a newspaper editorial. “The global entertainment company Zipney has signed an agreement with the government of the Eastern European county of Magdania to build a Zipneyland theme park there within the next five years. Since Zipneyland parks in Los Angeles and London attract between three and six million tourists per year, the years of waning tourism in Magdania are clearly over.” The author argues that the years of waning tourism in Magdania are clearly over. The author's argument is not entirely logically convincing, because it ignores certain crucial assumptions. First, the argument appears sound, but is in fact lacking information about the Eastern European country of Magdania. Since we don’t know the political and economical situation in this region we can not compare the described situations in Los Angeles and London with that of Magdania. Hence, we can not assume that it will attract the same number of tourists every year. The author could enhance his position by showing that this country shows economical and political stability, has a good transportation system and very friendly to tourists. Second, the author does not present sufficient grounds for the reader to believe that the tourism in Magdania is in a bad condition. He didn’t provide any information about how he evaluated the situation in tourism industry there. The author could have strengthened his position by showing some statistics or giving some examples of how bad the situation in this sector of economics was.

300


Finally, the argument omits the name of the newspaper where this information appeared. Since we don’t know whether this agreement is newspaper hoax or true we can not trust it. Any evidence that this newspaper is serious and trustable would strengthen the argument greatly. There are too many holes in the argument, so it is difficult to take the author seriously. Unless some more essential evidences are provided, the conclusion is not valid. Response 3: “The following appeared in a finance magazine. At the annual SpoonCorp shareholders' meeting in Alaska this year, several shareholders expressed their objection to the company's imminent merger with archrival LadleWorks. Since these individuals own only a collective 15 percent of the company's stock, SpoonCorp's board of directors should go ahead with the merger as planned.” This argument, forcing SpoonCorp’s board of directors to merger with LadleWorks, is unconvincing because it relies on too many assumptions that are not necessarily true. There are three points the board of directors should consider before making any decisions on the merger. My first problem with this argument is the author’s skepticism that the plan shall be realized because shareholders who objected the plan own only fifteen percent of company’s stock. Is there any me mentions about company’s ownership? It is not necessary that fifteen percent is a small number of shares. In some cases, fifteen percent of shares can give the major vote to shareholders. That is why the author’s remark that “these individuals collect only 15 percent of company’s stock”, therefore board of directors should go ahead with the plan” is really weak. Another problem with the argument is that this argument says nothing about the merger plan. It gives no information about either when and who proposed that plan or how long company’s specialist worked over that merger plan. Important thing to remember is that SpoonCorp is planning to merger with its archrival. The argument presents no information whether it is appropriate for SpoonCorp to merge with its rival, what benefits the shareholders of SpoonCorp will get, how it will affect the company’s reputation. We know nothing about this. According to the argument it seems that somebody dropped a thought about the merger and the board of directors should immediately take a decision to merger. This is nonsense. Lastly, I take the issue with the source of this article. Why should anybody believe that unknown finance magazine? If it was Wall Street Journal or Business Week, the information would sound much more valuable and true. Why do none of these respectful magazines say nothing about this news? Any evidence that this journal is real source to take this news true would strengthen this argument greatly. There are too many weak points in the argument, so it is difficult to take this information seriously. Unless some further evidence surfaces in each of the three problems I mentioned above, the premises that are presented are not sufficient to decide whether to merge or not. Response 4: The following appeared in an American medical journal. "A new study has shown that cardiac patients can reduce their chance of a heart attack by taking a reduced-potency aspirin pill once a day. Therefore, now would be a good time to invest in Malatet Pharmaceutical Corp., which happens to be Europe's largest producer of reduced-potency aspirin." This argument is poorly arranged. It uses assumptions and premises that are not useful and is therefore not very strong. For example, how do we know that this study is a good one? Maybe the scientists didn't know what they were doing and their information is incorrect. After all, people are capable of making mistakes and experimenting is always subject to margins of error. It's not a good idea to believe this argument until you get to know more about the scientists and the other work that they have done. It is

301


important to know that they have a history of excellence in their field and are able to determine when the tests they have performed are accurate. There have been many examples in the past of scientific failures, and this could be another one. Also, it is important to know about "reduced-potency" aspirin. We need to know how "reduced" the potency is and test it with aspirin that is more potent. Is it possible for a "high-potency" aspirin to have an even better effect? Because if the company makes "reduced potency" aspirin but not "highpotency" aspirin, the company could be in trouble. In conclusion, this argument can’t be a good one because we don't know if the study was properly conducted and that "high-potency" aspirin isn't better for you.

Response 5: The following appeared in an American medical journal. "A new study has shown that cardiac patients can reduce their chance of a heart attack by taking a reduced-potency aspirin pill once a day. Therefore, now would be a good time to invest in Malatet Pharmaceutical Corp., which happens to be Europe's largest producer of reduced-potency aspirin." Human behavior tells us that this argument is flawed on many levels because it relies so much on assumptions about how people will react to the news of this new study. There is an announcement that a weaker form of aspirin can guard against heart disease, and right away we are to believe that Malatet Pharmaceuticals will make a lot of money. This author forgets that reduced-potency aspirin won't be successful unless people trust the product and buy it over and over again. We humans are suspicious by nature, and it takes a lot to win over our trust when it comes to what we put in our bodies. What do we know about this Malatet company? It obviously chose a clever name because mal a la tete means "headache" in French, but that isn't enough. Humans need and appreciate humor in life, but we need more than just whimsy from drug companies because our health is serious business. First, Malatet has to convince us to buy its products, and then to buy its stock. Both are a tall order by themselves; together, it is an even more formidable challenge. The author also assumes that when we buy the aspirin, we will actually have the self-discipline to take it regularly. Humans can be lazy and forgetful, and this aspirin is unlikely to have the effect it portrays itself to have if no one bothers to take it. It's impossible to trust this argument. The fact that Malatet is a European company is also especially deceptive, because Americans often associate Europe with cutting-edge medicinal professionals, especially for its prowess among Swiss neurosurgeons. It is tempting to group this company with the many other fine European institutions, but sometimes the exception does not prove the rule. We humans must be alert and not jump to any unlikely conclusions.

302


Writing Bank ARGUMENT Below you can see Analysis of an Argument questions used on the GMAT® exam. Each question is followed by this statement: Discuss how well reasoned you find this argument. In your discussion be sure to analyze the line of reasoning and the use of evidence in the argument. For example, you may need to consider what questionable assumptions underlie the thinking and what alternative explanations or counterexamples might weaken the conclusion. You can also discuss what sort of evidence would strengthen or refute the argument, what changes in the argument would make it more logically sound, and what, if anything, would help you better evaluate its conclusion. 1. The following appeared as part of an annual report sent to stockholders by Olympic Foods, a processor of frozen foods: “Over time, the costs of processing go down because as organizations learn how to do things better, they become more efficient. In color film processing, for example, the cost of a 3-by-5-inch print fell from 50 cents for five-day service in 1970 to 20 cents for one-day service in 1984. The same principle applies to the processing of food. And since Olympic Foods will soon celebrate its 25th birthday, we can expect that our long experience will enable us to minimize costs and thus maximize profits.” Discuss how well reasoned . . . etc. 2. The following appeared in a memorandum from the business department of the Apogee Company: “When the Apogee Company had all its operations in one location, it was more profitable than it is today. Therefore, the Apogee Company should close down its field offices and conduct all its operations from a single location. Such centralization would improve profitability by cutting costs and helping the company maintain better supervision of all employees.” Discuss how well reasoned . . . etc. 3. The following appeared in a memorandum issued by a large city’s council on the arts: “In a recent citywide poll, 15 percent more residents said that they watch television programs about the visual arts than was the case in a poll conducted five years ago. During these past five years, the number of people visiting our city’s art museums has increased by a similar percentage. Since the corporate funding that supports public television, where most of the visual arts programs appear, is now being threatened with severe cuts, we can expect that attendance at our city’s art museums will also start to decrease. Thus some of the city’s funds for supporting the arts should be reallocated to public television.” Discuss how well reasoned . . . etc. 4. The following appeared in an announcement issued by the publisher of The Mercury, a weekly newspaper: “Since a competing lower-priced newspaper, The Bugle, was started five years ago, The Mercury’s circulation has declined by 10,000 readers. The best way to get more people to read The Mercury is to reduce its price below that of The Bugle, at least until circulation increases to former levels. The increased circulation of The Mercury will attract more businesses to buy advertising space in the paper.” Discuss how well reasoned . . . etc.

303


5. The following appeared in a report presented for discussion at a meeting of the directors of a company that manufactures parts for heavy machinery: “The falling revenues that the company is experiencing coincide with delays in manufacturing. These delays, in turn, are due in large part to poor planning in purchasing metals. Consider further that the manager of the department that handles purchasing of raw materials has an excellent background in general business, psychology, and sociology, but knows little about the properties of metals. The company should, therefore, move the purchasing manager to the sales department and bring in a scientist from the research division to be manager of the purchasing department.” Discuss how well reasoned . . . etc. 6. The following appeared as part of an article in a magazine devoted to regional life: “Corporations should look to the city of Helios when seeking new business opportunities or a new location. Even in the recent recession, Helios’s unemployment rate was lower than the regional average. It is the industrial center of the region, and historically it has provided more than its share of the region’s manufacturing jobs. In addition, Helios is attempting to expand its economic base by attracting companies that focus on research and development of innovative technologies.” Discuss how well reasoned . . . etc. 7. The following appeared in the health section of a magazine on trends and lifestyles: “People who use the artificial sweetener aspartame are better off consuming sugar, since aspartame can actually contribute to weight gain rather than weight loss. For example, high levels of aspartame have been shown to trigger a craving for food by depleting the brain of a chemical that registers satiety, or the sense of being full. Furthermore, studies suggest that sugars, if consumed after at least 45 minutes of continuous exercise, actually enhance the body’s ability to burn fat. Consequently, those who drink aspartame-sweetened juices after exercise will also lose this calorie-burning benefit. Thus it appears that people consuming aspartame rather than sugar are unlikely to achieve their dietary goals.” Discuss how well reasoned . . . etc. 8. The following appeared in the editorial section of a corporate newsletter: “The common notion that workers are generally apathetic about management issues is false, or at least outdated: a recently published survey indicates that 79 percent of the nearly 1,200 workers who responded to survey questionnaires expressed a high level of interest in the topics of corporate restructuring and redesign of benefits programs.” Discuss how well reasoned . . . etc. 9. The following appeared in the opinion column of a financial magazine: “On average, middle-aged consumers devote 39 percent of their retail expenditure to department store products and services, while for younger consumers the average is only 25 percent. Since the number of middle-aged people will increase dramatically within the next decade, department stores can expect retail sales to increase significantly during that period. Furthermore, to take advantage of the trend, these stores should begin to replace some of those products intended to attract the younger consumer with products intended to attract the middle-aged consumer.” Discuss how well reasoned . . . etc. 10.

304

The following appeared in the editorial section of a local newspaper:


“This past winter, 200 students from Waymarsh State College traveled to the state capitol building to protest against proposed cuts in funding for various state college programs. The other 12,000 Waymarsh students evidently weren’t so concerned about their education: they either stayed on campus or left for winter break. Since the group who did not protest is far more numerous, it is more representative of the state’s college students than are the protesters. Therefore the state legislature need not heed the appeals of the protesting students.” Discuss how well reasoned . . . etc.

305


Part V Putting it all together Test 1 In terms of its prevalence, obesity is the leading disease in the United States. There is no universally accepted standard for obesity, defined generally as an excess of adipose tissue, but a common rule of thumb classifies people who are more than 20 percent above their desirable weight as obese. By this measure, 30 percent of men and 40 percent of women in America are obese. Although studies show that few of these people will ever recover fully and permanently from the disease, the incidence of obesity in future generations can be reduced. Adipose tissue is a triumph of evolution. Fat yields 9 calories per gram, while protein, like carbohydrates, yields only 4 calories per gram. Fat also contains much less water than protein does. Therefore, fat is much more efficient for storing excess energy than is protein. Primitive humans, with uncertain food sources, had a great need for excess fat, and their bodies adapted accordingly. Modern humans, with a predictable food supply and a sedentary life-style, are burdened by this vestige of evolution. Although they need some adipose tissue to provide insulation and protect internal organs from injury, modern humans need much less than their primitive ancestors did. In an attempt to shed excess adipose tissue, many Americans turn from one fad diet to another, and a billion-dollar diet industry has grown up to aid them in their efforts. Nevertheless, the five-year cure rate for obesity is very low. In fact, by comparison, cancer is more curable. The reasons for this are psychological as well as physiological. From a physical standpoint, losing a pound or two a week for a few weeks is not difficult because most of the loss is in the form of protein and water, and protein carries four times its weight in water. However, protein is also the only source of nitrogen in the body, and when the body loses too much nitrogen, it acts to correct the imbalance by excreting less nitrogen than it takes in. Hence beyond a certain point additional weight loss must come from adipose tissue, which, because of its compactness, takes longer to shed. The body’s tendency to return to nitrogen balance and to protect its energy reserves can be so strong that dieters may stop losing or even gain weight while still expending more calories than they ingest. As a result, they frequently suffer not only from hunger, weakness, and a decreased metabolic rate, but also from depression and inactivity, all of which lead them to abandon their diets. Probably because of numerous psychological factors as well as physiological factors such as increased lipid synthesis, they then tend to regain weight rapidly. While vigorous attempts to reduce obesity in America should be aimed at all affected, the most successful efforts are likely to be those directed toward children. If the advertising and food industries stop trying to sell high-calorie, nutritionally deficient food to children, and if parents understand that the feeding patterns they impose on their children can determine the adolescent and adult eating habits those children will develop, the future generation may not be as fat as ours is.

1.

The primary purpose of the passage is to

(A) explain why prevention is the best defense against obesity (B) criticize the food and advertising industries for encouraging bad eating habits and thus contributing to the prevalence of obesity in the United States (C) recruit volunteers for a national crusade against obesity (D) discourage dependency on fad diets as a method for losing weight (E) argue that obesity is a genetic disorder that is virtually impossible to cure

306


2. Which of the following, if true, best helps to explain why a greater percentage of women than men in the United States are considered obese by the measure mentioned in the passage? (A) The measure is the same for both men and women, but it is normal for women to carry a greater percentage of fat at any weight. (B) The average man is heavier than the average woman; therefore, men generally need to gain more weight than women do before they are considered obese. (C) There are more women in the United States than there are men. (D) Because of the influence of the fashion and film industries, many American women consider themselves obese even though they are not. (E) Women burn off fewer calories than men because their bodies contain a lower percentage of muscle, and muscle burns calories at a faster rate than fat.

3. According to the passage, modern humans do not need to store as much fat as primitive humans because (A) (B) (C) (D) (E)

modern humans work fewer hours than their primitive forebears did the diet of modern humans is higher in protein than was the diet of primitive humans modern humans eat more regularly than primitive humans did primitive humans had to insulate their bodies from the cold whereas modern humans do not the food consumed by modern humans has a higher nutritional value than that consumed by primitive humans

4. Which of the following, if it could be demonstrated, would most strengthen the claim that the feeding patterns of children “can determine the adolescent and adult eating habits those children will develop�? (A) The incidence of obesity in children whose parents are both obese is 80 percent. (B) Thirty-six percent of infants who are obese during their first half year of life are also overweight between the ages of 20 and 36. (C) Excessive weight gain during the teen years can set the stage for a lifetime battle against the bulge. (D) Up to 80 percent of youngsters who are fat as 5-year-olds end up fat as adults. (E) The average child witnesses more than 15,000 commercials a year for snacks, candy, and soft drinks laden with fat, sugar, and calories.

5. Cardiopulmonary resuscitation should begin between one to four minutes after a cardiac arrest in order to be a success. (A) Cardiopulmonary resuscitation should begin between one to four minutes after a cardiac arrest in order to be a success. (B) Between one and four minutes after a cardiac arrest, cardiopulmonary resuscitation should begin to be successful. (C) Successful cardiopulmonary resuscitation should be begun from one to four minutes after a

307


cardiac arrest occurs. (D) If it is to be successful, a cardiac arrest should be treated with cardiopulmonary resuscitation one to four minutes afterward. (E) To be successful, cardiopulmonary resuscitation should begin within one to four minutes after a cardiac arrest.

6. A researcher discovered that people who have low levels of immune-system activity tend to score much lower on tests of mental health than do people with normal or high immune-system activity. The researcher concluded from this experiment that the immune system protects against mental illness as well as against physical disease. The researcher’s conclusion depends on which of the following assumptions? (A) High immune-system activity protects against mental illness better than normal immunesystem activity does. (B) Mental illness is similar to physical disease in its effects on body system. (C) People with high immune-system activity cannot develop mental illness. (D) Mental illness does not cause people’s immune-system activity to decrease. (E) Psychological treatment of mental illness is not as effective as is medical treatment.

7. Charles Lindbergh, for his attempt at a solo transatlantic flight, was very reluctant to have any extra weight on his plane, he therefore refused to carry even a pound of mail, despite being offered $1,000 to do so. (A) Charles Lindbergh, for his attempt at a solo transatlantic flight, was very reluctant to have any extra weight on his plane, he therefore (B) When Charles Lindbergh was attempting his solo transatlantic flight, being very reluctant to have any extra weight on his plane, he (C) Since he was very reluctant to carry any extra weight on his plane when he was attempting his solo transatlantic flight, so Charles Lindbergh (D) Being very reluctant to carry any extra weight on his plane when he attempted his solo transatlantic flight was the reason that Charles Lindbergh (E) Very reluctant to have any extra weight on his plane when he attempted his solo transatlantic flight, Charles Lindbergh

8. Chinese, the most ancient of living writing systems, consists of tens of thousands of ideographic characters, each character a miniature calligraphic composition inside its own square frame. (A) (B) (C) (D) (E)

308

each character a miniature calligraphic composition inside its all the characters a miniature calligraphic composition inside their all the characters a miniature calligraphic composition inside its every character a miniature calligraphic composition inside their each character a miniature calligraphic composition inside their


9. Are you still reading the other newspaper in town? Did you know that the Daily Bugle is owned by an out-of-town business syndicate that couldn’t care less about the people of Gotham City? Read the Daily Clarion, the only real voice of the people of Gotham City! Which of the following most directly refutes the argument raised in the advertisement above? (A) Over half of the advertising revenues of the Daily Clarion come from firms whose headquarters are located outside of Gotham City. (B) The Daily Clarion usually devotes more of its pages to out-of-town news than does the Daily Bugle. (C) Nearly 40 percent of the readers of the Daily Clarion reside outside the limits of Gotham City. (D) The editor-in-chief and all the other members of the editorial staff of the Daily Bugle have lived and worked in Gotham City for ten years or more. (E) The Daily Bugle has been published in Gotham City for a longer time than has the Daily Clarion.

10.

Which of the following best completes the passage below?

The most serious flaw in television’s coverage of election campaigns is its tendency to focus on the horse-race side of politics—that is, to concentrate on the question “Who’s winning?” at the expense of substantive coverage of the issues and the candidates’ positions on them. The endless interviews with campaign managers, discussions of campaign strategies, and, especially, the obsession with opinion polls have surrounded elections with the atmosphere of a football game or a prizefight. To reform this situation, a first step might well be______ (A) (B) (C) (D) (E)

a shortening of the length of election campaigns to a period of six weeks a stringent limit on campaign spending a reduction in the television coverage of opinion polls during election campaigns the publication and distribution of voter-education literature to inform the public about each candidate’s position on the major issues a limit on the length and number of political advertisements broadcast on television

The number of women directors appointed to corporate boards in the United States has increased dramatically, but the ratio of female to male directors remains low. Although pressure to recruit women directors, unlike that to employ women in the general work force, does not derive from legislation, it is nevertheless real. Although small companies were the first to have women directors, large corporations currently have a higher percentage of women on their boards. When the chairs of these large corporations began recruiting women to serve on boards, they initially sought women who were chief executive officers (CEO’s) of large corporations. However, such women CEO’s are still rare. In addition, the ideal of six CEO’s (female or male) serving on the board of each of the largest corporations is realizable only if every CEO serves on six boards. This raises the specter of director overcommitment and the resultant dilution of contribution. Consequently, the chairs next sought women in business who had the equivalent of CEO experience. However, since it is only recently that large numbers of women have begun to rise in management, the chairs began to recruit women of high

309


achievement outside the business world. Many such women are well known for their contributions in government, education, and the nonprofit sector. The fact that the women from these sectors who were appointed were often acquaintances of the boards’ chairs seems quite reasonable: chairs have always considered it important for directors to interact (to act upon one another) comfortably in the boardroom. Although many successful women from outside the business world are unknown to corporate leaders, these women are particularly qualified to serve on boards because of the changing nature of corporations. Today a company’s ability to be responsive to the concerns of the community and the environment can influence that company’s growth and survival. Women are uniquely positioned to be responsive to some of these concerns. Although conditions have changed, it should be remembered that most directors of both sexes are over fifty years old. Women of that generation were often encouraged to direct their attention toward efforts to improve the community. This fact is reflected in the career development of most of the outstandingly successful women of the generation now in their fifties, who currently serve on corporate boards: 25 percent are in education and 22 percent are in government, law, and the nonprofit sector. One organization of women directors is helping business become more responsive to the changing needs of society by raising the level of corporate awareness about social issues, such as problems with the economy, government regulation, the aging population, and the environment. This organization also serves as a resource center of information on accomplished women who are potential candidates for corporate boards.

11. According to the passage, the pressure to appoint women to corporate boards differs from the pressure to employ women in the work force in which of the following ways? (A) Corporate boards are under less pressure because they have such a small number of openings. (B) Corporate boards have received less pressure from stockholders, consumers, and workers within companies to include women on their boards. (C) Corporate boards have received less pressure from the media and the public to include women on their boards. (D) Corporations have only recently been pressured to include women on their boards. (E) Corporations are not subject to statutory penalty for failing to include women on their boards.

12. All of the following are examples of issues that the organization described in the last paragraph would be likely to advise corporations on EXCEPT (A) (B) (C) (D) (E)

long-term inflation health and safety regulations retirement and pension programs the energy shortage how to develop new markets

13. It can be inferred from the passage that, when seeking to appoint new members to a corporation’s board, the chair traditionally looked for candidates who (A) had legal and governmental experience

310


(B) (C) (D) (E)

had experience dealing with community affairs could work easily with other members of the board were already involved in establishing policy for that corporation had influential connections outside the business world

14.

Which of the following best describes the organization of the passage?

(A) A problem is described, and then reasons why various proposed solutions succeeded or failed are discussed. (B) A problem is described, and then an advantage of resolving it is offered. (C) A problem is described, and then reasons for its continuing existence are summarized. (D) The historical origins of a problem are described, and then various measures that have successfully resolved it are discussed. (E) The causes of a problem are described, and then its effects are discussed.

15. A newly discovered disease is thought to be caused by a certain bacterium. However, recently released data notes that the bacterium thrives in the presence of a certain virus, implying that it is actually the virus that causes the new disease. Which of the following pieces of evidence would most support the data’s implication? (A) In the absence of the virus, the disease has been observed to follow infection by the bacterium. (B) The virus has been shown to aid the growth of bacterium, a process which often leads to the onset of the disease. (C) The virus alone has been observed in many cases of the disease. (D) In cases where the disease does not develop, infection by the bacterium is usually preceded by infection by the virus. (E) Onset of the disease usually follows infection by both the virus and the bacterium.

16. Local phone companies have monopolies on phone service within their areas. Cable television can be transmitted via the wires that are already in place and owned by the phone companies. Cable television companies argue that if the telephone companies were to offer cable service, these telephone companies would have an unfair advantage, because their cable transmissions could be subsidized by the profits of their monopolies on phone service. On the basis of the information provided in the passage above, which of the following questions can be answered? (A) Are phone companies as efficient as cable companies in providing reliable and inexpensive service? (B) If phone companies were allowed to provide cable service, would they want to do so? (C) Do the cable companies believe that the local phone companies make a profit on phone service? (D) Are local phone companies forbidden to offer cable service?

311


(E)

Is it expected that phone companies will have a monopoly on cable service?

17. In 1846 about 80 percent of the towns in New York State banned the sale of alcoholic beverages. A recent article about the bans concludes that mid-nineteenth-century supporters of the temperance movement were not residents of remote rural areas, as has often been asserted; rather, they were concentrated in centers of economic opportunity. Which of the following, if true, best supports the conclusion reached in the article? (A) After 1846 the temperance movement grew rapidly and it flourished until the turn of the century. (B) Support for the ban on alcohol was strongest in New York towns where the economy was based on new, growing industries. (C) Many young New York State farmers supported the ban on alcohol. (D) The most adamant opponents of the ban included several affluent factory and mill owners. (E) In New York City, which was a commercial center in 1846, the sale of alcoholic beverages was not banned.

18. Clark and Florence Wallace, a husband-and-wife medical team, worked steadily and efficiently through the night, but sipping their coffee the next morning, she noticed that he seemed disoriented. (A) (B) (C) (D) (E)

sipping their coffee the next morning, she noticed that he seemed disoriented sipping their coffee the next morning, he seemed to be disoriented, she noticed as they sipped their coffee the next morning, she noticed that he seemed disoriented as they were sipping their coffee the next morning, he seemed, she noticed, disoriented he seemed disoriented, she noticed, sipping their coffee the next morning

19. Climatic shifts are so gradual as to be indistinguishable at first from ordinary fluctuations in the weather. (A) (B) (C) (D) (E)

so gradual as to be indistinguishable so gradual they can be indistinguishable so gradual that they are unable to be distinguished gradual enough not to be distinguishable gradual enough so that one cannot distinguish them

20. Interest rates on mortgages have declined steadily during the first six months of this year but virtually remained unchanged during the next three months.

312


(A) (B) (C) (D) (E)

have declined steadily during the first six months of this year but virtually remained unchanged declined steadily during the first six months of this year but virtually remain unchanged steadily declined during the first six months of this year but remain virtually unchanged declined steadily during the first six months of this but have remained virtually unchanging declined steadily during the first six months of this year but have remained virtually unchanged

21. Many economists predict that the next recession, when it comes, will be caused by Federal Reserve action taken to prevent an inflationary upsurge that would result if the economy were to expand at an annual rate of three percent or more. (A) (B) (C) (D) (E)

taken to prevent an inflationary upsurge that would result they took for preventing an inflationary upsurge that would result taken to prevent an inflationary upsurge resulting they took to prevent an inflationary upsurge resulting taken for preventing an inflationary upsurge that will result

22. Many environmentalists believe that the widespread planting of trees, along with the conservation of existing forests, would be one of the surest, easiest, and least expensive ways to begin to halt or even to reverse the buildup of carbon dioxide in the air. (A) (B) (C) (D) (E)

one of the surest, easiest, and least expensive ways to begin to halt or even to reverse one of the most sure, easy, and least expensive ways to begin a halt or even reverse one of the surest, easiest, and least expensive ways that would begin halting or even reversing a most sure, easy, and inexpensive way beginning the halting and even reversing of the most sure, easiest, and inexpensive way that would begin halting or even reversing

23. According to a recent study, fifteen corporations in the United States that follow a credo of social responsibility are also very profitable. Because of their credos, these fifteen corporations give generously to charity, follow stringent environmental-protection policies, and have vigorous affirmative-action programs. Which of the following can be correctly inferred from the statements above? (A) Following a credo of social responsibility helps to make a corporation very profitable. (B) It is possible for a corporation that follows a credo of social responsibility to be very profitable. (C) A corporation that gives generously to charity must be doing so because of its credo of social responsibility. (D) Corporations that are very profitable tend to give generously to charity. (E) Corporations that have vigorous affirmative-action programs also tend to follow stringent environmental-protection policies.

24.

Continue the argument below

313


Some observers have concluded that the rise in the price of pepper means that the switch by some growers from pepper to cocoa left those growers no better off than if none of them had switched; this conclusion, however, is unwarranted because it can be inferred to be likely that___ (A) (B) (C) (D) (E)

those growers could not have foreseen how high the price of pepper would go the initial cost involved in switching from pepper to cocoa is substantial supplies of pepper would not be as low as they are if those growers had not switched crops cocoa crops are as susceptible to being reduced by bad weather as are pepper crops as more growers turn to growing cocoa, cocoa supplies will increase and the price of cocoa will fall precipitously

In Forces of Production, David Noble examines the transformation of the machine-tool industry as the industry moved from reliance on skilled artisans to automation. Noble writes from a Marxist perspective, and his central argument is that management, in its decisions to automate, conspired against labor: the power that the skilled machinists wielded in the industry was intolerable to management. Noble fails to substantiate this claim, although his argument is impressive when he applies the Marxist concept of “de-skilling”—the use of technology to replace skilled labor—to the automation of the machine-tool industry. In automating, the industry moved to computer-based, digitized “numerical-control” (N/C) technology, rather than to artisan-generated “record-playback” (R/P) technology. Although both systems reduced reliance on skilled labor, Noble clearly prefers R/P, with its inherent acknowledgment of workers’ skills: unlike N/C, its programs were produced not by engineers at their computers, but by skilled machinists, who recorded their own movements to “teach” machines to duplicate those movements. However, Noble’s only evidence of conspiracy is that, although the two approaches were roughly equal in technical merit, management chose N/C. From this he concludes that automation is undertaken not because efficiency demands it or scientific advances allow it, but because it is a tool in the ceaseless war of capitalists against labor.

25.

The author of the passage is primarily concerned with

(A) (B) (C) (D) (E)

reexamining a political position and defending its validity examining a management decision and defending its necessity analyzing a scholarly study and pointing out a central weakness explaining a trend in automation and warning about its dangers chronicling the history of an industry and criticizing its development

26.

According to information in the passage, the term “de-skilling” refers to the

(A) (B) (C) (D) (E)

loss of skills to industry when skilled workers are replaced by unskilled laborers substitution of mechanized processes for labor formerly performed by skilled workers labor theory that automation is technologically comparable to skilled labor process by which skilled machinists “teach” machines to perform certain tasks exclusion of skilled workers from participation in the development of automated technology

314


27.

The author of the passage commends Noble’s book for which of the following?

(A) Concentrating on skilled as opposed to unskilled workers in its discussion of the machine-tool industry (B) Offering a generalization about the motives behind the machine-tool industry’s decision to automate (C) Making an essential distinction between two kinds of technology employed in the machine-tool industry (D) Calling into question the notion that managers conspired against labor in the automation of the machine-tool industry (E) Applying the concept of de-skilling to the machine tool industry

28. Which of the following best characterizes Forces of Production as it is described in the passage? (A) (B) (C) (D) (E)

A comparison of two interpretations of how a particular industry evolved An examination of the origin of a particular concept in industrial economics A study that points out the weakness of a particular interpretation of an industrial phenomenon A history of a particular industry from an ideological point of view An attempt to relate an industrial phenomenon in one industry to a similar phenomenon in another industry

29. Surveys indicate that 52 percent of all women aged eighteen to sixty-five are in the labor force (employed outside the home) in any given month. On the basis of these surveys, a market researcher concluded that 48 percent of all women aged eighteen to sixty-five are full-time homemakers year-round. Which of the following, if true, would most seriously weaken the researcher’s conclusion? (A) More women are in the labor force today than during any other period since the Second World War. (B) Many workers, both men and women, enter and exit the labor force frequently. (C) Although only a small sample of the total population is surveyed each month, these samples have been found to be a reliable indicator of total monthly employment. (D) Surveys show that more women than ever before consider having a rewarding job an important priority. (E) Women who are in the labor force have more discretionary income available to them than do women who are not.

30. Kansas Republican Nancy Kassebaum, one of only two women in the U.S. Senate in 1992, said she did not so much wish for more women senators but more moderate Republican ones. (A) did not so much wish for more women senators but more moderate Republican ones (B) wished not so much for more senators who were women than moderate Republicans

315


(C) did not wish so much for more women senators as for more moderate Republicans (D) did not wish for more women senators so much as moderate Republicans (E) wished for more senators who are moderate Republicans than women

31. After observing the Earth’s weather patterns and the 11-year sunspot cycle of the Sun for 36 years, scientists have found that high levels of sunspot activity precede shifts in wind patterns that affect the Earth’s weather. One can conclude that meteorologists will be able to improve their weather forecasts based on this information. Which of the following, if true, most seriously weakens the argument above? (A) Weather forecasts are more detailed today than they were 36 years ago. (B) Scientists can establish that sunspot activity directly affects the Earth’s weather. (C) Evidence other than sunspot activity has previously enabled meteorologists to forecast the weather conditions that are predictable on the basis of sunspot activity. (D) Scientists have not determined why the sunspot activity on the Sun follows an 11-year cycle. (E) It has been established that predictable wind patterns yield predictable weather patterns.

32. Mannis Corporation’s archival records are stored in an obsolete format that is accessible only by its current computer system; thus they are inaccessible when that system is not functioning properly. In order to avoid the possibility of losing access to their archival records in the case of computer malfunction, Mannis plans to replace its current computer system with a new system that stores records in a format that is accessible to several different systems. The answer to which of the following questions would be most helpful in evaluating the effectiveness of the plan as a means of retaining access to the archival records? (A) Will the new computer system require fewer operators than the current system requires? (B) Has Mannis Corporation always stored its archival records in a computerized format? (C) Will the new computer system that Mannis plans ensure greater security for the records stored than does Mannis’ current system? (D) Will Mannis’ current collection of archival records be readily transferable to the new computer system? (E) Will the new computer system be able to perform many more tasks than the current system is able to perform?

33. No matter how patiently they explain their reasons for confiscating certain items, travelers often treat customs inspectors like wanton poachers rather than government employees. (A) travelers often treat customs inspectors like wanton poachers rather than government employees. (B) travelers often treat customs inspectors as wanton poachers instead of government employees. (C) travelers often treat customs inspectors as if they were not government employees but wanton poachers.

316


(D) customs inspectors are often treated by travelers as if they were wanton poachers rather than government employees. (E) customs inspectors are often treated not like government employees but wanton poachers by travelers.

34. One noted economist has made a comparison of the Federal Reserve and an automobile as racing through a tunnel, bouncing first off one wall, then the other: the car may get where it is going, but people may be hurt in the process. (A) made a comparison of the Federal Reserve and an automobile as racing through a tunnel, bouncing (B) made a comparison between the Federal Reserve and an automobile racing through a tunnel, bouncing (C) compared the federal Reserve with an automobile as racing through a tunnel and which bounced (D) compared the Federal Reserve to an automobile racing through a tunnel, bouncing (E) compared the Federal Reserve with an automobile that races through a tunnel and it bounces

35. Puritan fanatics brought to civil and military affairs a coolness of judgment and mutability of purpose that some writers have though inconsistent with their religious zeal, but which was in fact a natural outgrowth of it. (A) (B) (C) (D) (E)

but which was in fact a natural outgrowth of it but which were in fact a natural outgrowth of it but which were in fact natural outgrowths of it but it was in fact a natural outgrowth of them which was in fact a natural outgrowth of it

36. Numismatist: In medieval Spain, most gold coins were minted from gold mined in West Africa, in the area that is now Senegal. The gold mined in this region was the purest known. Its gold content of 92 percent allowed coins to be minted without refining the gold, and indeed coins minted from this source of gold can be recognized because they have that gold content. The mints could refine gold and produced other kinds of coins that had much purer gold content, but the Senegalese gold was never refined. Which one of the following inferences about gold coins minted in medieval Spain is most strongly supported by the information the numismatist gives? (A) Coins minted from Senegalese gold all contained the same weight, as well as the same proportion of gold. (B) The source of some refined gold from which coins were minted was unrefined gold with a gold content of less than 92 percent. (C) Two coins could have the same monetary value even though they differed from each other in the percentage of gold they contained.

317


(D) No gold coins were minted that had a gold content of less than 92 percent. (E) The only unrefined gold from which coins could be minted was Senegalese gold.

37. A discussion of our nation’s foreign policy must begin with the fact of there being an independent Western Europe which now thinks of itself in trans-nationalist terms. (A) (B) (C) (D) (E)

A discussion of our nation’s foreign policy must begin with the fact of there being Beginning any discussion of our nation’s foreign policy must be the fact of there being Any discussion of our nation’s foreign policy must begin with the fact that there is Any discussion of our nation’s foreign policy must begin by acknowledging the existence of To begin discussing our nation’s foreign policy there must be an acknowledgment of the fact that

38. A private house in New York City is a building owned by an individual or individuals having less than eight units and no commercial space. (A) a building owned by an individual or individuals having less than eight units and no commercial space. (B) one that an individual or individuals own with fewer than eight units and no commercial space. (C) a building with fewer than eight units, no commercial space, and is owned by an individual or individuals. (D) one that has fewer than eight units, no commercial space and it is owned by an individual or individuals. (E) one that has fewer than eight units, is owned by an individual or individuals, and has no commercial space.

39. Asset allocators create portfolios, often in the form of mutual funds, with the intention to turn in good results in both “bull” and “bear” markets. (A) (B) (C) (D) (E)

with the intention the intention of which is intended and intending so intended as

40. Nearly all mail that is correctly addressed arrives at its destination within two business days of being sent. In fact, correctly addressed mail takes longer than this only when it is damaged in transit. Overall, however, most mail arrives three business days or more after being sent. If the statements above are true, which one of the following must be true? (A) A large proportion of the mail that is correctly addressed is damaged in transit.

318


(B) (C) (D) (E)

No incorrectly addressed mail arrives within two business days of being sent. Most mail that arrives within two business days of being sent is correctly addressed. A large proportion of mail is incorrectly addressed. More mail arrives within two business days of being sent than arrives between two and three business days after being sent.

41.

An advertisements states:

Like Danaxil, all headache pills can stop your headache. But when you are in pain, you want relief right away. Danaxil is for you—no headache pill stops pain more quickly. Evelyn and Jane are each suffering from a headache. Suppose Evelyn takes Danaxil and Jane takes its leading competitor. Which one of the following can be properly concluded from the claims in the advertisement? (A) (B) (C) (D) (E)

Evelyn’s headache pain will be relieved, but Jane’s will not. Evelyn’s headache pain will be relieved more quickly than Jane’s. Evelyn’s headache will be relieved at least as quickly as Jane’s. Jane’s headache pain will be relieved at the same time as is Evelyn’s. Jane will be taking Danaxil for relief from headache pain.

Test 2 1. According to a recent study, advertisements in medical journals often contain misleading information about the effectiveness and safety of new prescription drugs. The medical researchers who wrote the study concluded that the advertisements could result in doctors prescribing inappropriate drugs to their patients. The researchers' conclusion would be most strengthened if which of the following were true? (A) Advertisements for new prescription drugs are an important source of revenue for medical journals. (B) Editors of medical journals are often unable to evaluate the claims made in advertise ments for new prescription drugs. (C) Doctors rely on the advertisements as a source of information about new prescription drugs. (D) Advertisements for new prescription drugs are typically less accurate than medical journal articles evaluating those same drugs. (E) The Food and Drug Administration, the government agency responsible for drug regulation, reviews advertisements for new drugs before the ads have been printed.

2. To prevent some conflicts of interest, Congress could prohibit high-level government officials from accepting positions as lobbyists for three years after such officials leave government service.

319


One such official concluded, however, that such a prohibition would be unfortunate because it would prevent high-level government officials from earning a livelihood for three years. The official’s conclusion logically depends on which of the following assumptions? (A) Laws should not restrict the behavior of former government officials. (B) Lobbyists are typically people who have previously been high-level government officials. (C) Low-level government officials do not often become lobbyists when they leave government service. (D) High-level government officials who leave government service are capable of earning a livelihood only as lobbyists. (E) High-level government officials who leave government service are currently permitted to act as lobbyists for only three years.

3. Uninformed about students' experience in urban classrooms, critics often condemn schools' performance gauged by an index, such as standardized test scores, that are called objective and can be quantified and overlook less measurable progress, such as that in higher-level reasoning. (A) an index, such as standardized test scores that are called objective and can be quantified and overlook less measurable progress such as that (B) an index, such as standardized test scores that are called objective and can be quantified and overlook less measurable progress such as what is made (C) an index, such as standardized test scores, that is called objective and can be quantified and overlook less measurable progress such as what is made (D) a so-called objective index, such as standardized test scores, that can be quantified; and overlook less measurable progress, such as what is made (E) a so-called objective index, such as standardized test scores, that can be quantified and overlook less measurable progress, such as that

4. A Common social problem in the workplace occurs when workers accept supervisory positions, and it causes them to lose the trust of their former co-workers. (A) (B) (C) (D) (E)

when workers accept supervisory positions, and it causes them to lose by a worker accepting supervisory positions, which causes him to lose when workers accept supervisory positions, and so lose when a worker who accepts a supervisory position, thereby losing if a worker accepts a supervisory position, they would lose

An important feature of the labor market in recent years has been the increasing participation of women, particularly married women. Many analysts suggest, however, that women comprise a secondary labor market where rates of pay and promotion prospects are inferior to those available to men. The principal reason is that women have, or are assumed to have, domestic responsibilities which compete with paid employment. Such domestic responsibilities are strongly influenced by social values which require women to give priority to |home and family over paid employment. The difficulties which women face in the labor market and in their ability to reach senior

320


positions in organizations are accentuated with the arrival of children. In order to become full-time employees, women with children must overcome the problems of finding good, affordable childcare and the psychological barriers of workplace marginality. Some women balance domestic and workplace commitments by working part-time. However, part-time work is a precarious form of employment. Women part-timers are often the first laid off in a difficult economy. These workers are often referred to as the "reserve army" of female labor. One researcher has found that approximately 80 percent of women in their twenties who have children remain at home. Such women who later return to work represent another sector of the workforce facing difficulties. When the typical houseworker returns to the labor market she is unsure of herself in her new environment. This doubt is accentuated by her recent immersion in housework, a very private form of work. Without recent employment experience, these women confront a restricted range of opportunities and will almost certainly be offered low-status jobs with poor prospects. Even women professionals who interrupt their careers to have children experience difficulties. Their technical skills may become rusty or obsolete, important networks of business contacts are broken, and their delayed return to work may mean that they are likely to come up for promotion well after the age that would be otherwise normal. Consequently, women, even those of high ability, may find themselves blocked in the lower echelons of an organization, overlooked, or even "invisible" to senior management.

5.

The author of the passage is primarily concerned with

(A) (B) (C) (D) (E)

advocating changes in employers' practices towards women with children examining some of the reasons women rarely reach the higher echelons of paid labor describing the psychological consequences for women of working outside the home taking issue with those who believe women should not work outside the home analyzing the contribution of women to industry and business

6. The passage provides information to support which of the following statements about women workers? (A) It is the responsibility of employers to provide childcare accommodations for women workers with children. (B) Women in high-status positions are easily able to integrate career and children. (C) Conditions for working mothers are much better today than they were twenty years ago. (D) The decision to work outside the home is often the source of considerable anxiety for women with children. (E) With the expense of childcare, it is often not profitable for women with children to work.

7.

The author's discussion of women professionals in the last paragraph serves to

(A) show that the difficulties of integrating careers and motherhood can be overcome (B) indicate that even women of higher status are not exempt from the difficulties of integrating careers and children (C) defend changes in the policies of employers towards working mothers

321


(D) modify a hypothesis regarding the increased labor force participation of women (E) point out the lack of opportunities for women in business

8. According to the passage, men generally receive higher salaries and have a better chance of being promoted because women (A) (B) (C) (D) (E)

tend to work in industries that rely almost exclusively on part-time labor lack the technical and managerial experience of their male counterparts have responsibilities outside of the workplace that demand considerable attention are the first to be laid off when the economy grows at a very slow pace suffer discrimination in the male-dominated corporate environment

9. A state legislator argues that her state's ban on casino gambling is inconsistent and impractical, since other forms of gambling such as bingo and the state lottery are legal. She claims that instead of vainly attempting to enforce the ban, the legislature should simply legalize all gambling, and that to do so would also have the positive effect of reducing the crime rate. Which of the following, if true, most seriously weakens the legislator's argument above? (A) Since many people enjoy the thrill of participating in illegal practices, legalizing gambling would probably cause a decline rather than an increase in this activity. (B) Because prosecutors rarely seek prison terms for illegal gamblers, legalizing gambling would not result in a significant savings of money. (C) Long-term studies have shown that the number of people who participate in the lottery is higher now than it was when the lottery was prohibited. (D) Legalizing gambling would entice gamblers from states where it is still banned, and many of them are involved in other illegal activities such as drug smuggling. (E) Many people who participate in illegal gambling claim that they would risk their money on the stock market if they had more disposable income.

10. A researcher studying cats discovered that during the dream state of sleep, the cerebral cortex of a cat's brain fires messages to its body as rapidly as it does during wakefulness. In an effort to determine why the sleeping cat's body does not respond to the messages being fired by the brain, the researcher removed a cluster of neurons from a sleeping cat's brain stem, the part of the brain that connects the cerebral cortex to the spinal cord. After he had done so, the still sleeping cat got up, pounced as if it were chasing a mouse, and arched its back. Which of the following, if true, taken together with the information above, best supports the conclusion that the sleeping cat was acting out its dreams? (A) (B)

322

The neurons that were removed from the brain stem normally serve to trigger the dream state of sleep and the rapid brain activity that accompanies it. The cerebral cortex is able to receive and transmit sensory information even when the brain is in a sleeping state.


(C) (D) (E)

The neurons that were removed from the brain stem are normally responsible for transmitting messages from the cerebral cortex. The neurons that were removed from the brain stem normally prevent messages fired by the cerebral cortex during sleep from being received by the spinal cord. The types of brain waves produced by the cerebral cortex during sleep have distinctly different properties from those produced during a wakeful state.

11. Glaciologists believe that the frozen corpse recently found in a melting Alpine glacier, apparently that of a shepherd who is thought to have lived about 4,600 years ago, was preserved uncrushed by snow and ice because of the body's unique topographical position. (A) (B) (C) (D) (E)

apparently that of a shepherd who is thought to have lived that of a shepherd, apparently, who was thought to live that of an apparent shepherd who was thought to live that of a shepherd who is thought of as apparently living that of a shepherd who was apparently thought to live

12. In contrast to Walt Whitman, Ezra Pound considered that late 19th-century American is not a distinct formal repertoire informed by its own ideology, and is essentially an outgrowth the British poetic tradition. (A) Ezra Pound considered that late 19th-cetury American poetry is not a distinct formal repertoire informed by its own ideology, and is (B) Ezra Pound considered late 19th-century American poetry not as a distinct for repertoire informed by its own ideology but (C) Ezra Pound considered late 19th-century American poetry not a distinct formal repertoire informed by its own ideology, but (D) it was considered by Ezra Pound that lat 19th-century American poetry is not a distinct formal repertoire informed by its own ideology, but (E) late 19th-century American poetry was considered by Ezra Pound not to be a distinct formal repertoire informed by its own ideology, and is

13. Reva: Using extraneous incentives to get teenagers to change their attitude toward school and schoolwork won’t work. Take the program in West Virginia, for instance, where they tried to reduce their dropout rate by revoking the driving licenses of kids who left school. The program failed miserably. Anne: It’s true that the West Virginia program failed, but many schools have devised incentive programs that have been very successful in improving attendance and reducing discipline problems. According to Anne, the weak point in Reva’s claim is that it

323


(A) fails to consider the possibility that the majority of potential dropouts in West Virginia do not have driving licenses (B) doesn’t provide any exact figures for the dropout rate in West Virginia before and during the program (C) ignores a substantial body of evidence showing that parents and employers have been using extrinsic incentives with positive results for years (D) assumes that a positive incentive—a prize or a reward—will be no more effective than a negative incentive, like the revoking of a driving license (E) is based on a single example, the incentive program in West Virginia, which may not be typical

14. It’s time we stopped searching for new statistics to suggest that we are not spending enough on education. In fact, education spending increased 30 percent overall during the last decade. Which of the following, if true, would most weaken the argument above? (A) Despite increased spending on education, enrollment in our elementary and secondary schools declined about 4 percent during the last ten years. (B) Our spending on gasoline increased more than 100 percent during the last decade. (C) When adjusted for inflation, our per-pupil expenditure on education this year is less than it was ten years ago. (D) Eleven other economically developed nations spend more on education than we do. (E) The achievement levels of our students have been declining steadily since 1960, and the last decade produced no reversal in this trend.

15. Local reporters investigating the labor dispute reported that only half of the workers in the plant were covered by the union health plan; at least as much as a hundred and more others had not any health insurance whatsoever. (A) (B) (C) (D) (E)

at least as much as a hundred and more others had not any at least as much as more than a hundred others had no more than a hundred others had not any more than a hundred others had no there was at least a hundred or more other without any

16. According to a commonly held archaeological theory, the Neanderthals of Europe, an archaic version of Homo sapiens, competed with and were eventually replaced by modern humans, with little or no interbreeding between the two populations. A rival theory, developed more recently, suggests that Neanderthals were more similar to modern humans than previously supposed — that, in fact, modern humans evolved from them and from other archaic versions of Homo sapiens. Evidence that would strongly support the more recent theory concerning the relationship between Neanderthals and modern humans would be

324


(A) DNA analyses indicating that modern humans appeared in Africa 200,000 years ago, before migrating to Europe and other continents (B) archaeological evidence that Neanderthals and modern humans developed similar cultures, shared stone tools, and performed similar burial rituals (C) skulls of early modern humans in central Europe that exhibit a bone near the mandibular nerve that is a typical Neanderthal characteristic (D) evidence that the stone tools of Neanderthals remained unchanged for thousands of years, while the tools of modern humans in Europe were more specialized (E) biological evidence that Neanderthals had unique physical traits that enabled them to survive ice-age temperatures in Europe

17. Archaeologists have shown that ingesting lead in drinking water was a significant health hazard for the ancient Romans, like that of modern Americans. (A) (B) (C) (D) (E)

like that of modern Americans. as that for modern Americans. just as modern Americans do. as do modern Americans. as it is for modern Americans.

18. Born Nathan Weinstein in New York City on October 17, 1903, Nathanael West's first novel, The Dream Life of Balso Snell, was written during a stay in Paris and published when the author was twenty-eight. (A) Nathanael West's first novel, The Dream Life of Balso Snell, was written during a stay in Paris and published when the author (B) Nathanael West's first novel, The Dream Life ofBalso Snell, written while he was staying in Paris, was published when the author (C) Nathanael West's The Dream Life of Balso Snell, his first novel, was written while the author was staying in Paris and published when he (D) Nathanael West wrote his first novel, The Dream Life of Balso Snell, during a stay in Paris and published it when he (E) when Nathanael West was staying in Paris, he wrote his first novel, The Dream Life of Balso Snell, publishing it when he

19. Aggressive fertility treatments are not responsible for the rise in the incidence of twin births. Rather, this increase can be attributed to the fact that women are waiting longer to become mothers. Statistically, women over 35 are more likely to conceive twins, and these women comprise a greater percentage of women giving birth than ever before. The argument above is flawed in that it ignores the possibility that (A) many women over 35 who give birth to twins are not first-time mothers (B) women over 35 are not the only women who give birth to twins

325


(C) the correlation between fertility treatments and the increased incidence of multiple births may be a coincidence (D) on average, women over 35 are no more likely to conceive identical twins than other women are (E) women over 35 are more likely to resort to the sorts of fertility treatments that tend to yield twin births

Modern methods of predicting earthquakes recognize that quakes, far from being geologic anomalies, are part of the periodic accumulation and discharge of seismic energy. As continents receive the horizontal thrust of seafloor plates, crustal strains develop. Accumulation of crustal strains can take anywhere from 100 years in certain coastal locations to over a millennium in a inland regions before a critical point is reached and a rupture occurs. In both areas, the build of strain is accompanied by long- and short-range precursory phenomena that are crucial earthquake prediction. Quakes along active faults—like those along Pacific coasts—are usually frequent; scientists designate such areas as quake-prone. Geological scars of past subsidence, cracks, and offsets are useful in determining potential quake locations, as are seismicity gaps, areas where no small quakes have been recorded. Seismologists may also consult the historical record. Primary sources range from eyewitness accounts of ancient quakes to recent official documentation of quake-related damage. Once the perimeters of a quake-prone zone are established, a network of base stations can monitor precursory phenomena. Stations must extend over a wide area, yet be placed at measured intervals to obtain precise readings. Changes in geochemical readings (electric rents, radon concentrations) and in groundwater levels, as well as the occurrence of micro earthquakes, are valuable precursors. Crustal movements—tilting, rising, and expansion or contraction of the ground surface – can be read through triangulation and leveling surveys taken over the course of decades. Theoretically, if an area’s critical strain is known — the magnitude of strain necessary to produce a rupture — subtracting the measured accumulated crustal strain from the critical strain will indicate a time frame for an impending quake. Violent tilting and foreshocks are among phenomena classified as short-term precursors. Many are still being identified as new quakes occur. Such precursors are valuable since their appearance can permit prediction of a quake to within hours of the primary rupture. Here, too, historical documents are useful. Seismologists recognized the liquefaction of sand as a precursor after a 1964 quake in Japan.

20. According to the passage, a major difference between coastal regions and inland regions is that in coastal regions (A) (B) (C) (D) (E)

crustal strain does not occur earthquakes are less numerous critical points are reached more quickly precursory phenomena are seldom observed seafloor plate action is less powerful

21.

The primary purpose of the passage is to

(A) clarify the way in which earthquakes develop in inland locations

326


(B) (C) (D) (E)

show that earthquakes are a result of the normal accumulation and discharge of seismic energy discuss the accumulation of crustal strain in coastal regions argue that precursory phenomena should be disregarded in attempts at quake prediction describe methods of earthquake prediction and explain the importance of precursory phenomena

22.

The primary function of the third paragraph is to

(A) (B) (C) (D) (E)

explain the relationship between accumulated and critical strain describe the use of precise intervals in establishing networks of base stations summarize the differences between earthquakes in coastal and inland regions outline some of the methods used by seismologists to predict earthquakes suggest that critical strain is not spread evenly along most major fault lines

23.

According to the passage, knowledge of an area's critical strain can help seismologists

(A) (B) (C) (D) (E)

estimate the date of a future earthquake calculate the severity of an initial rupture measure the seismic force along a fault revise the distances between base stations predict the rate of future crustal movement

24. Until the Federal government began providing low-cost flood insurance to coastal property owners, construction along beaches was limited by owners' fears that their property would be washed away. Since the insurance was made available, however, beachfront construction has boomed and land erosion has increased at a dangerous rate. Which of the following, if feasible, offers the best prospects for the Federal government to put a stop to the problem of land erosion along beaches? (A) prohibiting beachfront property owners from embellishing or adding to existing buildings (B) utilizing computer science techniques to obtain detailed information on the extent and rapidity of land erosion along beaches (C) enacting building codes requiring new beachfront structures in flood-threatened areas to be elevated above the high water level of a storm (D) compensating beachfront property owners for moving to a new location off the coast while canceling flood insurance benefits for any new or remaining beachfront construction (E) requiring beachfront property owners receiving flood insurance coverage to adopt construction standards that will protect their buildings from inundation

25. People with Williams syndrome, a rare mental disorder, are often highly articulate and sensitive. Not uncommonly, they are gifted in music and possess rich vocabularies. Yet these same people because of their lack of ability in basic arithmetic and difficulty distinguishing left from right, are

327


misleadingly labeled mentally retarded. As evaluated by conventional means such as IQ tests their intelligence is no higher than that of people with Down's syndrome, despite the fact that people with Down's syndrome have uniformly limited cognitive abilities and show no specialized aptitudes. The author is arguing that (A) conventional methods of measuring intelligence, such as IQ tests, are inadequate for evaluating the capabilities of people with certain mental disorders such as Williams syndrome (B) people with Down's syndrome usually have less verbal and musical ability but more mathematical and spatial ability than do people with Williams syndrome (C) conventional methods of measuring intelligence tend to consider basic mathematical and spatial ability to be more important than verbal and musical skills (D) people with Williams syndrome are only rarely given the opportunity to develop their unique musical and verbal abilities (E) people with Williams syndrome need greater encouragement if they are to develop their mathematical spatial skills

26. When the nineteenth-century German bacteriologist Robert Koch identified a particular bacterium as responsible for cholera, Max von Pettenkoffer, a physician, expressed his skepticism by voluntarily drinking an entire bottle of the allegedly responsible bacteria. Although von Pettenkoffer took his failure to come down with the disease as a refutation of Koch's hypothesis that cholera was caused by bacteria, Koch argued that von Pettenkoffer had been protected by his own stomach acid. The acid secreted by the stomach, Koch explained, kills most ingested bacteria. Which of the following, if true, provides the most evidence to support Koch's counterargument? (A) Peptic ulcers, often associated with excessive secretions of stomach acid, are common in certain areas characterized by high rates of cholera. (B) As von Pettenkoffer later admitted that he had previously had cholera, it is probable that he had developed antibodies that protected him from a second attack. (C) Cholera is endemic in areas in which poor sanitation results in high concentrations of cholera bacteria in drinking water. (D) Although stomach acid kills most ingested bacteria, large numbers of E. coli bacteria nonetheless manage to make their way to the lower intestine of the digestive tract. (E) Cholera bacteria ingested with bicarbonate of soda, a neutralizer of stomach acid, is more likely to result in cholera than if the bacteria is ingested alone.

27. The Limon Dance Company believes that, since the death of Jose" Lim6n in 1972, they have and will continue to perpetuate the shared artistic vision of Limon and his mentor and collaborator Doris Humphrey, who both choreographed works in the company's active repertory. (A) they have and will continue to perpetuate the shared artistic vision of Limon and his mentor and collaborator Doris Humphrey, (B) they have and will continue to perpetuate Lim6n and his mentor and collaborator Doris Humphrey's shared artistic vision,

328


(C)

it has and will continue to perpetuate the shared artistic vision of Limon and his mentor and collaborator Doris Hurnphrey, (D) it has perpetuated and will continue perpetuating the artistic vision that Lim6n and his mentor and collaborator Doris Humphrey shared, (E) it has continued to perpetuate the shared artistic vision of Limon and his mentor and collaborator Doris Humphrey,

28.

Agencies studying discrimination in housing have experimentally proved that minority clients are often discouraged as prospective buyers of residential real estate and the antidiscrimination legislation of recent decades were only mitigating, rather than abolishing, inequity in housing practices.

(A) the antidiscrimination legislation of recent decades were only mitigating, rather than abolishing, inequity in housing practices (B) in recent decades, the antidiscrimination legislation only mitigated, rather than abolishing, inequity in housing practices (C) that antidiscrimination legislation of recent decades has only mitigated, rather than abolished, inequity in housing practices (D) that, in recent decades, antidiscrimination legislation has only mitigated, rather than abolishing, housing practices' inequity (E) that recent decades' antidiscrimination legislation only were mitigating, rather than abolishing, housing practices' inequity

29. Citing the legal precedent set by asbestos exposure cases, a state judge agreed to combine a series of workplace disability cases involving repetitive stress injuries to the hands and wrists. The judge's decision to consolidate hundreds of suits by data entry workers, word processors, newspaper employees, and other workers who use computers into one case is likely to prove detrimental for the computer manufacturing companies being sued, notwithstanding the defense's argument that the cases should not be combined because of the different individuals and workplaces involved. Which of the following, if true, casts the most serious doubt on the validity of the judge's decision to consolidate the cases? (A) Unlike asbestos exposure cases, in which the allegedly liable product is the same in each situation, the type and quality of the allegedly liable office equipment is different in each case. (B) The fact that consolidation will accelerate the legal process may prove advantageous for the defense, as it limits the number of witnesses who can testify for the plaintiffs. (C) One of the most common causes of repetitive stress injuries is companies' failure to allow its employees adequate rest time from using computer keyboards. (D) Whereas exposure to asbestos often leads to fatal forms of cancer, repetitive stress injury typically results in personal discomfort and only rarely in unemployability. (E) The issue of responsibility for repetitive stress injury cannot be resolved without first addressing the question of its existence as an actual medical condition.

329


30. Each of William Kennedy's novels in the "Albany Trilogy"—Ironweed, Legs, and Billy Phelaris Greatest Game—are set in the area around Albany, New York, a region whose history also suggested some details of the novels' plots. (A) Each of William Kennedy's novels in the "Albany Trilogy"—Ironweed, Legs, and Billy Phelaris Greatest Game—are set in the area around Albany, New York, (B) Ironweed, Legs, and Billy Phelaris Greatest Game—each of them novels in William Kennedy's "Albany Trilogy"—are set in the area around Albany, New York, (C) William Kennedy's "Albany Trilogy" novels—Ironweed, Legs, and Billy Phelaris Greatest Game—are all set in the area around Albany, New York, (D) Novels by William Kennedy—Ironweed, Legs, and Billy Phelaris Greatest Game— each one of the "Albany Trilogy" novels, is set in the area around Albany, New York, (E) Novels by William Kennedy—Ironweed, Legs, and Billy Phelaris Greatest Game— every one of the "Albany Trilogy" novels are set in the area around Albany, New York,

31. Between 1977 and 1989, the percentage of income paid to federal taxes by the richest one percent of Americans decreased, from 40 percent to 25 percent. By the end of that same period, however, the richest one percent of Americans were paying a larger proportion of all federal tax revenues, from 12.7 percent in 1977 to 16.2 percent in 1989. Which of the following, if true, contributes most to an explanation of the discrepancy described above? (A) Between 1977 and 1989, the Internal Revenue Service increased the percentage of its staff members responsible for audits and tax collection. (B) Between 1977 and 1989, the before-tax income of the richest one percent of Americans increased by over 75 percent when adjusted for inflation. (C) Between 1977 and 1989, many of the richest one percent of Americans shifted their investments from taxable to untaxable assets. (D) Between 1977 and 1989, the tax rate paid by middle-income Americans was reduced, but several tax loopholes were eliminated. (E) Between 1977 and 1989, the amount of federal taxes paid by the richest one percent of Americans increased by $45 billion, while the amount paid by all Americans rose by $50 billion. Contamination is the unintended presence of harmful substances or organisms in food. While it is true that recent scientific advances have Line resulted in safer foods, better methods of preservation, and improved storage practices, it is still necessary to guard against the practices that can increase the likelihood of food contamination. Because foodborne illness poses a potentially serious threat to public health, preventing contamination of safe food needs to be a prime objective of every foodservice manager. Furthermore, a foodservice manager must possess accurate information on the different hazards associated with the contamination of food in the event that a foodborne illness crisis does arise. A full understanding of the biological, chemical, and physical hazards allows the food-service manager to implement the control measures necessary to minimize the health risks associated with food, and thus to decrease the possibility of contamination. The most serious risk associated with food is the biological hazard. Biological hazards are dangers to food from pathogenic (disease-causing) microorganisms, such as bacteria, viruses, parasites, and fungi, and from toxins, that occur in certain plants and fish. When biological hazards

330


result in foodborne illnesses, these illnesses are generally classified as either infections or intoxications. A foodborne infection is a disease that results from eating food containing living harmful microorganisms. One of the most frequently reported diseases of this type is Salmonellosis, which results from the consumption of food contaminated with live pathogenic Salmonella. The other major form of biologically induced foodborne illness is the foodborne intoxication, which results when toxins, or poisons, from bacterial or mold growth are present in ingested food and cause illness in the host (the human body). These toxins are generally odorless and tasteless, and are capable of causing disease even after the microorganisms have been kills; Staphylococcus food intoxication is one of the most common types of foodborne illness reported in the United States. 32.

Which of the following best expresses the idea of the passage?

(A) Despite recent scientific advances, foodborne illness continues to present a serious risk to public health (B) Although chemical and physical hazard can cause a foodborne illness, biologic hazards pose the most serious risk of food contamination. (C) Knowledge of contamination sources is essential for a foodservice manager to safely operate a food establishment. (D) Biological, chemical, and physical hazard represent the main sources of food contamination. (E) The illnesses caused by the contaminatior of food by biological hazards take the form of either a foodborne infection or a foodborne intoxication.

33. The author of the passage would most likely agree that a foodservice manager's comprehension of the nature of potential food hazards is (A) (B) (C) (D) (E)

crucial to the safety of a foodservice operation necessarily limited due to the complexity of contamination sources the primary factor in an employer's decision to hire that manager utilized exclusively for the prevention of foodborne illness vitally important but nearly impossible to attain

34.

According to the passage, pathogenic microorganisms

(A) (B) (C) (D) (E)

are the most common form of biological hazard can only trigger a foodborne illness when alive are toxins that occur in certain plants and fish include life forms such as bacteria and parasites are difficult to detect because they are odorless and tasteless

35. Still employing the system of binomial nomenclature devised in the 18th century by Linnaeus, new technology enables modern day biological taxonomists to not only classify species, but to sort their evolutionary relationships by an approach that analyzes the sequences of DNA.

331


(A) new technology enables modern day biological taxonomists to not only classify species, but to sort (B) modern day biological taxonomists using new technology can not only classify species, but also sort (C) using new technology enables modern day biological taxonomists to not only classify species, but they also sort (D) using new technology not only in classifying species, modern-day biological taxonomists are enabled to also sort them for (E) when modern day biological taxonomists are enabled by new technology, not only do they classify species, they also can sort

36. American executives, unlike their Japanese counterparts, have pressure to show high profits in each quarterly report, with little thought given to long-term goals. (A) (B) (C) (D) (E)

have pressure to show are under pressure to show are under pressure of showing are pressured toward showing have pressure that they should show

37. The impact of the 1930s crisis on the different regions of Country X varied depending on the relationship of each region's economy to the international marketplace, with Region A most drastically affected. Interestingly, demand in foreign markets for Region As tropical crops was only slightly affected by the drop in income levels after 1929; the same was true of foreign demand for the temperate-zone basic foodstuffs produced by Region B. However, Region B was better able to survive the crisis because it could adjust the supply of its crops. Since Region A could not, its economy was devastated by the slight decrease in foreign demand. Which one of the following provides the most reasonable explanation for the fact that Region A's economy was more drastically affected by the slight decrease in demand than was Region B`s? (A) Tropical crops like those produced by Region A usually command higher prices on the world market than do basic foodstuffs like those produced by Region B. (B) Region B"s economy was dependent on annual crops, the supply of which is easily adjusted because the plants are renewed each year, in contrast to the perennial crops grown in Region A. (C) Because tropical goods are generally bought by more affluent consumers, demand for these products rarely declines even when overall income levels drop. (D) The temperate-zone basic foodstuffs produced in Region B directly competed with similar crops produced by the countries that imported Region Fs goods. (E) Because Region B's economy was dependent on the export of basic foodstuffs, there was only a slight decline in demand for its goods even after income levels dropped.

38. Two groups of laboratory mice were injected with cancerous cells. One group’s cages were rotated in a disorienting manner. Two-thirds of these mice developed cancers. One-tenth of

332


the mice in stationary cages developed cancers. The researchers concluded that stress enhances the development of cancer in laboratory mice. The researchers’ conclusion logically depends on which of the following assumptions? (A) (B) (C) (D) (E)

Rotating the cages in disorienting manner produced stress in the mice in those cages. The injections given to the two groups of mice were not of equal strength. Injecting the mice with cancerous cells caused stress in the mice. Even without the injections the mice in the rotated cages would have developed cancers. Even the rotation of cages in a manner that is not disorienting is likely to produce stress in mice in those cages.

39. In the late 19th century, when Vassar was a small recently founded women's college, founding professor and astronomer Maria Mitchell taught as many Astronomy majors in a given year as there are today, when Vassar is a larger, coeducational college. (A) when Vassar was a small recently founded women's college, founding professor astronomer Maria Mitchell taught as many Astronomy majors in a given year as there are today, when (B) when Vassar was a small, recently founded women's college, in a given year, founding professor and astronomer Maria Mitchell taught just as many Astronomy majors as there are in a given year today, when (C) while Vassar was a small, recently founded women's college, founding professor and astronomer Maria Mitchell taught a number of Astronomy majors in a given year such as there are today, when (D) while Vassar was a small, recently founded women's college, founding professor and astronomer Maria Mitchell taught such a number of Astronomy majors in a given year as are there today, whereas (E) when Vassar was a small, recently founded women's college, founding professor and astronomer Maria Mitchell taught a number of Astronomy majors just as large in a given year as the number that is there today, while

40. The work of short fiction writer Charles Chesnutt reflects characteristic interests of his contemporary "local colorists" as much as the intellectual ferment and historical reassessments of Black American culture during the late 19th century. (A) as much as the intellectual ferment and historical reassessments of Black American culture during the late 19th century (B) as much as it did the intellectual ferment in, and historical reassessments of, Black American culture in the late 19 th century (C) as much as it had reflected, during the late 19th century, the intellectual ferment and historical reassessments of Black American culture (D) as much as it was reflective and characteristic of the intellectual ferment and historical reassessments of late 19th century Black American culture (E) as much as it does the intellectual ferment and historical reassessments of late 19th century Black American culture

333


41. Just as the various languages contributing to English broaden and enrich its expressive range with words as diverse as the Arabic simoon, the Greek zephyr, and the Native American chinook, so the many musical traditions coexisting in U.S. culture create unlimited possibilities for the fusion of musical styles. (A) so the many musical traditions coexisting in U.S. culture create unlimited possibilities for the fusion of musical styles (B) similarly, the coexistence of many musical traditions in U.S. culture create unlimited possibilities in the fusing of musical styles (C) the many musical traditions coexisting in U.S. culture are creating unlimited possibilities in musical styles' fusion (D) in the same way, possibilities for the fusion of musical styles are unlimited, owing to the many musical traditions that coexist in U.S. culture (E) so it is in U.S. culture, where the many coexistent musical traditions make it possible that unlimited fusion of musical styles may be created

334


Turn static files into dynamic content formats.

Create a flipbook
Issuu converts static files into: digital portfolios, online yearbooks, online catalogs, digital photo albums and more. Sign up and create your flipbook.